Download as pdf or txt
Download as pdf or txt
You are on page 1of 192

stportal.mahendras.

org

Quantitative
Aptitude
SALIENT FEATURES:

## Chapterwise Discussion
## Topicwise Rules and Examples
## Exercise based on previous papers.
## Easy and comprehensible language

Mahendra Publication Pvt. Ltd.


New Delhi

QUANTITATIVE APTITUDE 3
stportal.mahendras.org

Mahendra Publication Pvt. Ltd.

© Copyright Reserved
##No part of this issue can be printed in whole or in part without the written permission of the
publishers.
##All the disputes are subject to Delhi jurisdiction only.

Registered Office
Mahendra Publication Pvt. Ltd., 103, Pragatideep Building, Plot No. 08, Laxminagar,
District Centre, New Delhi - 110092, TIN-09350038898, w.e.f. 12-06-2014

Branch Office
Mahendra Publication Pvt. Ltd. E-42,43,44, Sector-7, Noida (U.P.)

For queries regarding promotion, distribution & advertisement, contact:-


E-mail-sales@mahendras.org
Ph.: 09208037962

Owned, printed & published by N.K. Jain


103, Pragatideep Building, Plot No. 08, Laxminagar, District Centre, New Delhi - 110092

Please send your suggestions and grievances to:-


Mahendra Publication Pvt. Ltd., CP-9, Vijayant Khand, Gomti Nagar Lucknow - 226010
E-mail:info@mahendras.org

4 QUANTITATIVE APTITUDE
stportal.mahendras.org

PREFACE
This gives us immense pleasure to present you the latest edition of this book. We thank you from the depth of our
hearts, for the love and affection given by you just from the beginning. Dear friends, change is the law of nature.
We must learn to tolerate the blows of time with patience and learn not only to endure, but also to expect, welcome,
and enjoy both the joys as well as the sorrows of life. We must do what we can to "get success." We have to operate
with the information and skills that are necessary for winning.
Today’s era is governed by technology. The technology has increased the pace of changing the world we see day by
day, and so the pattern of examination and criteria of selection has also changed. As we are aware that interview is
no more in various examinations so the written/objective exams are going to be tougher than earlier.
Every choice you make — including the thoughts you have — has consequences. When you start choosing the right
behaviour and thoughts — which will take a lot of discipline — you'll get the right outcomes.
Be aware of the factors that influence the way you see the world, so that you can deal with them and react against
them. You are your own most important resource for making your life work. Success is a moving target that must
be tracked and continuously pursued.
In this context we have updated this book keeping the forthcoming examination pattern in mind. This edition caters
to need of Quantitative Aptitude that is asked in Preliminary and Mains stages of most of the Banking and Insur-
ance examinations for various posts. This book is thoroughly based on the latest pattern in which time limit is given
for each section. Every chapter in this book describes the concept with the help of various examples and at the
end gives miscellaneous examples to clear the concepts. Examples have been solved with standard as well as short
methods where needed. At the end of each chapter exercises has been given to master the topic by solving variety
of exam oriented question and also include questions asked in recent Banking Exams. So we have now given a
complete focus on concept building and mastering each topic through this book.
We are truly dedicated to provide you the best among the rest. This book is an initiative from our side to make you
perfect in this subject.
We will be highly gratified, if this book helps students in getting selection.
Any suggestions related to the this book shall always be welcomed and we shall endeavor to incorporate them in
our upcoming issues.

Research Team

QUANTITATIVE APTITUDE 5
stportal.mahendras.org

CONTENTS
(Sr. No.) (Topic) (Page No.)
1. Simplification 7-18

2. Number Series 19-24

3. Percentage 25-32

4. Profit and Loss 33-41

5. Simple & Compound Interest 42-50

6. Average 51-56

7. Ratio & Proportion 57-64

8. Problems on Ages 65-71

9. Partnership 72-78

10. Mixture and Alligation 79-85

11. Time and Work 86-96

12. Pipe & Cistern 97-105

13. Speed, Time and Distance 106-113

14. Problems Based on Trains 114-121

15. Boat and Stream 122-126

16. Inequality 127-135

17. Permutation and Combination 136-145

18. Probability 146-152

19. Data Sufficiency 153-159

20. Data Interpretation 160-183

21. Mensuration 184-194

6 QUANTITATIVE APTITUDE
stportal.mahendras.org

CHAPTER

1 SIMPLIFICATION

(vii) Composite number : Natural numbers which have


Types of Numbers
more than two distinct factors are called composite
(i) Natural Numbers : Counting numbers are called numbers eg. (4, 6, 8, 10, 12, 14)
natural numbers. Thus 1, 2, 3, 4,5, 6,.......etc. are all
Note :
natural numbers.
1. 1 is neither prime nor composite.
(ii) Whole Numbers : All counting numbers together
with zero form the set of whole numbers. 2. 2 is the only even number which is a prime number.

Note : (viii) Co-Prime Numbers : Two natural numbers a and b


are said to be co-prime, if their H.C.F is 1 as-
(a) 0 is the only whole number which is not a natural
(2, 3), (4, 5), (7, 9), (8, 11), (9, 14)
number
Note:
(b) Except 0 every natural number is a whole
number. it is not necesssary to have a pair consisting prime
number to be co prime number.
Thus 0, 1, 2, 3, 4, 5, 6,........are whole numbers.
(iii) Integers : When negative value of natural number Important Algebric Identities
are included with the whole number these number (a+b)2 = a2 +2ab +b2
are known as integers. in other words numbers which
(a-b)2 = a2-2ab+b2
have no fractional part are integer
a2-b2 = (a+b).(a-b)
e.g. {--&4] &3] &2] &1] 0] 1] 2] 3] 4] 5--}
(a+b)3 = a3 +b3 + 3ab (a+b)
(a) Positive Integers : All integers greater than 0
(a-b)3 = a3 -b3 - 3ab (a-b)
are known as positive integers.
a3+b3 = (a+b) (a2 +b2 -ab)
{1, 2, 3, 4, 5.............}
a3 - b3 = (a-b) (a2 +b2 +ab)
(b) Negative Integers : All the integers less than
0 is known as negative integers. Fraction
{-1, -2, -3, -4......} When we divide 50 rupees among 10 people each will get 5.
But what will happen if number of people increased by 2?
Note :
Every one will get 4 rupees after which 2 rupees will left. If
0 is called null integer as it is neither positive nor we divide this 2 rupees among 12 people each one will get
negative some part of rupees i.e. some paise.
(iv) Even Numbers : A natural number which is divisible This is fraction.
by 2 is called an even number. If unit digit a number is
0, 2, 4, 6 then the number is divisible by 2 and hence 5 2
Example ,23
even number 7

Thus 0, 2, 4, 6, 8, 10........etc. are all even numbers.


Classification of Fraction :
(v) Odd Numbers : A natural number which is not
(i) Proper fraction - A fraction whose numerator is less than
divisible by 2 is called an odd number.
denominator or in others words a fraction whose value is less
Thus 1, 3, 5, 7, 9.......etc. are odd numbers. than one is called proper fraction.
(vi) Prime Numbers : A natural number which has only 1 2 15
e.g. , ,
two factor (one and itself) are called prime number. 2 5 23
2 is smallest prime number (ii) Improper fraction - A fraction whose numerator is more
There are 15 prime numbers upto 50 and 25 prime than denominator or in others words a fraction whose value
is more than one is called improper fraction.
numbers upto 100.

QUANTITATIVE APTITUDE 7
stportal.mahendras.org

12 100 214 281 281


e.g. , , 4 2 6 + = 426
5 3 5 385 385
(iii) Mixed fraction - When a fraction is denoted as
combination of integer and fraction is called mixed fraction. 2 1 3 5
Ex. 19 - 33 - 26 + 45 = ?
3 2 4 6
e.g. 2 2 , 4 5
3 9
Only improper fraction may be converted into mixed fraction. 2 1 3 5
Sol. (19 - 33 - 26 + 45) + ( - - + )
3 2 4 6
e.g. 1 1 ⇒ 3 2
3 3  8 − 6 − 9 + 10  3 1
?= 5 +   = 5+ 1 2 = 5 4
12
Addition & Subtraction
(i) Addition & subtraction of whole Number: 2 1 2
Ex. 67 5 + 13 5 - 74 3 = ?
Ex. 8456 + 3891 + 4560 + 9823 = ?
Sol. ? = 26730 2 1 2
Sol. ?= (67 + 13 - 74) + ( 5 + 5 - 3 )
Ex. 94532 - 6754 - ? = 75432 - 2346
Sol. 94532 - 6754 - 75432 + 2346 = ?  6 + 3 −10  1
?= 6 +   = 6 -
? = 14692  15 15
(ii) Addition and Subtraction of Decimal : 1 14
To solve the questions based on addition and subtraction of ?= 5 + 1- 1 5 = 5 1 5
decimal numbers, first we put zeros after the decimal equal
to maximum digit after demical present in total numbers and Multiplication
then we operate addition and subtraction. (i) Multiplication of 2 digit Numbers:
Ex. 87.25+125.235-1234.63+14.2+2018 = ? Ex. 48 × 72
Sol. 87.250+125.235-1234.630+14.200 +2018 = ? Step ⇒ 1.
? = 1010.055
Ex. 266.230 + 934.600 - 16.256 = ? + 6.658
Sol. ? = 1177.916

Addition & subtraction of Mixed Fraction:
∴ Unit digit of result = 6
To solve questions related to addition and subtraction of
mixed fraction whole numbers are solved seperately and Step ⇒ 2.
added and subtracted together and then we solve fractions
are solved seperately then both are combined to get answer
3 1 7 1 1
Ex. 24 +18 −13 − 2 = ?− 2
4 2 8 6 4
∴ Ten's digit of Result = 5
3 1 7 1 1 
Sol. ?= (24 + 18 - 13 -2+2) +  + − − +  Step ⇒ 3.
4 2 8 6 4 

18 +12 − 21 − 4 + 6
?=29+
24

11 11 ⇒
?= 2 9 + = 29
24 24
∴ Hundred digit of Result = 4

3 3 1 Result = 3456
Ex. 215 + 711 − 500 = ?
5 11 7 (ii) Multiplication of 3 digit Numbers:
432 × 763
Sol. ?= (215 + 711 -500) + 2 3 1 + 1 0 5 − 5 5
385 Step ⇒ 1.
8 QUANTITATIVE APTITUDE
stportal.mahendras.org
82
= 64 332 = 1089
92
= 81 34 2
= 1156
102 = 100 352 = 1225

112 = 121 362 = 1296
Unit digit of result = 6
12 2
= 144 37 2
= 1369
Step ⇒ 2.
13 2
= 169 38 2
= 1444
142 = 196 392 = 1521
152 = 225 402 = 1600

16 2
= 256 41 2
= 1681
∴ Ten's digit of Result = 1
17 2
= 289 42 2
= 1764
Step ⇒ 3.
182 = 324 432 = 1849
4 3 2
192 = 361 442 = 1936
20 2
= 400 45 2
= 2025
7 6 3
21 2
= 441 46 2
= 2116
14 +18 +12 = 44

222 = 484 472 = 2209
⇒ 44 + 2 = 46
232 = 529 482 = 2304
Hundred digit of Result = 6
24 2
= 576 49 2
= 2401
Step 4.
4 3 2
25 2
= 625 50 2
= 2500
Type-I. Formula Method:
7 6 3 We know that
21 +24 = 45
(a+b)2 = a2 + 2ab + b2 i.e. (a/b)2

= a2 / 2ab / b2
∴ Thousand's digit of Result = 9 Ex. (56)2
Step ⇒ 5.
= (5/6)2

= 52 / 2 × 5 × 6/ 62
6 3

=
25 / 60 / 36

⇒ = 31 / 3 / 6

∴ Ten thousand's digit of Result = 2 = 3136

Required result = 329616 We break number in two parts i.e. 5 & 6 and follow
the rule of (a+b)2 = a2 /2ab/ b2
Square and Square Roots
Ex. (92)2
Square of (1-50) numbers
= (9/2)2
Students you should remeber these squares to speed your
= 81 / 3 6 /04
calculation
12 = 1 262 = 676 = 8464
22 = 4 272 = 729 Ex. Find the square of 112
32 = 9 282 = 784 I. Method :
42 = 16 292 = 841 (We break 112 in two parts like that (11/2)
52 = 25 302 = 900 (11/ 2)2 = 112 / 2 × 11 × 2 / 22
62 = 36 312 = 961
= 125 / 4 / 4
7 2
= 49 32 2
= 1024 12544
QUANTITATIVE APTITUDE 9
stportal.mahendras.org
II. Method : Ex. 3

(We break 112 in two parts like that 1 / 12) (103) 2 = 103 +3 / 3 2

(1/ 12)2 = 12 / 2 × 1 × 12 / 122 = 106 / 09



= 1 / 2 4 / 1 44 (103)2 = 10609
= 1 / 25 / 44 Since base is 100 therefore right part always be in two
Ans = 12544 digit.
12
Ex. Find the square of 211 Ex.
(112) 2 = 112 +12 / 12 2
(2 / 11)2 = 22 /2 × 2 × 11 / 112
+1
= 4 / 44 / 12 1 = 124 / 144
Ans. 44521 (112)2 = 125 44
In (998) — the above rule is failed
2
200 - Base Method (Near about 200)
Now we can use Base-method.......... Type-III :
100-Base Method (When the number is near about 100). (When the number is less than 200)
Type-I : Ex. 8
= (192) 2 = 1 92-8 / 8 2
(When number is less than 100).
Ex. (96)2
In this example we can use above method but we can = 184 / 64
also use 100 base method. ×2
= 368 / 64
Let see in this example 96 is less than 100 by 4.
(192)2 =368 64
Find the square of 4
Because of 200 base we multiply 184 × 2
Subtract 4 from 96.
Ex. 2
2
(1 9 8 ) = 1 9 8 -2 / 2
2
4
96 2 = (96 -4 ) 42

= 196 / 04
962 = 92 / 16
= 392 / 04
962 = 9216
(198) = 39204
2

Ex. 2
Ex.
98 = (98 -2 )
2
2 2
2
15

(1 8 5 ) = 1 8 5 -1 5 / 15
2

= 96 / 04
= 170 225
982 = 9604
×2
Ex. 15
(85) 2 = (85-15) 15 2 = 340 / 225
2
= 342 / 25
= 70 25
= 34225
= 72 / 25
Similarly we can use 200, 300--------base method.
852 = 7225 Type-IV :
Type-II : 1000 base (When the number is near about 1000)
(When the number is more than 100) Ex. 2
2
(9 9 8 ) = 9 9 8 -2 / 2
2
Ex. 8

(1 0 8 ) 2 = 1 0 8 + 8 82 = 996 / 004
= 116 / 64 (998)2 = 996004
(108)2 =11664 Since base is 1000 therefore Right hand side part one
Since 108 is more than 100 by 8 we add 8 to 108. in three digit.

10 QUANTITATIVE APTITUDE
stportal.mahendras.org
Ex.
2
12
4 4 1 =21
(1 0 1 2 ) = 1 0 1 2 + 1 2 / 12
2

Non-perfect square :
= 1024 / 144
If square root of any number is not integer but fraction
(1012)2 = 1024144 or decemal then number is said to be Non-perfect square
Ex. 2
8 number.
(1 0 0 8 ) = 1 0 0 8 + 8 / 8
2

5, 200
= 1016 / 064 How to find square root of perfect square number
(1008)2 = 1016064
Ex. Find the 2601
Special Type :
When unit place of the number is 5.
(x5)2 = x (x+1)/25
Ex. (75)2 = 7 × 8 / 52

= 56 / 25
(75) = 5625
2

Ex. (85)2 = 8 × 9 / 52

= 72 / 25 Ans = 51
(85)2 = 7225 Rules :
Ex. (105) = 10 × 11 / 5
2 2
 For unit place of the answer we see that last digit of
2

= 110 / 25 the question i.e. 1 (see the above table)
(105) = 11025
2 For the ten’s place of the ans. we always take smaller
number whose square is most nearest to 26.
Square Roots:
(i.e. 5)
See the given table :
 In the above process we find two ans. i.e. 51 or 59
By oberving unit place of any number we can find
the unit place of its square root.  We multiply ten digit number to next number
1 i.e. 5 × 6 = 30)
1  now compare 26 to 30 (we find that 26 is less than
9
4 30).
4  Therefore in both the ans. 51 and 59 the least ans is
8
correct.
5 5
Other example.
4
6 Ex.
6
3
9
7

0 0
Note: We can see a perfect square number may have 0, 1,
4 , 5, 6, 9 at unit place It means if a number has 2, 3, 7, 8
at its unit place it will not be a perfect square number Since 92 is greater than 90 therefore in both the ans.
96 is correct.
If a number has 0, 1, 4, 5, 6 or 9 at its unit place it has
a chance to be a perfect square number. Ex.
Perfect square :
If square root of any number is integer then number
is said to be perfect square number.
Ex. 36 = 6
QUANTITATIVE APTITUDE 11
stportal.mahendras.org
Now 11 × 12 = 132
Surds and Indices
125 is less than 132 An index (plural: indices) is the power, or exponent, of a
∴ 112 is correct answer. number. For example, a3 has an index of 3.
A surd is an irrational number that can be expressed with
Cube & Cube Root
roots, such as 2 or 1 9 .
5

Special Note for cube : Learn cube of 1 to 25 only for the


competitive exams. Important formulas of surds and indices.

Cube of the numbers: Indices Surds


13 = 1 143 = 2744 a × an = am+n
m 1
n
a =an
23 = 8 153 = 3375
33 = 27 163 = 4096 am n
ab = n a × n b
= a m −n
an
43 = 64 173 = 4913
5
3
= 125 18
3
= 5832
(a )
n
m
= amn a n
a
6
3
= 216 19
3
= 6859 n =
b n
b
73 = 343 203 = 8000
(ab)n = an­bn
( a)
n
83 = 512 213 = 9261 n
=a
93 = 729 223 = 10648
n
a  an m n
a − mn a
10 3
= 1000 233
= 12167 b  =
  bn
113 = 1331 243 = 13824
a0 = 1
( a)
m
12 3
= 1728 253
= 15625 n
= n am
133 = 2197
−2 −4

How to find cube root of perfect cube Number


Ex. FG 1 IJ ÷ FG 1 IJ = ?
3 3

While calculating Cube root of any number we will divide


H 216 K H 27 K
? = b216 g ÷ b27 g
it in two parts. 2/3 4 /3
Sol.
Second part (first three digit of number) will be used to 2 6 ×6 44
calculate the unit place of cube root and first part will be use ? = b6 g ÷ b3 g

3 , ?= 3×
?= 3
9 ×9 9
to calculate remaining part of the cube root.
Ex. −3
Ex. Cube Root of 50653 6 6 × 63 ÷ 6 2 = 6 ? +2
1 3 1 3
Sol. 6?+2 = 6 × 6 2 × 6 3 × 6 2 6?+2 = 6 1+ 2 + 3 + 2
Unit Place : To find second part will be used. As we can see
the unit place of Second part is 3 which comes at the end of ? + 2 = 6, ? = 4
cube of 7 so the unit place of cube root will be 7. Simplification
Remaining part : To find remaining part of cube root first VBODMAS Rule:
part will be used. As we can see first part is between cube of
3 and 4 hence cube root will be between 30 and 40. V → Vinculum means bar as (—)
Cube root : Combining above two statements we can say B → Bracket- () {} and then [ ]
that required cube root will be 37. O → of same as mulipilication but having greater priorty
Ex. than divison
D → Division [÷]
M → Multiplication [×]
A → Addition [+]
S → Subtraction [-]

12 QUANTITATIVE APTITUDE
stportal.mahendras.org
The word ‘VBODMAS’ represents the order of calculation Ex. 2415 × 655 ÷ 9600 = ?
i.e. order of signs
1
Sol. 49 × 26 × = ? , ? ≈ 13
Ex. 15 ÷ 5 (18 - 1 5 − 2 1 ) = ? 98
Sol. ? = 18 ÷ 5 (18-(-6)) Ex. 30.02 % of 261 + 59.98 % of 512 - 104.001 = ?
? = 18 ÷ 5 (24) = 18 ÷ 120 30 510
Sol. 260 × + × 60 – 104 = ? ,
18 3 100 100
?= =
120 20 ? ≈ 280
2 1 1 1 1  Ex. 249 299 14
Ex. ÷ − − = ? × ÷ =?
3 3  3 6 2  15 19 99

250 300 98
2 1 1 1  Sol. × × = ? , ? ≈ 1850
?= ÷ + 15 19 14
3 3  3 3 
Ex.
( )
2
9 0 0 .0 1 5 – 8 .9 6 6
2 1 2 3 =?
÷ [2 ] = × = 1
?=
( )
2
3 3 3 2 5 7 5 .7 5 7 – 4 4 1 .1 1 1

Approximation:
( )
2

Sol. ? ≈
900 – 9

(3 0 – 3 )2 ≈ 81
Approximation is the way of calculation in which we take
( ) (2 4 – 2 1)2
2

estaimated value is place of exact or real value. Now a days 576 – 441
approximation question are asked usually in PO exams and
8 9 8 × (1 2 .0 0 4 ) + ? = 5 0 0 0
2
clerk mains exams Ex.
There is no fix rule to calculate but depends on you which Sol. 900 ×122 + ? = 5000
technique suits you.
? = 5000–4320 = 680
In following examples we would try to find the find value by
Ex. 125% of 8475 + 88 × 14.995 = ?
taking nearest value and answer of these questions depend
upon the choices of given options. Sol. ? = 10595 + 1320 = 11900 (Approx)
What approximate value will come in place of question Ex. 721.001 × 7.998 + 6.05 × 8.010 = ?
mark (?) in the questions given below? Sol. ? = 5760 + 48 = 5800 (Approx)

QUANTITATIVE APTITUDE 13
stportal.mahendras.org

EXERCISE
Q.1-30. What will come in place of question mark (?) in the following questions?
Q.1. 18.5(17+18–3)–12.52=?
(1) 465.75 (2) 435.75 (3) 435.25 (4) -425.75 (5) None of these
Q.2. 175% of 480–87.5% of 288=?
(1) 544 (2) 588 (3) 584 (4) 455 (5) None of these
3 1
Q.3. 12 × ÷ + 12 ÷ 30 = ?
4 2
(1) 17.4 (2) 18.5 (3) 18.6 (4) 15.75 (5) None of these
Q.4. 17.52+192+123–133=?×5
(1) 39.35 (2) 37.65 (3) 36.65 (4) 39.65 (5) None of these
Q.5. 225% of 660 – 175% of 224= ?% of 800
(1) 136.625 (2) 136.25 (3) 132.625 (4) 132.25 (5) None of these
Q.6. 196 × 948 ÷ ? = 1176
(1) 148 (2) 158 (3) 128 (4) 160 (5) None of these
Q.7. (91) + (41) -
2 2
? = 9858
(1) 11236 (2) 10816 (3) 10404 (4) 9604 (5) None of these
Q.8. 42% of 445 - 25% of 354 =?
(1) 95.8 (2) 98.4 (3) 102.4 (4) 89.4 (5) None of these
Q.9. 8442 ÷ 576 − ? = 351
(1) 1.50 (2) 0.75 (3) 1.75 (4) 0.55 (5) None of these
Q.10. ?% of 340 + 48% of 480 = 451.40
(1) 95 (2) 85 (3) 130 (4) 65 (5) None of these
Q.11. (75)2 ÷ ? − 113 = 12
(1) 45 (2) 1225 (3) 2025 (4) 35 (5) None of these
Q.12. (3 ) = 19683
? ?

(1) 3 (2) 6 (3) 9 (4) 27 (5) None of these


Q.13. 76% of 845 + 102% of 234 = ?
(1) 862.88 (2) 812.84 (3) 792.82 (4) 880.88 (5) None of these
Q.14. (4.5)3.7 × (20.25)1.3 ÷ (91.125)1.8 = (4.5)?+5.2
(1) 6.5 (2) 5.6 (3) 6.3 (4) 5.9 (5) 6
Q.15. (207) +20% of 200 × 1 2 2 5 - 25% of 160 = ?
2

(1) 41409 (2) 42105 (3) 40219 (4) 39319 (5) None of these
Q.16. 24 + 96 + 216 + 384 = 5 2 × ?

(1) 5 3 (2) 3 3 (3) 4 3 (4) 2 3 (5) 3

Q.17. ?% of 800 = 293 – 22% of 750


(1) 16 (2) 18 (3) 20 (4) 24 (5) 25
Q.18. 3
12167 ÷ 46 × 8 + (2 2)6 − 250 = ?

(1) 250 (2) 260 (3) 240 (4) 270 (5) 280

14 QUANTITATIVE APTITUDE
stportal.mahendras.org

Q.19. (4 × 4)3 ÷ (512 ÷ 8) × (32 × 8) = ( 2 × 2)


4 4 ?+4

(1) 6 (2) 8 (3) 10 (4) 12 (5) 2


2 1 5
Q.20. of of of 12096 = ?% of 1600
7 8 9
(1) 10 (2) 12.5 (3) 15 (4) 17.5 (5) None of these
Q.21. 65% of 8800 + 18% of 720 = ? – 345.25
(1) 6468.50 (2) 6074.25 (3) 6284.85 (4) 6194.85 (5) None of these
Q.22. 625 × 1296 ÷ 324 = ?
(1) 10 (2) 50 (3) 30 (4) 70 (5) 90
2 2
1 3 4 5 6 × 7 3 9 6 + (? ) = 7 2 1 6 + (7 9 )
Q.23.
(1) 69 (2) 96 (3) 95 (4) 59 (5) None of these
2
Q.24. 4
×
80
× ? =  2 3 / 2
3  +4

32 560  

(1) 338 (2) 332 (3) 330 (4) 336 (5) None of these
Q.25. 1 1 4
1 + ?− 3 = 1÷ 2
3 9 7

2 1 1 2
(1) 1 (2) 2 (3) 3 6 (4) 2 (5) None of these
6 6 3

1
Q.26.
3 3 7 5 3 × 8 5 4 × 5 14 ÷  4 5 4 × 5 × 3 −3  = 1 7 ?
 

(1) 4 (2) 12 (3) 8 (4) 10 (5) 16


Q.27. 4 × (0.31 ×0.8 × 1.8) = 28.5696
?

(1) 3 (2) 4 (3) 2 (4) 5 (5) None of these

(7 0 )
2
Q.28. 2
−? ×12.5 = 850 ×72

(1) 2 (2) 4 (3) 6 (4) 4896 (5) None of these


Q.29. 33 +44 = ?
2 2 2

(1) 35 (2) 45 (3) 55 (4) 65 (5) None of these


4096 × 59
Q.30. (7414 + 3698 + 1257 + 1869) + =?
759 − 641

(1) 14272 (2) 14274 (3) 14270 (4) 14278 (5) None of these
Q.31-42.What approximate value should come in place of question mark (?) in the questions given below?
Q.31. 0.98 × 10.9 + 48% of 56 – 46.34 = ?
(1) 35 (2) 40 (3) 50 (4) 45 (5) 30
Q.32. 71.8% of 419-11.972=?% of 399
(1) 31 (2) 52 (3) 45 (4) 37 (5) 27
Q.33. 432+532-153=400% of ?
(1) 320 (2) 350 (3) 375 (4) 420 (5) 280

QUANTITATIVE APTITUDE 15
stportal.mahendras.org
Q.34. 77.92×93.1-44.87×40.996=?
(1) 5200 (2) 5400 (3) 5700 (4) 6100 (5) 4900
Q.35. ? =479% of 121- 400% of 105
2

(1) 9 (2) 13 (3) 5 (4) 17 (5) 22


Q.36. 892+452=362+?2
(1) 93 (2) 86 (3) 81 (4) 99 (5) 112
Q.37. 213-249% of 644=?3
(1) 10 (2) 14 (3) 18 (4) 20 (5) 8
Q.38. 1010 ÷ 36 + 187 × 20.05 = ?
(1) 3770 (2) 3800 (3) 3740 (4) 3700 (5) 3680
Q.39. (559% of 816) + 1449 = ?
(1) 6000 (2) 7000 (3) 5000 (4) 3000 (5) 8000
5
Q.40. 1654×24.62÷ of 15.86 = ?
8
(1) 4100 (2) 4700 (3) 3100 (4) 2700 (5) 2100
Q.41. 3 4 1
4 ×5 ×6 = ?
10 7 3

(1) 120 (2) 180 (3) 150 (4) 295 (5) 157
Q.42. 5 2 8 .6 4 + 8 4 1 .4 6 = ?
(1) 45 (2) 57 (3) 42 (4) 63 (5) 52

16 QUANTITATIVE APTITUDE
stportal.mahendras.org

EXERCISE EXPLANATION
Q.1.(2) 592-156.25=? Q.17.(1) 800 × ? = 293 − 750 × 22
100 100
?=435.75
? = 16
Q.2.(2) 840-252=? Q.18.(2) 3
12167 ÷ 46 × 8 + (2 2)6 − 256 = ?
1
?=588 23 × × 8 + 512 − 256 = ?, 4 + 512 − 256 = ?
46
3 1
Q.3.(5) 12 × ÷ + 12 ÷ 30 = ? ? = 260
4 2
Q.19.(1) (4 × 4)3 ÷ (512 ÷ 8)4 × (32 × 8)4 = ( 2 × 2)? + 4
2 46−12+16 = 4? + 4
18 + = 18.4
5 ?=6
Q.4.(4) 306.25+361+1728–2197=?×5
Q.20.(3) 240 = 16×? , ? = 15
?=39.65
Q.21.(4) 5720+129.6+345.25=?, ? =6194.85
Q.5.(1) 1485–392=?×8 Q.22.(2) ?= 25×36÷18 = 50 (approx)
?=1093/8 Q.23.(4) 116 × 86 + (?)2 = 7216 + 6241
?=136.625 (?)2 = 7216 + 6241 - 9976
Q.6.(2) ? = 196 × 948 ÷ 1176 (?)2 = 3481, ? = 59
? = 158 Q.24.(4) 3 3 
2
4 80
Q.7.(2) (8281 + 1681 – 9858)2 = ? × ×? =  22  + 4
32 560  
 
? = 1042 = 10816
Q.8.(2) ? = 186.9 - 88.5 (2 + 4 ) × 5 6 0 × 3 2
?= = 336
4 × 80
? = 98.4
Q.25.(2) 4 28 7 , 7 4 28
Q.9.(2) ? = 351.75 - 351 + ?− =1× ?= − +
3 9 18 18 3 9
? = 0.75
7 − 24 + 56 39 13 1
34 ?= = = =2
Q.10.(4) ?× = 451.40 − 230 .40 18 18 6 6
10
221 Q.26.(3) 1
? = 34 × 10 3 3 7 5 3 × 8 5 4 × 5 14 ÷  4 5 4 × 5 × 3 −3  = 1 7 ?
 
? = 65
15 ×17 4 × 5 4 ×17 4 × 3 4 × 33
75 × 75
= 1 7 ? , ? =8
= ? 154 × 34 × 5
Q.11.(3) 125
? = 45 × 45, ? = 2025 Q.27.(1) 4? × (0.31 × 0.8 × 1.8) = 28.5696
Q.12.(1) (3?)? = (3)9= (33)3 2 8 .5 6 9 6
4? =
?=3 0 .3 1 × 0 .8 × 1 .8
Q.13.(4) ? = 880.88 4? = 64 = 43

Q.14.(1) (4.5)3.7+ 2 × 1.3 + 3 × 1.8 = 4.5?+ 5.2 ?=3

(4900 − ? )
2

? = 3.7 + 2.6 + 5.4 - 5.2, ? = 6.5 Q.28.(2) ×12.5 = 850 ×72
Q.15.(1) 42849 + 40 × 35 - 40 = ?, ? = 41409 850 × 72
(4900 − ? )
2
= ,
Q.16.(3) 2 6 +4 6 +6 6 +8 6 =5 2 ×?
1 2 .5
(4900 − ? )
2
= 4896
20 6
?= =4 3
5 2 4900 - ? = 4896, ? = 4

QUANTITATIVE APTITUDE 17
stportal.mahendras.org
Q.29.(3) ?2 = 3025 Q.36.(1) ?2=8650
? = 55 ?=93 (approx)
64 × 59
Q.30.(3) (7414 + 3698 + 1257 + 1869) + 7 5 9 - 6 4 1 = ? Q.37.(4) 9261 – 1610 = ?3
14238 + 32 = ?, ? = 14270
?3 =7651
Q.31.(4) 11+41 – 7 = 45 (approx)
? = 20 (approx)
Q.32.(4) ?=(302-144)/4
Q.38.(1) ? = 28 + 3740 = 3770 (Approx)
?=37 (approx)
Q.33.(1) 1849+2809-3375=4×? Q.39.(1) ? = 4560 + 1449 = 6000 (Approx)

?=1283/4 Q.40.(1) 41350 ÷ 9.9125 = ?


?=320 (approx) ? ≈ 4100
Q.34.(2) ?=7254–1845=5409=5400 (approx) 43 39 19 31863
Q.41.(3) × × ⇒ = 151 ≈ 150
Q.35.(2) ? =581–420=161
2 10 7 3 210

?=13 (approx) Q.42.(5) ? = 23+29 = 52

Notes

18 QUANTITATIVE APTITUDE
stportal.mahendras.org

CHAPTER

2 Number Series

Number series is a order of numbers which are not arranged 3. Geometric Series : Under this category, each successive
randomly but follow a pattern. Here in this not we will number is obtained by multiplying (or dividing) the previous
understand how to identify which kind of pattern is following number with a fixed number.
because without this it is next to impossible to have a Ex. 5, 35, 245, 1715, ?
command on number series.
Sol. Here previous number is multiplied by 7, hence answer
Types of Series is 12005
There are different type of number series, which is based on Ex. 43923, 3993, 363, 33, ?
some of the important rules or order.
Sol. Here previous number is divided by 11, hence answer
1. Pure Series : In this type of number series, the number is 3.
itself obeys certain order so that the character of the series
Ex. 336, 168, 84, 42, 21, ?
can be found out.
Sol. Here previous number is divided by 2,
The number itself may be:
hence answer is 10.5.
(i) Even Number : 4. Two-tier Arithmetic Series : Under this category, the
Ex. 2, 4, 6, 8, 10, ? differences of successive numbers form an arithmetic series.
Sol. 12
Ex. 4, 5, 9, 16, 26, 39, 55 ?
(ii) Odd Number :
Sol.
Ex. 1, 3, 5, 7, 9, ? 4 5 9 16 26 39 55 74
Sol. 11
(iii) Prime Number : +1 +4 +7 +1 0 +1 3 +1 6 +1 9

Ex. 2, 3, 5, 7, 11, 13, ?


+3 +3 +3 +3 +3 +3
Sol. 17
5. Mixed Series: Here, the numbers obeying various orders
(iv) Perfect square of two or more different type of series are arranged alternately
Ex. 121, 144, 169, 196, 225, ? in a single number series.
Sol. 256 Ex. 7, 15, 32, 67, 138, ?
(v) Perfect cube Sol. ×2 + 1, ×2 + 2, ×2 + 3, ×2 + 4, ×2 + 5,
Ex. 6859, 5832, 4913, 4096, 3375, ? Hence answer is = 281
Sol. 2744 Ex. 8, 15, 42, 141, 580, ?
2. Difference Series: Under this category, the change in Sol. +7×1, +6×2, +5×3, + 4×4, +3×5,
order for the differences between each consecutive number Hence answer is = 2915
of the series is found out.
Ex. 7, 4, 5, 9, 20, ?
Ex. 13, 18, 28, 43, 63, ?
Sol. ×.5+0.5, ×1+1, ×1.5+1.5, ×2+2, ×2.5+2.5
Sol. 13 18 28 43 63 88
Hence answer is = 52.50

+ 5 + 10 + 15 + 20 + 25 Types of Question:
There are 3 type of questions usually asked in Banking
Ex. 1348, 1338, 1318, 1288, 1248, ? examination.
Sol. 1. Missing Number series.
1348 1338 1318 1288 1248 11 9 8
2. Wrong number series

-20 -30 -40 -50


3. Coding and decoding series.
-10
QUANTITATIVE APTITUDE 19
stportal.mahendras.org
1. Missing Number series: In this type of series one number ∴ (c) = 17.5×3.5+3.5 = 61.25+3.5 = 64.75
is missing in given series. Such type of series, find the given
Ex. 15, 9, 8, 12, 36, 170
pattern and solved the missing number.
19 (a) (b) (c) (d) (e)
Ex. What value come in place of question mark (?).
Ex. 6 11 21 36 56 ? What will come in place of (b) ?
Sol. +5 +10 +15 +20 Sol.
Hence, Answer is = 56 + 25 = 81
Ex. 3 7 15 31 63 ? Similarly,
Sol. +4 +8 +16 +32
Hence, Answer is = 63 + 64 = 127
Ex. 1 6 15 ? 45 66 91

Sol. +5 +9 +21 +25
∴ (b) = 13×2 - (5 × 2) = 26 - 10 = 16
Hence, Answer is = 15 + 13 = 28
Ex. 504 336 210 120 ? 24 Ex. 7, 6, 10, 27, 104, 515
Sol. 83-8 73-7 63-6 53-5 43-4 9 (a) (b) (c) (d) (e)
Hence, Answer is = 43-4 = 60 What will come in place of (d) ?
2. Wrong number series: In this type of series one number Sol.
is odd man out which does not follow the sequence.
Ex. Find the wrong number in given series:
Ex. 2, 3, 7, 22, 89, 440, 2677, 18740 Similarly,
Sol. ×1+1, ×2+1, ×3+1, ×4+1, ×5+1 .........
So, 440 is replaced by 446
Ex. 5, 6, 14, 40, 89, 170, 291
Sol. +12, +32, +52, +72, +92 ............ ∴ (d) = 39×4 - 4 - 4 = 156 - 4 = 152
So, 14 is replaced by 15. Ex. 6, 16, 57, 244, 1245, 7506
Ex. 445, 221, 109, 46, 25, 11,4 3 (a) (b) (c) (d) (e)
Sol. -3÷2, -3÷2..................
What will come in place of (d) ?
So, 46 is replaced by 53.
Sol.
Ex. 12, 26, 56, 116, 244, 498, 1008
Sol. ×2+2, ×2+4, ×2+6.............
So, 116 is replaced by 118 Similarly,
Ex. 8, 27, 64, 125, 217, 343
Sol. 23 , 33 , 43 , 53 ..........
So, 217 is replaced by 216

3. Coding decoding series: In the following number series
question a series a given. First you have to find out the pattern ∴ (d) = 196×5 + (5)2 = 980 + 25 = 1005
given in the series then followed by this pattern you make Ex. 8, 9, 20, 63, 256, 1285
new series whose first term is given.
5 (a) (b) (c) (d) (e)
Ex. 5, 9, 25, 91, 414, 2282.5
What will come in place of (e) ?
3 (a) (b) (c) (d) (e)
Sol.
What will come in place of (c) ?
Sol.
Similarly,

Similarly,


∴ (E) = 184×5 + (5) = 920 + 5 = 925

20 QUANTITATIVE APTITUDE
stportal.mahendras.org

EXERCISE
Q.1-25. What should come in the place of question mark (?) in the following number series?
Q.1. 5 25 7 ? 9 19
(1) 23 (2) 22 (3) 25 (4) 32 (5) None of these
Q.2. 7 18 40 84 172 ?
(1) 326 (2) 328 (3) 330 (4) 332 (5) None of these
Q.3. 100 50 52 26 28 ?
(1) 30 (2) 32 (3) 14 (4) 16 (5) None of these
Q.4. 980 392 156.8 ? 25.088 10.0352
(1) 62.72 (2) 63.85 (3) 65.04 (4) 60.28 (5) None of these
Q.5. 113 225 449 ? 1793
(1) 789 (2) 786 (3) 897 (4) 987 (5) None of these
Q.6. 5 12 39 160 805 ?
(1) 4163 (2) 4153 (3) 4181 (4) 4836 (5) None of these
Q.7. 2 10 30 68 ?
(1) 126 (2) 130 (3) 140 (4) 150 (5) None of these
Q.8. 45 46 70 141 ? 1061.5
(1) 353 (2) 353.5 (3) 352.5 (4) 352 (5) None of these
Q.9. 33 321 465 537 573 ?
(1) 600 (2) 591 (3) 585 (4) 498 (5) None of these
Q.10. 2 9 ? 105 436 2195
(1) 25 (2) 27 (3) 30 (4) 33 (5) None of these
Q.11. 4 6 14 44 ? 892
(1) 176 (2) 172 (3) 178 (4) 1821 (5) None of these
Q.12. 126 64 34 20 14 ?
(1) 12 (2) 14 (3) 16 (4) 18 (5) None of these
Q.13. 16 8 12 30 ? 472.5
(1) 100 (2) 105 (3) 205 (4) 300 (5) None of these
Q.14. 25 34 30 39 35 ?
(1) 45 (2) 44 (3) 46 (4) 50 (5) None of these
Q.15. 10 11 13 21 69 ?
(1) 384 (2) 490 (3) 453 (4) 390 (5) None of these
Q.16. 8 4 4 6 12 30 ?
(1) 80 (2) 85 (3) 90 (4) 95 (5) None of these
Q.17. 32 ? 92 134 184
(1) 55 (2) 38 (3) 45 (4) 58 (5) None of these
Q.18. 11 16 26 ? 86
(1) 46 (2) 56 (3) 76 (4) 86 (5) None of these
Q.19. 198 194 185 169 ?
(1) 154 (2) 165 (3) 144 (4) 134 (5) None of these
Q.20. 9050 5675 3478 2147 ?
(1) 3478 (2) 1418 (3) 2428 (4) 3678 (5) None of these
QUANTITATIVE APTITUDE 21
stportal.mahendras.org
Q.21. 5 17 37 65 ? 145.
(1) 95 (2) 99 (3) 97 (4) 101 (5) None of these
Q.22. 95 115.5 138 ? 189.
(1) 154.5 (2) 164.5 (3) 162.5 (4) 166.5 (5) None of these
Q.23. 3 4.5 9 22.5 67.5 ? 945
(1) 265.25 (2) 236.25 (3) 225.36 (4) 150 (5) None of these
Q.24. 8544 1420 280 ? 18 5
(1) 33 (2) 44 (3) 56 (4) 66 (5) None of these
Q.25. 812 398 ? 90 40 16
(1) 192 (2) 182 (3) 172 (4) 162 (5) None of these
Q.26-35. In the following number series one number is wrong. Find the wrong number.
Q.26. 6072 1000 200 48 14 5
(1) 6072 (2) 1000 (3) 5 (4) 48 (5) 14
Q.27. 198 165 148 117 104 77
(1) 165 (2) 148 (3) 198 (4) 104 (5) 77
Q.28. 445 221 109 53 23 11
(1) 445 (2) 221 (3) 23 (4) 53 (5) 11
Q.29. -1/2 0 1/6 1 3/2 2
(1) 1/2 (2) 3/2 (3) 2 (4) 1/6 (5) -1/2
Q.30. 3 9 39 113 265 577
(1) 3 (2) 113 (3) 39 (4) 577 (5) 265
Q.31. 7 15 42 126 231
(1) 42 (2) 231 (3) 126 (4) 15 (5) 7
Q.32. 8 19 41 65 118
(1) 65 (2) 8 (3) 41 (4) 65 (5) 118
Q.33. 13 13 27 21 33 53
(1) 13 (2) 33 (3) 27 (4) 21 (5) 53
Q.34. 7 8 17 42 90
(1) 90 (2) 7 (3) 17 (4) 42 (5) 91
Q.35. 10 11 24 65 304
(1) 10 (2) 24 (3) 65 (4) 11 (5) 304
Q.36-40. In the following number series questions a series a given. First you have to find out the pattern given in the series
then followed by this pattern you make new series whose first term is given.
Q.36. 2, 15, 92, 463, 1856, 5561
3 (a) (b) (c) (d) (e)
Which of the following numbers will come in place of (c) ?
(1) 670 (2) 672 (3) 673 (4) 675 (5) None of these
Q.37. 512, 256, 128, 64, 32, 16
998 (a) (b) (c) (d) (e)
Which of the following numbers will come in place of (d) ?
(1) 61.275 (2) 62.375 (3) 63.475 (4) 60.275 (5) None of these
22 QUANTITATIVE APTITUDE
stportal.mahendras.org
Q.38. 3, 5, 12, 38, 154, 772
7 (a) (b) (c) (d) (e)
Which of the following numbers will come in place of (c) ?
(1) 56 (2) 60 (3) 61 (4) 62 (5) None of these
Q.39. 9, 22, 24, 37, 39, 52, 54
11 (a) (b) (c) (d) (e)
Which of the following numbers will come in place of (d) ?
(1) 39 (2) 41 (3) 45 (4) 50 (5) None of these
Q.40. 4, 4, 6, 12, 30, 90
18 (a) (b) (c) (d) (e)
Which of the following numbers will come in place of (e) ?
(1) 405 (2) 403 (3) 404 (4) 402 (5) None of these
Q.41. The number series 1 , 3 , 9 , 31, 129 follow a certain pattern then 5, ______,17, ---------1131. If 1131 is nth term
then the value of n is-
(1) 5 (2) 4 (3) 8 (4) 10 (5) None of these
Q.42. 4, 2, 2, 3, 6, 15,..………., 2835. If 2835 is nth term then the value of n is
(1) 9 (2) 8 (3) 10 (4) 11 (5) 12
Q.43. If one series is 1.5, 6, 22, 93, 471, 2833 and another series which follows same pattern as given series is 2.5, __,
__, __, __, x then x = ?
(1) 2369 (2) 3553 (3) 2961 (4) 4145 (5) 945
Q.44. A series is given as 6, 3, 3, 4.5, 9, 22.5, …. and it’s pth term is 4252.5. Find the value of P.
(1) 10 (2) 11 (3) 9 (4) 8 (5) 12
Q.45. A series is 113, 170, 232, 303, 399, 556, 838. Another series is 93, __, __, __, __, __, m. Which follows same
pattern as given number series. Then m= ?
(1) 808 (2) 443 (3) 626 (4) 818 (5) 909

Notes

QUANTITATIVE APTITUDE 23
stportal.mahendras.org

EXERCISE EXPLANATION
Q.1.(2) Q.29.(4) (+1/2), (+1/2), (+1/2), (+1/2), (+1/2)
5 25 7 22 9 19
Q.30.(2) -3 9 41 113 265 577
+2 +2
-3 -3 +1 2 +3 2 +7 2 +1 52 +3 12
Q.2.(5) +11, +22, +44, +88 +2 0 +4 0 +8 0 +1 60
Q.3.(3) 100 50 52 26 28 14 Q.31.(3) 23, 33, 43, 53
Q.32.(4) +11, +22, +33, +44
÷2 ÷2 ÷2 Q.33.(3) +(02+0), +(12+1), +(22+2), +(32+3)

980 392 1568 62.72 25.088 Q.34.(1) +(1)2, +(3)2, +(5)2, +(7)2
Q.4.(1)
Q.35.(3) ×1+1, ×2+2, ×3+3, ×4+4
2 2 2 2 Q.36.(3) ×7+1, ×6+2, ×5+3, ×4+4
×5 × × ×
5 5 5
Q.37.(2) ÷2, ÷2,÷2, ÷2, ÷2
Q.38.(4) ×1+2, ×2+2, ×3+2, ×4+2
Q.5.(3)
113 225 449 897 179 3 Q.39.(2) +13, +2, +13, +2, +13, +2
×2-1 ×2-1 ×2-1 ×2-1
Q.40.(1) ×1, ×1.5, ×2, ×2.5
Q.41.(1) 5×1+2=7
Q.6.(4) ×2+2, ×3+3, ×4+4, ×5+5, ×6+6 7×2+3=17
Q.7.(2) 2 10 30 68 130 17×3+4=55
55×4+5=225
225×5+6=1131
(1) 3 +1 (2) 3 +2 (3) 3 +3 (4) 3 +4 (5) 3 +5 Q.42.(1) 4×0.5=2
Q.8.(2) ×1+1, ×1.5+1, ×2+1, ×2.5+1, ×3+1 2×1=2
Q.9.(2) +288, +144, +72, +36, +18 2×1.5=3
3×2=6
Q.10.(3) ×1+(1×7), ×2+(2×6), ×3+(3×5), ×2+(4×4)
6 × 2.5 = 15
Q.11.(3) (×1+2) (×2+2) (×3+2) (×4+2) (×5+2) 15 × 3 = 45
Q.12.(1) ÷2+1, ÷2+2, ÷2+3, ÷2+4, ÷2+5 45 × 3.5 = 157.5
Q.13.(2) ×0.5, ×1.5, ×2.5, ×3.5, ×4.5 157.5 × 4 = 630
Q.14.(2) +9, -4, +9, -4, +9 630 × 4.5 = 2835
Q.15.(3) +1, +2, +8, +48, +384 10th term = 2835
Q.43.(2) 1.5×2+3 = 6 ,6×3+4=22 ,
Q.16.(3) ×0.5, ×1, ×1.5, ×2
22×4+5=93,93×5+6 = 471, 471×6+7= 2833
Q.17.(4) +26, +34, +42, +50 Similarly for next series x = 3553
Q.18.(1) +5, +10, +20, +40 Q.44.(1) 6×0.5 = 3, 3×1=3 , 3×1.5=4.5 ,4.5×2=9 ,
Q.19.(3) -22, -32, -42, -52...... 9×2.5=22.5……945×4.5 =4252.5
Q.20.(2) -153, -133, -113, -93...... 113 170 232 303 399 556 838
Q.21.(4) 2 +1, 4 +1, 6 +1, 8 +1, 10 +1, 12 +1
2 2 2 2 2 2
Q.45.(4) +57 +62 +71 +96 +157 +282
Q.22.(3) +20.5, +22.5, +24.5, +26.5
+5 +9 +25 +61 +125
Q.23.(2) ×1.5 ×2 ×2.5 ×3 ×3.5 ×4
+4 +16 +36 +64
Q.24.(4) ÷6-4, ÷5-4, ÷4-4, ÷3-4, ÷2-4
Second Series
Q.25.(1) ÷2-8, ÷2-7, ÷2-6, ÷2-5, ÷2-4 93 150 212 283 379 536 818
Q.26.(2) -12÷6, -10÷5, -8÷4, -6÷3, -4÷2, -2÷1
+57 +62 +71 +96 +157 +282
Q.27.(3) (142+4), (132-4), (122-4), (112-4), (102-4), (92-4),
(82-4) +5 +9 +25 +61 +125

Q.28.(3) (-224), (-112), (-56), (-28), (-14), (-7) +4 +16 +36 +64
24 QUANTITATIVE APTITUDE
stportal.mahendras.org

CHAPTER

3 Percentage

Introduction: Percentage is a fraction whose denominator 100 ×100


is always 100. x percentage is represented by x%. Sol. Ist Method : y = ×x
80
x ∴ y = 125% of x
I. To express x% as a fraction : We know x% =
100 IInd Method : Let y is 100 then x = 80
.
20 100
Thus 20% = .......(means 20 parts out of 100 parts) Required % = 8 0 × 1 0 0 = 125%
100
1 Ex. K is what % of N ?
= .............(means 1 part out of 5 parts)
5 Sol. K K
20 1 ×100 = %
N N
and 20% = 100 = 5
V. If A is R% more than B, then B is less than A by-
II. To express
x
y
as a percentage: LM R × 100 OP%
MN b100 + R g PQ
x FG
=
x IJ
× 100 %

We know that
y Hy K If A is R% less than B, then B is more than A by-
Change in % LM R × 100 OP%
1 F1 I MN b100 − R g PQ
Thus = GH × 100 JK % = 25%
4 4
Ex. If Akash income is 10% more than that of Vikas
8 F 8 × 100 IJ % = 80%
=G income. How much % Vikas income is less than that
10 H 10 K
and 0.8 =
of Akash income?
III. Remember it : I. Method.
1 = 100%
By using formula-
1 1 1
= 50% = 33 % r 10
2 3 3 less % = × 100 = × 100
100 + r 100 + 10
1 1
= 25% = 20% 10 1
4 5 = 110 × 100 = 9 11 %
1 2 1 2
= 16 % = 14 % II. Method.
6 3 7 7
1 0 0%
1 1 1 1 Since 10% more Tw o to o ls
= 12 % = 11 % 11 0%
8 2 9 9
100
1 1 1 Less % = 10 × ........
= 10% =9 % 110
10 11 11
1 1 1 9 (To decrease any number, we, multiply with small
=8 % =7 %
12 3 13 13 number and divide with large number.)
IV. Comparison between two values x and y. 1
(i) If x is compare to y then we assume always y is = 9 11 %
equal to 100% Ex. If the income of A is 40% less than that of B, How
(ii) When any question ask y is a what percent of x much percent B’s income is more than that of A’s?
then x is always write in the denominator 40
Ex. If x is 80% of y, what percent of x is y? Sol. More% = 6 0 × 1 0 0 = 66.66%

QUANTITATIVE APTITUDE 25
stportal.mahendras.org
VI. If the price of a commodity increases by R%, then % Ex. If the present population of a town is 72600 and it
reduction in the consumption as not to increase the is decreased by 10% per annum. What will be its
expenditure is- population 2 years hence?
LM R × 100 OP% Sol.

Population after n years = P  1 −
R 
n


MN b100 + R g PQ Here P = 72600
 1 0 0 

If the price of a commodity decreases by R%, then R = 10%


the % increase in consumption as not to decrease the n = 2yrs.
expenditure is- Population after 2 years
LM R × 100 OP%
MN b100 − R g PQ FG
= 72600 1 −
10 IJ 2
= 72600 ×
90
×
90
= 58806

Ex. If tax on a commodity is reduced by 10%, total revenue
H 100 K 100 100

remain unchanged. What is the percentage increase in Trick:


its consumption? 90 90
Population after 2 yrs = 72600 × × = 58806
Sol. Percentage increase 100 100
Note :
10 100 1
= ×100 = = 11 % These formula is also used for the depreciation value
100 – 10 9 9 of machine.
VII. Result on population : (a) Let the population of a VIII. Net percentage change :
town be P now and suppose increases at the rate of Assume your pocket money is Rs 500 per month. If
R% per annum, then : it is increased by 20 % then what will be your new
pocket money?
1. Population after n years = P 1 +
FG R IJ n
Of course your new pocket money will be 600. But

H 100 K what if it is further decreased by 20% ?
Will your pocket money will become same as before??
P No.Why??
2. Population n years ago =
FG1 + R IJ n
This is because of successive process.
H 100 K So next we are going to study is net percentage change.
Ex. Present population of a town 1.21 crore & it increases xy
x+ y+
at 10% per annum. Then find out population after two 100
years as well as before 2 year.
But always remember one thing x and y always come
Sol. Present Population = 1.21 crore with sign. Means if quantity is increased we will take
n 2
 r   10  + and if it is decreased we will take -.
Two years after = P  1 +
 1 0 0 
= 1.21  1 + 1 0 0  So the answer of the question asked will be-
= 1.21 × 1.21= 1.4641 crore 20 × 20
20 − 20 − = −4%
P 100
Two years before = n But what if there are three changes??
 r 
 1 + 1 0 0  We also have formula for that
xy yz zx xyz
1 .2 1 1 .2 1 x+ y+z+ + + −
= 2 = 100 100 100 10000
 10  1 .2 1 = 1 crore
 1 + 1 0 0  But using this formula is not a great idea instead this
we can apply first formula thrice Like
(b) Results on Population : Let the population of a town be P Ex. Find the net percentage change of 40% increase.25%
now and suppose decreases at the rate of R% per annum, then : decrease and 10% increase?
1. Population after n year Sol. Very important point it doesn’t matter which change is

= P 1−
FG R IJ n applied first final change will be always same.

H 100 K 40 − 25 −
25 × 40
100
=5

2. Population n years ago
5 ×10
P Again 5 + 10 + = 15.5
=
FG
1−
R
n
IJ 100
H100 K So final change will be increase of 17.5%.

26 QUANTITATIVE APTITUDE
stportal.mahendras.org

EXERCISE
Q.1. If the income of Ram is 20% more than Shyam’s income. How much percent Shyam’s income is less than that of Ram’s
income?
2
(1) 12% (2) 20% (3) 30% (4) 40% (5) 1 6 %
3
Q.2. When numerator of a fraction is increased by 40% and denominator increased by 80% the resultant fraction becomes
14
2 7 . Find the original fraction ?
2 1 4 3
(1) 3 (2) 3 (3) 3 (4) 3 (5) 4
Q.3. A reduction of 5% in the price of tea a person would enable to obtain 5 kg. more for Rs. 2000. Find the reduced
rate per kg.(new rate) of tea.
(1) Rs. 30 (2) Rs. 40 (3) Rs. 20 (4) Rs. 50 (5) Rs. 10
Q.4. Due to increase in the price of sugar by 25%, a man able to purchase 1/2 kg less sugar for Rs.96. Find the original
rate per kg.
(1) Rs.40 (2) Rs.50 (3) Rs.38.4 (4) Rs.52 (5) Rs.30.4
Q.5. In an examination, a student who gets 20% of the maximum marks fails by 5 marks. Another student who scores
30% of the maximum marks gets 20 marks more than the pass marks. The necessary percentage required for
passing is
(1) 32% (2) 23% (3) 22% (4) 20% (5) None of these
Q.6. In a town, 60% registered voters cast their votes in the election. Only two candidates (A and
B) were contesting the election. A won the election by 1860 votes. Had B received 40% more
votes, the result would have been a tie. How many registered votes are there in the town?
(1) 10580 (2) 10880 (3) 10550 (4) 10850 (5) 10250
Q.7. lf A has 4/5 of the number of books that shelf B has. If 25% of the books A are transferred to B and then 25
% of the books from B are transferred to A then the percentage of the total number of books that in shelf A is:
(1) 25% (2) 50% (3) 75% (4) 100% (5)
Q.8. In an election survey, 30% people promised to vote candidate A and remaining promised to vote for candidate B.
If on the day of election x% percent of people who promised to vote for A, voted for B and 40% of people who
promised to vote B voted against him and in the end B lost by 10 votes. What is value of x, if total 250 votes were
there?
(1) 20 (2) 30 (3) 50 (4) 70 (5) None of these
Q.9. Mr. Ram Niwas Singh spent 20% of his monthly income on food and 15% on children's education. 40%
of the remaining, he spent on entertainment and transport together and 30% on the medical. He is left with
an amount of Rs.8775 after all these expenditures. What is Mr. Ram Niwas Singh's monthly income?
(1) Rs.40000 (2) Rs.35000 (3) Rs.42000 (4) Rs.38000 (5) None of these
Q.10. When the price of rice was increase by 25%, a family reduced its consumption in such a way that the expenditure
of rice was only 10% more than before. If 50 kg. were consumed before, find the new consumption?
(1) 44 kg. (2) 45 kg. (3) 43 kg. (4) 46 kg. (5) 42 kg.
Q.11. If the price of salt decrease by 20%, then by what percent consumption should be increase to keep the expenditure
same?
(1) 27 % (2) 26 % (3) 25 % (4) 36 % (5) 30 %
Q.12. A number wrongly divided by 5, instead of multiplied by 5. Find the percent error in result.
(1) 92% (2) 93% (3) 97% (4) 96% (5) 60%
Q.13. If the radius of a circle increased by 20%, then what is the percent change in area?

QUANTITATIVE APTITUDE 27
stportal.mahendras.org
(1) 44% (2) 45% (3) 43% (4) 46% (5) 42%
Q.14. In an examination 45% student passed mathematics, 50% students passed in english and 15% failed in both subjects.
If 150 students passed in both subjects, find out the total no. of students?
(1) 1200 (2) 1100 (3) 1400 (4) 1500 (5) 1600
Q.15. In a class of 40 students and 2 teachers, each student got toffee that are 10% of the total number of students and
each teacher got toffee that are 15% of the total number of students. How many toffee were there?
(1) 166 (2) 176 (3) 172 (4) 177 (5) 173
Q.16. A 120 ltrs solution of milk and water contains 20% water, what quantity of water must be added with that solution
to get 25% water?
(1) 8 liter (2) 6 liter (3) 3 liter (4) 4 liter (5) 5 liter
Q.17. A person spent 40% of his income. If his income is increased by 20% then his expenditure also increased by 50%.
Find the % change in his saving?
(1) 30% (2) 40% (3) 45% (4) 50% (5) No Change
Q.18. A man income is increased by Rs.5000 and at the same time, the rate of tax to be paid reduced from 10% to 9%.
He now pays the same amount of tax as before. What is his increased income if 20% of his income is exempted
from tax in both cases?
(1) Rs.30000 (2) Rs.60000 (3) Rs.40000 (4) Rs.50000 (5) Rs.20000
Q.19. 75 Litres of mixture contains 20% spirit and rest of water. If 5 ltrs of water be mixed with it, the percent of spirit
in the new mixture.
2 3
(1) 16 % (2) 18 % (3) 15% (4) 25% (5) 26%
3 4

Q.20. Total salary of A and B is Rs. 1500. A spends 90% while B spends 80% of his salary. If ratio of their savings are 3
: 4 then what is the salary of A and B?

(1) Rs. 900, Rs. 600 (2) Rs.600, Rs. 900 (3) Rs.800, Rs. 600 (4) Rs. 700, Rs. 900 (5) Rs. 600, Rs. 800
Q.21. Two numbers are less than the third number by 50% and 54% respectively.By how much percent is the second
number less than the first number?
(1) 13% (2) 10% (3) 12%
(4) Cannot be determined (5) None of these
Q.22. Aman's expense is 30% more than Vimal's expense and Vimal's expense is 10% less than Raman's expense. If the
sum of their expense is Rs. 6447, then what would be the Aman's expense?
(1) Rs. 2,200 (2) Rs. 2,457 (3) Rs. 1,890 (4) Rs. 2,100 (5) None of these
Q.23. Sumitra has an average of 56% on her first 7 examinations. How much she should make on her eighth examination
to obtain an average of 60% on 8 examinations?
(1) 88% (2) 78% (3) 98%
(4) Cannot be determined (5) None of these
Q.24. Two Candidates A and B participate in election. 80% of the total people, cast the votes, in which A received 55%
of the total votes caste. Difference between votes recieved by A and B is 1344. What is the number of registered
votes?
(1) 14200 (2) 18200 (3) 16500 (4) 16800 (5) None of these
Q.25. On republic day, in a class only 45 student appear thus it is decided that each student will get number of chocolate
equal to 40% of the number of student present in class. After distribution of chocolate 45 more student join the
class so it is decided that now the new student will get only number of chocolates equal to the 10% of the total
student present in class. Find the total number of chocolate distributed in class.

28 QUANTITATIVE APTITUDE
stportal.mahendras.org
(1) 1200 (2) 1210 (3) 1215 (4) 1220 (5) Data inadequate
Q.26. Mr Mishra spent 20% of his monthly income on petrol and household . Out of the remaining he spent 15% on
children’s education ,25% on transport and 20% on entertainment .He is left with an amount of Rs7,200 after
incurring above expences .What is his monthly income?
(1) Rs 1,44,000 (2) Rs 36,000 (3) Rs 7,2,000 (4) Rs 1,14,000 (5) None of these
Q.27. Vijayshree got 273 marks in an examination and got 5% more than the pass percentage. If Subhi got 312 marks,
then by what percentage above the pass marks did she pass the exam ?
(1) 9% (2) 12.5% (3) 20% (4) 25% (5) None of these
Q.28. Fresh fruits contain 75% while dry fruits contain 20% water. If the weight of dry fruits is 300 kg, what was its total
weight when it was fresh ?
(1) 900 kg (2) 850 kg (3) 920 kg (4) 960 kg (5) None of these
Q.29. The monthly income of Shyama and Kamal together is Rs. 28000. The income of Shyama and Kamal is increased
by 25% and 12.5% respectively. The new income of Kamal becomes 120% of the new salary of Shyama. What is
the new income of Shyama ?
(1) Rs. 12000 (2) Rs. 18000 (3) Rs. 14000 (4) Rs. 16000 (5) Rs. 15000
Q.30. Chhaya bought a watch costing Rs. 1404 including sales tax at 8% She asked the shopkeeper to reduce the price
of the watch so that she can save the amount equal to the tax. The reduction of the price of the watch is ?
(1) Rs. 108 (2) Rs. 104 (3) Rs. 112 (4) Rs. 120 (5) None of these

Notes

QUANTITATIVE APTITUDE 29
stportal.mahendras.org

EXERCISE EXPLANATION
Q.1.(5) Method-I. 100
By using formula- = 48 × = 3 8 .4 Rs.
125
r 20 Q.5.(3) According to the question,
less % = ×100 = ×100
100 + r 100 + 20 20% + 5 = 30% – 20
10% = 20 + 5 = 25
20 2 100% = 250
= ×100 = 16 %
120 3 Passing marks = 75 – 20 = 55
Method-II.
55
1 0 0% Required percentage = ×100
100 = 22%
Since 20% more Tw o to ols 250
1 2 0%
Q.6.(4) According to question,
100
Less % = 2 0 × ....... (To decrease any A – 0.4B = 1.4B
110
A = 1.8B
number, we, multiply with small number and
A:B=9:5
divide with large number.)
Difference (9 – 5 = 4) = 1860
2
= 16 %
3 14 = 1860 ×14= 6510
Q.2.(1) Method-I: 4
5
x Total registerted vote = 6510 × = 10850
Let the original fraction be then- 3
y
Q.7.(2) Let the number of books in shelf B be 100.
 x × 40  x ×140 ∴ Number of books in shelf A = {(100 × 4)/5} =
 x + 1 0 0  1 4 100 14
= ⇒ = 80
 y × 80  27 y × 180 27 On transferring 25% i.e. of books of shelf A to
 y + 1 0 0  100
shelf B, the books in shelf B = {100 + (80 ×
25)/100}
x 14 18 B = 100 + 20 = 120
= ×
y 27 14 Books left in shelf A = 80–20 = 60
x 2 Again, on transferring 1/4th books of shelf B to
= shelf A, the books in
y 3 Ans.
Method-II shelf A = {60 + (120/4)} = 90
Total no of books in A and B = 120 +60 = 180
14 Required percentage of books in shelf A =
Given fraction →
27 (90/180) × 100
2 = 50%
14 18
Original fraction = × = Q.8.(1) A = 75 – 20% of (75) + 40% of (175) = 130
27 14 3
B = 175 + 20% of (75) – 40% of (175) = 120
Q.3.(3) 5% of 2000 = Rs. 100
Q.9.(5) Let his monthly income be Rs.10000
Rs.100 is the rate of 5 kg. of tea.
Amount spent on Food= Rs.2000
1 kg of tea = Rs.20 /kg.
Amount spent on Children's education= Rs.1500
25
Q.4.(3) Since = 96× = 24 Amount spent on Entertainment and Transport=
100
Rs.2600
According to question
Amount spent on Medical= Rs.1950
1
Rs. 24 = kg.
2 Remaining= 10000 – (2000 + 1500 + 2600 + 1950)
1
1 kg = Rs. 48= 24 = 48 Rs. 1950 ratio= 8775
2
Hence Original rate 1 ratio= 4.5

30 QUANTITATIVE APTITUDE
stportal.mahendras.org
100 ratio= 45000 96
100% = ×100= 128 ltrs.
Q.10.(1) Expenditure = Price × Consumption 75
new expenditure = 110% of 50 So, water should be added = 128 - 120 = 8 ltrs
So, Short Trick: (by alligation)

110 125 20% 100%


× 50 = ×x
100 100 25
x = 44 75 5
So, new consumption is 44 kg. 15 : 1

R 20 15 unit = 120 ltrs.


Q.11.(3) % change = ×100 = ×100
100 – R 1 0 0 − 20 1 unit = 8 ltrs.
Q.17.(5) Incom e E xp S aving
20 100 40 60
= × 1 0 0 = 25%
80 +20 +20% +20 +50% 0%
120 60 60
Q.12.(4) Let take number = 20
That means no change in his saving.
Actual answer = 20 × 5 = 100
Q.18.(4) Since, same percentage of his income is exempted
20
New Answer = =4 from tax in both cases, this data is not to be
5 considered.
Error = 100 - 4 = 96 Ratio of interest = 10 : 9
96 So, Ratio of income = 9 : 10
% error = × 100 = 96%
100
20 × 20 difference 1 unit = 5000
Q.13.(1) % change = + 20 + 20 + 10 unit = 50000
100
= + 20 + 20 + 4 = + 44% 20
Q.19.(2) spirit = 75 × = 15 ltrs
Increased by 44% 100
Q.14.(4) % of students passed in one subject or both New Water = 65 ltrs
subjects = 45% + 50% = 95% New mixture = 15 + 65 = 80 ltrs
15% failed in examination, 15
% of spirit in new mixture = ×100
So passed % = 100 - 15 = 85% 80
75 3
So, students passed in both subject = = 18 %
4 4
= (95 - 85) = 10% Q.20.(1) Lets A's salary = 100 A
So, 10% = 150 B’s salary = 100 B
1% = 15
100% = 1500 10 A 3
savings =
2 0B 4
Q.15.(3) Each student got = 10% of 40 = 4
A 3
each teacher got = 15% of 40 = 6 =
B 2
So, total toffee = 4 × 40 + 6 × 2= 160 + 12 = 172
3
Q.16.(1) Quantity of water in solution = 20% of 120 A’s income is = × 1 5 0 0 = R s.9 0 0
5
Quantity of milk = 120 - 24 = 96 ltrs 2
B’s income is = × 1 5 0 0 = R s.6 0 0
Now solution contain 25% water, 5
So, milk % = 100% - 25% = 75% Q.21.(5) Let the third number be 100.
So, 75% = 96 ltrs ∴ First number = 50
96 and Second number = 100–54 = 46
1% =
75 Decrease = 50 –46 = 4
QUANTITATIVE APTITUDE 31
stportal.mahendras.org

4 Q.24.(4) Let the total no. of registered votes =x


∴ Required percentage= ×100 = 8%
50 So total casted votes = 80% of x = 0.8x
Q.22.(2) Let Vimal's expense be Rs. 100. Now (55-45)% of 0.8x = 1344
∴ Aman's expense = Rs. 130 1344 ×10
x= = 16800
and Raman's expense 0 .8
10
100 1000 Q.25.(3) 45×45×+45×90× 1 0 0 = 810+ 405
= × 1 0 0 =Rs.
90 9
∴ Ratio of the expenses of Vimal. Aman and = 1215
Raman respectively Q.26.(5) Remaining salary = 80 × 40%=32
1000 7200
= 100 : 130 : = 90 : 117 : 100 100% = ×100 = 22500
9 32
∴ Aman expense
117 Total monthly income = Rs 22500
= × 6447
90 + 117 + 100 100
Q.27.(3) Passing marks = 2 7 3 × = 260
117 105
= × 6 4 4 7 = Rs. 2457 Required %
307
or 3 1 2 −2 6 0 52
= × 100 = × 100 = 20%
1000 260 260
100+130+ = 6447
9 Q.28.(4) Quantity of water in 300 kg dry fruits
3070 = (20/100)×300 = 60 kg
i.e. = 6447
9 Quantity of fruit alone = 300 - 60 = 240 kg.
25 kg fruit piece in 100 kg fresh fruits
6447 × 9 ×130
⇒ 130 = = Rs. 2457 For 240 = (100 × 240)/25 = 960 kg.
3070
Q.23.(4) The question cannot be answered because total Q.29.(5) The monthly income of Shyama and kamal
marks are not givven. S + K = 28000......(1)
Total maximum marks obtained in 8 exams Shyama's income= x; Kamal's income= 28000-x
= 700+50 = 750 K = 120/100× S........(2)
For the average percentage in 8 exams to be 60%, S's new income
the total marks obtained should be = (28000 -x)× 112.5/100
60 (28000-x)×112.5/100
= × 750 = 450
100 = x × 125/100
x = 12000
∴ Marks to be obtained in the eighth exam should
New income of Shyama
be
= 125% of 12000 = Rs. 15000
450–392 = 58
Q.30.(2) 1.08x = 1404
But, 58 > 50 x = 1300
It means the marks to be obtained would be greater The reduction of the price of the watch
than the maximum marks. This is impossible. = 1404-1300 = Rs. 104

32 QUANTITATIVE APTITUDE
stportal.mahendras.org

CHAPTER

4 Profit and Loss

Profit and loss → 85% → represent selling price (S.P.)

I. Cost price : The price at which an article is purchased → 15% → represent loss i.e.,
is called its Cost Price and denoted by CP. 10 0% (C P )
or i.e. Loss 15%
85 % (SP )
For any person, Cost Price is an amount or quantity
which firstly releases from the pocket of that person. II. In the case of discount, the relation is generated
II. Selling Price : The price at which an article is sold, between SP/MP
is called its selling price, and denoted by SP. If 20% discount accrued on an article it means :
or → 100% → represent marked price (MP)
For any person, Selling Price is, an amount or quantity → 80% → represent selling price (SP)
which come to the pocket of that person.
→ 20%→ discount.
III. Profit or Gain : If S.P. is greater than CP, then Seller
is said to have a profit or gain. i.e. 10 0% (M P )
i.e. D iscou nt 2 0%
Profit = S.P - C.P
80 % (S P)
IV. Loss : If S.P.is less than C.P. the seller is said to have
a loss. i.e. Ex. A man bought a cycle for Rs. 330. For how much
loss = C.P - S.P should he sell it so as to gain 10%?
V. Profit and loss are always counted on C.P. Sol. Profit percentage = x% = 10%
VI. CP is always 100% in the case of profit and loss. Cost Price = 100
VII. For any transaction there are two persons involved Selling Price = 100 + x = 100 + 10 = 110
one is called buyer and second one is called seller. Required answer
VIII. Marked Price : MRP of an article is known as Marked 330
Price or labelled price or listed price and denoted by = × 1 1 0 = 363
3 6 .3
100
MP.
Ex. A merchant professes to sell his goods at a loss of 10%
IX. Discount always carried on MRP (MP) but weighs 750 gm in place of 1 kg. Find his real loss
X. MRP is always 100% in the case of discount. or gain percentage?
Observation: Sol. Let cost price of 1 kg = 1000
I. In the case of profit or loss, the relation is generated But his giving product worth 750
between CP and SP. So it's very clear that he is getting a gain.
If 25% profit accrued on an article it means: SP=900
→100% represent cost price (CP) CP=750
→125% represent selling price (S.P.) Now gain percent
→25% represent profit = 900 − 750 ×100% = 20%
750
10 0% (C P ) Ex. A person got 5% loss by selling an article for Rs. 1045.
i.e. P rofit 25 %
12 5% (S P ) At what price should the article be sold to earn 15%
profit ?
If 15% loss accrued on an article, it means : Sol. Let CP = 100 SP = 95
→ 100% → represent cost price (CP) New SP = 115

QUANTITATIVE APTITUDE 33
stportal.mahendras.org

115 ×1045 S P 1 + S P2
Required Answer ⇒ x = = Rs. 1265 CP =
95 2
Short Trick : 480 + 240
CP = = 360
2
120
SP = × 360 = 432
Ex. A person sold an article at profit of 15%. If he had 100
sold it Rs. 480 more, he would have gain 18%. What Ex. A vendor bought bananas at 5 for Rs. 4 and sold them
is the cost price ? at 4 for Rs. 5. Find his gain or loss percent.
Sol. Let the CP of an article be x Rs, then Sol. Suppose, number of bananas bought
115% of x + 480 = 118% of x = LCM of 5 & 4 = 20
118% of x - 115% of x = 480
FG 4 × 20 IJ =16;SP=Rs. FG 5 × 20 IJ = 25
3% of x = 480 CP=Rs.
H5 K H4 K
480
x= × 1 0 0 = Rs. 16000 25 − 16 9
3 % Profit = × 100 = × 100
16 16
Short Trick :
D iffe re n ce in S .P . = 56.25%
CP = D iffe re n ce in P ro fit % × 1 0 0
Short Trick
480
CP = × 1 0 0 = Rs. 16000
3
Ex. If the CP of 15 articles is equal to the SP of 12 articles.
Find the gain or loss %.
Sol. Let the C.P of each article be Rs. 1.
Then CP of 12 articles = Rs. 12
SP of 12 articles = Rs. 15

3 Special Case: If a'th part of some items is sold at x% loss,
Gain % = × 1 0 0 = 25% then required gain per cent in selling rest of the items
12
in order that there is neither gain nor loss in whole
Ex. A person bought articles at 20 for a rupee. How many
for a rupee must to sell to gain 25% ? ax
transaction, is %
Sol. CP of 20 articles = Rs. 1, SP of 20 article = 125% of (1 − a )
Rs. 1 Ex. A shopkeeper purchased medicines worth Rs. 9000
5 form a company. He sold 1/3 part of the medicine at
For Rs. , article sold = 20 30% loss. On which gain he should sell his rest of the
4
medicines, so that he has neither gain or loss?
4
For Rs. 1 article sold = 2 0 × = 16 Here a = 1/3 , x = 30 %
5
Short Trick : 1
× 30
Required gain % = 3 = 15%
 1
1 − 
Special Case:  3
If 'a' part of an article is sold at x% profit/loss, 'b'
part at y% profit/loss and c part at z% profit/loss
and finally there is a profit/lossof Rs.R, then Cost
Ex. The percentage profit earned by selling an article for price of entire article
Rs. 480 is equal to the percentage loss incurred by
R
selling the same article for Rs. 240. At what price = ×100
should the article be sold to make 20% profit? ax + by + cz
Ex. 33.33% of a commodity is sold at 15% profit,25% is
Sol. This question is easy as profit and loss both are same
sold at 20% profit and the rest at 24% profit. If the
so CP will lie exactly between then
34 QUANTITATIVE APTITUDE
stportal.mahendras.org
Total profit is Rs. 80 is earned then find the value of Ex. A tradesman marks his goods at 25% above his
commodity? (in Rs.) 1
cost price and allows a discount of 1 2 % for cash
Sol. Part sold at 24% profit 2
 1  1 5 purchases. What profit does he make?
= 1−  +  =
 3 4  12 Sol. Let CP=100
80 × 100
Value of commodity = 1 = 400
1 5
×15 + × 20 + × 24 MP=125
3 4 12
SP=87.5% of 125=109.375
Ex. If 2/3 part of an article is sold at 30% profit, 1/4 part
at 16% profit and remaining part at 12% profit and Now required answer
finally, there is a profit of Rs.75, then find the cost
1 0 9 .3 7 5 − 1 0 0
price of the article. = × 1 0 0 = 9.375%
100
Here a = 2/3 , x = 30 % , b =1/4 , y = 16 % , z = 12 %
and R = 75 Rs Ex. A seller gives 2 oranges free on purchase of 10 oranges.
Find equivalent discount.
Required CP of article
75 Sol. Clearly, the retailer get 1 dozen out of 6 dozens free.
= ×100 = 300
2 1 1 1
× 30 + × 16 + × 12 Hence discount% = ×100
3 4 12 6
Equivalent Discount : When two or more than two discount
are given by seller then equivalant discount may be calculated
by following formula. 2
=16 %
xy 3
x +y −
100 Short Trick
Note : 2
×100%
For more than two discount you can use this formula 10 + 2
further.
2
( order is not important) = 16 %
3
Ex. Find the equivalent discount of 10% and 20% Ex. A shopkeeper earns a profit of 12% on selling a book
Sol. Let the MP be = Rs. 100 at 10% discount on printed price. The ratio of the cost
price to printed price of the book is ?
90 80
Then, Net SP = 1 0 0 × × = Rs.72 Sol. C.P. of the book = Rs. x
100 100
Printed price = Rs. y
Required discount = (100 -72)%=28%
Short Trick y × 90 x ×112
=
xy 100 100
x+y-
100
x 45
=
10 × 20 y 56
10 + 20 - = 28%
100

QUANTITATIVE APTITUDE 35
stportal.mahendras.org

EXERCISE
Q.1. If the CP is 60% of the selling price. What is the profit%?
(1) 33. 60% (2) 66.67% (3) 63.27% (4) 60.67% (5) None of these
Q.2. If on selling 12 Notebooks any seller makes a profit equal to the selling price of 4 Note books. What is his percent
profit?
2
(1) 50% (2) 25% (3) 1 6% (4) Data inadequate (5) None of these
3
Q.3. Arun purchased a house for Rs. 75000 and a site for Rs. 15000 respectively. If he sold the house for Rs. 83000 and
the site for Rs. 10000. Find the resultant% of gain?
1 1
(1) 3% (2) 3 % (3) 30% (4) 3 3 % (5) None of these
3 3
Q.4. One fifth of the CP, one-seventh of the marked price and one-sixth of the selling price are equal. What is the gain%
or loss% to trader?

2 2
(1) 20% gain (2) 1 6 % less (3) 1 4 % profit (4) 10% loss (5) None of these
3 7
Q.5. Shri Ramlal purchased a TV set for Rs. 12500 and spent Rs. 300 on transportation and Rs. 800 on installation. At
what price should he sell it so as to earn an overall profit of 15%
(1) Rs. 14560 (2) Rs. 14375 (3) Rs. 15460 (4) Rs. 153758 (5) None of these
Q.6. A shopkeeper bought two radios for Rs. 1008, he sold one at a loss of 20% and other at a profit of 44%. If he sell
both radios at same price, then find the cost price of the radio which is sold at loss.
(1) Rs. 648 (2) Rs.360 (3) Rs.368 (4) Rs.640 (5) Rs.540
Q.7. A shopkeeper gets 20% more from wholesaler when he buys sugar. He uses a false weight of 750 gram while selling
1 Kg of sugar. Find the profit he earn if he had paid 10000 Rs to the wholesaler.
(1) Rs. 5000 (2) Rs. 7500 (3) Rs. 6000
(4) Cannot be determined (5) None of these
Q.8. A table is sold at a profit of 20%. If both the cost price and the selling price of the table are decreased by Rs. 100,
the profit would be 5% more, original cost price of the table is-
(1) Rs. 600 (2) Rs. 550 (3) Rs. 650
(4) Cannot be determined (5) None of these
Q.9. A shopkeeper marks up his goods by 20% and cheats by 10% while selling it. Find his total profit percent.
1
(1) 30% (2) 32% (3) 3 3 % (4) 35% (5) None of these
3
Q.10. A man sells an article at a profit of 25% if he had bought it at 20% less and sold it for Rs. 1050 loss, he would have gained
30% find the cost price of the article.
(1) Rs. 10000 (2) Rs. 5000 (3) Rs. 12000 (4) Rs. 10500 (5) None of these
Q.11. Marked price of an article is 60% more than the cost price. A shopkeeper allows X% discount and earns ____%
profit, if a shopkeeper allows 2X% discount and earns ___% profit. Which of the following options satisfies the
two blanks in the question?
A) 44, 28 B) 36, 12 C) 30, 16 D) 20, 12
(1) Only B (2) Only A, B (3) Only C) (4) Only D (5) Only A
Q.12. Divya went to a textile shop. When Divya asked the price of a saree the shopkeeper told her the price by increasing
18% of the original cost. But Divya insisted to decrease the price by 18% so the shopkeeper sold it by decreasing
the price by 18% what is the loss or profit percen of shopkeeper?
(1) 3.25% (2) 2.5% (3) 3%
(4) 3.225% (5) None of these
36 QUANTITATIVE APTITUDE
stportal.mahendras.org
Q.13. The market price of an article is 40% more than the cost price. There is a profit of x% after giving the discount of
15%. If the cost price of an article is Rs 1160 and its market price is 30% higher, then how many Rs will be the
benefit after giving the discount of x%?
(1) Rs 61.48 (2) Rs 42.25 (3) Rs 85 (4) Rs 48.75 (5) None of these
Q.14. A shopkeeper bought an article marked at Rs. 22800 at two successive discounts of 15% and 20%. He spent Rs.
1064 on transportation and sold the article for Rs. 20710. Then, what was the profit percentage of the shopkeeper?
(1) 35% (2) 30% (3) 25% (4) 20% (5) None of these
Q.15. A fruit seller wants to make a profit of 20%, after giving the discount in cash of 10% and giving 1 dozen banana
free on the purchase of 4 dozen bananas. What percentage above the cost price should he listed on bananas?
(1) 76.47% (2) 75% (3) 65% (4) 83.5% (5) None of these
Q.16. A book seller sells a book at a profit of 10%. If he had bought it at 4% less and sold it for 6 more, he would have
3
gained 18 %. The cost price of the book is-
4
(1) Rs.130 (2) Rs.140 (3) Rs.150 (4) Rs.160 (5) Rs.165
Q.17. A shopkeeper purchased 25 identical pieces of mobile phones at the rate of Rs. 1500 each. He spent an amount of
Rs. 2500 on transport and packing. He fixed the labeled price of each phone at Rs. 2000. However, he decided to
give a discount of 5% on the labeled price. What is percent profit earned by him?
(1) 18.75 (2) 16.75 (3) 14.75 (4) 20.75 (5) None of these
Q.18. A shopkeeper bought 30 kg of wheat at the rate of Rs. 45 /kg. He sold 40% of the total quantity at the rate of Rs.
50/kg. Approximately, at what price/kg should he sell the remaining quantity to make 25 percent overall profit?
(1) Rs. 54 (2) Rs. 52 (3) Rs. 50 (4) Rs. 60 (5) Rs. 56
Q.19. Rajani purchased a mobile phone and a refrigerater for Rs. 12000 and Rs. 10000 respectively. She sold the refrig-
erator at a loss of 12% and mobile phone at a profit of 8%. What is his overall loss/profit?
(1) Rs. 240 Loss (2) Rs. 250 Loss (3) Rs. 240 Profit
(4) Rs. 280 Profit (5) Rs. 360 Loss
Q.20. A trader purchases two watches. He makes the first one up by Rs.200 over the cost price and gives a discount of 20%
on it. The second one he marks up by 50% and gives a discount of Rs. 160. If he gains 15% on both the watches
put together of 8% on the first alone, what is the percent of profit on the second watch?
(1) 21% (2) 22% (3) 18.5%
(4) Can’t be determined (5) None of these
Q.21. A man sold two articles, each for the same price of Rs. 840. He earns 20% profit on the first and 40% profit on the
second. Find his overall profit percent.

1 1 3 3
(1) 29 % (2) 2 8 % (3) 2 9 % (4) 2 8 1 3 % (5) None of these
2 2 13
Q.22. An article is listed at Rs. 65. A customer bought this article for Rs. 56.16 and got two successive discount of which
one is 10%. Find the other discount of this scheme.
(1) 4% (2) 5% (3) 3% (4) 6% (5) 8%
Q.23. A shopkeeper sold two article for Rs. 15000 each on first he gains 50% but on the other a loss of 25%. Find the
overall profit percent or loss percent.
2 2
(1) 2% (2) 1 6 % (3) 25% (4) 6 3 % (5) No profit no loss
3 5
Q.24. A discount of 15% is given on the marked price of an article. The shopkeeper charges sales tax of 6% on the
discounted price. If the selling price be Rs. 1081.20 What is the marked price of the article?
(1) Rs. 11850.20 (2) Rs. 1250.20 (3) Rs. 302
(4) Rs. 1200 (5) None of these

QUANTITATIVE APTITUDE 37
stportal.mahendras.org
Q.25. A trader who marks his goods upto 50% and offered discount of 20%. What percent profit the trader makes after
offereing the payment?
(1) 15% (2) 20% (3) 25% (4) 30% (5) 40%
1
Q.26. Article are marked at a price which gives a profit of 25%. After allowing a certain discount the profit reduces 1 2 %
,then find the discount percent. 2

(1) 10% (2) 15% (3) 20% (4) 18% (5) 25%
Q.27. A person purchased two chairs for Rs. 900. He sold first article at 4/5 of cost price but another at 5/4 of cost price
He gains overall Rs. 90. Find the cost price of Lesser valued chair.
(1) Rs.300 (2) Rs. 400 (3) Rs. 500
(4) Rs. 600 (5) Can’t be determined
Q.28. A property dealer sells a house for Rs.9,00,000 and in this he makes 12.5% profit. Had he sold it for Rs. 6,00,000,
then what percentage of loss or gain have be made ?
1 2
(1) 25% loss (2) 10% loss (3) 12 % gain (4) 16 % gain (5) None of these
2 3
Q.29. A shopkeeper sells one transistor for Rs. 960 at a gain of 20% and another for Rs. 1152 at a loss of 4%. His total
gain or loss percent is :
3 3 2 2
(1) 5 % loss (2) 5 % gain (3) 6 % gain (4) 6 % loss (5) None of these
5 5 3 3
Q.30. Profit earned by selling an article for Rs. 1060 is 20% more than the loss incurred by selling the same article for
Rs. 950. At what price should the article be sold to earn 20% profit ?
(1) Rs. 980 (2) Rs. 1080 (3) Rs. 1800 (4) Rs. 1200 (5) None of these

Notes

38 QUANTITATIVE APTITUDE
stportal.mahendras.org

EXERCISE EXPLANATION
Q.1.(2) Let S.P of the article =x x
60 Now, 5 ×100 = 25
C.P. of the article = x
100 x −100
Then profit = x−
60
x =
40 x 2500
100 100 25x – 2500 = 20x, x = = Rs. 500
5
Hence profit percent
Q.9.(3) C.P. = 100, M.P. = 120, C.P. = 100–10=90
40 x
P 120 – 90 1
= ×100 = 00 ×100
1
P% = ×100 = 33 %
CP 60 x 90 3
100
Q.10.(2) Case - I
200
= 6 6 .6 7 % C.P S.P. Profit%
3
100 125 25%
Q.2.(1) Let S.P. of l notebook = x
80 104 30%
S.P. of 12 Notebook = 12x
According to question,
Profit = 4x
21 = 1050
then
1 = 50, 100 = 5000
CP = 12x - 4x = 8x
Q.11.(2) Cost price = 100 Marked price = 160
4x
Hence profit percent = × 1 0 0 = 50% Option (A): 1st case:
8x
Q.3.(2) Total CP = Rs. 75000 + Rs.15000= Rs. 90,000 SP = 100 + 44/100 × 100 = 144
Total SP = Rs. 83000 + Rs. 10000= Rs. 93000 Discount percentage = (160–144)/160 × 100
Profit = Rs. 93000 - Rs. 90000= Rs. 3000 = 10%

3000 1 2nd case: SP = 100 + 28/100 × 100 = 128


Profit percent = ×100 = 3 %
90000 3 Discount percentage = (160 – 128)/160 × 100
Q.4.(1) According to question = 20%
CP 1 1
= MP = SP⇒ C P = 5 This satisfies the given condition
5 7 6 SP 6 Option (B): 1st case:
1
Profit = 5 × 1 0 0 = 2 0 % SP = 100 + 36/100 × 100 = 136
Q.5.(5) Total Cost price Discount percentage = (160–136)/160 × 100
= 12500 + 300 + 800 = Rs. 13600 = 15%
115 2nd case: SP = 100 + 12/100 × 100 = 112
Selling price= 1 3 6 0 0 × = Rs. 15640
100 Discount percentage = (160–112)/160 × 100
Q.6.(1) 80% of x = 144% of (1008 - x), x = 648 = 30%
Q.7.(3) (100 + 20) − (100 − 25) This satisfies the given condition
P% = × 100 = 60%
100 − 25
Q.12.(5) Loss = x2/100 = 182/100 = 3.24%
60 Q.13.(1) Let cost price of the article is 100.
P = 10000 × = Rs.6000
100
So market price = 140
Q.8.(5) Let the cost price of table be x. Selling price
Now discount = 15% of 140 = 21
120 x 6 x
= = Now profit = 140 -21 = 119
100 5
So, x% = 19%
(II) Cost price = Rs. (x–100) Now according to the question,
 6x  x Market price of article whose cost is Rs 1160
Profit =  − x  = Rs.,
 5  5 = 130% of 1160= Rs 1508

QUANTITATIVE APTITUDE 39
stportal.mahendras.org
Then discount = 19% of 1508 = Rs 286.52 1900 − 1600
profit % = × 100% = 18.75
So selling price = Rs 1221.48 1600
Now, profit = 1221.48-1160 = Rs 61.48 Q.18.(4) C.P. of 30 kg wheat = 30 × 45 = Rs.1350

Q.14.(3) Marked price of the article = Rs. 22800 total S.P. for an overall profit of 25%

Cost price of the article 125


= 1350 × = 1687.5
100
 15  80
S.P of 12 kg of wheat = 12 × 50 = Rs. 600
=  22800 − 22800 × ×
100  100
=Rs. 15504 Expected S.P of 18 kg of remainig wheat

Actual cost price = 15504 + 1064 = Rs. 16568 = 1687.5-600 = Rs. 1087.5
1 0 8 7 .7
Required S.P. /kg = = Rs. 60
( 20710 − 16568) × 100 = 25% 18
Profit % = Q.19.(1) Total C.P= Rs (12000 + 10000)
16568
Q.15.(5) Discount percentage when bought 4 dozen = Rs. 22000
bananas = 1/5 ×100 Total S.P
= 20%
 12000 ×108 88 
=  +10000 ×
Total discount percentage =28%  100 1 0 0 
Let the M.P. of banana = Rs 100 = Rs. 21760
S.P. = 72 Since CP > SP
Hence,
100
C.P. = 72× = 60 Loss = Rs. 22000 - Rs. 21760 = Rs. 240
120
Q.20.(4) With this information P% can’t be determined.
40 Q.21.(3) SP of 1st Article = 840
required % = ×100 = 66.66%
60 840
CP of 1st Article = 1 2 0 × 1 0 0
MP : CP = 120 : 72 = 5 : 3 = 700
Hence = 66.66% SP of IInd Article = 840
Q.16.(3) Let the C.P. of the book = Rs.x 840
CP of IInd Article = × 1 0 0 = 600
110x 140
Initial S.P. = Rs. = 1.1 x Net Profit Percent
100
380
New C.P. = Rs.0.96x = × 1 0 0 = 29 3 %
1300 13
 75  Q.22.(1) According to question
New S.P. = Rs. 100 +  % of Rs.0.96x
 4
9 0 (1 0 0 − x )
475 65 × × = 5 6 .1 6
= × 0.96x = Rs.1.14x 100 100
400
100 − x
5 8 .5 × = 5 6 .1 6
= 1.14 x – 1.1 x = 6 100

0.4x = 6 5 6 .1 6 × 1 0 0
(1 0 0 − x ) =
6 5 8 .5
x= = Rs.150
0.04 100 - x = 96
Q.17.(1) Total cost of 25 mobile sets x = 4%
= 1500 × 25 + 2500 = 40000. Q.23.(5) SP of 1st Article = 15000
40000
Cost price of 1 mobile set = = 1600 15000
25 CP of 1st Article = × 1 0 0 = 10,000
Labeled price = 2000. 150
S.P. 1 mobile = 95% of 2000 = 1900 SP of IInd Article = 15000

40 QUANTITATIVE APTITUDE
stportal.mahendras.org

15000 Q.27.(1) Let CP of 1st chair is x


SP of IInd Article = × 1 0 0 = 20000
75 According to question
Overall P/L = overall SP - overall CP
= 30,000 - 30000 = 0 Then x × 4 + (9 0 0 – x ) × 5 = 9 9 0
5 4
that means No profit no loss
4x 5x
Short trick – = 990 – 1125
5 4
When SP1 = SP2
16 x − 25 x
Profit percent ≠ Loss Percent = −1 3 5
20
Then
x = Rs. 300
Profit or Loss
100 (P − L ) − 2PL 100 ( 50 − 25 ) − 2 × 50 × 25 FG 100 × 900000 IJ = Rs. 800000
=
200 + P − L
=
200 + 50 − 25
= 0% Q.28.(1) C.P.=Rs.
H 112.5 K
Q.24.(4) Let the marked price of the Article be
= x then according to question
F 200000 I
∴Required loss% = GH 800000 × 100 JK = 25%
85 106
x× × = 1 0 8 1 .2 0
100 100 F 100 I
C.P. of 1 transistor = Rs. GH 120 × 960 JK

Q.29.(2) st

1 0 8 1 .2 0 × 1 0 0 × 1 0 0
x= = Rs. 1200 = Rs. 800
85 ×106
Q.25.(2) Let CP = 100 MRP = 150 C.P. of 2nd transistor
80 FG 100 × 1152 IJ = Rs. 1200
SP = 150×
100
Rs. 120 = Rs.
H 96 K
Hence Profit percent = 20%
Short trick So, total C.P. = Rs. (800 + 1200) = Rs. 2000

By successive formula Total S.P. = Rs. (1152+960) = Rs. 2112

x +y +
xy
= 50 − 20 +
5 0 × −2 0 FG 112 × 100 IJ % = 5 3 %

100 100
= 30 - 10 ∴Gain% =
H 2000 K 5
= 20% Q.30.(4) Let C.P. be Rs. x
Q.26.(1) Let CP = Rs. 100 120
MRP = Rs. 125 Then (1060-x)= 100 (x-950)
Since proift percent = 12.5% ⇒ 106000 - 100x = 120(x– 950)
So SP = 112.5
⇒ 220x = 220000 ⇒ x = 1000
then required discount %
FG 120 × 1000 IJ =Rs. 1200
=
1 2 5 − 1 1 2 .5
125
×100 =
1 2 .5
125
× 1 0 0 =10%
∴ Desired S.P. = Rs.
H 100 K

QUANTITATIVE APTITUDE 41
stportal.mahendras.org

CHAPTER Simple &


5 Compound
Interest
Principal: When you first deposit money in a saving acount x ×R ×5
or When you borrow some money from another person, bank, S.I. = x, x = R = 20%
100
or any financial institute is known as principal. ...............(1)
Rate of Interest: The extra money paid by the borrower to the Again P = x
lender for the use of money lent is called Interest. The rate of A = 4x
interest is the percent charge, or paid, for the use of money.
S.I. = 3x
Simple Interest: When principal remains constant
x ×R × T
irrespective of time period for which interest is being 3x = 100
calculated.
300 = R × T ..............(2)
This interest is always same for same period at the same rate From eq. (1) and (2)
of interest. So it is known as simple interest denoted as S.I.
T = 15 years
Formula :
SHort Trick :
P ×R × T
S.I = n1 − 1 n 2 − 1 2 −1 4 −1
100 = =
t1 t2 5 t2
Where P → Principal
R → Rate percent per annum 1 3
=
T → Number of years 5 t2

When S.I. is added to principal. It is change into amount. t2 = 15 years
P + S.I. = A By figure :
S.I. = A - P
5 year 5 year 5 ye ar
Case I : If S.I, R & T are known, P-x 2x 3x +x 4x
+x +x +x
S . I × 100
P= Total years = 5 + 5 + 5 = 15 year
R×T
Ex. The simple interest is the 9/16 of the principal & the
Case II : If S.I., P & T, are known, number of years is equal to rate %. Find the rate %.
S . I × 100 Sol. Let P = x
R=
P ×T
PRT
Case III : When S.I. P, R are known S.I. =
100
S . I × 100
T= 9 x ×R ×R
P ×R x =
16 100
Ex. Find simple interest on Rs. 5000 at the rate of interest
900 30 1
1 R2 = ⇒R= = 7 %
4% for 2 years. 16 4 2
2
5 Ex. A sum was invested at a certain rate of simple interest
5000 × 4 × for two year. If the sum were invested at 3% more rate
Sol. S.I. = 2 = Rs. 500
of interest, it would have fetehed Rs. 72 more. What
100
is the sum?
Ex. A sum of money doubles itself in 5 years. In how many
years it will become 4 times? Sol. According to question change in interest
Sol. P = x, A = 2x = 3% per annum in years

42 QUANTITATIVE APTITUDE
stportal.mahendras.org
Change in interest in 2 years *Note
= 3% × 2 = 6% Type Rate Time
that means Yearly R n
6% = 72 Half yearly R/2 2n
72 Quaterly R/4 4n
100% = × 1 0 0 = Rs. 1200
6 compount intrest may also be calculated by SI formula
Ex. Kumar borrowed a total amount of Rs. 30000 part of and change of principal after each time period.
it on S.I. rate of 12 p.c.p.a. and remaining on simple
interst of 10 p.c.p.a.. at the end of 2 year he paid in Note:
all Rs. 36480 to settle the loan amount. What was the We can also use successive percentage net percentage
amount borrowed at 12 p.c.p.a? method to calculate equivalent simple rate of interest.
Sol. Given p = 30000 xy
x +y +
A = 36480 100
SI. = A-P = 36480 - 30000 = 6480 If r = 20%
Let sum borrowed is x at 12 p.c.p.a for 2 year equvilant rate of interst
according to question 20 × 20
20+20 = 44%

12
×2 +
(3 0 0 0 0 − x ) × 1 0 × 2 = 6480
100
100 100 For 3 years
⇒ 24x + 600000 - 20x = 648000 44 × 20
44 + 20 + = 7 2 .8 % and son on.
4x = 48000 100

48000 Ex. Find C.I. on Rs. 16000 at the rate 5% p.c.p.a. for 2
x= = Rs. 12000 years, compounded annually.
4
Sol. By. Formula
Compound Interest
LMF r IJ − 1OP
n
As we discussed the S.I. is same for same time but in the case
GMH
C.I. = P 1 + 100 K PQ
of compound Interest this is not happened. In the case of C.I. N
The interest varies according to time eg.

L
= 16000 MFG 1 + 5 IJ − 1P
O
2

NMH 100 K PQ
Suppose we take 100 Rs. at the rate of compound interest
10% after one year we can not able to pay amount so, here
C.I. is imposed on amount rate Ist year. ie. 10% is imposed
on 110 and so on...... 16000 × M
L 21 × 21 − 1OP
= N 20 × 20 Q
So. C.I. is equal to S.I. in first year
but after 1 year C.I. > S.I.
L 441 O
= 16000 × M 400 − 1P
In ather words we can that principal varies with each time

N Q
period = 16000 × 41 = 40 × 41
400
Basic Formula :
= Rs. 1640
F r IJ
A = P G1 +
n
Short Trick :
H 100 K P = 16000
5%
800
for Ist year S.I. = C .I
1st year
LF r I
C.I. = P MG 1 +
n O
− 1P
5% rate of interest

MNH 100 JK
IInd year 800 + 40
PQ 5%
C .I. = 8 00 + 8 00 + 40
A → Amount = R s. 164 0
P → Principal Ex. Find C.I. on Rs. 10,000 at the rate of interest 6% for,
r → Rate of Interest 1
1 2 years, compounded annually.
n → Number of years

QUANTITATIVE APTITUDE 43
stportal.mahendras.org

LF r I − 1OP
P MG 1 +
n LF R I − 1OP
P MG 1 +
n

Sol. C.I. = MH 100 JK PQ Sol. C.I. = MH 100 JK PQ


N N
LF 6 I − 1OP LF 5 I O 3

= 6000 MGH 1 + 100 JK − 1P


3 /2

= 10,000 MGH 1 + 100 JK


MN PQ

MN PQ
LF 6 I F1 + 6 × 1 I − 1OP
1 F 105 105 105 I
= 6000 GH 100 × 100 × 100 − 1JK
= 10,000 × MG 1 +
MNH 100 JK GH 100 2 JK PQ

L106 × FG1 + 3 IJ − 1PO = 6000 × M


L 21 × 21 × 21 − 1OP
= 10,000 × M N 20 × 20 × 20 Q

N100 H 100 K Q F 9261 − 1IJ
= 6000 × G
= 10,000 × M
L106 × 103 − 1OP H 8000 K
N100 100 Q F 9261 − 8000 IJ
= 6000× G
= 10,000 × M
L10918 − 10 ,000 OP = Rs 918
H 8000 K
N 10000 Q =
6 × 1261 7566
= = Rs. 945.75
Short trick : 8 8
Ist Year Important Formula
6%
P = 10,0 00 600 If difference between C.I. & S.I. is to find out.
6% rate of interest Let the principal = P
6% 300 + 18 Time = T
1/2n year
Total = R s. 918 Rate % p.a. = R
(1) When time is 2 years
Ex. Mr. Bashu invested a certain sum in Scheme X for 4
year. Scheme X offers simple interest at the rate 12
C.I - S.I = P
FG R IJ 2

p.c.p.a. for the first 2 years and compound interest at


the rate 20% per annum for next two year. The total
H 100 K
(2) When time is 3 years
interest earned by him after 4 years is Rs. 11060. What
was sum invested.
C.I - S.I = P
FG R IJ × FG 300 + R IJ
2

Sol. Let sum invested be - x


H 100 K H 100 K
According to question. Ex. The difference between the C.I. and S.I. for a period
of 2 years at the rate of 10% per annum is 50. Find
S.I. + C.I. = 11060
the Principal.
C.I for 2 year by successive effect Sol. Method I.
20 × 20 By Slab. Method
20+20+ = 44%
100 10 % for Ist year S .I. = C .I
P=? 50 0
x ×12 × 2 x × 44 1st year
+ = 11060 10 % rate of interest
100 100 IIn d year 50 0 + 50
10 %
24 x × 44
x× 1 0 0 + 1 0 0 =11060 Hence principal = 5000
Method II
68
x× = 11060 By formula
100
2
11060  r 
x = 6 8 × 1 0 0 = Rs. 16200 D = P  
100 
2
Ex. Find the C.I. on Rs. 6000 at the rate of interest 5% for  10 
50 = P 
3 years, compounded yearly.  1 0 0 
44 QUANTITATIVE APTITUDE
stportal.mahendras.org

500000 Again,
P=
100
8P = P 1 +
FG r IJ n

P = Rs. 5000 H 100 K


Ex. The difference between S.I. & C.I. on a certain sum
of money for 3 years at 10% is Rs. 31. Find the sum F r IJ
8 = G1 +
n

of money.
H 100 K .............(II)
Sol. From formula for 3 years. using (I) and (II)
FG r IJ FG 300 + r IJ 2
R|F r I U| 3
CI. - S.I. = P
H 100 K H 100 K FG1 + r IJ n
= 8 = 2 = SG 1 +
15

|TH 100 JK V|
3
H 100 K W
F 10 IJ × 310
31 = P GH
2
100 K

100
FG1 + r IJ = FG1 + r IJ
n 45

31 = P
100
×
310 H 100 K H 100 K
100 × 100 100
x = 45 years
P = Rs 1000
IInd method :
Short Trick:
10% LMn = bn g OP t2
1 t1
N 2
Q
P=? 1st 100 10% n2, n1 — number of times
t1, t2 — number of years.
2nd + 10 10%
t2 t2 t2 3
3rd year + 10 + 10 + 1
8= 2 bg 15 23 = 2 bg 15
15 = 1
t2 = 45 years

Ex. SI of 2 years is 300 and CI is 320. Find
Ex. A sum of money doubles itself at compound interest
in 15 years. In how many years will it become eight (i) Rate of Interest
times. (ii) Principal

A =P 1+
FG r IJ n
Sol. 150
Sol.
H 100 K 150 20

2P = P 1 +
FG r IJ 15
20 40 1

H 100 K (i) r =
150
×100% =
3 =
13 %
3


FG
2 = 1+
r IJ 15
.............(1) 3
H 100 K (ii) 1 5 0 ×
40
× 1 0 0 = Rs. 1125

QUANTITATIVE APTITUDE 45
stportal.mahendras.org

EXERCISE
Q.1. A sum of Rs 7930 is divided into 3 parts and given an loan at 5% S.I. to A, B and C for 2, 3 and 4 years respect. If
amounts of all three are equal of the their respective periods of loan, A received a loan of -
(1) Rs. 2760 (2) Rs. 3560 (3) Rs. 2660 (4) Rs. 3450 (5) Rs. 2680
Q.2. A sum of Rs. 1550 was lent partly 5% and partly at 8% SI. The total interest received after 3 years is Rs. 300. The
ratio of money lent at 5% to that at 8% is -
(1) 15 : 14 (2) 15 : 13 (3) 16 : 15 (4) 5 : 6 (5) None of these
Q.3. A man invested equal sum of money at 12% per annum on compound interest and simple interest. After three years
the difference between the interest is Rs. 4449.28. Find the total amount he invested.
(1) Rs.11000 (2) Rs.10000 (3) Rs.12000 (4) Rs.12100 (5) Rs.15000
Q.4. If the simple interest on a certain sum of money at the rate of 4% per annum for 3 years is Rs. 1875. What would
be the C.I. at the same rate for the same time?
(1) Rs. 1950 (2) Rs. 1951 (3) Rs. 1975 (4) Rs. 1976 (5) Rs. 1900
Q.5. What is the sum of money at compound interest will give interest Rs. 6238 in 3 years, if the rate of interest is 3%
for the first year, 4% for the second year and 5% for the third year?
(1) Rs. 5000 (2) Rs. 10000 (3) Rs. 25000
(4) Rs. 50000 (5) Cannot be determined.
Q.6. Two partners X and Y invested their capital. X invested Rs. P at 10% per annum, Y invested Rs. (P – 4000) at the
rate of 12% per annum. If they both get total compound interest Rs 6412.8 at the end of two years, then find capital
invested by Y?
(1) 16000 Rs. (2) 12000 Rs. (3) 10000 Rs. (4) 14000 Rs. (5) 18000 Rs.
Q.7. A sum of Rs.7000 is deposited in two schemes. One part is deposited in Scheme A which offers 8% rate of interest.
Remaining part is invested in Scheme B which offers 10% rate of interest compounded annually. If interest
obtained in scheme A after 4 years is Rs.226 more than the interest obtained in scheme B after 2 years, find the
part deposited in scheme B.
(1) Rs.2800 (2) Rs.3850 (3) Rs.3580 (4) Rs.3800 (5) None of these
2
Q.8. The difference between the compound interest and simple interest on a certain sum of money for 3 years at 6
3
% per annum is Rs.184. Then find out the sum.
(1) Rs.13500 (2) Rs.12500 (3) Rs.11500 (4) Rs.10500 (5) None of these
Q.9. What will come at place of blank?
Rs_ is invested at 12.5% compound interest. After 3 years the sum becomes Rs. 5467.5.si?
(1) 3840 Rs. (2) 3520 Rs. (3) 4080 Rs. (4) 4250 Rs. (5) 3675 Rs.
Q.10. A man spends 85% of his monthly income, and the remaining money is deposited in the savings account for 5
years and he gets a simple interest of Rs. 2160 at the rate of 12% per annum. Out of monthly expenditure, he
spends 20% on rent, 15% on food. Out of the remaining he spends 50% on clothes and remaining on education.
Find the 33.33% of money that is spent on food.
(1) 1230 Rs. (2) 1530 Rs. (3) 1020 Rs. (4) 640 Rs. (5) None of these
Q.11. ` 600 becomes ` 708 in 3 years at certain simple rate of interest. If the rate of interest is increased by 5% what
amount will ` 600 become in 3 years?
(1) ` 758 (2) ` 798 (3) ` 804 (4) ` 850 (5) None of these
Q.12. A sum of Rs.31500 was lent partly at 7% and partly at 11% p.a. simple interest. The total interest received after 4
years was Rs.12708. Find the ratio of money lent at 7% to that lent at 11%.
(1) 4: 25 (2) 7: 29 (3) 11: 25 (4) 8: 27 (5) 13: 27
Q.13. Arun invested a sum of money at a certain rate of S.I. for a period of 4 year. Had he invested the same sum for a
period of 6 years, the total interest earned by him would have 50% more than earlier interest amount. what was the
rate of interest per annum?
(1) 4% (2) 8% (3) 5%
(4) can't be determined (5) None of these
46 QUANTITATIVE APTITUDE
stportal.mahendras.org
Q.14. Mukesh borrowed some money at the rate of 4 p.c.p.a for the first 3 year, at the rate of 8 p.c.p.a. for the next 2 year
and at the rate of 9 p.c.p.a for the period of beyond 5 year. If he pays a total S.I. of Rs. 19550 at the end of 7 years.
How much money did he borrow?
(1) Rs. 39500 (2) Rs. 42500 (3) Rs. 41900 (4) Rs. 43000 (5) Rs. 42000
Q.15. Dhoni investes two equal amounts in two banks giving 10% and 12% rate of interest respectively. At the end of
the year the interest earned is Rs. 1650 /- Find the sum invested in each bank.
(1) Rs. 8500 (2) Rs. 15000 (3) Rs. 7500 (4) Rs. 17000 (5) None of these
Q.16. The simple interest accrued on a certain principal is Rs 6400 in four years at 8 p.c.p.a.What would be the C.I accrued
on that principal at 2 p.c.p.a in 2 years?
(1) Rs 808 (2) Rs 750 (3) Rs 980 (4) Rs 720 (5) None of these
Q.17. What annual installment will discharge a debt of Rs. 2360 due in 4 years. at 12% p.a. simple interest ?
(1) Rs. 400 (2) Rs. 500 (3) Rs. 600 (4) Rs. 300 (5) Rs. 700
Q.18. A sum is invested for 3 years at the rate of 20% p.a. The difference between the simple interest and the compound
interest on same principal is Rs. 408. Find the principal.
(1) Rs. 5525.5 (2) Rs. 2550 (3) Rs. 3187.5 (4) Rs. 3400 (5) None of these
Q.19. The simple interest accrued on an amount of Rs. 40,000 at the end of 3 years is Rs. 12000. What would be C.I.
accrued on the same amount at the same rate in the same period?
(1) Rs. 18765 (2) Rs. 15350 (3) Rs. 21555 (4) Rs. 13240 (5) None of these
Q.20. The compound interest accrued on an amount of Rs. 22000 at the end of two years is Rs. 5596.8. What would be
S.I. accrued on the same amount at the same rate in the same period?
(1) Rs. 5420 (2) Rs. 5360 (3) Rs. 5280 (4) Rs. 5140 (5) None of these
Q.21. Mr. Basant invested Rs. 20,000 with rate of interest at 20 p.c.p.a. If the interest was compounded half yearly for
first year and in the next year it was compounded yearly. What will be the total interest earned at the end of two
years?
(1) Rs. 8800 (2) Rs. 9040 (3) Rs. 8040 (4) Rs. 9800 (5) Rs. 9500
Q.22. A sum of money Rs. 10200 is invested in 2 schemes. I scheme has offered compound intrest at the rate 10% for
st

2 years and IInd scheme has offered S.I. at the rate 5% per annum for 6 years. From both scheme equal interest is
obtained. Find the sum invested at simple interest.
(1) Rs. 4200 (2) Rs. 4800 (3) Rs. 6000 (4) Rs. 5800 (5) Rs. 5000
Q.23. A person invested equal amounts in two schems A and B at the same rate of interest. Scheme A offers S.I. while
scheme B offers compound interest. After two years he got Rs. 1920 from scheme A on interest and Rs. 2112
from scheme B. If the rate of interest is increased by 4%. What will be the total interest after two years from both
schemes?
(1) Rs. 4884.48 (2) Rs.4888.48 (3) Rs.4884.84 (4) Rs. 4384.48 (5) None of these
Q.24. A man has a debt of Rs. 820 which he undertakes to pay back with compound interest at the rate of 5% per annum
in 2 equal yearly installments at the end of first year and second year. What is the amount of each installment?
(approximately)
(1) Rs. 370 (2) Rs. 381 (3) Rs. 390 (4) Rs. 395 (5) Rs. 400
Q.25. The compound interest accrued on an amount at the end of two years at the rate of 12% per annum is Rs. 2862.
What is the amount?
(1) Rs. 11250 (2) Rs. 1220 (3) Rs.13500 (4) Rs. 10000 (5) None of these

QUANTITATIVE APTITUDE 47
stportal.mahendras.org

EXERCISE Explanation
Q.1.(1) According to question. 46.44 P = 743040
110% a = 115% of b = 120% of C = K P= Rs. 16000
Hence Capital invested by Y = (16000-4000)
= Rs. 12000
1 1 1 1 1 1
a:b:c= : : = : : _7000 - x i ×8×4
>d1 + 10 n - 1H + 226
2
110 115 120 2 2 2 3 2 4 Q.7.(4) = x
100 100
= 23 × 24 : 22 × 24 : 22 × 23
32x 21x
=276 : 264 : 253 70×8×4 – 100 = 100 + 226
Loan recieved by
53x
7930 2240 - 226 = 100
A= × 276 = Rs. 2760 x = Rs.3800
793
Q.8.(1) According to the question,
Q.2.(3) x × 5 × 3 (1 5 5 0 − x ) × 8 × 3 100
2
100
− = 300
100 100 P = 184× f 20 p × f 300 + 20 p
3 3
x = 800
800 16 = Rs.13500
Required Ratio = = 100 100 100
750 15 Q.9.(1) 5467.5× 112.5 × 112.5 × 112.5 = 3840
By alligation Q.10.(3) 5Y=2160
1Y=432
12%=432
100%=3600 (saving)
now 15% of income=3600
85% of income=20400 (expenditure)
= 16 : 15 expenditure on food=15% of 20400=3060
33.33% of 3060=1020
2
Q.3.(2) d = P  r   3 0 0 + r  600 × 5 × 3
100   100  Q.11.(2) S.I. on 5% interest = = ` 90
100
2
12   300 +12 
449.28 = P    
Total S.I. = (708–600) + 90= ` 198
100   100 
P = 10000 Total amount = 600+198= ` 798
1875 Q.12.(4) Let the sum lent at 7% be x
Q.4.(2) ×12.4864 = 1951
12 then,

Q.5.(4) 3× 4 x × 7 × 4 (3 1 5 0 0 − x ) × 1 1 × 4
3+ 4+ = 7.12 + = 12708
100 100 100
7.12 × 5 2 8 x (3 1 5 0 0 − x ) × 4 4
7.12 + 5 + = 12.476 + = 12708
100 100 100
6238
P= × 100 = Rs.50000 28 x + 31500 × 44 − 44 x
12.476 = 12708
100
Q.6.(2) X invested = P Rs.
- 16x = 1270800 - 1386000
Y invested = (P-4000)

10×10
2 years CI on 10% = 100 = 21% 115200
x= = 7200
16
12×12
2 year CI on 12% = 12+12 + 100 = 25.44%
Required ratio
21P 25.44 ] - 7200 7200
100 + 100 P 4000 = 6412.8
g =
=
21P + 25.44 P- 101760 = 641280 (3 1 5 0 0 − 7 2 0 0 ) 2 4 3 0 0 = 8 : 27
48 QUANTITATIVE APTITUDE
stportal.mahendras.org
Q.13.(4) A ccording to the question. 10%

P × R × 6 P × R × 4 150
= ×
100 100 100 P = 40 00 0 1st 400 0 10%

This relation give no result. 2n d 40 00 4 00 10 %


Q.14.(2) If the amount borrowed be Rs x then.
According to question. 3rd ye ar 40 00 40 0 + 40 0 + 40

x ×4 ×3 x ×8 ×2 x ×9 ×2
+ + = 19550
100 100 100 CI = Rs. 13240
12x+16x+18x = 1955000 Q.20.(3) According to question,
46x = 1955000 Rate of interest of C.I. for 2 years
x = Rs. 42500 5 5 9 6 .8
= × 1 0 0 = 25.44%
Q.15.(3) Let sum be Rs. x 22000
According to question that means

x × 1 0 × 1 x × 1 2 x1 Rate of interest / anum = 12%


+ = 1650
100 100 Now
1650 ×100 S.I. for 2 years = 24%
x= = Rs. 7500
22 22000 × 24
S.I. = = Rs. 5280
100
6400 ×100 Q.21.(2) According to formula
Q.16.(1) P = = Rs 20000
8 ×4  R 
n 2
A = P 1 +  = 20000  1 + 1 0   1 + 2 0 
2  100   100   100 
 2  
C.I = P  1 0 0  − 1
 1 +
  
= 20000 ×
11 11 6
× × = 29040
10 10 5
= 20000 × 0.0404= Rs. 808
C.I = 29040- 20000 = Rs. 9040
Q.17.(2) Let the installment be Rs. 100:
Installments paid at the end of 1, 2, 3 and 4 yers Short Trick :
10%
earn a simple interest at 12% p.a. for 3, 2, 1, 0
year respectively.
P = 20000 1st 2000 10%
Hence the respective installments amount to -
(100+3×12) + (100+2×12) + (100+1×12) + 100 2nd 2000 200 20%

= 236
3rd year 40 000 4 0 + 400 + 40
136+124+112+100 = 2360

472 = 2360
C.I = Rs. 9040
2360 Q.22.(1) Let amount X is invested at C.I.
100 = 4 7 2 × 1 0 0 = Rs. 500
and amount Y is invested at S.I.
2
100   100  then
Q.18.(3) Principal = 408 ×    
 20   20 + 300   r 
2

C.I. = P  1 0 0  − 1
1 +
408 ×100 ×100 ×100  
= = Rs. 3187.5
20 × 20 × 320  10 
2

Q.19.(4) According to question =  1 0 0  − 1 ---- (1)
x 1 +
 
S.I. × 1 0 0 1 2 0 0 0 × 1 0 0 again
R= = = 10%
P ×T 40000 × 3
y ×5 ×6
Now for C.I. We use Slab Method. S.I. = - (2)
100
QUANTITATIVE APTITUDE 49
stportal.mahendras.org
According to question So. Total interest = 2304 + 2580.48= Rs. 4884.48.
from eq (1) and (2) Q.24.(2) By formula
 10 
2
 y ×5 ×6 D
x  1 +  − 1 = I=  1 2
  100   100 r   r 
 1 +  + 1 + 
100   100 
x : y = 10 : 7
Hence 820
= 1 2
10200  5   5 
sum invested at S.I. = × 7 = 4200  1 +  +  1 + 
17 100 100 
820
Short Trick :
= 21 21 21
S.I. for 6 years = 5% × 6 = 30% + ×
20 20 20
10 ×10
C.I. for 2 year = 10 + 10 × = 21% 820 820 820 × 400
100
According to question = 21 441 = 420 + 441 = 861
+
21% of x = 30% of y 20 400 400

x : y = 10 : 7
= 380.95 ≈ 381
10200 × 7
Hence sum invested at S.I. = Q.25.(1) According to formula
17
n
= Rs. 4200  r  
Q.23.(1) Difference between S.I. and C.I. for 2 years C.I. = P  1 0 0  − 1
  1 + 
 
= 21112 - 1920 = 192 2
 12  
1920 
2862 = P 
1 +  − 1
S.I. for 1 year = = 960 100
2  

192  2 8  2 
Hence r = × 1 0 0 = 20% per annum. − 1
960 2862 = P   
 2 5  
Now,
S.I. (20%) = 960 2862 × 625
P= = Rs. 11250
960 159
P (100%) = 2 0 × 1 0 0 = 4800 Method II
When rate is increased by 4% then By Successive formula, net C.I rate person for 2
4800 × 24 × 2 years
New S.I. = = 2304
100 xy
24 × 24 12 ×12
= x +y+
again rate of C.I. for 2 year = 24 + 24 + 1 0 0 = 12 + 12 + 1 0 0 = 25.44%
100
= 48 + 576 = 53.76% Now given.
Hence 25.44% = 2862
5 3 .7 6 2862
New C.I. = 4800 × = 2580.48 100% = ×100 = Rs. 11250
100 2 5 .4 4

50 QUANTITATIVE APTITUDE
stportal.mahendras.org

CHAPTER

6 Average

Average is equal distribution of the overall value among all Sol. Required average
the things or persons present there. So the formula for finding 3 8 .9 × 1 0 − 4 2 × 6
the average is as follows : = 34.25
4
A verage =
FG S um of observations IJ Ex. The average age of 80 girls was 20 years, the average
H N um ber of observations K age of 20 of them was 22 years and another 20 of them
was 24 years. Find out average age of the remaining
If there are average is also known as mean observation x1, girls?
x2,x3....xn
Sol. Total age of remaining 40 girls
x 1 + x 2 + ... + x n
Average = =(80 × 20 - 20 × 22 - 20 × 24) = 680 years
n
Average = 680 = 17 years
Ex. What is average of 40, 42, 38, 48. 40
40 + 42 + 38 + 48
Sol. Average = = 42 Short Trick:
4
Ex. There are two sections A and B of a class, consisting of 22 + 24
Average of 40 girls= = 23 years
10 and 20 students respectively. If the average weight 2
of sections A is 60 kg and that of section B is 45 kg. Average of 40 girls is 3 more than actual average so average
Find the average weight of the whole class (in kg). of remaining 40 girls is 3 less than actual average 20 - 3
60 ×10 + 45 × 20 = 17 years
Sol. Average =
10 + 20
Concept of Consecutive Term
600 + 900 (A) To solve consecutive terms problem, we will use exact
30 = 50
mid concept. Exact mid term is average of given term.
Ex. A batsman makes a score of 97 runs in the 21th inning  n u m b e r + 1
and thus increases his average by 2. Find his average Exact mid = 
  te rm s
2
after 21th inning.
6 +1
Sol. Let the average after 21th innings = x. Then, Exact mid of 6 terms = = 3.5
2
Average after 20th innings = (x - 2)
3.5 many in between 3rd and 4th number exact mid of
Average 9 +1
9 terms = = 5th term
= 20 (x-2)+97 = 21x, 20x-40+97 = 21x 2
x = 57 Ex. Average of 5 consecutive terms is 43.
Ex. A student was asked to find the arithmetic mean of the So smallest no is-
numbers 3,11, 7, 9,15,13, 8,19, 17, 21,14 and x. He 5 +1
found the mean to be 12. What should be the number Sol. Exact mid of 5 terms = =3
2
in place of x? 1 2 3 4 5
Sol. 12=(3+11+7+9+15+13+8+19+17+21+14+x)/12 41 42 exact mid term = 43
Number in place of x is Smallest is 41
137+x=144 , x = 144-137 (B) Average of consecutive terms:
x=7. 1st te rm + la st te rm
=
Ex. The average score of a cricketer for 10 matches is 38.9 2
runs. If the average for the first six matches is 42 then Ex. Find the average of all the numbers from 20 to 90
find the average for the last four matches. which is divisible by 3.

QUANTITATIVE APTITUDE 51
stportal.mahendras.org
Sol. First term is 21 and last is 90. That means excluded no is 4 more than average.
21 + 90 So, excluded number = 21 + 4 = 25
So, average = = 55.5
2 Ex. The average of 12 no. is 45 when one no is excluded
Ex. Find the average of all the numbers from 20 to 90 then average becomes 46. What is the excluded
which is divisible by 8. number?
Sol. First term 24 and last is 88. Sol. In this case average increased by = (46-45) = 1
24 + 88 Total increased = 1 × 11 = 11
So, average = = 56
2 That means excluded number is 11 less than average.
Ex. Find the average of all the numbers from 20 to 90
which divisible by 6. Number is 45 - 11 = 34
Sol. First term is 24 and last is 90. Ex. Average marks of 35 students is 70. If marks of one
student is excluded then average increased by 2 marks.
24 + 90
So, average = = 57 What is the marks of excluded student?
2
Ex. The sum of eight consecutive even number 288. What Sol. Excluded student marks
is the sum of different set of six consecutive numbers = 70 – (35 - 1) × 2= 70 – 68 = 2
whose lowest number is 10 more than the mean of set
A? Ex. The average age of 12 persons is decreased by 2 years
when one of them having age 39 years is replaced by
288 a new person. Then what is the age of new person?
Sol. Mean of set A = =36
8
Sol. Age of new person = 39 - (12 ×2)
So, lowest number of other set = 36 + 10 = 46
Required sum = 46 + 47 + 48 + 49 + 50 + 51 = 291 = 39 - 24 = 15 years

Concept of addition or removal of items and change in Ex. The average weight of 10 persons is increased by 2.5
average: kg when two of them having weight 60 and 75 kg
replaced by two new persons. Then what is the average
The excluded or included item= Average + extra
weight of new persons?
(Extra means total change in average)
Sol. Sum of age of new persons = (60+75) + (2.5×10) =
Ex. Average of 5 no. is 21 one no is excluded then average 135 + 25 = 160
becomes 20. What is the excluded no.
Average age = 160/2 = 80 kg
Sol. Average is 21.
So, if the excluded number is 21, then there will be Ex. The average of 50 numbers is 38. It was found that
no change in average. one number is misread as 89 instead of 39. The correct
mean is?
But new average is 20.
Average decreased by = 21 - 20 = 1  89 − 39 
Sol. New average = 38 -   = 38 - 1 = 37
Total decreased = 1 × 4 = 4  50 

52 QUANTITATIVE APTITUDE
stportal.mahendras.org

EXERCISE
Q.1. Find the average of first 10 prime numbers.
(1) 12.1 (2) 11.9 (3) 12.9 (4) 30.25 (5) None of these
Q.2. Out of four numbers, the average of first 3 numbers is thrice the fourth number. If average of all four numbers is
5, what is fourth number?
(1) 4.5 (2) 4 (3) 5 (4) 2 (5) None of these
Q.3. Average age of students in section A of 55 students is 12 and the average age of students in section B of 45 students
is 14 years. Find the average age of students in both section taken together?
(1) 13 years (2) 12.5 years (3) 13.2 years (4) 12.9 years (5) 13.5 years
Q.4. The average monthly income of a family was Rs. 5000 during first 2 months, Rs. 6000 during next 4 months and
Rs. 7000 during last 6 months of the years. If total saving is Rs. 10000, Find average monthly expenditure.
(1) Rs. 550 (2) Rs. 5000 (3) Rs. 5500 (4) Rs. 5550 (5) Rs. 5555
Q.5. In an art exhibition average visitors from Monday to Friday is 620 and 900 in remaining days. Find out average
visitor in per day?
2
(1) 700 (2) 750 (3) 800 (4) 666 (5) None of these
3
Q.6. The average of six consecutive odd no is 48. Then find out largest and smallest no.
(1) 57,47 (2) 53,41 (3) 55, 45 (4) 53, 45 (5) None of these
Q.7. Find the average of numbers lies between 100 and 300 as well as divisible by 7.
(1) 199.5 (2) 200 (3) 205 (4) 203 (5) 2015
Q.8. Set A, in which 6 consecutive odd number, has average 76. The smallest number of Set B in which 6 consecutive
even number, is 33 less than the highest number of set A, what is the average of the numbers of set B?
(1) 53 (2) 67 (3) 93 (4) 89 (5) None of these
Q.9. The average weight of 29 students is 40 kg. If the weight of teacher be included the average weight increased by
400 gms.The weight of the teacher is-
(1) 52 kg (2) 55 kg (3) 50 kg (4) 49 kg (5) None of these
Q.10. A cricketer whose bowling average is 24.85 runs per wicket, takes 5 wickets for 52 runs in last match and there by
decrease his average by 0.85. The number of wickets taken by him before the last match was?
(1) 85 (2) 75 (3) 95 (4) 80 (5) None of these
Q.11. Batting average of Rohit Sharma in three ODI series is 48, 37.5 and 51 respectively. Of the number of matches in
these ODI series are in ratio 3:4:2. Find his overall average in these three series.
(1) 50.5 (2) 49.25 (3) 44.4 (4) 46.75 (5) None of these
Q.12. The average age of a family consisting 15 person is 42 years, when two person aged 36 years and x years is replaced
by 2 new person of average age 21 years of another family then average increases by 1 year. Find the value of x .
(1) 13 (2) 11 (3) 7 (4) 9 (5) None of these
Q.13. Find the average of first 120 natural numbers which is multiple of 7.
(1) 29600 (2) 72800 (3) 92820 (4) 50820 (5) None of these
Q.14. Virat played three matches in a tournament .The respective ratio between the scores of 1st and 2nd match was 6: 5
and that between the scores of 2nd and 3rd match was 3: 2.The difference between the 1st and 3rd match was 128
runs. What was Virat’s average score in all the three matches?
1 1 1 1 1
(1) 223 (2) 226 (3) 229 (4) 230 (5) 231
3 3 3 3 3
Q.15. The average age of a family of 4 member is 25 years. If the present age of youngest member is 7 years, then what
was the average age of the family at the time of birth of the youngest member?
(1) 18 years (2) 24 years (3) 20 years (4) 22 years (5) None of these

QUANTITATIVE APTITUDE 53
stportal.mahendras.org
Q.16. The average of six number is 40. If average of first three is 35 and last four is 45. Then third number is ?
(1) 35 (2) 40 (3) 45 (4) 50 (5) 60
Q.17. The average of 10 numbers is 43. Average of first four is 40 and last five is 47. Then 5 number is-
th

(1) 35 (2) 38 (3) 32 (4) 36 (5) 40


Q.18. The sum of 7 consecutive odd numbers is 539, also average of four consecutive even no. is 93. What is sum of
smallest even number and 3rd largest odd number.
(1) 629 (2) 631 (3) 627 (4) 169 (5) None of these
Q.19. The average marks of Rahul, Manu and Sheetal is 73. Rahul's marks is 18 less than Ankit and 12 more than Manu.
If Ankit got 30 marks more than the average marks of Rahul, Manu and Sheetal, what is the sum of the marks of
Manu and Sheetal?
(1) 130 (2) 132 (3) 134 (4) 138 (5) None of these
Q.20. The average marks obtained by 120 students was 37. If the average of passed candidate was 42 and failed candidates
is 18, the number of candidates who failed in the examination is
(1) 85 (2) 70 (3) 75 (4) 95 (5) 25
Q.21. The average score of a class of boys and girls in an examination is 42. The ratio of boys and girsl is 3 : 2. If the
average score of boys is 46, the average score of girls?
(1) 40 (2) 39 (3) 36 (4) 48 (5) 38
Q.22. In the first 10 overs of a cricket game, the run rate was 6.8. What should be the run rate in the remaining 40 overs
to reach the target of 350 runs?
(1) 6.05 (2) 5.05 (3) 7.05 (4) 8.05 (5) 9.05
Q.23. The average salary of all the workers in a workshop is Rs. 8000. The average salary of 12 officer is 15000 and
average salary of non officer is 7000. The total number of non-officers is
(1) 82 (2) 84 (3) 86 (4) 88 (5) 90
Q.24. A number of friend decided to go on a picnic and planned to spend Rs.96 on eatables. Four of them did not turn
up. As a consequence the remaining ones had to contribute Rs. 4 each extra. The number of those who attend the
picnic was-
(1) 12 (2) 16 (3) 6 (4) 8 (5) 24
Q.25. In a school with 300 students, the average age of boys is 16 years and that of the girls is 14 years. If average age
of class is 14 years 8 month then the number of boys in the school is
(1) 60 (2) 80 (3) 100 (4) 120 (5) 140

54 QUANTITATIVE APTITUDE
stportal.mahendras.org

EXERCISE Explanation
Q.1.(3) Sum =
2 + 3 + 5 + 7 + 11 + 13 + 17 + 19 + 23 + 29  30 × 400 
Q.9.(1) Weight of teacher =  4 0 + kg.
10  1 0 0 0 
129 = 52kg
Average = = 1 2 .9
10 Q.10.(4) 24.85 52
Q.2.(4) Let the numbers are a, b, c and d. 5
24
According to the question,
13.6 0.85
a +b +c
= 3d, a + b + c = 9d …(I) 16 : 1
3
Number of wickets before last match .
a +b +c
and = 5, a + b + c + d = 20 …(II) 5
4 = × 1 6 = 80
From (I) and (II), we get 1
3×48×4×37.5 + 2×51
Q.11.(5) Required answer= =44
9d + d = 20, 10d = 20, d = 2 3+4+2

Q.3.(4) Average =
55 ×12 + 45 ×14 Q.12.(4) 42×15 -_36 + x i + 42 = 43
15
55 + 45
x = 645-636 = 9
Q.13.(5) Sum of first 120 natural numbers which is multiple
660 + 630 1290
= = = 12.9 years of 7,
100 100
Q.4.(3) Total income = 5000× 2 + 6000 × 4 + 7000×6 7+ 14+21………..+840 = 7 (1+2+3……..+120),
= 10000 + 24000 + 42000 = 76000
n (n +1)
Total expenditure = 76000 - 10000 Sum of first n natural numbers =
2
= 66000
121
66000 So, 7 (1+2+3……..+120) = 7×120× = 50820
Average expenditure = = 5500 2
12 50820
Now average = =423.5
Q.5.(1) Total vistors = 620 × 5 + 900 × 2 = 3100 + 1800 120
= 4900 Q.14.(3) Match 1: Match 2 = 6: 5
4900
Average visitors = 7 = 700 Match 2: Match 3 = 3: 2
Therefore,
6 +1
Q.6.(5) Exact mid of 6 no= means in between 3rd Match 1: Match 2: Match 3 = 18: 15: 10
2
and 4 term
th
Now ,
1 2 3 4 5 6
18x – 10x = 128
48
x = 16
Largest no = 48 + 5 = 53
Total runs scored in three matches
Smallest no. 48 - 5 = 43
= 18x + 15x +10x = 43x
Q.7.(1) First no is = 105
= 43×16 = 688
Last no is 294
Required average = (688/3) = 229(1/3)
105 + 294
Average = = 199.5 Q.15.(2) Present age sum = 25 × 4 = 100
2
Sum of age 7 years age = 100 -7 × 4= 72
Q.8.(1) Set A = 71, 73, 75, 77, 79, 81
72
Set B = 2x-4,2x-2, 2x,2x+2,2x+4,2x+6 Average = = 24 years
3
According to questions- 81 -(2x-4) = 33
SHORT Trick:
⇒ x=26
So set (B) = 48, 50, 52, 54, 56, 58 (2 5 − 7 ) × 4
= 24 years
3
Average of set B = 53

QUANTITATIVE APTITUDE 55
stportal.mahendras.org
Q.16.(3) Sum of six no. = 40 × 6 = 240 42 − x 3
= =
Sum of 1st three no. 35 × 3 = 105 4 2
Sum of last four no. = 45 × 4 = 180 x = 36
Sum of 1st three and last four = 105 + 180 = 285 Q.22.(3) Total run in first 10 over = 6.8 × 10 = 68
So, 3rd number = 285 - 240 = 45 Required run rate in the remaining 40 overs
Q.17.(1) Sum of 10 numbers = 43 × 10 = 430 350 − 68
= 40
= 7.05
Sum of 1st four numbers = 40 × 4 = 160
SHORT Trick:
Sum of last five numbers = 47 × 5 = 235
350
Sum of 1st four and last five = 160 + 235 = 395 Required average of game = =7
50
So, 5th number = 430 - 395 = 35
539 6.8 x
Q.18.(4) Average of odd number = = 77
7 7
So, 3rd largest number = 77 + 2 = 79
(x-7) : 0.2
Average of four even number = 93
So, smallest number = 93 (x − 7 ) 1
= or, 4x - 28 = 0.2
So, smallest number = 93 - 3 = 90 0 .2 4
Required sum = 79 + 90 = 169 So, x = 7.05
Q.19.(3) Rahul + Manu + Sheetal = 219 -----(I) Q.23.(2) By Alligation
Rahul = Ankit - 18 ------------ (II) 15000 7000

Rahul = Manu + 12 ------ (III) 8000

R ahul + M anu + S heetal 1000 7000


A nkit = + 30 ---(IV)
3 Ratio between officer and non - officer= 1 : 7
From eqn. (I), (II),(III) and (IV) No. officres is = 12
Ankit = 103 So, non officers = 12 × 7 = 84
Rahul = 85 Q.24.(4) Let there were x friends.
Manu = 73 96
So, contribution =
Required sum = 61 + 73 = 134 x
Four friends left
Q.20.(5) Using alligation
96
Passed : Failed = 19 : 5 So, new contribution =
x −4
96 96
∴x −4 - = 4, x = 12
x
So, number of friend who attended the picnic
= 12 - 4 = 8
5 Q.25.(3)
So, failed students = 120 × = 25 B oys G irls
24
16 14

B oys G irls 44 /3
Q.21.(3) x
46
2 : 4
1 : 2
42
1
(42-x) 4 So, boys = 300× =100
3

56 QUANTITATIVE APTITUDE
stportal.mahendras.org

CHAPTER
Ratio &
7 Proportion
Comparison of Ratio:
Ratio
Let a : b and c : d, be two ratios, then
Ratio is a comparison of two quantities. The ratio of a to b
is written as- a : b > c : d if ad > bc
a a c
a:b= = a ÷b ie. > if ad > bc
b b d
Types of Ratio: Similarly,
(i) Compounded Ratio: When two or more than two a : b < c : d if ad < bc
ratios are multiplied with each other, then it is called a : b = c : d if ad = bc
as Compounded Ratio.
Two Important Results
a
2 4 6 48 2 4 If > 1 , it is implied that a > b
Ex. × × = is the compounded ratio of , b
3 5 7 105 3 5
Ist term > 2nd term
6. a
and <1⇒a<b
7 b

a c e k
Important Facts
So × × ........... = (a) If A : B = x : y and B : C = p : q then
b d f l
(1) A : C = x × p : y × q
(ii) Duplicate Ratio: Square of any ratio is known as (2) A : B : C = (x : y) ×p : y × q
Duplicate Ratio.
=x×p:y×p:y×q
Duplicate Ratio of a : b = a2 : b2. Or
Ex. Duplicate ratio of 7 : 5 = 49 : 25 A:B =x:y
(iii) Triplicate Ratio: Cube of any ratio is known as
Triplicate Ratio. B :C =p:q
Triplicate Ratio of a : b = a : b . x×p :y× p:y× q
3 3

Ex. Triplicate ratio of 9 : 2 = 729 : 8 (b) If A : B = x : y, B : C = p : q and C : D = m : n, then


(iv) Sub Duplicate Ratio: Square root of any ratio is (1) A : D = x × p × m : y × q × n
known as Sub Duplicate Ratio: (2) A : B : C : D = (xp : yp : yq) ×m:y×q×n
Sub Duplicate Ratio of a : b = a1/2 : b1/2. = xpm : ypm : yqm : yqn
Ex. Sub duplicate ratio of 9 : 16 = 3 : 4 Or

(v) Sub Triplicate Ratio: Cube root of any ratio is known x:y
as Sub Triplicate Ratio:
Sub Triplicate Ratio of a : b = a1/3 : b1/3. p:q
Ex. Sub triplicate ratio of 27 : 125 = 3 : 5 m :n
Properties of Ratio : xp m :yp m :yq m :yq n
1. In a ratio, two quantities are compared, so the Ex. If A : B = 3 : 4 and B : C = 8 : 9 then A:B:C is
quantities must be of the same kind. Sol. A : B = (3 : 4) × 2
2. The ratio of two quantities determines how many times B:C=8:9
one quantity is contained by the other.
A:B:C=6:8:9
QUANTITATIVE APTITUDE 57
stportal.mahendras.org
SHORT Trick b2
A:B =3:4 ∴c =
a
B :C =8:9 Ex. Find the third proportion of 0.36 & 0.48.
A : B : C = 24 : 32 : 36 b 2 0 .4 8 × 0 .4 8
Sol. Third proportion = c = = 0 .6 4
A:B:C=6:8:9 a 0 .3 6
2 1 2 5 (ii) Fourth Proportion: If a : b :: c : d, then d is called
Ex. If a : b = : ,b:c= : and the 4th proportion to a,b and c. d will be calculated
9 3 7 14
as below:
7 3
d:c= : then a:b:c:d is- a : b :: c : d or a : b = c:d
10 5
2 1 ⇒ a×d = b×c
Sol. a : b = : = 2:3
9 3 bc
∴d =
2 5 a
b:c= : = 4:5 Ex. Find the fourth proportion of 3,6, and 12.
7 14
7 3 bc 6 ×12
d:c= : = 7:6 Sol. Fourth proportion= d = = = 24
10 5 a 3
⇒c:d=6:7 (iii) Mean Proportion: If a and b are two numbers then
mean proportion is a b
a:b=2:3
if mean proportion between a and b is A then :
b:c= 4:5
a:A :: A:b
c:d= 6:7
⇒ A2 = ab
a:b:c:d = 2×4×6 : 3×4×6 : 3×5×6 :3×5×7
or
= 16 : 24 : 30 : 35
a ×b = A ×A
Ex. If a : b = c : d = e : f = 1 : 2 then (pa+qc + re) : (pb +
qd +rf) is equal to ∴A = a b

Sol. a c e 1 Ex. Find the mean proportion between 9 and 16.


= = =
b d f 2
Sol. Required mean proportion = ab = 9 × 16 = 12
p a q c re 1
⇒ = = = Continued Proportion :
p b q d rf 2
Three quantities a, b, c of same kind are said to be in
p a + q c + re 1 continued proportion, when a : b = b : c
⇒ = or 1 : 2
p b + q d + rf 2
The middle number b is said to be a mean proportional
Proportion to two extreme numbers a & c
A proportion is an expression which states that two So, in such case of continued proportion.
ratios are equal.
b2 = ac
3 1
eg. = is a proportion (middle number)2 =First number × Last number
12 4
It can also be expressed as Ex. If 3, x, 27 are in continued proportion, then find the
value of x.
3 : 12 = 1 : 4 or 3 : 12 : : 1 : 4
Sol. Since 3, x, 27 are in continued proportion.
Each quantity in proportion is called a term
x2 = 3 × 27
proportional. The first & the last terms are called the
extreme term whereas the second & the third terms x2 = 81
are called middle term. x=9
Types of Proportion : Relation Among the Quantities More than two:
(i) Third Proportion :- If a : b :: b : c, then c is called Given a : b = x : y
the 3rd proportional to a and b. c will be calculated
as below a : b :: b : c or a : b = b : c b:c=m:n

⇒ a ×c = b × b then these three quantites are related as,


58 QUANTITATIVE APTITUDE
stportal.mahendras.org
a:b= x :y Ex. Cost of a diamond varies directly as the square of
its weight. A diamond broke into four pieces with
b:c= m :n their weight in the ratio 1:2:3:4. If the loss in the
total value of the diamond was Rs.70,000, the price
a : b : c = xm : m y : yn of the original diamond was-
Sol. Let the weight of the four pieces of diamond be
a:c = xm : yn x, 2x, 3x & 4x units respectively.
Useful Results on Proportion : Total weight of the original diamond
If four quantities a, b, c and d are said to be proportion = x + 2x + 3x + 4x
if and only if = 10x units
Price of the original diamond
a:b=c:d
= K (10x)2 = K 100x2
Where K is a constant
a×d=b×c
Cost of four pieces of diamond
⇒product of extrems = product of means
= K (x2 + 4x2 + 9x2 + 16x2)
Componendo and Dividendo:
= K.30X2
If a : b is equal to c : d
Loss in the cost of diamond
a c
= = K (100x2 - 30x2)
b d
= K.70x2
a +b c +d ∴ K.70x2 = 70000
i) Componendo Rule - =
b d
Kx2 = Rs. 1000
a −b c −d ∴ Cost of the original diamond
ii) Dividendo Rule - =
b d
= K (100x2)
iii) Componendo and Dividendo Rule -
= 1000 × 100 = 1,00,000 Rs.
a +b c +d
= Ex. The price of sugar is increased by 20%. If the
a −b c −d
expenditure is not allowed to increase, the ratio
Ex. A bag contains 50 P, 25 P and 10P coins in the ratio between the reduction in consumption and the
5 : 9 : 4. amounting to Rs. 206. Find the number of original consumption is:
coins of 50 paise. Sol. When price is increased by 20%
Sol. Let the number of 50P, 25P and 10 P coins be 5x, 9x then expenditure is decreased by
and 4x respectively.
r
5x 9x 4x ×100
+ + = 206 100 + r
2 4 10
20
×100
50x + 45x + 8x = 4120 120
103x = 4120 2
x = 40 16 %
3
No. of 50 P coins = 5 ×40 = 200 Hence required Ratio = 1 : 6
Ex. A mixture contains alcohol and water in the ratio of Ex. The largest and the smallest angles of a triangle
4 : 3. If 5 litres of water is added to the mixture the are in the ratio of 3:1 respectively. The second
ratio becomes 4 : 5. Find the quantities of alcohol in largest angle of the triangle is equal to 440. What
the given mixture. is the value of 150% of the largest angle of the
Sol. Let the quantity of alcohol and water be 4x litres and triangle?
3x litres respectively. Sol. Sum of largest and smallest angle
4x 4 = 1800 - 440 = 1360
= ⇒ 8x = 20
3x +5 5 136
Largest angle = × 3 = 34×3 = 1020
x = 2.5 4
150
Quantity of alcohol = 4 × 2.5 = 10 litres 150% of largest angle= 102× =1530
100

QUANTITATIVE APTITUDE 59
stportal.mahendras.org

EXERCISE
Q.1. A watermelon is cut into two pieces in the ratio of 3 : 5 by weight. The bigger of the two is further cut in the ratio
of 5 : 7 by weight . Find the ratio of weight of each of the three pieces.
(1) 3 : 5 : 7 (2) 15 : 25 : 56 (3) 36 : 25 : 35 (4) 3 : 2 : 3 (5) None of these
Q.2. If b : (a+c) = 4 : 9 and c : (a+b) = 3 : 7 then find the ratio of a :(b+c)
(1) 53 : 77 (2) 5 : 8 (3) 51 : 79
(4) Can't be determined (5) None of these
Q.3. Divide Rs. 3600 among P, Q, R, in the ratio 7 : 2 : 9. Find the amount received by Q.
(1) Rs. 1000 (2) Rs. 400 (3) Rs. 500 (4) Rs. 1800 (5) Rs. 600
Q.4. What least number must be subtracted from each of the numbers 14, 17 , 34 and 42 so that the remainders may be
proportional ?
(1) 0 (2) 1 (3) 2 (4) 7 (5) 4
Q.5. 20 boys and 25 girls form a group of social workers. During their member ship drive, the same number of boys
and girls joined the group. How many members does the group have now if the ratio of girls to boys is 8:7?
(1) 75 (2) 65 (3) 70 (4) 60 (5) 80
Q.6. What will come at place of blank?
A solution has _ liter milk and 35 liter water. When 25.5 liter milk is poured in solution percentage of milk be-
comes 60%.
(1) 15 (2) 27 (3) 24 (4) 30 (5) 36
Q.7. The value of a diamond is Rs 40000, which is directly proportional to the square of its weight. For some reason
the diamond fall and is broken into two parts in the ratio of 1: 3. Find out the loss percentage on diamond.
(1) 0% (2) 32% (3) 35% (4) 37.5% (5) None of these
Q.8. In a mixture 200 litres of milk and 40 litres of water. ____ litres of mixture is drawn and ____ litres of pure water
is added, the milk in the mixture is 124 litres more than the water. Which of the following options satisfies the two
blanks in the question?
A) 36, 12 B) 24, 8 C) 18, 20 D) 12, 6
(1) Only B (2) Only C (3) Only A (4) Only D (5) Only B) and C
Q.9. A mixture contains milk and water in ratio 5: 1. On adding _______liters of water, the ratio of water to milk be-
come 2 : 3. The quantity of Milk in the mixture is ______liters. Which of the following options is /are possible for
the value of missing place in the same order.
A. 6, 15 B.7,12 C.14, 30 D. 21,30
(1) Only A (2) Only B (3) Any Two (4) Any Three (5) None of these
Q.10. A vessels contains mixture of 3 types of milk A, B, C. The ratio of their quantity is 3 : 4 : 9 respectively. If 10
litres of the mixture is taken out and 3 litres of A, B, C each are filled in the vessels due to which the quantity of
C became twice that of B. Find the initial quantity of mixture?
(1) 48 (2) 58 (3) 40 (4) 50 (5) 68
Q.11. A milkman gives 1.5 lt. milk at first flat and add same amount of water before giving milk at second flat. He follows the
same process with each flat. On the fourth flat, in on purity test it is found that the ratio of milk and water become 343 :
169. If he starts the distribution with pure milk then find the initial quantity of milk.
(1) 12 lit. (2) 15 lit. (3) 18 lit.
(4) 9 lit. (5) Can't be determined
Q.12. The monthly incomes of two persons are in the ratio 2 : 3 and their monthly expenses are in the ratio 5 : 9. If each
of them saves Rs 600 per month then the total of their monthly incomes are -
(1) Rs 4500 (2) Rs 4000 (3) Rs 4800 (4) Rs 6000 (5) None of these
1
Q.13. Two litre of pure milk is added to 6 liter of mixture of milk and water, contains 8 3 3 % of water. Find the ratio
between water and milk in resultant mixture?
(1) 3 : 5 (2) 5 : 3 (3) 2 : 3 (4) 3 : 2 (5) None of these
Q.14. The incomes of Sonu and Prasant are in the ratio 3:2 and their expenditure are in the ratio 5 : 3. If each saves Rs. 7200.
Then Sonu's income is what percent the expenditure of Prashant?
(1) 100% (2) 200% (3) 300% (4) 350% (5) None of these

60 QUANTITATIVE APTITUDE
stportal.mahendras.org
Q.15. Number of students in Arts and Science in an institute are in the ratio of 5:8 respectively. If 150 more students join
Arts faculty while 80 more students join Science faculty, the respective ratio becomes 3:4. Originally what was
the total number of students in both faculties together?
(1) 1200 (2) 1400 (3) 1150
(4) Cann't be determined (5) 1170
Q.16. 729 ml of combination has coffee and water in the ratio 7:2. How much more water to be included to get a new
combination of coffee and water in the ratio 7:3?
(1) 162 (2) 81 (3) 75 (4) 360 (5) 400
Q.17. The length and breadth of a rectangle are raised in the ratio 3:4 and 4:5 corresponding. What is the ratio of the
previous area to the new area?
(1) 5:3 (2) 3:5 (3) 5:8 (4) 8:5 (5) 3 : 1
2
Q.18. Rs. 6800 is divided among three person A, B & C in such a way that A received of total shared B & C together.
Find the share of B. 5

(1) 1800 (2) 2000 (3) 2200 (4) C.N.D. (5) None of these
2 3
Q.19. Rs.1540 is distributed among A,B & C such that A receives as much as B and C together and B receives
9 11
as much A & C together. Find the share of C.
(1) 390 (2) 280 (3) 330 (4) 930 (5) 1020
Q.20. The income of A & B are in the ratio of 5:7 & their expenditure are in the ratio of 3:5. If each saves Rs.1500. Find
the income of B.
(1) 4250 (2) 5250 (3) 3550 (4) 4050 (5) 5000
Q.21. An amount of money is distributed among A, B&C in the ratio 6:19:7. If C gives Rs.400 from his share to B, then
new ratio becomes 3:10:3. Find the total amount of money.
(1) Rs.18600 (2) Rs.1450 (3) Rs.12800 (4) Rs.14800 (5) Rs.15800
Q.22. The ratio of A's and B's income last year was 3:4. The ratio of their own income of last year and this year is 4:5
and 2:3 respectively. If the sum of their present income is Rs.6565. Find the present income of A.
(1) Rs.2525 (2) Rs.5025 (3) Rs.2500 (4) Rs.5000 (5) Rs. 2200
Q.23. Rs. 525 is divided among 4 men, 5 women and 6 boys such that share of a man, a woman and a boy may be in the
ratio of 9:8:4. What is the share of a women?
(1) Rs.36 (2) Rs.38 (3) Rs.40 (4) Rs.42 (5) Rs. 48
Q.24. A vessel contains 60 litres of mixture of milk and water in the ratio 7:3 respectively. 8 litres of mixture is replaced
by 12 litres of milk. What is the ratio of milk and water in the resulting mixture?
(1) 67 : 121 (2) 121 : 39 (3) 164 : 123 (4) 165 : 121 (5) 63 : 121
Q.25. Mr. X invested a certain amount in Debit and Equity funds in the ratio of 4:5 respectively. At the end of the one year,
he earned a total dividend of 30% of his investment, after one year he reinvested the amount including dividend in
the ratio of 6:7 in Debit and Equity funds. If the amount reinvested in Equity funds was Rs.94500. What was the
original amount invested in Equity Funds?
(1) Rs.75000 (2) Rs.81007 (3) Rs.60000 (4) Rs.65007 (5) Rs. 63121
Q.26. Three containers have their volume in the ratio 1:4:7. They are full of mixture of milk and water. The mixture
contains milk and water in the ratio (4:1), (3:1) and 5:2 respectively. The contents of all these three containers are
poured into fourth container. The ratio of milk and water in the fourth container is:
(1) 11:15 (2) 15:11 (3) 14:11 (4) 11:4 (5) 13:15

QUANTITATIVE APTITUDE 61
stportal.mahendras.org

EXERCISE Explanation
5 7 = 46 litres
Q.1.(3) 3:5× :5× = 36 : 25 : 35
12 12 Difference of milk and water in final solution =
170 – 46 = 124 litres
Q.2.(3) b : (a+c) = (4:9) × 10
This satisfies the given condition.
c : (a+b) = (3:7) × 13 Q.9.(5) 5 = 15 liter
a b c 1 = 3 liter
51 : 40 : 39 M : W = 15 : (3+6) = 15 : 9 = 5 : 3
Hence it is not followed
51 : 79 option :- B
Q.3.(2) Amount received by Q 5 = 12 liter
1 = 2.4 liter
Its related ratio term
= × T otal am ount M : W = 12 : (2.4+7) = 12 : 9.4 = 60 : 47
S um of ratio term s Hence it is not followed.
2 option :- C
= × 3 6 0 0 = Rs. 400 5 = 30 liter
7 +2 +9
1 = 6 liter
Q.4.(3) Let no. x is substracted from each number. Hence,
M : W = 30 : (6+14) = 3 : 2
14 − x 34 − x
= Hence it is followed
17 − x 42 − x option :- D
x=2
5 = 30 liter
Q.5.(1) Let x boys & x girls each are added in each group. 1 = 6 liter
According to question, M : W = 30 : (6+21) = 30 : 27 = 10 : 9
Hence it is not followed
20 + x 7
=
25 + x 8 Q.10.(2) Suppose, after 10 litre of the mixture is taken out

⇒ 160+8x = 175 + 7x Quantity of A = 3x, Quantity of B = 4x


⇒ x = 15 Quantity of C = 9x
Hence, total no. of person According to question, 9x+3=2(4x+3), 9x+3
= (20+x)+(25+x) = 8x+6, x = 3
= 20 + 15 + 25 + 15 Mixture in the initial stage
= 75 = 3x+4x+9x+10 = 16×3+10= 58 litre.
Q.6.(2)  1 .5 
3

x + 25.5 Q.11.(1) x 1 –
3  x  343 1 .5 343 7
= = 1– = 3 =
x + 25.5 + 35 5 x 343 +169 x 512 8
x = 27 x = 8×1.5=12 lit.
Q.7.(4) Let the diamond breaks into the ratio of 1k and Q.12.(2) Let the expenditure of first person and second
3k person is Rs. 5x, and Rs. 9x respectively.
Initially total weight of diamond = total weight
of broken diamond=4k 5 x + 600 2
According to equation, =
According to the question, 9 x + 600 3
16k2 = 40000, k = 50 15x + 1800 = 18x + 1200, 3x = 600, x
The price of the broken diamond = 1k2+9k2 =
10k2 = Rs 25000 = 200
So loss percentage First person's monthly income = Rs 1600
40000 − 25000 Second person's monthly income= Rs 2400
× 100 = 37.5%
40000
Total income = Rs. (1600 + 2400)= Rs. 4000
Q.8.(3) Milk and water ratio initially = 200: 40 = 5: 1
36 litres of mixture is drawn and 12 litres of 5 1
Q.13.(2) 6 × :6 × +2 = 5 :3
water added 6 6
According to the question, Milk in the final Q.14.(2) According to question
solution = 200 – (36 × 5/6) = 170 litres 3x- 5y = 7200 ... (i)
Water in the final solution = 40 – 36×1/6 + 12
62 QUANTITATIVE APTITUDE
stportal.mahendras.org
2x -3y = 7200 ....(ii) A : (B+C) = 2 : 9 ..........(1)
On solving (i) and (ii) we get 3
and B = (A+C)
x = 14400, y = 7200 11
Sonu's income = 3 ×14400 = 43200 B : A + C = 3 : 11 ........(2)
Prashant's expenditure = 3 × 7200 Now from eq.(1)
= 21600 1540
share of A = × 2 = 280
11
43200 1540
Req. % = ×100 = 200 Share of B= × 3 = 330
21600 14
Q.15.(5) According to question, Hence,
5 x + 150 3 Share of C= 1540 - (280 + 330)= Rs.930
=
8 x + 80 4 Q.20.(2) IA : IB = 5 : 7
= 24x + 240 = 20x + 600 EA : EB = 3 : 5
4x = 360 Since saving = Income - Exp.
x = 90 and each saves Rs.1500
Original number of students = 5x + 8x = 13x Hence change in ratio of both must be save.
= 13 × 90 = 1170 1500
So, IB = × 7 = 5250
Q.16.(2) Method-I. 2
729 Q.21.(3)
0 0 0
Coffee = ×7=567 ml A : B : C = (6 : 19 : 7) × 1 = 6:19:7
9
0 +4 0 0 −4 0 0
729 A : B : C = (3 : 10 : 3) × 2
Water = ×2 = 162 ml
9 +1 −1
Let x lt water be added then, = 6 : 20 : 6
567 7 Since C gives Rs. 400 to B
=
162 + x 3 Hence total amount of

x = 81 ml 400
A+B+C = × 32
1
Method-II (Short Trick): = Rs. 12800
C:W=7:2 Q.22.(1) −1 −1
..............(I)
A :B = 3 :4
C:W=7:3
−1 0
729 A : A = 4 : 5 ..............(II)
Added Water = 9 ×1 = 81 ml
−1 0
Q.17.(2) According to question B : B = 2 : 3 ..............(III)
Real length = 3a From eq. (I) & (II)
Real breadth = 4a 0 3
A = ×5
Real Area = 3a × 4a = 12a 2
4
Now, 0 15
Increased length = 4a A =
4
Increased breath = 5a
from eq. (I) & (III)
New Area = 4a × 5a = 20a2 0 4
Required Ratio = 12a : 20a
2 2 B = ×3
2
=3:5 0

Q.18.(4) CND (because ratio b/w B&C is not known). B =6


Q.19.(4) According to question, So,
2 0 0 15
A= (B+C) A :B = : 6 = 15 : 24 = 5 : 8
9 4
QUANTITATIVE APTITUDE 63
stportal.mahendras.org
Hence, Dividend at the end of the year
6565 30 27 x
Income of A = × 5 = 2525
13 9x × =
100 100
Q.23.(4) Ratio of 4 men, 5 women & 6 boys Total investment after 1 year
= 4 × 9 : 5 × 8 : 6 × 4= 9 : 10 : 6
27 x 117 x
= 9x + =
525 10 10
Share of 5 women = 2 5 × 10 = 210 According to question, ratio between debit and
equity is 6:7.
210
Share of a woman = = 42 Rs.
5 7 117 x
∴ × = 94500
Q.24.(2) Since 8 litres mixture is drawn. Hence remaining 13 10
mixture = 60 - 8 = 52 94500 ×13 ×10
x=
Now, 7 ×117
= 15000
52
Milk in remaining mixture= × 7 = 36.4 Hence, invested amount in equity fund=5x
10
Water in remaining mixture = 5×15000 = Rs.75000

52 Q.26.(4) According to question:


= × 3 = 15.6
10 4 3 5
1× +4 × +7 ×
New ratio of milk & water when 12 ltrs more milk M ilk 5 4 7
=
is added. W a te r 1 1 2
1× + 4 × +7 ×
5 4 7
= (36.4 + 12) : 15.6 4
+3 +5
= 48.4 : 15.6= 121 : 39 5 44
⇒ = = 11 : 4
1 16
Q.25.(1) Let the original amount invested in Debit and +1+ 2
5
Equity funds are 4x & 5x respectively.

64 QUANTITATIVE APTITUDE
stportal.mahendras.org

CHAPTER
PROBLEMs
8 ON AGES
SHORT Trick :
Introduction
A0 . B0 = 3 : 5
Problems based on ages are generally asked in
most of the competitive examinations. To solve (A+B)0 = 48 years
these problems, the knowledge of linear equations A-5 = ?
is essential. In such problems, there may be three
situations: B-5 = ?

(i) Age some years ago 48


A0 = 8 × 3 = 18 years
(ii) Present age
A-5 = 18 - 5 = 13 years
(iii) Age some years hence
48
Some Important Solved examples : B0 = × 5 = 30
8
Ex. The ratio of present age of A & B is 2 : 3. The present B-5 = 30 - 5 = 25 years
age of A is 20 years. Find the age of B after 5 years.
Ex. The ratio of ages of A & B before 5 years was 2 : 3. If
Sol. Let the age of A is 2x years and age of B is 3x years. the sum of ages of A & B at present is 45 years. Find
2x = 20 the present ages of A & B .
x = 10 Sol. Let the ages of A & B 5 years before are 2x years and
3x years respectively.
then present age of B is
then, (2x + 5) + (3x + 5) = 45 years
3 × x = 3×10 = 30 years
5x + 10 = 45
After 5 years, Age of B = 30 + 5
5x = 35
= 35 years
x=7
SHORT Trick :
The age of A before 5 years = 7 × 2 = 14 years
A0 : B0 = 2 : 3, A0 = 20 year At present age of A = 14+5 = 19 years
20 The age of B before 5 years= 7 × 3 = 21 years
B0 = × 3 = 30 years
2
At present age of B = 21 + 5 = 26 years
B+5 = 30 +5 = 35 years
short trick :
Ex. The present ratio of age of A & B is 3 : 5. If the sum -5 -5
A : B =2:3
of present age of A & B is 48 years. Find the ages of
A & B before 5 years. (A+B) = 45 years
Sol. Let the ages of A & B are 3x years and 5x years (A+B)-5 = 45 - 10 = 35 years
respectively. 35
3x + 5x = 48 A-5 = 5 × 2 = 14 years
8x = 48
A0 = 14 + 5 = 19 years
x = 6 years
35
Then the present age of A is B-5 = 5 × 3 = 21 years
= 3 × x = 3 × 6 = 18 years B0 = 21 + 5 = 26 years
Before 5 years the age of A Ex. The ratio of ages of A & B after 5 years will be 3 :
= 18 - 5 = 13 years 5. If the sum of ages of A & B at present is 38 years.
The present age of B is Find the ages of A & B before 5 years.
= 5 × x = 5 × 6 = 30 years Sol. Let the ages of A & B are 3x years and 5x years
respectively after 5 years.
Before 5 years the age of B.= 30 - 5 = 25 years
QUANTITATIVE APTITUDE 65
stportal.mahendras.org
then, Solving eq. (1) & (2)
(3x - 5) + (5x - 5) = 38 years x = 30 years and y = 36 years
8x - 10 = 38 years SHORT Trick :
8x = 48 years, x = 6 years A0 : B0 = 5 : 6
The age of A after 5 years A-6 : B-6 = 4 : 5
= 6 × 3= 18 years
6
Then the age of B after 5 years A0 = × 5 = 30 years
1
= 6 × 5 = 30 years
6
The age of A before 5 years B0 = × 6 = 36 years
= 18 - 10 = 8 years 1
Ex. The ratio of age of A & B at present is 6 : 5 & the sum
The age of B before 5 years
of their ages is 54 years after 5 years. What will be the
= 30 - 10 = 20 years ratio of their ages after 8 years ?
SHORT Trick : Sol. A0 : B0 = 6 : 5
A+5 - B+5 = 3 : 5 (A+B)+5 = 54
(A+B)0 = 38 years A+8 : B+8 = ?
(A+B)+5 = 38 + 10 = 48 years (A+B)0 = 44
48 44
A+5 = × 3 = 18 years × 6 = 24
8 A0 =
11
A-5 = 18 - 10 = 8 years
44
A+8 = 32, B0 = × 5 = 20
48 11
B+5 = × 5 = 30 years B = 28
+8
8
A+8 : B+8 = 32 : 28 = 8 : 7
B-5 = 30 -10 = 20 years
Ex. The present ratio of ages of A & B is 6 : 7. After 5 Ex. If Sita’s mother was 4 times as old as Sita 10 years
years this ratio will be changed into 7 : 8. Find the ago. After ten years mother will be twice as old as
present age of A & B. Sita. How old is mother & Sita at present?
Sol. Let the present age of A is x years & B is y years. Sol. Sm–10 : S–10 = [4 : 1]×1 = 4 : 1
Sm+10 : S+10 = [2 : 1]×3 = 6 : 3
x 6 ............(1)
=
y 7 20
Sm+10 = × 6 = 60
2
x+5 7 Sm0 = 50 years
= ...........( 2 )
y +5 8
20
Solving eq. (1) & (2) S+10 = × 3 = 30
2
We get, x = 30 years S0 = 20 years
y = 35 years Ex. Jayesh is as more younger to Amit as Jayesh is older
SHORT Trick : to Prashant. If the sum of the ages of Amit & Prashant
is 48 years. What is the age of Jayesh in years ?
A0 : B0 = 6 : 7, A+5 : B+5 = 7 : 8
Sol. Let the age of Jayesh = x years
5
A0 = 1 ×6 = 30 years Amit = y years
5 Prashant = z years
B0 = × 7 = 35 years y–x = x–z
1
Ex. The ratio of ages of A & B is at present 5 : 6. 6 years 2x = y+z = 48
before this ratio was 4 : 5. Find the present age of A age of Jayesh (x) = 24 years
& B.
Ex. Neeraj is as younger to Gopal as he is older to Deepak.
Sol. Let present age of A is x and B is y years. If the sum of the ages of Gopal & Deepak is 58 years.
x 5 What is Neeraj’s age ?
Then, y = 6 ....................(1)
Sol. G – N = N–D, G+D = 2N
x −6 4 58 = 2N
Again, = ................(2)
y −6 5 N = 29 years
66 QUANTITATIVE APTITUDE
stportal.mahendras.org

EXERCISE
1
Q.1. A man's age is 125% of what it was 10 years ago, but 83 % of what it will be after 10 years. After how many
3
years his age will become 64 years?
(1) 12 years (2) 16 years (3) 15 years (4) 14 years (5) 10 years
Q.2. The ratio of the ages of the husband and the wife five years ago was 11: 9 whereas at the same time the ratio of
the ages of the husband and the son was 5: 1. Five years hence the ratio of the ages of the husband and the wife
will become 13 : 11. What is the sum of the present ages of all the three persons of the family?
(1) 128 years (2) 127 years (3) 126 years (4) Data inadequate (5) None of these
Q.3. The ratio of sum and the difference of the present age of Vivek and Arvind are 2 : 1. Four years ago, this ratio was
3 : 2. What will be the ratio of their ages after 12 years.
(1) 9 : 5 (2) 5 : 9 (3) 8 : 3 (4) 4. 5 (5) None of these
Q.4. The ratio of ages of Ravi 5 years ago and that of Rohan's age after 5 years is 2:3. The ratio of their present age is
3:4. What is the age of Ravi before 3 years and that of Rohan's age after 7 years?
(1) 39 : 49 (2) 75 : 10 (3) 72 : 107 (4) 3 : 5 (5) None of these
1
Q.5. A man's is 125% of what it was 10 years ago, but 83 % of what it will be after 10 years. What is his present age?
3
(1) 40 years (2) 50 years (3) 60 years (4) 30 years (5) 45 years
Q.6. The ratio of present age of A and B is 11:13 and sum of their ages is 72 years. Then find the present age of C who's
age is twice that of A?
(1) 56 years (2) 12 years (3) 78 years (4) 66 years (5) None of these
Q.7. The ratio of present age of Manoj and Waseem is 3 : 11. Waseem is 12 years younger than Rehana 7 years hence
Rehana will be 85 years old. What will be the present age of Manoj's father who is 25 years older than Manoj?
(1) 42 years (2) 43 years (3) 44 years (4) 45 years (5) 46 years
Q.8. Rohan's present age is 3 years less than twice his age 12 years ago. Also the ratio of present ages of Mohan and
Rohan is 4:9. What will be the Mohan's present age.
(1) 4 years (2) 12 years (3) 8 years
(4) 16 years (5) Cannot be determined
Q.9. Ratio of present ages of A and B is 2:3. B's age after 16 years will be twice the age of A after 4 years. Find the
difference between present ages of A and B?
(1) 7 years (2) 9 years (3) 11 years (4) 2 years (5) 8 years
Q.10. The average age of Ram and his two sons is 17 years. While the average age of Ram's wife and his two son's is 16
years. If the age of Ram is 33 years, find the age of Ram's wife.
(1) 31 years (2) 32 years (3) 35 years (4) 30 years (5) None of these
Q.11. The average age of two boys and their fathers exceed by 3 years than the average age of those two boys and their
1
mother. The average of those 4 persons is 19 years. If the average of these two boys is 5 2 years, then find the age
of father and mother?
(1) 37 years & 28 years (2) 47 years & 38 years (3) 50 years & 41 years
(4) 35 years & 32 years (5) None of these
Q.12. If 6 substracted from the present age of A and the remainder is divided by 18, the resultant is the age of B. C's
age is 5 years and B is 2 years younger than C. What is the present age of A?
(1) 60 years (2) 48 years (3) 84 years
(4) 64 years (5) None of these
Q.13. The average age of a family of five members is 24. If the present age of the youngest member is 8 years, what was
the average age of the family at the time of the birth of the youngest member?
(1) 20 years (2) 16 years (3) 12 years (4) 18 years (5) 21 years
QUANTITATIVE APTITUDE 67
stportal.mahendras.org

2
Q.14. Yashika got married 8 years ago Today her age is 1 times her age at the time of her marriage At present her
7
daughters age is one-sixth of her age What was her daughter’s age 3 years ago?
(1) 2 years (2) 3 years (3) 4 years (4) 5 years (5) None of these
Q.15. The ratio of the present ages of a son and his father is 1 : 5 and that of his mother and father is 4:5. After 2 years
the ratio of the age of the son to that of his mother becomes 3:10. What is the present age of the father?
(1) 30 years (2) 28 years (3) 37 years (4) Data inadequate (5) None of these
Q.16. The respective ratio between the present age of Manisha and Deepali is 5 : X. Manisha is 9 years younger than
Parineeta. Parineeta's age after 9 years will be 33 years. The difference between Deepali's and Manisha's age is
same as the present age of Parineeta. What will come in place of X?
(1) 23 (2) 39 (3) 15
(4) Cannot be determined (5) None of these
5
Q.17. Sush was thrice as old as Poonam 6 years back. Sush will be times as old as Poonam 6 years hence, How old
is Poonam today? 3
(1) 20 (2) 14 (3) 12 (4) 15 (5) None of these
Q.18. The sum of the ages of a father and his son is 4 times the age of the son. If the average age of the father and the
son is 28 years, what is the son’s age?
(1) 14 years (2) 16 years (3) 12 years (4) Data inadequate (5) None of these
Q.19. Radha's present age is three years less than twice her age 12 years ago. Also, the ratio of Raj's present age to
Radha’s present age is 4 : 9. What will be Raj’s age after 5 years?
(1) 12 yrs (2) 7 yrs (3) 21 yrs
(4) Can’t be determined (5) None of these
Q.20. Neeraj's age is 1/5 of his father's age. Neeraj's father's age will be twice Vinod’s age after 10 years, if Vinod’s
eighth birthday was celebrated two years ago, then what is Neeraj’s present age ?
(1) 14 years (2) 6 years (3) 30 years
(4) 10 years (5) None of these
Q.21. After 5 yrs, the age of a father will be thrice the age of his son, whereas five years ago. he was 7 times as old as his
son was. What are their present ages?
(1) 30 yrs (2) 40 yrs (3) 50 yrs
(4) 60 yrs (5) None of these
Q.22. In a family, a couple has a son and daughter. The age of the father is three times that of his daughter and the age
of the son is half of his mother. The wife is nine years younger to her husband and the brother is seven years older
than his sister. What is the age of the mother?
(1) 40 years (2) 45 years (3) 50 years
(4) 60 years (5) None of these
Q.23. Abhay’s age after six years will be three-seventh of his father’s age. Ten years ago, the ratio of their ages was 1 :
5. What is Abhay’s father’s age at present?
(1) 30yrs. (2) 40yrs. (3) 50yrs. (4) 60yrs. (5) None of these
Q.24. Tanya’s grandfather was 8 times older to her 16 years ago. He would be 3 times of her age 8 years from now. Eight
years ago, what was the ratio of Tanya’s age to that of her grandfather?
(1) 1:2 (2) 1:5 (3) 3:8 (4) 11:53 (5) None of these
Q.25. Eighteen years ago, a father was three times as old as his son. Now the father is only twice as old as his son. Then
the sum of the present ages of the son and the father is:
(1) 54 years (2) 72 years (3) 105 years (4) 108 years (5) None of these

68 QUANTITATIVE APTITUDE
stportal.mahendras.org

EXERCISE Explanation
Q.1.(4) Let his present age be x years Q.5.(2) Age 10 years before = 100%
According to the question, Present Age= 125%
x = 125% of (x – 10) 25% ................... 10 years
10
4x = 5x – 50 125% .............. × 1 2 5 = 50 years
25
x = 50 years OR
Required number of years = 64 – 50 = 14 years Age 10 years after = 100%
Q.2.(3) Five years ago, 1
Present Age = 83 %
H : W = 11 : 9 3
2
and H : S = 5 : 1 16 % ................ 10 years
3
Hence, H : W : S = 55 : 45 : 11 1 10 × 3 250
83 % ........ × = 50 years
55k + 10 13 3 50 3
= Q.6.(4) A0 : B0 = 11 : 13
45k + 10 11
Sum of A + B = 72 years
Five years hence,
24 parts ....... 72
k=1
72
Required value = 55 + 45 + 11 + 15 = 126 years 11 parts ....... 2 4 ×11 = 33 years
Q.3.(1) Let the present age of Vivek and Arvind be x & Present age of A = 33 years
y. According to question Present age of C = 2×33 = 66 years
x+y 2 Q.7.(2) Ratio of Age of Manoj : Waseem = 3 : 11
x − y = 1 , x = 3y.......(I) Waseem present age = 78-12 = 66 years
4 years ago their ages will be (x - 4) & (y- 4)
66
Again according to question Age of Manoj = ×3
11
(x − 4 ) + ( y − 4 ) 3 Present age of Manoj's father = 18+25= 43 years
= (x − 4 ) − (y − 4 ) = 2
Q.8.(2) Let Rohan's is present age be = x years
or 2x + 2y - 16 = 3x - 3y
According to question:
5y - x = 16 ...............(II)
x = 2 (x - 12) - 3
On solving eq. (i) & (ii)
x = 2x - 24 - 3
x = 24
x = 27
y = 8, Age of Vivek after 12 years
Rohan's present age = 27 years
= 24+12 = 36 years
4
Age of Arvind after 12 years= 8+12 = 20 years Mohan's present age = 27× = 12 years
9
Required Ratio = 36 : 20 = 9 : 5 Q.9.(5) Let the present ages of A and B be 2x & 3x.
Q.4.(3) Let the present age of Ravi and Rohan be 3x and A's age after 4 years = (2x + 4)
4x respectively. B's age after 16 years = (3x + 16)
According to question: (3x + 16) = 2 (2x + 4)
3x − 5 2 3x + 16 = 4x + 8
=
4x + 5 3
4x - 3x = 8
9x - 15 = 8x + 10 x = 8 years
x = 25 Required Difference = 3x - 2x = 3×8-2×8
Present age of Ravi = 3x = 3×25 = 75 = 24 - 16 = 8 years
Present age of Rohan = 4x = 4×25 = 100 Q.10.(4) Total age of Ram and his son = 17×3
Ravi age before 3 years = 72 = 51 years ........ (i)
Rohan age after 7 years 107 Total age of Ram's wife and his son
Required ratio = 72 : 107 = 16×3 = 48 years ........ (ii)
QUANTITATIVE APTITUDE 69
stportal.mahendras.org
So (i) – (ii) 5
(Ram + son)-(Ram's wife + son) = 3 years 5x = z
4
33 - Ram's wife = 3 years
4x = z................(3)
Ram's wife = 30 years
Q.11.(1) Let the ages of boys be P and Q and that of mother
x+2 3
After 2 years, =
and son be x and y respectively. z + 2 10
According to question:
Y +P +Q X +P +Q 10x + 14
− = 3 .........(I) 10x + 20 = 3z +6 ⇒ = z.......(4 )
3 3 3
X + Y +P +Q
Also = 1 9 .......... (II) from 3 & 4
4
P + Q 11 10 x + 14
Also = 4x = ⇒ x=7
2 2 .................. (III) 3
On solving eq. (II) and (III)
Putting x = 7
x + y = 65 ....................... (IV)
y = 35
On solving eq. (I) and (III)
Q.16.(5) Let age of Manisha = M, Deepali = D, Parineeta
y - x = 9 ........................ (V)
=P
On solving equation (IV) & (V) M 5
= ........(1 )
x = 28, y = 37 D x
Q.12.(1) According to question
M = P - 9.......(2)
A −6
=B P + 9 = 33
18
P = 24
A = 18 B + 6 .................... (I)
Putting P = 24 in eq. (2)
B = C - 2 = 5 - 2 = 3 years.......(II)
M = 24 - 9 = 15
Putting value of B in eq. .... (I)
D - M = P ........(3)
A = 18×3 + 6= 54 + 6 = 60 years
D - 15 = 24
Q.13.(1) Total age of 5 person = 24 × 5 = 120 years
D = 39
Before 8 year 5 × 8 = 40 years
Putting M = 15 and D = 39 in eq. (1)
120 - 40 = 80 years
15 5
Now the family member is 4 = 80/4= 20 years =
39 x
Q.14.(2) Yashika's age today = x
5 × 39
Yashika's age of marriage = x-8 ⇒x= = 13 ⇒ x = 13
15
9
x= ( x − 9) Q.17.(3) Let Present age of Sush= S and Poonam = P
7
(S-6) = 3 (P-6)
7x = 9x - 72 Þx = 36 years
S = 3P - 12.......(1)
1
Daughter's present age = 3 6 × = 6 5
6 (S + 6) = (P + 6 )
3 years age = 6 - 3 = 3 years 3
Q.15.(5) Let present age of son = x, father y, mother = z 35 + 18 = 5P + 30.........(2)
x:y=1:5 From (1) and (2)
5x = y.........(1) 3 (3P-12) + 18 = 5P + 30
z 4 9P - 36 + 18 = 5P + 30
z:y=4:5= =
y 5 P = 12 years
5z = 4y ..........(2) Q.18.(1) Let present age of father = F and Son = S
F + S = 4S.......(1)
70 QUANTITATIVE APTITUDE
stportal.mahendras.org

F+S From (A) and (B)


= 28 M 
2 3  - 7 = M + 9
2 
3M
F + S = 56..........(2) - 2 1 = M + 9 , M = 60 years
2
4S = 56
Q.23.(3) 3
S = 14 years ( A + 6) = ( F + 6) .....(1)
Q.19.(5) Let present age of Radha = x and Roy = y 7
1
x = z (x-12) - 3 ( A − 10) = ( F − 10) .......(2)
x = 27 years.......(1)
5

y 4 From (1) and (2)


= .......(2 ) 3F 18 F
x 9 + -6 = - 2 + 10
7 7 5
From (1) and (2)
y 4 3F F 18 8F
= ⇒ - = 14 - ⇒
27 9 7 5 7 35
y = 12 years F = 50 years
y + 5 = 12 + 5 = 17 years Q.24.(4) (G - 16) = 8 (T-16)
Q.20.(2) Let present age of Neeraj = N, Father = F,. Vinod G = 8T - 112.......(1)
=V (G+8) = 3 (T+B)
F G = 3T + 16.......(2)
N= ⇒ 5 N = F ..........(1 )
5 From 1 and (2)
8T - 112 = 3T + 16
(F+10) = 2(V+10) ÞF = 2V +10..........(2)
5T = 128
(V-2) = 8 Þ V = 10
128
Putting V = 10 in eq.....(2) T= .....(A )
5
F = 2 × 10 + 10 = 30 3 × 128
G= + 16
Putting F = 30 in eq. (1) 5

5N = 30 384 + 80 464
N = 6 years G= = ......(B )
5 5
Q.21.(2) (F+5) = 3 (S+5)
F = 3S + 10.......(1) T -8
=?
(F-5) = 7 (S-5) G -8
F = 75 - 30.......(2)
From (A) & (B)
From (1) and (2)
128
S = 10 Years, F = 40 years -8
T -8 5 128 - 40
Q.22.(4) F = 3D.......(1) = =
G - 8 464 464 - 40
M -8
S = ......(2 ) 5
2
T -8 11
=
F = M + 9..........(3) G -8 53
S = D + 7.......(4)
Q.25.(4) (F-18) = 3 (S-18), F = 3S - 36.........(1)
M=?
F = 25.............(2)
From (1) and (3)
2S = 3S - 36, S = 36 Putting S = 36 in (2)
3D = M + 9.........(A)
F = 72
From (2) and (4)
F+S = 36 + 72= 108 year
D + 7 = M/2.........(B)

QUANTITATIVE APTITUDE 71
stportal.mahendras.org

CHAPTER

9 PARTNERSHIP

Ex. A,B and C start a business each investing Rs.20,000.


Partnership
After 5 months A withdraws Rs.5000, B withdraws
When two or more than two persons run a business then they Rs.4000 and C invested Rs.6000 more. At the end of
are called partners and when they agree to share profit and the year, a total profit of Rs.69900 was recorded. Find
losses, the deal is known as Partnership. the share of each.
Elements of Partnership:
Sol. Ratio of the capital of A, B and C
Capital: The sum of money invested by the partners to start
any business is known as capital. A : B : C

Equivalent Capital: When the capital invested by the (20000×5+15000×7):(20000×5+16000×7):(20000×5+26000×7)


partners is multiplied by the time of investment, the product 205000 : 212000 : 282000
thus obtained is known as Equivalent Capital. 205 : 212 : 282
Categorisation of Partnership:
 69900 
Case 1 : When the investment and the time of investment is A’s share= Rs.  2 0 5 + 2 1 2 + 2 8 2 × 2 0 5  = Rs. 20500
equal, the profit and loss is distributed equally.  
Ex. A, B and C are three partners. Each invested Rs.20,000  69900 
in a business. After a year the profit of the business B’s share=Rs.  × 2 1 2  = Rs. 21200
205 + 212 + 282 
was 90,000. Then find the share of each in the profit.
Sol. Ratio of shares of A, B and C = Ratio of their  69900 
investment C’s share = Rs.  × 2 8 2  = Rs. 28200
205 + 212 + 282 
A : B : C
20000×12 : 20000×12 : 20000×12 Case 4: When capital and time are given in form of ratio.
1 : 1 : 1
Ex. A, B and C become partners in a business. A contributes
Now Profit share of A
1 1 1
rd of the capital for th of the time B contributes th
 90000  90000 ⇒ 3 4 5
=  1 + 1 + 1 × 1 ⇒ 30000/- 1
  3 of the capital for th of the time and C the rest of the
6
Similarly B and C will also get = 30000/-each. capital for the whole time. If the profit is Rs. 1820,
Case 2: When the investment is different but the time of then the A's share in profit is :
investment is same. Then the profit and loss is shared
in the ratio of their capital. Sol. 1 1 1 1 7 , 1 1 7
× : × : ×1 : :
Ex. P, Q and R started a business each investing Rs. 20,000, 3 4 5 6 15 12 30 15
30,000, 50,000 respectively. After 1 year the profit 5 : 2 : 28
recorded by the business was 1,50,000. Then find the
share of each in the profit? 1820
Sahre of A = × 5 = 260
Sol. A : B : C 35
1 1 7
20000×12 : 30000×12 : 50000×12 Ex. A, B and C share profit in the ratio of : : if C
4 6 12
2 : 3 : 5 retires, they share the profit of C in the ratio of 4 : 5
150000
P's share in profit = × 2 =30000/ respectively.
2 +3 +5
Q's share in profit = 45000/- The new profit shareing ratio of A and B will be :
R's share in profit = 75000/- 1 1 1
Sol. A : B : C = : : =3:2:1
Case 3 : When the investment in a business are different 4 6 12
then the profit & loss is shared in the ratio of their 1× 4 1 3 1 2 3 = 31 : 23
A:B:C= 3 + : 2 + ×5 = :
equivalent capital. 9 9 9 9
72 QUANTITATIVE APTITUDE
stportal.mahendras.org

EXERCISE
Q.1. Three men A, B and C start a business together. They invest Rs. 30000, Rs. 24000, and Rs.42000 respectively in
the beginning. After 4 months, B took out Rs.6000 and C took out Rs.10000. They get a profit of Rs.11960 at the
end of the year. B's share in the profit is-(approximate value)
(1) Rs. 2700 (2) Rs. 2600 (3) Rs. 2800 (4) Rs. 2500 (5) Rs. 2900

Q.2. A starts a business with an investment of Rs.16000. After 3 months B joins the partnership with an investment equal
5
to 8 th of A's investment and A withdraws Rs.2000 from the business. After 3 months more C joins the partnership
with an investment of Rs.9000. In the end of the year, C gets Rs.1026 as his share in profit. Find A's share in the

profit.
(1) Rs. 3300 (2) Rs. 3306 (3) Rs. 3406 (4) Rs. 3506 (5) None of these
Q.3. A,B and C started a business with an investment in the ratio 5:6:8 respectively. After one year C withdraw 50%
of his capital and A increased his capital by 60% of his investment. After two years in what ratio should the earned
profit be distributed among A,B and C respectively?
(1) 2 : 3 : 3 (2) 4 : 3 : 2 (3) 13 : 12 : 12
(4) Cannot be determined (5) None of these
Q.4. Prakash, Sunil and Anil started a business jointly investing Rs.11 lakh, Rs.16.5 lakh and Rs.8.25 lakh respectively.
The profit at the end of 3 years was Rs.19.5 lakh. What will be the 50% of Anil's share in the profit?
(1) Rs.4.50 Lakh (2) Rs.2.25 Lakh (3) Rs.2.50 Lakh (4) Rs.3.75 Lakh (5) None of these
Q.5. Sonu invested 10% more than Mona. Mona invested 10% less than Raghu. If the total sum of their investment is
Rs.5780. How much amount did Raghu invested?
(1) Rs.2010 (2) Rs.2000 (3) Rs.2100 (4) Rs.2210 (5) None of these
Q.6. Gita invests Rs.48000 to start a business. Four months later Shreya joins her by investing Rs.62000 and another
two months later Deepika joins then both by investing Rs.80000. At the end of one year the business earn a profit
of Rs.20661. What is Deepika's share in the profit?
(1) Rs.7668 (2) Rs.6603 (3) Rs.7240 (4) Rs.6390 (5) None of these
Q.7. Three friends Naimish, Ranveer and Arjun together started a business by investing Rs.250, Rs. 400 and Rs. 300
respetively. After 4 months Naimish invested Rs. 150 more and after 6 months Ranveer withdraw Rs. 100. While
after 8 months Arjun invested Rs. 200 more . At the end of the year, a profit of Rs. 1280 is obtained. Find the share
of Ranveer?
(1) Rs. 360 (2) Rs. 380 (3) Rs. 395 (4) Rs. 420 (5) Rs. 440
Q.8. Mr. Shiv Kumar started a business investing Rs.25000 in 1996. In 1997 he invested an additional amount of Rs.10000
and Mr. Rakesh joined him with an amount of Rs.35000. In 1998, Mr. Shiv Kumar invested another additional
amount of Rs.10000 and Mr. Suresh joined them with amount of Rs.35000. What will be Rakesh's share in the
profit of Rs.150000 earned at the end of three years from the start of the business in 1996?
(1) Rs.70000 (2) Rs.50000 (3) Rs.45000 (4) Rs.75000 (5) None of these
Q.9. Mrs. Raksha invested an amount of Rs.60,000 to start a software business. After six months, Kamal joined here
with an amount of Rs.90,000. After one year from the commencement of business, Raksha put in an additional
amount of Rs.20,000. At the end of three years, they earned a profit of Rs.71,20,000. What is Raksha's share in the
profit?
(1) Rs.35,20,000 (2) Rs.26,40,000 (3) Rs.27,00,000 (4) Rs.3,84,00,000 (5) None of these
Q.10. X, Y and Z are sharing profit in the ratio 4 : 3 : 2. Y retired from the firm and X and Z decide to share profit in the
ratio of 3:2. Calculate the gaining ratio.
(1) 7 : 8 (2) 5 : 9 (3) 4 : 7 (4) 5 : 8 (5) None of these
Q.11. A, B and C started a business with their investment in the ratio 1:3:5. After 4 months. A invested the same amount
as before and B as well as C withdraw half of their investment. The ratio of their profit at the end of the year was:
(1) 5 : 6 : 10 (2) 6 : 5 : 10 (3) 10 : 5 : 6 (4) 4 : 3 : 5 (5) Can't be determined
QUANTITATIVE APTITUDE 73
stportal.mahendras.org
Q.12. A, B and C are partners in a business. A, whose money has been used for 4 months, claims 1 /8 of the profit. B,
whose money has been used for 6 months, claims 1/3 of the profit. C had invested Rs. 1560 for 8 months. How
much money did A and B contribute together?
(1) Rs. 720 (2) Rs. 1560 (3) Rs. 2000 (4) Rs. 1280 (5) None of these
Q.13. A and B started a business by investing Rs. 20000 and Rs. 25000 respectively. After 4 months B left and C joined
by invested Rs. 15000. At the end of the year, there was a profit of Rs. 23000. What is C’s share?
(1) Rs. 8000 (2) Rs. 9000 (3) Rs. 6000 (4) Rs. 12000 (5) None of these
7 4 6
Q.14. A, B and C enter into a partnership in the ratio : : . Alter 4 months, A increase his share 50%. If the profit
3 3 5
earned by C at the end of one year be Rs. 21,600; then find the total profit.
(1) Rs. 102100 (2) Rs. 102400 (3) Rs. 103600 (4) Rs. 104000 (5) None of these
Q.15. Avinash, Manoj and Arun started a business in partnership investing in the ratio of 3:2:5 respectively. At the end
of the year they earned a profit of Rs. 45,000 which is 15% of their total investment. How much did Manoj invest?
(1) Rs. 60,000 (2) Rs. 1,80,00 (3) Rs. 30,000 (4) Rs. 90,000 (5) None of these
Q.16. A and B started a business by investing Rs. 35,000 and Rs. 20,000 respectively. B left the business after 5 months
and C joined the business with a sum of Rs.15,000. The profit earned at the end of the year is Rs. 84,125. What is
B’s share of profit?
(1) Rs.14133 (2) Rs. 15,000 (3) Rs.13,460
(4) Cannot be determined (5) None of these
Q.17. Ninad. Vikas and Manav enter into a partnership. Ninad invests some amount at the beginning. Vikas invests
double the amount after 6 months and Manav invests thrice the amount invested by Ninad after 8 months. They
earn a profit of Rs. 45,000 at the end of the year. What is Manav’s share in the profit?
(1) Rs. 25,000 (2) Rs. 15,000 (3) Rs. 12,000 (4) Rs. 9,000 (5) None of these
Q.18. A invests Rs. 64,000 in a business. After few months B joined him with Rs. 48,000. At the end of year, the total
profit was divided between them in the ratio 2:1. After how many months did B join?
(1) 7 (2) 8 (3) 4 (4) 6 (5) None of these
Q.19. Geeta press makes a profit of 9,00,000, 20% of which is paid as taxes. If the rest is divided among the partners P
1
Q and R in the ratio of 1 : 1 : 2, then shares of P, Q and R are respectively.
2
(1) Rs. 2,40,000, Rs. 3,20,000, Rs. 1,60,000 (2) Rs. 3,20,000, Rs. 2,40,000, Rs. 1,60,000
(3) Rs. 2,60,000, Rs. 3,20,000, Rs. 2,40,000 (4) Rs. 1,60,000, Rs. 2,40,000, Rs. 3,20,000
(5) None of these
Q.20. A, B and C started a business with their investments in the ratio 1 : 2 : 4. After 6 month A invested the half amount
1
more as before and B invested same the amount as before which C withdrew th of the their investments. Find
the ratio of their profits at the end of the year. 4
(1) 5 : 12 : 13 (2) 5 : 11 : 14 (3) 5 : 12 : 14 (4) 5 : 6 : 11 (5) None of these
Q.21. A, B and C started a business by investing Rs. 24,000, Rs. 32000 and Rs. 18000 respectively. A and B are active
partners and get 15% and 12% of total profit and remaining profit is to be distributed among them in the ratio of
their investment. If C got total Rs. 65700 as a profit, what was the total amount of profit ?
(1) Rs. 375000 (2) Rs. 367000 (3) Rs. 350000 (4) Rs. 360000 (5) Rs. 370000
1
Q.22. X, Y and Z enter into partnership. X invest part of total capital for one-fourth of the time. Y contributes one fifth
4
of the capital for half of the time. Z contributes the remaining capital for the whole time. How should they divided
a profit of Rs. 1140 ?
(1) Rs. 100, Rs. 160, Rs. 880 (2) Rs. 110, Rs. 140, Rs. 860
(3) Rs. 120, Rs. 150, Rs. 840 (4) Rs. 140, Rs. 170, Rs. 830
(5) None of these
74 QUANTITATIVE APTITUDE
stportal.mahendras.org

1
Q.23. A, B and C are three partners in a business A, whose money has been used for 4 months, claims of the profit,
1 8
B whose money has been used for 6 months, claims 3 of the profit. C had invested Rs. 1560 for 8 months. C had

invested Rs.1560 for 8 months. How much money did A and B contribute ?
(1) Rs. 740, Rs. 1250 (2) Rs. 730, Rs. 1240 (3) Rs. 720, Rs. 1280
(4) Rs. 750, Rs. 1260, (5) Rs. 750, Rs. 1200
1 1 1
Q.24. A, B and C started a business in partnership and invested in the ratio of : : . After 4 months A withdraw half
4 3 6
1
of his investment and after its 2 months B withdraw of its investment. If the total earned profit, at the end of
3
year is Rs. 14000. Find the share of thier profit.
(1) Rs. 2500, Rs. 2450, Rs.2145 (2) Rs. 3000, Rs. 4500, Rs.2100
(3) Rs. 4000, Rs. 3500, Rs.1254 (4) Rs. 4200, Rs. 5600, Rs.4200
(5) None of these
Q.25. A and B started a business in partnership by investing Rs. 10000 and Rs. 4000 respectively. condition of partnership
is that B got Rs. 100 per month for management of the business. After paying 5% interest on the capital, annual
profit has distributed in the ratio of their investment, Find the share of their profit, if the annual profit is Rs. 4000.
(1) Rs. 3000 each (2) Rs. 2500 each (3) Rs. 1500 each (4) Rs. 2000 each (5) None of these
Q.26. A and B started a business together, A invested 36,000 and B invested 60,000. A is active partner and B is sleeping
partner. A received 10% of the profit for managing the business, and remaining is divided in proportion to their
capitals. If A total received Rs 7000 of profit, then find the share of B in profit.
(1) Rs 10,000 (2) Rs 12,000 (3) Rs 8,000 (4) Rs 9,000 (5) None of these

QUANTITATIVE APTITUDE 75
stportal.mahendras.org

EXERCISE Explanation
Q.1.(5) Ratio of capital ∴ Deepika's share in profit
A : B : C 20661
= × 3 0 = Rs.6390
30000×12 : (24000×4 : (42000×4 36 + 31 + 30
+18000×8) +32000×8) Q.7.(2) Ratio of the profit
45 : 30 : 53 (250×4+400×8) :(400×6+300×6) : (300×8+500×4)
⇒ 21 : 21 : 22
Now,
11960 1280
B's share= × 3 0 = 2917.073 ∴ Ranveer share = × 2 1 = Rs.420
40 + 30 + 53 64
or Rs. 2900 (Approx.) Q.8.(2) Shiv Kumar's equivalent capital
16000 × 5 =(25000×3) + (10000×2) + (10000×1)
Q.2.(2) B's Investment = = 10000
8 = Rs.105000
Now,
Rakesh's equivalent capital =(35000×2)
Ratio of equivalent capital of A, B and C for 1
month. = Rs.70000

=(16000×3+14000×9) : (10000×9) : (9000×6) Suresh's equivalent capital= (35000×1)

=(48000+126000) : (90000) : (54000) = Rs.35000

29 : 15 : 9 Shiv Kumar : Rakesh : Suresh

Where, 3 : 2 : 1

9 ratio = 1026 Hence, Rakesh's share in the total profit

1026 15000
∴ 29 ratio = × 29 = 3 + 2 + 1 × 2 = Rs.50000/-
9
A's share of profit = Rs. 3306 Q.9.(1) Equivalent capital of Raksha
Q.3.(3) For profit sharing ratio =Rs. (60000×1+80000×2) = Rs. 2,20,000
A : B : C Equivalent capital of kamal
(5×12+8×12) : (6×24) : (8×12+4×12) 1  5
= 90,000×2 ⇒ 9 0 0 0 0 ×  = Rs. 225000
(60+96) (144) : (96+48) 2  2

156x 144x 144x Raksha : Kamal

= 13 : 12 : 12 44 : 45

Q.4.(2) Profit sharing ratio = Now, Raksha's share

Prakash Sunil Anil 7120000


= × 4 4 = Rs.3520000/-
44 + 45
=[1100000×3] : [1650000×3] : [825000×3]
3 4  2 2 
= 4 : 6 : 3 Q.10.(1) =  −  :  − 
5 9  5 9 
1950000
Anil's share = × 3 ⇒ 4,50,000/-
4 +6 +3  2 7 − 2 0  1 8 − 1 0 
=  : = 7 : 8
Hence, 50% of Anil's share = 2.25 lakh  45   45 
Q.5.(2) Capital Ratio : Q.11.(1) Let the initial investment of A,B and C be x, 3x,
5x
Sonu Mona Raghu
Now,
99 : 90 : 100
A B C
5780
Raghu's share in investment= 9 9 + 9 0 + 1 0 0 × 1 0 0 3 5
(x×4+2x×8) :(3x×4+ x×8): (5x×4+ x×8)
= Rs. 2000 2 2
Q.6.(4) Ratio of the profit of Gita, Shreya and Deepika = 20x : 24x : 40x
= 48000 × 12 : 62000 × 8 : 80000 × 6 = 5 : 6 : 10
76 QUANTITATIVE APTITUDE
stportal.mahendras.org
Q.12.(3) Ratio of their profit

1 1   1 1  Q.17.(2) x × 12 : 2x × 6 : 3x × 4
= : : 1 −  +   1:1:1
8 3   8 3 
45000
1 1 13 Share of Manav = × 1 = R s.1 5 0 0 0
= : : 3
8 3 24 Q.18.(3) 64000 × 12 : 48000 × x = 2 : 1
= 3 : 8 :13
16 2
Now, for A & C =
x 1
A × 4 : C × 8 = 3 : 13
x = 8 month
A × 4 : 1560 × 8 = 3 : 13
After = 12 - 8 = 4 month
1560 × 8 × 8 Q.19.(4) P : Q : R
A= = Rs.720
4 × 13 3
Capital — 1 : :2
For B & C, 2
Profit— 2 : 3 : 4
B × 6 : C × 8 = 8 : 13
Profit = (2x + 3x + 4x) = 9x units
B × 6 : 1560 × 8 = 8 : 13
According to the question,
1560 × 8 × 8
B= = Rs.1280 8
6 × 13 9x = 9,00,000 ×
100
Contribution by A & B together 9x = 7,20,000
= 720 + 1280= Rs. 2000 Profit of P = 2x = 2 ×80,000 = Rs. 160,000
Q.13.(3) A : B : C Profit of Q = 3x = 3 × 80,000 = Rs. 240,000
= 20000×12 : 25000×4 : 15000×8 Profit of R = 4x = 4 × 80,000= Rs. 3,20,000
= 12 : 5 : 6 Q.20.(3) Initial capital = 2x : 4x : 8x
23000 Total capital invested by A
Share of C = × 6 = R s.6 0 0 0
23 = (2x × 6 + 3x ×6) = 30x
Total capital invested by B
7 4 6 = (4x × 6 + 8x × 6) = 72x
Q.14.(5) A : B : C = : : = 3 5 : 2 0 : 1 8
3 3 5
Total capital invested by C= (6×8x + 6x × 6)
Ratio of profit
= (48x + 36x) = 84x
= [35×4+49×8]:20×12:18×12
New ratio of capitals :
532 : 261 : 216
A:B:C
21600
Total profit= × 1 0 0 9 = Rs. 100900 Capital — 30x : 72x : 84 x
216
Q.15.(1) Total investment Profit— 5 : 12 : 14
Required ratio of their profit = 5 : 12 : 14
= 1 0 0 × 4 5 0 0 0 = R s.3 0 0 0 0 0
15 Q.21.(5) 12 : 16 : 9
Avinash : Manoj : Arun = 3 : 2 : 5 65700 37
× × 1 0 0 = Rs. 370000
Investment of Manoj 9 73
 2  Q.22.(1) Let the total time = 8 years
=  × 3 0 0 0 0 0  = R s.6 0 0 0 0 Let the total capital = 20 units
10 
Q.16.(3) A : B : C = 35000 ×12 : 20000×5 : 15000 × 7 X :Y : Z
Capital— 5 : 4 : 11
84 : 20 : 21
¯× ¯× ¯×
84125 Time— 2 : 4 : 8
Share of B = × 2 0 = 13460
125 Profit— 10 : 16 : 88
QUANTITATIVE APTITUDE 77
stportal.mahendras.org
5 : 8 : 44 A:B:C
According to the question, Capital 24 x : 32x : 24x
(5+8+44) units = Rs. 1140 3x : 4x : 3 x
57 units = Rs. 1140 According to the question,
1140 (3x + 4x + 3x) = 14000
1 units = R s. = Rs. 20
57 10x = 14000
Profit of x = 20 ×5 = Rs. 100
x = 1400
Profit of Y = 20 × 8 = Rs. 160
Hence, Profit of A = 1400×3 = Rs. 4200
Profit of Z = 20 × 44 = Rs. 880
Profit of B = 1400 × 4 = Rs. 5600
Q.23.(3) Let total profit = 24 units
Profit of C = 1400×3 = Rs. 4200
1
Profit of A = × 2 4 = 3 units Q.25.(4) B's profit share in 1 years = 12×100
8
= Rs. 1200
1
Profit of B = × 2 4 = 8u n its
3 10000 × 5 ×1
Interest of A = 100 = Rs. 500
A:B:C
Capital— x : y : 1560 4000 × 5 ×1
Interest of B = = Rs. 200
Time— 4:6:8 100
Total profit of A and B = 1200 + 500 +200
Profit — 3 : 8 : 13 [24-(8+3)]
We know = Rs. 1900

Capital × Time = Profit Remaining profit = 4000 - 1900

P r o fit = Rs. 2100


= Capital
T im e A : B

1 3 units = 1560 Capital 10000 : 4000



8 5 : 2
1 unit = Rs. 960
2100
960 × 8 Share of A in remaining profit = ×5
y= (5 + 2 )
6 = Rs. 1500
y = Rs. 1280 Share of B in remaining profit
3 2100
x= × 9 6 0 = Rs. 720 = m (5 + 2 ) × 2 = R s.6 0 0
4
Capital of A = Rs. 720 Total profit of A
Capital of B = Rs. 1280 = 500 + 1500 = Rs. 2000
1 1 1 Total profit of B = 1200 + 600 + 200
Q.24.(4) A : B : C = : :
4 3 6
= Rs. 2000
Ratio of shares of A, B and C
Q.26.(4) Ratio of their profit share =36000: 60000 =3: 5
A:B:C
Let total Profit = 80
Capital 3x : 4x : 2x
3
Total capital invested by A in 1 year A will get 10% of 80 + of remaining profit
8
3x × 4 + 1.5x × 8 = 24 x
Total capital invested by B in 1 years
3
8 + 72 × = 35
8
4x
= 4x × 6 + 3 × 6 = 32x Now 35 = 7000
Total capital invested by C in 1 year 45
so 45 = 7000× = 9000
= 2x × 12 = 24x 35
78 QUANTITATIVE APTITUDE
stportal.mahendras.org

CHAPTER
MIXTURE and
10
ALLIGATION
Applying Rule of Alligation
General Rules
Ist type IInd type
I. Alligation : It enables us to find the ratio in which two
R s .6 0 R s .6 5
or more ingredients at the given price must be mixed
to produce a mixture of a desired price.
R s .6 2 (M ix ture )
II. Mean price : The cost price of a unit quantity of the
mixture is called the mean price.
III. Basic Formula : If two ingredients are A and B are 3 2
mixed of price x and y respectively, then the price of
Required Ratio = 3 : 2
resultant mixture is M mean price. The ratio in which
ingredients are mixed is given by rule of alligation Ex. 729 Litres of mixtures containing milk and water in
the ratio 7 : 2. How much more water should be added
M −y so that the new mixture contains milk and water in the
R =
x −M ratio 7 : 3 ?.
The above formula can be respented as 2
Sol. 729 litre quantity of water =
Ingre die nt A Ingre die nt B 9
y Pure water = 1
x
3
Quantity of water in mixture =
10
Applying Alligation:
m ea n price M
2
9 1

(M -y) : (x-M ) 3
10
Thus the required ratio is R
M −y y −M
= = 3 3 2
x −M M −x 1
10 10 9
Ex. In what ratio must rice costing Rs.8.50 per kg be mixed 7 7
with rice costing Rs.13 per kg so that the mixture be 10 90
worth Rs.10 per kg? Required Ratio = 9 : 1
Sol. O ne ty pe of R ice 2n d typ e o f R ic e 9 = 729 litre
R s .8 .50 R s .1 3.0 0 729
1= ×1= 81 Litres
9
10 R s . IV. Mixture of more than two elements:
These question may seems to be a little tricky at first,
but similar concept is applied repeatedly in order
3.00 1.50 to calculate final ratio of ingredients when mixture
contains more than two ingredients.
2 : 1
1. Take two ingredients in such away that first ingredient
Ex. In what ratio must a grocer mix two varities of sugr is lower than the mean value and the other one is
costing Rs.60 per kg and Rs.65 per kg, so that on higher than mean value.
selling the mixture at Rs.68.20 per kg he may gain 2. Calculate the ratio of ingredients.
10%? 3. Repeat for all possible pairs.
Sol. Cost price of 1 kg of mixture 4. Final ratio is the ratio obtained from step 2 (if an
100 ingredients is common in the ratios, add value for this
= 68.20× particular ingredient)
110
QUANTITATIVE APTITUDE 79
stportal.mahendras.org
Ex. Three types of Rice of Rs. 1.27, Rs. 1.29 and Rs. 1.32 Sol. Let initial quantity of wine in a cask = x lit
per kg are mixed together to be sold at Rs. 1.30 per
kg. in what ratio should this rice be mixed. So, After 4 process,
Sol. I III II III
L F 8I 4 OP
127 132 129 132 4 = M x GH 1 − x JK
130 130

MN PQ lit.
2 3 1 2
FG
x 1−
8 IJ 4

Hence final ratio is 2 : 1 : 3 + 2 = 2 : 1 : 5 ∴


H x K =
16
⇒ 1−
8 FG IJ = FG 2 IJ
4 4

FG x − 8 IJ = 2
x 81 x H K H3K H x K 3
V. Mixture containing 4 ingredients:
⇒ 3x - 24
Ex. How much a shop owner mixture 4 types of rice worth
Rs. 95, Rs. 60, Rs. 90 & Rs. 50 perkg so that he can
= 2x ⇒ x
make the mixture of these rice worth Rs. 80 perkg.
Sol. I II II III = 24 lit.
95 60 60 90 Ex. A shopkeeper has two types of article. The CP of 1st
article is 20 Rs./Kg. and other article is X Rs./kg. He
80 80 has quantity of 1st article is 10 kg and other article
is 20 kg. He sold the mixture of these article at Rs.
39/kg. with a profit of 30%. Find the value of X ?
20 15 10 20 Sol. 20 x
4 : 3 1 : 2
30
III IV
90 50
30-x : 10
80 Again
30 − x 10
=
10 10 20
30
3 : 1 On solving above equation
x = 25 Rs./Kg
I : II : III : IV = 4 : 4 : 5 : 1
Ex. The ratio of milk and water in a mixture is 5 : 3.
VI. Concept of replacement: How much fraction of the mixture must be drawn
off and substituted by water so that the ratio of milk
Suppose a container contains a - units of liquid from and water is the mixture becomes 1 : 1 ?
which b -units are taken out and replaced by water-
Sol. 3 1
after k-operations, the quantity of pure liquid
8 1
 k

= a  1 − b   units. 2
  a  
1 1
Ex. 8 litres are drawn from a cask full of wine and is then :
2 8
filled with water. This operation is performed three
more times. The ratio of the quantity of wine now left 4 : 1
in cask to that of the water is 16 :65. How much wine 1
did the cask hold originally ? Removed mixture =
5

80 QUANTITATIVE APTITUDE
stportal.mahendras.org

EXERCISE
Q.1. What quantity of tea costing Rs.8 per kg be mixed with 15 kg of tea costing Rs.20 per kg so that mixture be worth
Rs.12 per kg.?
(1) 15 kg (2) 20 kg (3) 25 kg (4) 30 kg (5) 40 kg
Q.2. Two alloys are both made up of copper and tin. The ratio of copper and tin in the first alloy is 1 : 3 and in the second
alloy is 2 : 5. In what ratio should the two alloys be mixed to obtain a new alloy in which ratio of tin and copper
be 8 : 3?
(1) 7 : 5 (2) 5 : 7 (3) 4 : 7 (4) 7 : 4 (5) 3 : 4
Q.3. A mixture contains milk and water in the ratio 3 : 2 and another mixture contains that of in the ratio 4 : 5. How
many litres of the latter must be mixed with 3 ltrs of the former so that the resulting mixture may contain equal
quantities of milk and water?
3 2 4 2 2
(1) 5 ltr. (2) 5 5 ltr. (3) 1 5 ltr. (4) 2 ltr. (5) 4 ltr.
5 5 5
Q.4. A mixture contains two liquids ‘A’ and ‘B’ are in the ratio 4:1. If 10 litres of mixture is withdrawn and replaced
with 10 litres of ‘B’, then the ratio becomes 2:3. What was the initial quantity of A?
(1) 16 ltr. (2) 20 ltr. (3) 25 ltr. (4) 15 ltr. (5) 18 ltr.
Q.5. A Jar contains a mixture of milk and water in the ratio 5:1, 18 litre of mixture is taken out from jar and 6 litre pure
water is added in the jar. If the resultant ratio between milk and water in the jar is 3:1, what was the initial quantity
of mixture in jar before the replacement?
(1) 54 ltr. (2) 36 ltr. (3) 60 ltr. (4) 72 ltr. (5) 76 ltr.
Q.6. Ajay purchased two different kinds of vinegar. In the first mixture, vinegar is 75% and in the second mixture, vin-
egar is 83.33%. If he mixes, the two given mixtures and makes a third mixture in which the percentage of vinegar
is 80%, what is the quantity of the first mixture that is required to make 20 litres of the third kind of mixture?
(1) 8 litres (2) 9 litres (3) 6 litres (4) 7 litres (5) None of these
Q.7. Two vessel contain petrol and kerosene in the ratio 7 : 3 and 1 : 4. In what ratio should both vessel is mixed so that
the mixture contain petrol and kerosene in the ratio 3 : 2?
(1) 4 : 1 (2) 1 : 4 (3) 2 : 3 (4) 3 : 2 (5) None of these
Q.8. If a mixture contains 60% of milk and the remaining is water. Another mixture contains 75% milk, 20% water.
The new mixture is formed by mixing these two mixtures such that the water contains 25%. In the new mixture
the remaining is honey. What is the % of honey in the new mixture?
(1) 11.25% (2) 15% (3) 3.75% (4) 12% (5) None of these
Q.9. Two tank contain petrol and diesel. In the first tank the ratio of petrol to diesel is 7:4 and in the second tank the ratio
is 4 : 5. A third tank of capacity 38L is filled with these two tanks so that the mixture contains the ratio of petrol to
diesel is 5 : 4. How many litres are taken from the second tank?
(1) 11L (2) 12L (3) 13L (4) 16L (5) None of these
Q.10. 1050 rupees are distributed among 1400 males and females such that each male got Rs.1 and each female got 50
paise. Find the number of females among the group.
(1) 600 (2) 700 (3) 800 (4) 550 (5) 640
Q.11. Weight of two friends Ram and Shyam are in the ratio of 4:5. Rams weight increases by 10% and the total weight of
Ram and Shyam together becomes 82.8 kg, with an increase of 15%. By what % did the weight of Shyam increase?
(1) 17% (2) 18% (3) 19% (4) 20% (5) 21%
Q.12. Mukesh earned Rs.4000 per month. From the last month, his income is increased by 8%. Due to rise in prices, his
expenditure also increased by 12% and his saving decreased by 4%. Find his increased expenditure.
(1) Rs.3000 (2) Rs.3200 (3) Rs.3360 (4) Rs.3060 (5) Rs.3150
Q.13. A man travelled a distance of 90 km in 9 hours, partly on foot at 8 km/hr and partly on bicycle at 17 km/hr. Find
the distance travelled on foot.
(1) 50 km (2) 55 km (3) 60 km (4) 56 km (5) 75 km

QUANTITATIVE APTITUDE 81
stportal.mahendras.org
Q.14. A cricketer whose bowling average is 24.85 run per wicket takes 5 wickets for 52 runs and there by decrease his
average by .85. The number of wickets taken by him before the last match was-
(1) 64 (2) 72 (3) 80 (4) 96 (5) 92
Q.15. A man lent Rs. 1500 partly at 6% and other at 5% simple interest. If the total interest earned at the end of 1 year is
Rs.85, then find the amount lent out at 5% simple interest?
(1) Rs.550 (2) Rs.500 (3) Rs.540 (4) Rs.640 (5) Rs.555
Q.16. Jaya purchased 150 kg of wheat at the rate of Rs.7 per kg. She sells 50 kg at a profit of 10%. At what rate should
she sell the remaining quantity so that she gains an overall profit of 20%?
(1) Rs.8 (2) Rs.8.25 (3) Rs.8.40 (4) Rs.8.75 (5) Rs.9.50
Q.17. The ratio of land to water on the whole of the earth is 1:2 and it is 2:3 on the northern hemisphere. The ratio of land
to water on the southern hemisphere-
(1) 11 : 4 (2) 4 : 11 (3) 15 : 4 (4) 4 : 15 (5) 11 : 15
Q.18. A mixture contains milk & water in the ratio 5 : 3. If 16 litres of mixture is replaced by 16 litres of water the ratio
of milk and water becomes 3:5. How much milk was there in the mixture initially?
(1) 8 ltr. (2) 16 ltr. (3) 24 ltr. (4) 25 ltr. (5) 28 ltr.
Q.19. A vessel contains mixture of spirit and water. The percent of spirit in the mixture is 18%, 6 litres of mixture are
replaced with pure water, if the vessel contains 15% spirit now, the capacity of vessel is-
(1) 25 ltr. (2) 30 ltr. (3) 36 ltr. (4) 40 ltr. (5) 45 ltr.
Q.20. How many kg of salt at 42 paisa per kg must be a man mixed with 25 kg of salt at 24 paisa per kg., so that 25%
gain on outlay on selling the mixture at 40 paisa per kg ?
(1) 15 kg (2) 16 kg. (3) 25 kg. (4) 30 kg. (5) 18 kg.
Q.21. A man has 90 pens. He sells some of these at a profit of 15% and the rest at 9% profit. On the whole transaction he
gets a profit of 11%. How many pens did he sell at 9% profit ?
(1) 60 (2) 50 (3) 40 (4) 70 (5) 30
Q.22. A mixture of a certain quantity of milk with 15 ltr of water is purchase at 100 paisa per ltr. If pure milk be worth
Rs. 1.15 ltr, then how much milk is there in the mixture ?
(1) 80 ltr (2) 90 ltr (3) 100 ltr (4) 110 ltr (5) 120 ltr
Q.23. Ram covered a distance of 200 km in 10 hrs. The first part of his journey is covered by auto, then he hired a car.
The speed of the auto and car is 15 km/hr. and 30 km/hr. respectively. Find the ratio of distance covered by auto
and that of car.
(1) 3 : 4 (2) 2 : 1 (3) 1 : 1 (4) 2 : 3 (5) None of these
Q.24. Two brands of detergents are to be combined detergent A contains 40% bleach and 60% soap. While detergent B
contains 25% bleach and 75% soap. If the combined mixture is to be 35% bleach. What % of the final mixture
should be detergent A ?
(1) 30% (2) 45.64% (3) 20% (4) 32.5% (5) 66.67%
Q.25. 12 litres of water is drawn out from a container full of water and replaced by milk. Again 12 litres of mixture are
drawn and the container is again filled with milk. The ratio of final quantity of water to milk in the container is 25
: 11. How much did the container hold ?
(1) 60 litres (2) 65 litres (3) 72 litres (4) 39 litres (5) None of these
Q.26. Tea worth Rs. 126 per kg and Rs. 135 per kg are mixed with a third variety of the ratio 1 :1 : 2 If the mixture is
worth Rs. 153 per kg, the price of the third variety per kg will be :
(1) 175.50 Rs./kg (2) 195.50 Rs./kg (3) 165.50 Rs./kg (4) 135.50 Rs./kg (5) None of these
Q.27. In what ratio must a person mix three kind fo tea each of which has a price of 70,80 & 120 rupees per/kg in such
a way that the mixture cost 100 Rs./kg
(1) 2 : 3 : 4 (2) 2 : 1 : 2 (3) 5 : 2 : 2 (4) 2 : 2 : 5 (5) 3 : 2 : 1

82 QUANTITATIVE APTITUDE
stportal.mahendras.org

EXERCISE Explanation
Q.1.(4) Required Ratio = 1 : 1
O ne type of Tea 2n d typ e of Te a 1 = 10 litre
R s.8 /kg R s.2 0/kg
2 = 20 litre
12 R s./kg (M ixture ) 20
Quantity of A = ×4 = 16 litre
5
1
8 4 Q.5.(4) Quantity of water in the Jar =
6
Quantity of water added = 1
2 : 1 (R a tio )
1
1 Part = 15 kg Quantity of water in the mixture = 4
2 Part = 15×2 = 30 kg. 1
3 6 1
Q.2.(3) Quantity of Tin in Ist alloy =
4 1
5 4
Quantity of Tin in 2nd alloy =
7
Quantity of Tin in the mixture = 3 1
Apply rule of Alligation: 4 12
3 5 Required Ratio = 9 : 1
4 7
1 =.6 litre
8 9 = 54 litre
11
Now = 54 + 18 litre = 72 litre
Q.6.(1) By allegation rule
8 5 3 8 3/4 5/6
11 7 4 11 4/5
1 1
77 44 1/30 1/20=2:3

Required Ratio = 4 : 7 Required answer =20×2/5 = 8 litres
3 Q.7.(1) 30% 80% Kerosene in first mixture
Q.3.(2) Quantity of milk in the Ist mixture=
5 = 3/10×100%= 30%
4
Quantity of milk in the IInd mixture = 40% Kerosene in final mixture
9
1 = 2/5×100%= 40%
Quantity of milk in the final mixture=
2 40% 10% Kerosene in second mixture
Applying Alligation:
3 4 = 4/5×100%= 80%
5 9 4 : 1
1 Hence, option(1) is correct.
2 Q.8.(3) Taking percentage of water
40 20
1 4 3 1 25
2 9 5 2 5 15
1 1 1:3
18 10
Now let first mixture be 100 ltr. and second
Required ratio = 5 : 9 mixture be 300 ltr.
5 Part = 3 litres
Quantity of Milk = 5% of 300 = 15 ltr.
3 27 2
9 Part = 5 ×9 = 5 = 5 5 litres 15
Percentage of Milk in final mixture = ×100
Q.4.(1) 400
1 = 3.75%
5 1 Q.9.(4) 7/11 4/9
3 5/9
5 1/9 : 8/99
11 : 8
2 2
Amount taken from the first tank = 11/19×38
5 5
QUANTITATIVE APTITUDE 83
stportal.mahendras.org
= 22L In this case we get ratio of time
Amount taken from the second tank = 8/19×38 Hence,
= 16L 9 = 90 hrs.
Hence, option(4) is correct. 7 = 7 hrs.
Q.10.(2) Amount received by each man = 1 Rs. D = S × T = 8 × 7 = 56 km/hrs.
Amount received by one women = 50 paise Q.14.(3) New Average = 24 run per wicket
Amount received by each person Old Average = 24.85
1050 52
= = 7 5 p a ise Average of last match = = 10.4
1400 5
Applying Alligation :
Applying alligation :
24.85 10.4
1 .5 0
24
.7 5

13.6 .85
.2 5 .2 5
Required Ratio = 16 : 1
Required Ratio = 1 : 1
1 = 5 wicket
2 = 1400
16 = 80 wicket
1400
1= 2 ×1 = 700 womens. Q.15.(2) Interest on 1500 at 6% = 90 Rupees
Q.11.(3) % increase in Ram’s weight = 10% Interest on 1500 at 5% = 75 Rupees
% increase in Shyam’s weight = x% 85 ×100 17
Overall Interest = = %
% increase in overall weight = 15% 1500 × 6 3
Ram Shyam on Rs.1500/-= 85 Rupees
10% x% 90 75 6% 5%

17%
15% 85
Or 3

2 1
10 5
x-15 : 5 3 3
2 1 2 1
x −15 4
= 1
5 5 Required Ratio = ×1500 = 500 Rs.
3
x = 19% Q.16.(4) S.P. at 10% Profit = 7.70
S.P. at 20% Profit = 8.40
Q.12.(3) % increase in Exp % decrease in Sa ving
12 -4 (50 kg) (100 kg)
7.7 X
8 % increase in incom e
8.4
12 4
1 2
Required Ratio = 3 : 1
3 8 .4 − 7 .7 2
Expenditure = × 4 0 0 0 =3000 Rs. =
4 x − 8 .4 1
112
Increase Expenditure = 3000 × = Rs.3360
100 x = 8.75 Rs.
Q.13.(4) Foot On cycle
8 km /hr 17 km /hr Q.17.(2) By Alligation:
2 x
5
10 km /hr O verall sp eed 1
3
= 90/9= 10 km .
1 1
7 : 2 2
2

84 QUANTITATIVE APTITUDE
stportal.mahendras.org

2 1 1 6 − 5 1 − 3x Distance ratio = 15×2 : 30×1 =1 : 1


⇒ − = − x = =
5 3 3 15 3 Q.24.(5) A B
40 25
1 = 5-15x
4 = 15x
4 35
x=
15
Required Ratio of land to water
10 : 5
= 4 : (15 - 4) = 4 : 11
2 : 1
Q.18.(4) By Alligation:
5 Therefore, % of detergent in A
0
8 = (2/3)×100 = 66.67%
3
8 2
11
2
Q.25.(3) x  1 − 1 2  =
3  x  25
8 8

3:2 ⇒ 5 x = 72 liter
2 Ratio = 16 Litre Q.26.(1) Since first and second varieties are mixed in equal
5 Ratio = 40 Litre proportions, so their average price
5
Initial quantity of Milk = × 4 0 = 25 Litre
FG 126 + 135 IJ =Rs. 130.50

Q.19.(3) 18 0
8
= Rs. H 2 K
So, the mixture is formed by mixing two varieties,
15 one at Rs. 130.50 per kg and the other at say, Rs.
15 : 3 x per kg in the ratio 2 : 2, i.e., 1 : 1. We have to
5 : 1 find x.
Now, By the rule of alligation, we have :
1=6 13 0.50 x
6 = 36 litre.
40 15 3
Q.20.(2) Cost Price of mixture= × 1 0 0 = 3 2 R s. / K g
125
42 24
2 2
32
8 : 10 1 : 1
4 : 5 1 5 3 − 1 3 0 .5 1
Hence, =
5 = 25 x −153 1
4 = 16 x = 175.50 kg.
Q.27.(4) I III
70 12 0
Q.21.(1) 15 9
11 10 0
2 : 4
Þ 1 : 2 20 30
Hence pen at 9% profit = 2/3 × 90 = Rs.60
Similarly
Q.22.(3) 115 0
II pa ir III pa ir
100 80 120
100 : 15
100
20 : 3
3 = 15 litre, 20 = 100 litre
20 20
Q.23.(3) 15 30
20 Thus find ratio
10 : 5 = I : II : III
2 : 1 = 20 : 20 : (30 + 20) = 2 : 2 : 5

QUANTITATIVE APTITUDE 85
stportal.mahendras.org

CHAPTER

11 TIME AND WORK

In most of the problems on time and work, one of the


Important Formulas
following basic parameters is to be calculated:
(i) If A can do a piece of work in X days and B
(a) Time : Time needed by one or more than one
can do the same work in Y days, then both of
person to complete a job or time for which a person(s)
them working together will do the same work
actually worked on the assigned job.
XY 
(b) Alone time : Time needed by single person to in   days
complete a job. X +Y 
(c) Work : The amount of total work (assigned) or (ii) A,B,C while working alone can complete a work
the part of total work actually done. in X,Y and Z days respectively then they will
together complete the work in
Basic Concepts
XY Z
(1) Total amount of a complete job (or assigned job) = 1, days
XY + Y Z + Z X
always, unless specified.
Ex. A can do a piece of work in 6 days and B in 9 days.
(2) If any person ‘M’ completes a job alone in t days, then
How many days will both take together to complete
alone time for ‘M’ = t
the work?
(3) 1 day’s work by any person
Sol. Method I:
F 1 IJ
= G
th
Here A = 6 and B = 9

H alone tim eK × part of total work
They together complete work in 1 days
(4) The reciprocal of 1 day’s work gives the alone time
i.e. = 1/6 + 1/9
1 3 +2 5
Alone time ∝ =
1 day ' s w ork = 18 18
(5) When more than one person working on the same 18
piece of work, then their combined 1 day’s work = They together complete the whole work=
5
sum of 1 day’s work by each person. i.e., If A, B and = 3.6 days
C are three persons working on a job, then (A+B+C)’s
Method II (L.C.M. method) :
1 day’s work = A’s 1 day’s work + B’s 1 day’s work +
C’s 1 day’s work.
(6) It is the application of concept (4) for more than one
person.
The reciprocal of combined 1 day’s work gives the
18
time for completion by the person working together. They together complete the whole work=
5
i.e., time for completion = 3.6 days
1 Formula Method:
=
com bined 1 day ' s w ork
They together complete the whole work
It implies that.
 XY  9 ×6
If three persons, say, A, B and C are working together =  X + Y  = 9 + 6 = 3.6 day
on a job, then
Ex. If man A complete a piece of work in 12 days and the
Time for completion by them
same work is completed by B and C in 15 days and
1 20 days respectively. In how many days the work is
=
bA + B + C g' s 1 day ' s w ork completed if they work together ?

86 QUANTITATIVE APTITUDE
stportal.mahendras.org
Sol. METHOD I : FORMULA METHOD :
1 1 1 by formula
Work/ day of A,B and C = + +
12 15 20  XY  8 ×5 1
5 +4 +3 1 = X − Y  = = 1 3 days
= =   8 −5 3
60 5
3. If A and B working together, can finish a piece of
Total days = 5 days work in X days, B and C in Y days, C and A in Z days
METHOD II (L.C.M. method) : then.
5w /d A 12 days A, B and C working together will finish the job in
4w /d B 15 days L.C .M .= 60 w ork 2 XY Z
3w /d C 20 days XY + Y Z + Z X days
Ex. A and B can complete a piece of work in 8 days B
Total work/day by (A+B+C) = 12 work
and C can do it in days 12 days, C and A can do it in
Total work is done by (A+B+C) 8 days, A B and C together can complete it in-
60 1
= =5 days
12 Sol. Work done by A and B in one day = 8
FORMULA METHOD :
1
They together complete the whole work work done by B & C in one day =
12
xyz
= 1
xy + yz + zx work done by C and A in 1 day = work done by
A,B and C together in 1 day 8
12 ×15 × 20
= = 5 days 1
12 ×15 + 15 × 20 + 20 ×12 11 1 1
=  + +  =
2. Two person A and B working together, can complete a 2  8 12 8  6
piece of work in X days. If A alone can complete the A,B and C together complete the whole job =6 days
work in Y days, then B working alone will complete
METHOD II (L.C.M. METHOD) :
 XY  3 w /d (A +B )
the work in   days 8d
X −Y  2 w /d (B +C ) 1 2 d
Ex. A and B together can do a piece of work in 5 days and 3 w /d (C + A ) 8d
A alone can do it in 8 days. B alone can do the same 8 w /d 2 (A + B + C )
piece of work in 4 w /d (A + B +C )
on e d ay w ork
Sol. METHOD I :
Time taken by A, B & C together to complete the work
Here (A+B) = 5 days, A = 8 days = 24/4 = 6 days
B alone can finish the work in one day FORMULA METHOD:
1 1 8 −5  A + B and C can do finish the whole work
= − = part
5 8  4 0 
2 × 8 ×12 × 8 2 × 8 ×12 × 8
40 = = = 6 day
B finish the whole work = days 8 ×12 + 12 × 8 + 8 × 8 96 + 96 + 64
3
1 Ex. A can complete a piece of work in 20 days & B can
= 1 3 days complete the same work in 25 days. If they start the
3
work together but After 5 days A left the work. In how
METHOD II. (L.C.M. METHOD) : many days the total work would be finished?
8 /d A + B - 5 d a y
w Sol. METHOD I:
5 w /d A - 8 day To ta l w o rk
Work/day of A and B
3 w /d B
(O ne d ay w ork) 1 1 9
= + =
20 25 100
B finish the whole work
5 ×9
40 1 A and B’s 5 day’s work = = 9/20
= = 1 3 days 100
3 3
Remaining work = 1 - 9/20 = 11/20
QUANTITATIVE APTITUDE 87
stportal.mahendras.org
Since A left after 5 days hence remaining work done 1
by B = 11/20 Total days So total days = 1 3 +3
5
11 / 20 11 25 3 1
= × = 1 3 days = 16 days
1 20 1 4 5
25 Ex. A can complete a piece of work in 9 days and B in 12
3 3 days respectively. If they work for a day alternately,
So, total days = 5 + 1 3 = 1 8 days In how many days the work would be finished, If A
4 4
begins the work?
METHOD II:
Sol. 4w /d A - 9 da ys
5w /d A 20 d ays LC M = 36 w o rk
10 0 w ork
4w /d B 25 d ays 3w /d B - 12 d ays
9w /d A+B Now,

A+B A B 55 = 1 3 3 da ys A B A B

5 da ys 4 4 7w
4w 3w 4w 3w
5× 9 = 45w 55 w 2 days 2 days

10 0 w 7 w → 2 days
3 3 5x 7w → 2 days ×5
Total Time = 5+13 = 18 days
4 4 35w → 10 days
Ex. A and B can complete a piece of work in 24 days &
Remaining work = 36 -35 = 1 w
36 days respectively. They start the work but 3 days
before completion of work, A left. In how many days 1w will be done by A (because A starts the work)
will the total work be completed. 1
Sol. METHOD I : ∴ 1w → 4 days----------- (done by A)
A’s 1 day’s work = 1/24
∴ Total time = 10 +
FG1
days
IJ
B’s 1 days’ work = 1/36
1
H4 K
∴A left before 3 days = 10 days
4
3 1
∴For last 3 days B’s work = = 4. Work efficiency of any worker inversely proportional
36 12
to time taken by him.
1 11
Remaining work = 1 − = 1
12 12 i.e, work efficiency ∝ T im e
Hence remaing work done by both so total days Ex. A is twice as good a work man as B & together they
finish a piece of work in 18 days. In how many days
11 11
1 will A alone finish the work ?
= 12 = 12 =13
1 1 5 5 Sol. A : B = 2 : 1
+
24 36 72 Time taken by A and B = 18 days
1 1 work done by A and B = 18 × 3 = 54 work
So total days = 1 3 5 +3 = 1 6 5 days
54
A will do alone = = 27 days
METHOD II: 2
3 w /d A −2 4 d ay Ex. If the ratio of work efficiency of A & B is 6 : 5 & that
2 w /d B −3 6 d ay To ta l w ork of B & C is 6 : 5. If A can complete the piece of work
5 w /d A+B in 2 days, In How many days the same work can be
(O n e d a y w ork ) completed by B separately ?

1 Sol. A : B = [6 : 5]
6 6 /5 = 1 3 5 d a y s A
Note : Here B : C is not required, this is given to
A+B B confuse you,
(3 da ys) time taken by A = 2 days
6 6 w o rk 3× 2= 6
w o rk work done by A = 2 × 6 = 12 work
7 2 w o rk
88 QUANTITATIVE APTITUDE
stportal.mahendras.org

12 2 ×
time taken by B = = 2 days a m en or b w om en or c children d days
5 5
5. MDH FORMULA:
(i) More men can do more work. i.e. M ∝ W. x m en & y w om en &
z children ? days
1
(ii) More men means less working hours. i.e. M ∝ H . a ×b ×c ×d
D =
(iii) More men can do same work in less number of xb c + ya c + za b
1 Ex. 3 Men or 4 Women can do a piece of work in 43 days.
days. i.e. M ∝ .
D In how many days 7 men & 5 women can do the same
W work ?
M∝
DH Sol. 3m or 4w = 43 days
MDH 7m + 5w = ?
= Constant
W From formula : M1 D1 = M2 D2
(iv) If M1 can do W1 work in D1 days working H1 hr/ 3 m × 43 = (7m + 5w) × D2
day for Rs. R1 and M2 man can do W2 work in 15
D2 days working H2 hr/day for Rs. R2 then 129 m = (7m + m) × D2
4
ForMULA is given as:
43 m
M 1D 1H 1 M 2 D 2 H 2 129 m =
4
× D2 ⇒ D2 = 12 days
=
W 1R 1 W 2R 2
SHORT Trick :
Here, R = wages (in Rs.) a ×b ×d
D =
Ex. If 10 men complete half work in 12 days when they xb + ya
work 8 hours per day, In how many days 18 men
3 × 4 × 43
complete the full work when they work 6 hours/day ? D = = 12 days
28 + 15
Sol. By formula :
Ex. 1 man or 2 boys or 3 girls can do a piece of work in
M 1D 1H 1 M 2D 2H 2 88 days. In how many days one man, one boy and one
=
W1 W2 girl can do the same work ?
Sol. 1m = 2b = 3g = 88 days
10 × 12 × 8 18 × D 2 × 6 (1m + 1b + 1g) = ?
=
1/ 2 1
By formula : m1 d1 = m2 d2
10 × 12 × 8 × 2 = 18 × D2 × 6 3g × 88 = (1m + 1b + 1g) × d2
160 7 3
D2 = = 17 days. 3g × 88 = (3g + g + 1g) d2
9 9
2
6. (i) If a men or b women can do a piece of work in
d days then x men and y women together finish 3g × 88 = FG 6 g + 3 g + 2 g IJ d
the whole work-

H 2 K 2

a m en or b w o m en d da ys 11
× 3g × 88 = g × d2
+ 2
x m en & y w om e n ? da ys d2 = 48 days
a ×b ×d SHORT Trick :
D =
xb + ya a ×b ×c ×d
D =
xb c + ya c + za b
(ii) If a men or b women or c children can do a piece of
work in d days then x men, y women and z children 1 × 2 × 3 × 88
D = = 48 days
together finish the whole work- 6 +2 +3

QUANTITATIVE APTITUDE 89
stportal.mahendras.org

EXERCISE
Q.1. A man can do a job in 15 days. His father takes 20 days and his son finishes it in 25 days. How long will they take
to complete the job if they all work together ?
15 17 18 12
(1) 5 days (2) 5 days (3) 6 days (4) 6 days (5) None of these
47 23 47 47
Q.2. A man can do a piece of work in 5 days, but with the help of his son, he can do the same work in 3 days. In what
time can the son do it alone ?
1 1
(1) 6 days (2) 7 days (3) 7 days (4) 8 days (5) None of these
2 2

Q.3. Two workers A and B are engaged to do a work. A working alone takes 8 hours more to complete the job than if both
1
worked together. If B worked alone, he would need 4 hours more to complete the job than they both working
2
together. What time would they take to do the work together ?
(1) 4 hours (2) 5 hours (3) 6 hours (4) 7 hours (5) None of these
Q.4. A and B can do a piece of work in 72 days; B and C can do it in 120 days while A and C can do it in 90 days. In
what time can A alone do it ?
(1) 80 days (2) 100 days (3) 120 days (4) 150 days (5) None of these
Q.5. A and B can do a piece of work in 5 days; B and C can do it in 7 days; A and C can do it in 4 days. Who among
these will take the least time if put to do it alone ?
(1) A (2) B (3) C (4) Data inadequate (5) None of these
Q.6. A can do a piece of work in 4 hours; B and C together can do it in 3 hours, while A and C together can do it in 2
hours. How long will B alone take to do it ?
(1) 8 hours (2) 10 (3) 12 hours (4) 24 hours (5) None of these
Q.7. A can do a certain work in the same time in which B and C together can do it, A and B together could do it in 10
days and C alone in 50 days, then B alone could finish it in :
(1) 15 days (2) 20 days (3) 25 days (4) 30 days (5) None of these
Q.8. A does half as much work as B in three-fourth of the time. If together they take 18 days to complete the work, how
much time shall B take to do it ?
(1) 30 days (2) 35 days (3) 40 days (4) 20 days (5) None of these
Q.9. A can finish a work in 24 days, B in 9 days and C in 12 days. B and C start the work but are forced to leave after
3 days. The remaining work was done by A in :
1
(1) 5 days (2) 6 days (3) 10 days days (4) 10 (5) None of these
2
Q.10. X and Y can do a piece of work in 20 days and 12 days respectively. X started the work alone and then after 4 days
Y joined him till the completion of the work. How long did the work last ?
(1) 6 days (2) 10 days (3) 15 days
(4) 20 days (5) None of these
Q.11. What will come at place of blank?
Aman completes a work in _ days. Brijesh and Dheeraj completes the same work in 22.5 days working together.
Brijesh is 33.33% more efficient than Aman whereas Dheeraj is 20% less efficient than Aman.
(1) 24 (2) 36 (3) 45 (4) 48 (5) None of these
Q.12. A person can complete a job in 91 days. He works alone on Day 1. On Day 2, he is joined by another person who
also can complete the job in exactly 91 days. On Day 3, they are joined by another person of equal efficiency. Like
this, everyday a new person with the same efficiency joins the work. How many days are required to complete the
job?
(1) 12 (2) 15 (3) 13 (4) 14 (5) 11
90 QUANTITATIVE APTITUDE
stportal.mahendras.org
Q.13. Ram, Lakhan and Krish are ready to do a work. Ratio of time taken to complete the work by Lakhan and Ram is
9 : 4 and ratio of time taken by Ram and Krish to finish the work is 3:2. If they all complete the work in 27 days
than find the number in which Krish will complete the work?
(1) 56 days (2) 46 days (3) 53 days (4) 37 days (5) None of these
Q.14. If 5 men or 9 women can do a piece of work in 19 days. In how many days will 3 men and 6 women do the same
work?
(1) 10 days (2) 12 days (3) 13 days (4) 15 days (5) None of these

Q.15. A can cultivate th of a land in 6 days and B can cultivate rd of the same land in 10 days. Working together A

and B cultivate th of a land in:


(1) 4 days (2) 5 days (3) 8 days (4) 10 days (5) 12 days
Q.16. A certain number of men can complete a job in 30 days. If there were 5 men more, it could be completed in 10 days
less. How many men were in the beginning?
(1) 10 (2) 15 (3) 20 (4) 25 (5) None of these
Q.17. A men undertakes to do a certain work in 150 days. He employes 200 men. He finds that only a quarter of the work
is done in 50 days. The number of additional men that should be appointed so that the whole work will be finished
in time is:
(1) 50 (2) 75 (3) 100 (4) 125 (5) 300
Q.18. In a fort, there was sufficient food for 200 soliders for 31 days. After 27 days, 120 soliders left the fort. For how
many extra days will the food last for the remaining soliders?
(1) 10 days (2) 6 days (3) 4 days (4) 12 days (5) None of these

Q.19. 12 men take 36 days to do a work while 12 women complete th of the same work in 36 days. In how many days

10 men and 8 women together will complete the same work ?


(1) 6 days (2) 27 days (3) 12 days (4) Data inadequate (5) None of these
Q.20. 12 men can complete a piece of work in 36 days. 18 women can complete the same piece of work in 60 days. 8
men and 20 women work together for 20 days. If only women were to finish the remaining work in 4 days, how
many total women would be required?
(1) 70 (2) 28 (3) 66 (4) 40 (5) None of these
Q.21. A alone can do a piece of work in 6 days and B alone in 8 days. A and B undertook to do it for Rs. 3200. With
the help of C they completed the work in 3 days. How much is to be paid to C?
(1) Rs. 375 (2) Rs. 400 (3) Rs. 600 (4) Rs. 800 (5) None of these
Q.22. Six men earn as much as seven women, two women earn as much as three boys, four boys earn as much as five
girls. If a girl earns Rs. 16 a week, what does a man earn per week?
(1) Rs. 35 (2) Rs. 20 (3) Rs. 40 (4) Rs. 30 (5) None of these
Q.23. It 8 men, working 9 hours per day can build a wall 18 metres long, 2 metres wide and 12 metres high in 10 days.
How many men will be required to build a wall 32 metres long, 3 metres wide and 9 metres high by working 6
hours a day in 8 days?
(1) 25 men (2) 18 men (3) 30 men (4) 16 men (5) None of these
Q.24. A can do a piece of work in 36 days. B in 54 days and C in 72 days All the three began the work together, but A
left before 8 days and B 12 days before the completion of the work. In how many days, the work was finished from
the day it started?
(1) 24 (2) 28 (3) 36 (4) 48 (5) None of these
Q.25. Two men and 7 boys can do a piece of work in 14 days, 3 men and 8 boys can do it in 11 days In how many days
can 8 men and 6 boys do a work 3 times as much as the first ?
(1) 21 (2) 27 (3) 28 (4) 35 (5) None of these

QUANTITATIVE APTITUDE 91
stportal.mahendras.org
Q.26. Kaveri takes twice as much time as Kanti and thrice as much as Kalpana to finish a place of work. They together
finish the work in one day. Find the time taken by by Kaveri to finish the work.
(1) 2 (2) 4 (3) 6 (4) 8 (5) None of these
Q.27. 1 man or 2 women or 3 boys can do a work in 44 days. Then in how many days will 1 man, 1 woman and 1 boy
do the work?
(1) 24 (2) 48 (3) 36 (4) 40 (5) None of these
Q.28. Two friends take a piece of work for Rs. 960. One alone could do it in 12 days, the other in 16 days with the
assistance of an expert they finish it in 4 days How much remuneration the expert should get ?
(1) Rs. 400 (2) Rs. 640 (3) Rs. 520 (4) Rs. 300 (5) None of these
1
Q.29. A,B and C working together can complete a piece of work in 7 days. If A is twice as fast as B, B is thrice as fast
2
as C then in how many days B & C can complete this work together.
3 3
(1) 17 days (2) 18 days (3) 17 days (4) 18 days (5) None of these
4 4
Q.30. Ram can do a piece of work in 6 days while Shyam can do the same piece of work in 8 days. Ram and Shyam work
together for 2 days and then Ram leaves the work while Shyam continues to work till the work gets completed. On
which day from the beginning, will the work be completed?
(1) 6th (2) 5th (3) 7th (4) 4th (5) None of these
Q.31. A group of certain number of men can complete a job in 5 days. On every alternate day starting from second day,
3 men are withdrawn from the job and on every alternate day starting from third day, 2 men are added to the job.
If the job is completed in 6 days, what is the number of persons who started the job?
(1) 10 (2) 12 (3) 15
(4) Cannot be determined (5) None of these
Q.32. Ashok can complete one-fourth of a piece of work in 6 days, Shiva can complete one-third of the same work in
8 days and Rahul can complete one-fifth of the work in 4 days. Ashok, Rahul, and Shiva start working on the
work together for 5 days after which Ashok and Rahul stop working. How many days more would Shiva take to
complete the work?
(1) 5 (2) 6 (3) 7 (4) 8 (5) None of these
Q.33. The ratio of efficiencies of Mohan and Shyam is 2:3. The ratio of the time taken by Shyam and Anil to complete
some work is 1: 3. If all three work together to finish a particular work, the work is completed in 24 days. How
much time will it take to finish the same work if Mohan and Anil work together?
(1) 30 (2) 24 (3) 48 (4) 36 (5) None of these

92 QUANTITATIVE APTITUDE
stportal.mahendras.org

EXERCISE Explanation
Q.1.(3) 1 day’s work of the three persons Adding we get : 2 (A +B +C)’s 1 day’s work
FG 1 + 1 + 1 IJ = 47 FG 1 + 1 + 1 IJ = 83
=
H 15 20 25 K 300 =
H 5 7 4 K 140
So, all the three together will complete the work 83
(A +B + C)’s 1 day’s work =
300 18 days 280
in ≈ 6
47 47
A’s 1 day’s work =
FG 83 − 1 IJ = 43 ,
Q.2.(3) Formula H 280 7 K 280
Son’s 1 day’s work
B’s 1 day’s work = G
F 83 − 1 IJ = 13 ,
=
bS + F gno. of days × father no. of days H 280 4 K 280
(father' s 1day w ork − both w ork together)
C’s 1 day’s work = G
F 83 − 1 IJ = 27

F 1 1I 2
Son’s 1 day’s work = GH − JK =

H 280 5 K 280
3 5 15 280
15 1 Thus time taken by A,B, C is days,
The son alone can do the work in = 7 days 43
2 2 280 280
Q.3.(3) Let A and B together take x hours to complete the days and days respectively.
13 27
work. Then, Clearly, the time taken by A is least.

A alone takes (x+8) hrs and B alone takes x +


9 FG IJ 1

hrs to complete the work
2 H K Q.6.(3) A’s 1 hour’s work =
4
’(B+C)’s 1 hour’s work
1 1
1 1 1 = ’ (A+C)’s 1 hour’s work = .
+ 3 2

b x+8 g FG x + 9 IJ =
x
FG 1 + 1 IJ = 7 .
H 2K (A+B+C)’s 1 hour’s work =
H 4 3 K 12
1 2 1
⇒ x + 8 + 2x + 9 = x
b g b g F 7 − 1 IJ = 1
B’s 1 hour’s work = G
⇒ x(4x+25) = (x+8)

H 12 2 K 12
∴ B alone will take 12 hours to do the work.
⇒ 2x2 = 72 ⇒ x2 =36 ⇒ x = 6 hours
1
Q.7.(3) (A+B)’s 1 day’s work = ,
9 10
Short trick, × 8 = 6 hour 1
2 C’s 1 day’s work = .
1 50
Q.4.(3) (A+B)’s 1 day’s = (A+B+C)’s 1 day’s work
72
(B + C)’s 1 day work =
1 FG 1 + 1 IJ = 6 = 3
120
1
=
H 10 50 K 50 25
(A + C)’s 1 day’s work = 90 Also, A’s 1 day’s work = (B +C)’s 1 days work.
Adding, we get : 2 (A+ B +C)’s 1 day’s work 3
Then, we get : 2 × (A’s 1 day’s work) =
25
1FG 1 1 12 1
= 72 + 120 + 90 = 360 = 30
IJ ⇒ A’s 1 day’s work = 3 .
H K
50
⇒ (A + B +C)’s 1 day’s work =
1
∴B’s 1 day’s work=
1

FG
3
=
2
=
1 IJ

60 10 50 H 50 25 K
FG 1 − 1 IJ = 1 So, B alone could do the work in 25 days.
So, A’s 1 day’s work =
H 60 120 K 120 Q.8.(1) Suppose B takes x days to do the work.
∴ A alone can do the work in 120 days.
1 ∴ A takes 2 ×
FG 3
x =
IJ3x
Q.5.(1) (A + B)’s 1 day’s work = ,
5

H 4 K 2
days to do it
1
1 (A + B)’s 1 day’s work =
(B + C)’s 1 day’s work = 7 , 18
1 2 1
1 ∴ + = or x = 30
(A+C)’s 1 day’s work = . x 3 x 18
4 So, B can do the same work in 30 days.
QUANTITATIVE APTITUDE 93
stportal.mahendras.org

FG 1 + 1 IJ = 7
95m × 9 = 57m × d2
Q.9.(3) (B + C)’s 1 day’s work =
H 9 12 K 36 95 × 9
Work done by B and C in 3 days = d2 ⇒ 15 days

57
FG 7 × 3 IJ = 7 2 1
Q.15.(3) One day work of A = × =
1
.
=
H 36 K 12 5 6 15

Remaining work = 1 −
FG 7 IJ
=
5
1 1
One day work of B = × =
1
H 12 K 12

3 10 30
One day work of A and B
1
Now, work is done by A in 1 day.
24 1 1 3 1
So,
5 FG
work is done by A in 24 ×
5 IJ
=10 days.
= + =
15 30 30 10
=
12 H 12 K 4 4
Q.10.(2) Work done by X in 4 days= G
F 1 × 4 IJ = 1 ∴ Days taken to do
5
th work= × 1 0 =8 days
5
H 20 K 5 Q.16.(1) Let men employed originally = x. Then


F 1I 4
Remaining work = GH 1 − 5 JK = 5
30x=20(x+5) ⇒ 10x=100 ⇒ x = 10 men
Alternative Method :

(X+Y)’s 1 day work= G


F 1 + 1 IJ = 8 = 2 Ratio of days = 30 : (30–10) = 3 : 2

H 20 12 K 60 15 ⇒ Ratio of workers=Inverse ratio of 3 : 2 = 2 : 3
2
So, work is done by X and Y in 1 day. Difference = (3–2) ratio = 1 ratio
15
4 = 5 men ⇒ Men (originally) = 2 ratio
So, work is done by X and Y in
5
FG
15 4
×
IJ = 2×5 = 10 men.

H2 5 K
= 6 days
Q.17.(3) Days remaining after 50 day's work
Hence, total time taken = (6 +4) days = 10 days. = 150–50=100 days
Q.11.(4) A:B=3:4 (eff) 1
50 men has completed work in 50 days.
A : D=5 :4 (eff) 4
1 3
A:B:D=15:20:12 (eff) Remaining work = 1 – =
4 4
32 which is equal to triple of work already
Required answer= 22.5 × = 48 days completed.
15
Q.12.(3) 91= n/2(2+(n-1))
Men Days Work
on solving n=13
200 50 1
Q.13.(3) Efficiency ratio of Lakhan: Ram: Krishna 8:18:27
x 100 3
Number of days Ratio of Lakhan: Ram: Krish -
27:12:8 50 3
∴ Men required = 2 0 0 × × = 300 men
100 1
Sum of one day work of Lakhan,Ram and Krish ∴ Additional men required = 300–200 = 100 men
1 1 1 1 Q.18.(2) After 27 days, the remaining food is sufficient for
+ + =
27k 12k 8k 27 200 soliders for 31-27=4 days.
Number of days of Krish = 538×8538×8 = 53 days Remaining soliders = 200–120=80 soliders
Q.14.(4) 5 m or 9 w ⇒ 19 days ⇒ 5 m=9m =19 days Soliders Days
3m and 6 w ⇒ ? ⇒ 3m + 6w ⇒ ? 200 4
Formula - m1d1 = m2 d2 80 x

5m × 19 = (3m + 6w)×d2 Less soliders, provision will last for more days,
5 i.e. inverse proportion
5m × 19 = (3m+ ×6)×d2

9 200
FG
27 m + 30 m IJ ∴ The food will be sufficient for 4 ×
80

95m = H 9 ×d2 K =10 days ⇒ Extra days = 10–4 = 6 days
94 QUANTITATIVE APTITUDE
stportal.mahendras.org
Q.19.(2) The number of persons and number of days are Alternative Method :
same in both the cases. 3
A's wages = × Rs.3200= Rs.1600. B's wages
3 6
⇒ 1 woman's work = man's work ⇒ 8 3
4 = × Rs.3200 = Rs.1200
3 8
women's work = 8 × =6 men's work C's wages = 3200–(1600+1200)= Rs. 400
4
∴ 10 men + 8 women = 10 men + 6 men= 16 men Q.22.(1) 6 men - 7 women;
12 men can do the work in 36 days. 2 women = 3 boys;
12 4 boys = 5 girls;
⇒16 men can do the work in 36× = 2 7 days
16 Now earning of 1 girls per week = Rs. 16.
Q.20.(1) (12×36) men's work= (18×60) women's work 5
Earning of 1 boy = × 16 = Rs. 20
18 × 60 5 4
⇒ 1 man = = women
12 × 36 2 3
5 Earning of 1 women = × 20 = Rs. 30
8 men + 20 women = 8 × + 2 0 women 2
2
= 40 women 7
Earning of 1 man = × 30 = Rs. 35.
Total work days = 18×60 woman days 6
= 1080 woman days
M 1D 1H 1 M 2D 2H 2
Work already done =40×20 = 800 woman days Q.23.(3) =
W1 W2
Remaining work = 1080-800 = 280 woman days
8 × 10 × 9 x×8×6
Required number of women = 280÷4= 70 women =
18 × 2 × 2 18 × 2 × 12
Alternative method :
x = 30 men
(12×36) men's = (18×60) women's
Q.24.(1) Let the total time taken be x days.
⇒ 2 men = 5 women
According to the given condition
8men + 20 women
x −8 x − 12 x
2 ⇒ + + =1
= 8 men + 20× men = 16 men 36 54 72
5
6 ( x − 8) + 4 ( x − 12) + 3x
12 men can do the work in 36 days. ⇒ =1
∴ 16 men work for 20 days 216
6x − 48 + 4x − 48 + 3x 13x − 96
66 × 20 20 ⇒ =1⇒ =1
= 12 × 36 = 27 216 216
20 7
Remaining work = 1 – 2 7 = 2 7 Þ13x - 96 = 216 Þ 13x = 216 + 96 = 312
312
7 x= = 24
Woman-days required to finish th of the work 13
7 27
= ×18×60=280 Q.25.(1) 2 men + 7 boys in 14 days
27
Women required to finish work in 4 days ⇒ 28 men + 98 boys in 1 day
= 280 ÷ 4 = 70 women. 3 men + 8 boys in 11 days
3 1 ⇒ 33 men + 88 boy s in 1 day
Q.21.(2) A's work for 3 days = = .
6 2
28 men + 98 boys - 33 men + 88 boys
1 3
B's work for 3 days = 3 × = 2 boys = 1 man
8 8
1 3 1 Now. 2 men + 7 boys = 11 boys; 8 men + 6 boys
C's work 1 – – =
2 8 8 = 22 boys
1
C's share = × Rs.3200=Rs. 400 More boys, fewer days, more work, more days
8
Boys
QUANTITATIVE APTITUDE 95
stportal.mahendras.org
B oys D ays W ork 75 3
11 14 1 Total time = = 18
4 4
22 x 3 Q.30.(1) R 6 4
24

x
∴ =
11 3
× S 8 3
14 12 1 Work done = 2(4+3) = 14
∴ Number of days = 21 days Remaining work = 10
Q.26.(3) More, the alone time of Kaveri is related to the 10
Remaining time = = 3.33
alone times of to other two persons, so assume (be 3
alone time of kaveri) =-x. Req. Answer = 2 + 4 = 6th day
x x
Then, alone time of Kanti= and of Kalpana= Q.31.(3) Let the number of men who worked on day-1 be
2 3 ‘n’.
Kaveri’s 1 day work + Kanti’s 1 day work +
The number of people working each day starting
Kalpana's 1 day
from day-1 will be,
work = combined 1 days work
n, n-3, n-1, n-4, n-2, n-5.
1 1 1 1
⇒ + + = ⇒x=6 This work is equal to the work done by ‘n’ men in
x x/2 x/3 1
5 days.
∴ Alone time for Kaveri = 6 days, for So, 6n-15 = 5n => n = 15
Kanti = 6/2 = 3 days, Q.32.(4) Let the total amount of work be 120 units.
Kalpana = 6/3 = 2 days. Ashok completed 1/4rd of the work i.e. 30 units
of work in 6 days. Thus, the amount of work
Q.27.(1) Number of required days
Ashok can complete in 1 day = 30/6 = 5 units
1 44 × 1 × 2 × 3
= = The amount of work Shiva completed in 8 days
1 1 1 6+3+ 2
+ + = 120/3 = 40 units
44 × 1 44 × 2 44 × 3
Thus, the amount of work done by Shiva in 1 day
= 24 days = 40/8 = 5 units
4 1 Similarly, the amount of work done by Rahul in
Q.28.(1) First friend’s 4 day’s work = (Since, the
12 3 1 day = 24/4 = 6 units
work is finished in 4 days, when expert assists) The work done by all three of them in 5 days
4 1 = (5 + 5 + 6) x 5 = 80 units
Second friends s 4 day ’s work =
16 4 Thus, amount of work remaining= 120 - 80
 1 1 5 = 40 units
The expert’s 4 day’s work= 1 −  +  =
3 4 12 Thus, the number of days taken by Shiva to
complete the remaining work = 40/5 = 8 days
Now. total wages of Rs.960 is to be distributed
among two friends and the expert in proportion Q.33.(3) The ratio of efficiency of Mohan and Shyam is
to the amount of work done by each of them 2:3. So let Mohan does 2 units of work in a day
So, 960 is to be divided in the proportion of while Shyam does 3 units of work in a day.

1 1 5 The ratio of time taken by Shyam and Anil is 1:3,


: : or 4 : 3 : 5 so the ratio of efficiency of Shyam and Anil is 3:1
3 4 12
=> Anil does 1 unit of work in a day
5 => So when all 3 are working together, the total
Share of expert = × 960 = Rs. 400
12 work done in one day = 3 + 2 + 1 = 6 units
Hence, the expert should get Rs. 400 Total work which they do= 6×24= 144 units
Q.29.(2) 6 + 3 + 1 = 10 Now if only Mohan and Anil are working, then
Total work = 7 × 10 = 75 they will finish 2+1 = 3 units of work in one day.
So, Total time required for them to finish the
(B & C) one day work = 3 + 1 = 4
work = 144/3 = 48 days.

96 QUANTITATIVE APTITUDE
stportal.mahendras.org

CHAPTER

12 PIPE & CISTERN

Pipe : Pipes are connected to a tank or cistern and are used 60


to fill or empty the tank, there are of two types. = 12 min
Time taken by ( A + B) =
5
Inlet : A pipe connected with a tank or cistern that fills it is Method iii: by formula-
known as inlet, means nature of pipe is positive.
Both Pipe A and B together fill the tank in
Outlet : A pipe connected with a tank or cistern emptying it
is known as outlet, means nature of pipe is negative. 20 × 30 600
= = = 12 min.
Important Points (2 0 + 3 0 ) 50
Ex. If pipe 'A' can fill the tank in 8 hr. and pipe 'B' can
(1) Chapter Pipe & cistern is more similar to chapter Time empty the tank in 16 hr. When both pipes are opened
& Work. simultaneously, how much time will be taken to fill
(2) If an Inlet can completely fill the empty tank in x hours, the tank ?
the part of the tank filled in 1 hour = 1/x. Sol. Method i:
(3) If an outlet can empty the full tank in y hours, the part Here A = 8 hour, B = 16 hour
of the tank empty in 1 hour = 1/y.
Part of cistern fill by A & B in 1 hour
(4) If both inlet and outlet are open, net part of the tank
1 1 1 1 2 −1 1
filled in 1 hour = − . = − = =
x y 8 16 16 16
Both pipe fill the tank = 16 hours
(5) If a pipe A alone can fill the tank in x hours and pipe
B can fill or empty the tank in y hours. If both pipe Method iI:
are working simultaneously then tank to fill or empty A = +8 h
+ 2 l/h 16 l
 xy 
by  hours. - 1 l/h B = -16 h
 x ± y 
+ 1 l/h A+B
For filling pipe, we take ‘+’ sign.
For empty or drain pipe we take ‘-’ sign. Time taken when A & B both are opened,

Ex. Two pipes A & B can fill a tank in 20 min. and 16


A+B= = 16 hour.
30 min. respectively. If both the pipes are opened 1
simultaneously. How much time will be taken to fill Method iii: by formula-
the tank ?
16 × 8
Both pipe fill the tank = = 16 hours
Sol. Method i: 16 − 8
Here A = 20 min, B = 30 min (6) Three pipes A,B,C can fill the tank in x,y and z hours
respectively.If all three pipes opened simultaneously
1 1
∴ Part of the tank filled by A & B in 1 min. = + the time taken to fill the cistern is given by -
20 30
5 1 xyz
= =
60 12 xy + yz + zx
Both Pipes A and B together fill the tank in 12 min. Ex. Three taps A,B,C, can fill the tank in 4,6 and 12 hour
Method ii: respectively. How long would these three taps take to
fill the tank if all of them are opened together?
+ 3 l/m A + 20m
60 l
+ 30m Sol. Method i:
+ 2 l/m B
A+B Here A = 4 hours, B = 6 hours and C = 12 hours
+ 5 l/m
All together fill tank in 1 hour

QUANTITATIVE APTITUDE 97
stportal.mahendras.org

1 1 1 Method iII: by formula-


= + +
4 6 12 C can empty the tank
3 + 2 +1 1 30 × 60 × 45
= = part, =
12 2 30 × 45 + 60 × 45 − 30 × 60
All fill the tank in 2 hours.
81000
= = 36 min.
Method iI: 1350 + 2700 −1800
+ 3 l/h A + 4h (8) One inlet pipe A is k times faster than the other inlet
B + 6h 12 l pipe B.
+ 2 l/h
+ 1 l/h C + 12h (a) If B can fill a cistern in x hrs., then the time in
+ 6 l/h A+ B+C which the cistern will be full, if both the inlet
12 pipes are opened together, is hrs.
All fill the tank = hours = 2 hours
6
(b) If A can fill a cistern in y hrs., then the time in
Method iII: by formula-
which the cistern will be full, if both the inlet
All together fill the tank  k 
pipes are opened together, is  y hrs.
4 × 6 ×12 4 × 6 ×12  k + 1 
= =
4 × 6 + 6 ×12 + 12 × 4 24 + 72 + 48 (9) One fill pipe A is k times faster and takes x minute
less time than the other inlet pipe B, then
4 × 6 ×12
= = 2 hours (a) The time taken to fill a cistern, if both the pipes
144
(7) Two pipes A & B can fill the tank in X, Y respectively.  kx 
are opened together is  2 mins.
There is also an outlet pipe C. If all three pipe opened  (k − 1) 
simultaneously tank will full in Z hours the time taken
by C to empty the full tank is given by -  x 
(b) A will fill the cistern in  min.
XY Z k − 1 
XZ + Y Z − XY  kx 
(c) B will fill the cistern in  min.
Ex. Two taps A & B, can fill the tank in 30 min and 60  k − 1 
min respectively. There is a third exhaust pipe C at Ex. One inlet pipe A is 9 times faster than second inlet pipe B.
the bottom of the tank. If all taps are opened together If B can fill a cistern in 40 mins, then find the time when the
then tank will be full in 45 minutes. In what time cistern will be full if both inlet pipes are opened together.
can exhaust tap C empty the cistern when tank is
completely full? Sol. Here k = 9 and x = 40

Sol. Method i:  x 
∴ Cistern will be full in = 
 k + 1  mins
Here A = 30 min, B = 60 min and A+B-C = 45 min.
 40 
=  mins or 4 mins.
C can empty the tank in one minute  9 + 1 
1 1 1 6 +3 −4 1
= + − = = part. Ex. One inlet pipe A is 3 times faster than second inlet
30 60 45 180 36 pipe B. If A can fill a cistern in 16 minute, then find
the time when the cistern will be full if both inlet pipes
All fill the tank in 36 minutes. are opened together.
Method iI: Sol. Here k = 3 and x = 16
+ 6 l/m A + 30m
B + 60m 18 0 l ∴ Cistern will be full in =  x  y minute
+ 3 l/m  k + 1
+ 4 l/m + 45m
 3 
-5 l/m =  16 = 12 minute.
C  3 + 1 
180 Ex. One inlet pipe A is 3 times faster than second inlet pipe
C can empty the tank= min. = 36 min
5 B. If together can fill the tank in 36 min, then in how
much time slower pipe can fill the tank.

98 QUANTITATIVE APTITUDE
stportal.mahendras.org
Sol. Method i: Here both the pipes fill the tank = 36 mins
Let time taken by faster pipe be x min then slower k = 3, so slower pipe x fill the tank
pipe take = 3x
= 36 × (3 + 1) = 144 mins
1 1 1
∴ + = Ex. One inlet pipe A is 5 times as fast as second inlet pipe
x 3 x 36
B and takes 32 min less than the inlet pipe B. When
36 × 4 will the cistern be full if both inlet pipes are opened
x= = 48 minute
3 together?
The time taken by slower pipe to fill the tank Sol. Here k = 5 and x = 32.
= 3×48 = 144 min.
 kx 
∴ Cistern will be full in =  2 min.
Method iI: by formula-  (k − 1) 
Both pipes fill the tank
5 × 32 20
 x  = min. = min.
=  (5 2 − 1) 3
 k + 1 

QUANTITATIVE APTITUDE 99
stportal.mahendras.org

EXERCISE
Q.1. Pipe A can fill a tank in 3 hrs while another pipe B can empty the same tank in 6 hrs. If both the pipes are opened
simultaneously. In what time will the tank be completely full?
(1) 4 hrs (2) 3 hrs (3) 2 hrs (4) 1.5 hrs (5) 6 hrs
Q.2. A vessel can be completely filled by a pipe in 16 hrs but due to a leak in the vessel, the vessel is now filled in 24
hrs. If the vessel is completely full, in what time the leak will empty the vessel completely.
(1) 48 hrs (2) 36 hrs (3) 44 hrs (4) 42 hrs (5) None of these
Q.3. Two pipes A and B can separately fill in a cistern15 and 10 minutes respectively and a waste pipe can discharge 7
litres per minute. If all the pipes are opened when the cistern is full, it in emptied in 2 min. How many litres does
the cistern hold?
(1) 21 ltr (2) 30 ltr (3) 40 ltr (4) 35 ltr (5) 45 ltr
Q.4. Two fill taps A and B can separately fill a tank in 45 and 40 min respectively. They started to fill tank together but
fill tap A is turned off after few minutes and fill tap B fill the rest part in 23 minutes. After how many minutes was
tap A turned off?
(1) 7 min (2) 8 min (3) 11 min (4) 15 min (5) 9 min
Q.5. Two pipes A and B can fill a tank in 15 hours and 20 hours respectively A third pipe C can empty the full tank in
25 hours. All the three pipes are opened in the beginning. After 10 hours, C is closed. Find in how much time will
the tank be full.
(1) 12 hrs (2) 8 hrs (3) 10 hrs (4) 14 hrs (5) 16 hrs
Q.6. Two taps A and B can fill a cistern in 15 min and 20 min respectively. Then both are turned on at the same time. If
the tap A is turned off after some time, after that cistern is filled in 12 min. After how much time tap A is closed?

3 3 2 1 3
(1) 4 min (2) 3 7 min (3) 5 min (4) 8 min (5) 7 4 min
5 5 9
Q.7. A, B and C can fill a tank in 6, 8, 12 hours respectively. If the pipes are opened in order at 10 am, 11 am, 12 am.
When will the cistern be filled? (approximately)
(1) 1 P.M. (2) 1.30 P.M. (3) 1.50 P.M. (4) 2 .00 P.M. (5) 2.20 P.M.
Q.8. The inlet to the tank can fill it in 4 hours while the outlet can empty it in 5 hr. Both the pipes were opened at 9 am
but after some time the outlet closed and it is observed that the tank was full at 5 pm. At what time was the outlet
closed?
(1) 12 P.M. (2) 2 P.M. (3) 3 P.M. (4) 8 P.M. (5) 6 P.M.
Q.9. Two pipes A and B can fill a tank in 30 min and 36 min respectively. Both pipes are open in the empty tank but due

5 9
to dust material present in the pipes A flows of original quantity and pipe B flows parts of original quantity.
6 10
31
After some time pipe is cleaned, then tank is filled in the min so after how much time pipes are ready to fill the
tank properly? 2

(1) 1 min (2) 2 min (3) 1.5 min (4) 5 min (5) 5.5 min
Q.10. A tank is filled by three pipes with uniform flow. The first two pipes operating simultaneously fill the tank in the
same time during which the tank is filled by the third pipe alone the second pipe fills the tank 5 hours faster than
the first pipe and 4 hours slower than the third pipe. The time required by the first pipe is:
(1) 11 h (2) 15 h (3) 4 h (4) 12 h (5) 18 h
Q.11. A tank is filled in 5 hours by three pipes P,Q & R. The pipe R is twice as fast as Q and Q is twice as fast as P. How
much time will pipe P alone take to fill the tank?
(1) 35 hr (2) 55 hr (3) 20 hr (4) 18 hr (5) 15 hr
Q.12. Three pipes P,Q and R can fill a tank in 6 hours. After working at it together for 2 hrs R is closed & P and Q can fill
the remaining part in 7 hours. The number of hours taken by R alone to fill the tank is-
(1) 10 (2) 12 (3) 13 (4) 14 (5) 16

100 QUANTITATIVE APTITUDE


stportal.mahendras.org
Q.13. 7 pipes attached with a tank out of which some are inlets and outlets. Every inlet can fill tank in 10 hours and every
30
outlet can empty tank in 15 hours when all the pipes are opened simultaneously the tank filled up hours. Find
the no. of inlets & outlets- 11

(1) 5,2 (2) 2,5 (3) 3,4 (4) 4,3 (5) None of these
Q.14. Two pipes A and B can fill a tank in 20 minutes and 40 minutes. If both pipes are opened simultaneously, after how
much time should B be closed  so that the tank is full in 18 minutes?
(1) 10 min (2) 9 min (3) 8 min (4) 7 min (5) 4 min
Q.15. Three pipes P, Q and R connected to a cistern. The first pipe (i.e) P can fill 1/2 part of the tank in one hour, second
pipe, Q can fill 1/3 part of the cistern in one hour. R is connected to empty the cistern. After opening all the three
pipes in a cistern which is 7/12 part filled. Then how much time required to empty the cistern completely?
(1) 2 hours (2) 3 hours (3) 4 hours (4) 5 hours (5) None of the Above
Q.16. In a tank there is a pipe which can be used for filling the tank as well as for emptying it. The capacity of the tank
is 1200 m³. The emptying of the tank is 10 m³ per minute higher than its filling capacity and the pump needs 6
minutes lesser to empty the tank than it needs to fill it. What is the filling rate of the pipe?
(1) 20 m³ / min. (2) 40 m³ / min. (3) 50 m³ / min. (4) 60 m³ / min. (5) None of the Above
Q.17. In what time would a cistern be filled by three pipes whose diameters are 1cm, 2 cm and 3 cm running together,
when the largest pipe alone can fill the tank in 21 minutes. The amount of water flowing through the pipe is directly
proportional to the square of its diameter?
(1)10.5 minutes (2) 11.5 minutes (3) 12.5 minutes (4) 13.5 minutes (5) None of these
Q.18. A pipe can empty a tank in 60 minutes alone. Another pipe whose diameter is twice the diam-
eter of first pipe is also opened. Now find the time in which both pipe will empty the tank together.
(1) 8 min (2) 10 min (3) 12 min (4) 14 min (5) None of These
Q.19. A fill pipe can fill the tank in 15 minutes and a drain pipe can drain the tank in 30 minutes. If a system of ‘x’ pipes
(includes both fill and drain pipes) fills the tank in 3 minutes. Find the possible value of ‘x’.
(1) 10 (2) 16 (3) 12 (4) 14 (5) None of these
Q.20. An inlet pipe can fill a tank in 8 hours. An outlet pipe can empty the same tank in 12 hours. Initially the tank is
completely empty. At this point of time, the inlet pipe is connected to the tank. After 3 hours, the outlet pipe is also
connected to the tank. After how much time from the beginning, will the tank be full?
(1) 15 hours (2) 18 hours (3) 20 hours (4) 24 houre (5) 22 hours

QUANTITATIVE APTITUDE 101


stportal.mahendras.org

EXERCISE Explanation
1 1 15 − 2 − 3
Q.1.(5) Part filled by pipe A in 1 hr. part + =−
3 C 30
1 1 −1 0
Part empty by pipe B in 1 hr part + =
6 C 30
Part empty by pipe (A+B) in 1hr
1 1
1 1 2 −1 1 =−
= − = = C 3
3 6 6 6
Full tank is empty in 3 min
Time taken by both pipes to fill the tank = 6 hr.
In 1 min ....... 7 lit is empty
AlternatIVE Method:
In 3 min ..... 7 × 3 = 21 lit
2 part/ hr 3 hr A
6 AlternatIVE Method :
3 part/ hr 6 hr B
2 part/m in 15 m in (A )
3 part/m in 30 10 m in (B )
1 part/hr ← A+B
15 pa rt/m in 2 m in (A +B +C )
6
Time taken to fill the tank = 1 = 6 hr 10 part/min C
30
1 C will empty full tank in = = 3 min
Q.2.(1) Part filled by inlet pipe A = 10
16
In 1 min ......... 7 lit is empty
1
Part filled by inlet pipe A+B = 2 4 In 3 min .... 7 × 3 = 21 lit.
where B is leakage. Q.4.(5) Let the tap A be turned off after x minutes
so accordingly ∴Tap B is opened for (x+23) minutes
1 1 1 x x + 23
+ = ∴ + =1
A B A +B 45 40
1 1 1 8 x + 9 x + 207
+ = =1
16 B 24 360

1 1 1 1 −2 1 17x = 153, x=9 min


= − , = =–
B 24 16 B 96 48 AlternatIVE method :
Time taken by B to empty the tank= 48 hrs part/m in 8 45 m in (A )
360
AlternatIVE Method : part/m in 9 40 m in (B )

6 part/ hr 16 h r (A )
96 A+B B
24 h r (A +B ) – 2 3 m in –
4 part/ hr
B worked for 23 mins
-2part/hr B
Work done by B in 23 mins = 23 × 9= 207parts
96
Time taken by B to empty the tank= = 48 hr Remaining part = 153 parts
2
Q.3.(1) According to question 153
A+B = = 9 min
1 1 1 1 17
+ − =
A B C A +B +C Pipe A was turned off after 9 min

 1 1 1  23
1 1 1 1 Q.5.(1) Tank fill in 10 hrs =  + − =
+ + =− 15 20 25  30
15 10 C 2
 23  7
1 1  1
+ = − − +
1  Remaining part =  1 − 3 0  = 3 0
 
C 2  1 5 1 0 
work done by (A+B) in 1 hr
102 QUANTITATIVE APTITUDE
stportal.mahendras.org

 1 1  7 AlternatIVE method:
15 + 20  = 60 4 l/m A 1 5 m in
 
60l
3 l /m B 2 0 m in
7
Now part filled by (A+B) in 1 hr.
60 Let after x min tap A is closed then according to
question
7
∴ part is will be filled by (A+B) in x m in A B 1 2 m in
30
A+B 60l
 60 7 
=  ×  hrs =2 hours 7x + 3 × 12 = 60
 7 30 
7x = 60 - 36 = 24
∴ Total time = 10+ 2 = 12 hours
24 3
AlternatIVE method : x= = 3 min
7 7
20 part/hr 15 hrs (A ) Q.7.(2) Since A,B & C are opened in order at 10 am, 11
15 part/hr 300 20 hrs (B )
am & 12 am
12 part/hr 25 hrs(C )
Hence,
C
1
A+B+C A+B Water filled by pipes A in 1 hour = liter
– 10 hr – 6
Again,
10×(20+15-12) Water filled by pipes A & B in next 1 hour
Work done by A+B+C in 10 hr = 10 × 23 1 1 
 6 + 8  liter
= 230 parts  
Remaining part = 300 - 230 = 70 4 +3 7
= = liter
24 24
70
Remainig part filled by A + B in = = 2 hr and water filled by pipes A,B & C in 1 hours
35
Total time = 10 + 2 = 12 hrs 1 1 1 4 +3 +2
= + + =
1 6 8 12 24
Q.6.(2) Water filled by tap A in 1 min =
15 9
= l
24
1
Water filled by tap B in 1 min = Now remaining water to be filled by A,B & C
20
together
Water filled by tap A and tapB in 1 min
Hence
1 1 4 +3 7
= + = = 1 7 
15 20 60 60 1−  + 
6 24 
Remaining time =
According to question, 9
24
Let after x min pipe A is closed
4 + 7  11
7x 1 1−   1−
So, +12 × =1  2 4  = 24 = 13 × 24 13
60 20 = = hours
9 9 24 9 9
7x 3 24 24
+ =1 4
60 5 = 1 hours
9
7x 3 2 cistern will be filled
=1− =
60 5 5 that means at 12 + 1.5 = 1 : 30 pm
2 60 Q.8.(2) + 5 l/h A + 4h
x= × 20
5 7 - 4 l/h B - 5h
QUANTITATIVE APTITUDE 103
stportal.mahendras.org
Given total time = 9 am to 5 pm = 8h 1 2 4 1
So, + + =
Since both pipes are opened at 9 am x x x 5
Let of tap 't' time B pipe closed then 7 1
=
t+(8-t) × 5 = 20 x 5

t+ 40 - 5t = 20
x = 35 hours
4t = 20
2 1
t = 5h Q.12.(4) Part of tank filled in 2 hr = =
6 3
Hence
1 2
Required answer = 9 am + 5 hours = 2 pm Remaining capacity = 1 − =
3 3
Q.9.(1) 6 l/m A 3 0 m in
180 2
5 l/m B 3 6 m in therefore, (P+Q)'s work =
3
According to question
2
5 (P+Q)'s 1 hour work =
efficiency of A = 6 × =5 l/m 21
6
Therefore, R's 1 hour work
9 = (P+Q+R)'s 1 hour work
effienciency of B = 5× = 4.5 l/m
10
1 2 = 1
efficiency of A + B = 9.5 l/m (P+Q)'s 1 hour work = −
6 21 14
again therefore R alone will fill the tank = 14 hours
Let after x min dust particle is cleaned Q.13.(1) + 3 A (In le t ty p e ) + 1 0 m in
So, -2 B (O u tle t ty p e ) -1 5 30

31 30
9.5x + 11 × = 180
2 But tank fill in 1 1 hrs
9.5 + 170.5 = 180 30
So, work done in one hour = = 11
9.5x = 180 - 170.5 30
11
9.5x = 9.5
With these total pipes
x = 1 min
Total inlets = (5) = 5 × 3 = 15 w/h
Q.10.(2) According to question
Total outlets = (2) = 2 × (2) = -4 w/h
Let the first pipe alone takes x hours to fill the
11w/h
tank then second pipe takes
Q.14.(5)   A fill the tank in 1 minute (20×2 = 40)
= x - 5 third pipe takes = x - 9
= 2 units
1 1 1
So, + = B fill the tank in 1 minute (40×1 = 40)
x x −5 x −9
=  1units
x −5 + x 1
= For 18 min(A) = 18×2 = 36 units
x (x − 5 ) x − 9
Remaining = 40 – 36 = 4 units
(2x - 5) (x-9) = x (x-5) Time for B to be closed  so that the tank is full in
x - 18x + 45 = 0
2
4
(x-15) (x-3) = 0 18 minutes = = 4 min
1
x = 15 hours. Q.15.(3) In 1 hour, P can fill = 1/2 Part
Q.11.(1) Suppose pipe p alone takes x hour to fill the tank Time taken to fill the Cistern by Pipe P = 2
hours
x x In 1 hour, Q can fill = 1/3 Part
Then pipes Q & R will take & hours
respectively 2 4 Time taken to fill the Cistern by Pipe Q = 3
hours
104 QUANTITATIVE APTITUDE
stportal.mahendras.org
[1/2 + 1/3 – 1/R] = 7/12 Q.19.(4) In a given time, one fill pipe cancels out two drain
1/R = 1/4 pipes, and we require 5 more fill pipes to fill the
Time required to empty the Cistern = 4 hours tank in 3 minutes. So the number of pipes would
Q.16.(2) 1200/x – 1200/(x+10) = 6 be in the form of 3x+5.
200/x – 200/(x+10) = 6 Among the given options only ‘14’ is in the form
x2 + 10x – 2000 = 0 of 3x+5.
x = 40 So the correct option to choose is 4.
Q.17.(4) More the diameter more will be the water flowing Q.20.(2) Let the volume of the tank be 48. The inlet pipe
through it and less will be the time taken. fills 6 litres in 1 hour
Means bigger pipe will take less time to fill the while the outlet pipe empties 4 litres in 1 hour.
tank
For the first three hours only inlet pipe is
21× 32 21× 9 connected, so it will fill
t= 2 =
1 + 22 + 32 14 = 6×3 = 18 litres in those 3 hours.
So total time = 13.5 minutes
Now after this both the pipes are operating
Q.18.(3) Time taken by pipe to empty the tank is inversely simultaneously.
proportional to cross-sectional area.
So, effectively only 2 litres will be filled in 1
So, time taken by second pipe to empty hour. We have 30 litres to fill. Hence time
= 60/4 = 15 min required = 30/2
Both pipe can empty the tank = 15 hours.
1 1 1 Total time from the beginning to fill the tank
= + = = 12 min
60 75 12 = 3 + 15 = 18 hours

QUANTITATIVE APTITUDE 105


stportal.mahendras.org

CHAPTER
Speed, Time AND
13
Distance
Ex. In covering a certain distance the speed of A and B are
Speed
in the ratio of 3 : 4. A takes 30 minutes more than B
The distance covered in a unit time interval is known as speed. to reach the destination. The time taken by A to reach
It is obtained by dividing the distance covered by an object the destination is-
by the time it takes to cover that distance.
Sol. Let the distance of destination be D km
D is ta n ce T ra v e lle d
Speed (S) = Let the speed of A= 3x km/hr.
T im e T a k e n
The speed of B = 4x km/hr.
Distance (d) = Speed ×Time
According to the question
D is ta n ce
Time (t) = D D 1
Speed − = 30 =
3x 4x 2
Unit conversion :
1. km/hr to m/sec conversion: D 1 D
= , =6
12x 2 x
F 5I
A km/hr = G A × J m/sec. D 6
H 18 K Time taken by A = =
3x 3
= 2 hours.
2. m/sec to km/hr conversion:
short Trick :
FG
A m/sec = A ×
18 IJ Speed ratio = 3 : 4

H 5 K km/hr.
Time ratio = 4 : 3
Ex. A plane is moving with the speed of 180 km/hr. Its
1 = 30 min
speed in metre per second is-
4 = 120 min =2 hours
5 Ex. If a person walks at 14 km/hr instead of 10 km/hr, he
Sol. 1 8 0 × = 5 0 m/s
18 would have walked 20 km more. The actual distance
travelled by the person is:
Important Points :
Sol. Let the actual distance travelled be x km.
1. If the time taken is constant, then the distance travelled
is directly proportional to the speed i.e. more speed, x x + 20
Then, = ⇒ 14x = 10x + 200
more distance can be travelled in the same time. 10 14
⇒ 4x = 200 ⇒ x= 50km.
S ∝ d → at constant time
short Trick :
2. If the speed is constant, the distance travelled is 20
directly proportional to the time taken, i.e. more the (14 - 10) =
T
distance travelled, more the time taken at the same
speed. T = 5 hrs.
Distance = 10 × 5 = 50 km
d ∝ t → at constant speed
Ex. Excluding stoppages, the speed of a bus is 54 kmph
3. If the distance travelled is constant, the speed is and including stoppages, it is 45 kmph. For how many
inversly proportional to the time taken, that is more minutes does the bus stop per hour?
speed, less the time taken for the same distance to be Sol. Due to stoppages, it covers 9 km less.
travelled.
Time taken to cover 9 km

S∝
1
→ at constant distance FG 9 × 60 IJ min = 10 min
t = H 54 K
106 QUANTITATIVE APTITUDE
stportal.mahendras.org
Relative Speed = s1-s2 (s1 >s2)
Average Speed
The average speed of an object is a measure of the distance A
←
s k m /h r 
covered by that object in a set period of time. 1

T o ta l D ista n ce B
Average Speed = ←
T o ta l T im e T a k e n s k m /h r 
2

(a) If A covers a distance d1 km at S1 km/hr and then
d2 km at S2 km/hr. then the average speed during Relative Speed = s1-s2 (s1 >s2)
whole journey is given by II. If two objects are travelling in the opposite direction
at s1 km/hr and s2 km/hr, respectively, then s1 +s2 is
S 1S 2 ( d 1 + d 2 )
Average Speed = called their relative speed.
S 1d 2 + S 2 d 1
A B
(b) A person goes certain distance (A to B) at speed 
s 1 km /hr
→ ←
s 2 km /hr

of S1 and return (B to A) at speed of S2 km/hr. If
he takes T hrs in all, then Relative Speed = s1+s2
2S 1 × S 2 A B
Average speed = ←
s k m /h r   →
S1 + S 2 1 s2 km / hr

Distance between A and B is
Relative Speed = s1+s2
 S1 × S 2  Ex. A thief is noticed by a policeman from a distance of
 t × S + S  200 m. The thief starts running and the policeman
1 2
chases him. The thief and the policeman run at the
(c) If a person travelled three equal distance by three rate of 10 km/hr and 11 km/hr. respectively. What is
different speed S1, S2 and S3 then the distance between them after 6 minutes.
3S 1 × S 2 × S 3
Average speed = Sol. P 11 km /hr. T
10km /hr.
S 1S 2 + S 2 S 3 + S 3 S 1
Ex. A man goes from place A to B at a speed of 12 km/ 200 m
hr. and return at the speed of 18 km/hr. The average Relative speed of police= 11-10 = 1 km/hr.
speed for the whole journey is- 5
2 ×12 ×18 2 ×12 ×18 = m/sec
18
Sol. Average Speed =
(1 2 + 1 8 ) = 3 0 ∴ Distance decreased in 6 minutes
2 5
= 14 km/hr. = × 6 × 6 0 = 100 m
5 18
Ex. One-third of a certain journey is covered at the rate of ∴ Distance remained between them
25 km/hr. one fourth at the rate of 30 km/hr. and the
= 200-100 = 100 m
rest at 50 km/hr. The average speed for whole journey.
Sol. Let total journey = x km Important Points
So, according to the question a
I. If the new speed is of the original speed, then the
x x 5x 3x b
Total time = + + =
3 × 25 4 × 30 12 × 50 1 00 change in time taken to cover the same distance is
given by
x 1
Average Speed = = 3 3 km/hr.
3x 3 b
Change in time =  -1 ×o rig in a l tim e
100 a 
Relative Speed :
II. If a man travels at the speed of x km/hr. reach t1 hr.
I. If two objects are travelling in the same direction at s1
late and if he travels at the speed of y km/hr. reach t2
km/hr. and s2 km/hr respectively such that s1 >s2 then
hr. early the travel distance.
s1-s2 is called the relative speed.
xy
 A
s 1 km /hr
→  B
s 2 km /hr

y −x
(t1 + t 2 )k m

QUANTITATIVE APTITUDE 107


stportal.mahendras.org
III. If two persons A and B start at the same time from x 1
two points P and Q towards each other and after =
20 5
crossing they take T1 and T2 hrs in reaching Q and P
respectively, then x= 4 km
A 's S p e e d T2 short Trick :
=
B ' s Speed T1 4 × 5 12
Distance = × = 4k m
1 60
3
Ex. If a man travels th of his original speed then he
4
Ex. Two places A and B are placed at a distance of 150
reaches 20 minutes late. Find his actual time : km on a highway. A car moves from A and another car
Sol. Let the actual speed be x km/hr and the actual time be moves from B at the same time. If both car travelled in
t minutes same direction with the different speed then they meet
after 5 hours. If they travelled in opposite directions
1 1 20 then they meet in 1 hour speed of the fast car is-
− =
3 x x 60
Sol.
4
When they are travelled in same direction (x>y)
4x 1 1
− = then (x-y) × 5 = 150
3x x 3
⇒ x - y = 30 ......... (i)
x = 1 km/hr.
When they are travelling in opposite directions
1
Actual time = = 1 hr. = 60 min. (x + y) = 150 ....... (ii)
1
Adding eq. (i) and (ii)
short Trick : 2x = 180
3 x = 180/2
Actual time = × 20
( − 3)
4 x = 90 km/hr.
= 60 min. Ex. Ram arrives at a Bank 15 minutes earlier than shedules
time if drives his car at 42 km./hr. He arrives 5 minutes
Ex. Ritu goes at the speed of 4 km/hr. reaches her school late if he drives his car at 35 km./hr. The distance of
6 minutes late and next day she goes at the speed of the Bank, from his starting point is:
5 km/hr. reaches her school 6 minutes earlier find the
Sol. Difference in time = (15+5) minutes = 1/3 hour
distance between her home and school ?
∴ Required distance
Sol. Let Distance = x
42 × 35 1
= × = 7 0 km.
x x 12 7 3
So, − =
4 5 60

108 QUANTITATIVE APTITUDE


stportal.mahendras.org

EXERCISE
Q.1. A bus moves 270 km at a speed of 45 km per hour and there after increases its speed upto 60 km/h. to travel another,
480 km. Find the average speed of the bus.
4 4 3 2 8
(1) 53 km/hr. (2) 55 km/hr (3) 58 km/hr. (4) 51 km/hr. (5) 47 9 km/hr.
7 7 7 8
Q.2. Walking 5/7 of his usual rate a boy reaches his school 6 min late. Find his usual time to reach school.
(1) 15 minutes (2) 12 minutes (3) 18 minutes (4) 20 minutes (5) 22 minutes
3
Q.3. Two friends x and y walk from A to B at a distance of 78 km at 1.5 km per hr and 1 per hour respectively. y
4
reaches B, returns immediately and meet x at C. Find the distance from A to C.
(1) 75 km (2) 90 km (3) 72 km (4) 80 km (5) None of these
Q.4. A person leaves a place A to place B at 6 am and reaches place B at 10 am. Another person leaves B at 8 am and
reaches A at 11.30 am. They will meet each other at.
(1) 8:56 am (2) 7:45 am (3) 8:20 am (4) 8:40 am (5) None of these
Q.5. It takes 4 hours for a 300 km journey of 60 km is covered by train and the rest by car. it takes 20 min more if 100
km is covered by train and the rest by car, the ratio of the speed of train to that of the car is-
(1) 4 : 3 (2) 3 : 4 (3) 2 : 3 (4) 3 : 2 (5) 5 : 3
Q.6. Ram and Shyam starts simultaneously from point A towards B with speed of 7.5 m/sec and 33 km/hr respec-
tively. When one of them reaches B, the person has to return back towards A immediately. If he meet the other
person after 2 hours of total journey. Find the distance between point A and B.
(1) 120 km. (2) 100 km. (3) 60 km.
(4) 50 km. (5) Cannot be determined
Q.7. Two trains P and Q start from Delhi and Mumbai towards Mumbai and Delhi respectively. After passing each
other they take 19 hours 30 minutes and 8 hours 40 minutes to reach Mumbai and Delhi respectively. If the train
from Delhi is moving at 80 km/hr then find the speed of the train from Mumbai?
(1) 110 kmph (2) 100 kmph (3) 90 kmph (4) 120 kmph (5) None of these
Q.8. A bus crosses a certain distance with out stoppage with the speed of 60 km/hr. The bus covers the same distance
with the stoppage speed of 40 km/hr. What is the time of stoppage of the bus per hour (in minutes)
(1) 20 (2) 15 (3) 12 (4) 40 (5) 18
Q.9. A monkey climbs 20 m. in a minute and slips down 6 m in next minute How much time he will reach upto a 60 m
high pole in this way.
(1) 6 min 59 sec (2) 6 min 54 sec. (3) 6 min 58 sec (4) 6 min 50 sec (5) 6 min 45 sec.
Q.10. A bus started from A to B then after covering two third distance some fault occurs. Driver mentioned the fault quickly
and bus will move again. But now the speed of the bus is reduced by one third and for this reason bus reaches at B
late by 1 hour what was the actual time in going B from A.
(1) 6.30 hr (2) 7 hr (3) 6 hr (4) 6.45 hr (5) 7.30 hr
Q.11. A bus from A to B started with the speed of 60 km/hr. at 6 am another bus from A to B started with the speed of 80
km/hr at 8 am. At which time will second bus catch the first bus ?
(1) 2 p.m. (2) 1 p.m. (3) 12 a.m. (4) 3 p.m. (5) 2.30 p.m.
Q.12. A man travels with the speed of 40 km/hr. then he reaches the particular place 10 min late but if he travels with the
speed of 50 km/hr. then he reaches 5 minutes before the stimulated time. Then how much time will he take to reach
the place.
(1) 1 hr. 10 min. (2) 1 hr. 15 min. (3) 1 hr. 12 min. (4) 1 hr. 18 min. (5) 1 hr. 5 min.
Q.13. A bus started its journey from Ramgarh and reached Devgarh in 88 minutes with its average speed of 100 km/hr.
If the average speed of the bus is increased by 10 km/hr. How much time will it take to cover the same distance?
(1) 60 minutes (2) 70 minutes (3) 75 minutes (4) 80 minutes (5) 72 minutes
QUANTITATIVE APTITUDE 109
stportal.mahendras.org
Q.14. A faster train takes 2 hrs less than a slower train in a journey of 192 km. If the average speed of the slower train is
16 km/hr less than that of the faster train then the average speed of the faster train is-
(1) 36 km/hr. (2) 64 km/hr. (3) 36 km/hr. (4) 48 km/hr. (5) None of these
Q.15. Two guns were fired from the same place at an interval of 12 minutes. A person approaching the firing place in his
car, hears the second shot 11 minutes after the first. If the speed of sound is 330 m/s, find the speed of the car.
(1) 50 km./h. (2) 72 km./h. (3) 90 km./h. (4) 108 km./h. (5) None of these
Q.16. A man covered a certain distance at some speed. Had he moved 3 km per hour faster, he would have taken 40 minutes
less. If he moved 2 km per hour slower, he would have taken 40 minutes more. The distance (in km) is:

(1) 20 (2) 35 (3) 36 (4) 40 (5) None of these


Q.17. A thief running at 8 km/hr is chased by a policeman whose speed is 10 km.hr. If the thief is 100 m ahead of the
policeman, then the time requiared for the policeman to catch the thief will be-
(1) 2 min (2) 3 min (3) 4 min (4) 6 min (5) None of these
Q.18. A person started his journey in the morning. At 11 a.m., he covered 3/8 of the journey and on the same day at 4.30
p.m., he covered 5/6 of the journey. He started his journey at:
(1) 6 a.m. (2) 5.30 a.m. (3) 7 a.m. (4) 6.30 a.m. (5) None of these
Q.19. Two guns were fired from the same place at an interval of 10 minutes and 30 seconds but a person in the train ap-
proaching the place hears the second shot 10 min. after the first. Find the speed of the train supposing that sound
travels at 330 m/s.
(1) 111.8 kmph (2) 59.4 kmph (3) 58.6 kmph (4) 19.8 kmph (5) None of these
Q.20. In a race of 600 m. A can beat B by 50 m. and in a race of 500 m, B can beat C by 60 m. By how many metres
will A beat C in a race of 400 m ?
(1) 16 m (2) 50 m (3) 77 m (4) 84 m (5) None of these
Q.21. Two cars 'M' and 'N' are running in the same direction, car 'M' had already covered a distance of 60 Kms. when Car
'N' started running. The cars meet each other in 3 hours after Car 'N' started running. What was the speed of Car
'M'?
(1) 40 kmph (2) 45 kmph (3) 60 kmph (4) Data inadequate (5) None of these
Q.22. Sandeep, Deepak and Naimish drive with a speed of 30km/hr, 40 km/ hr and 60 km/hr respectively. If they all start
from a same point but not together, Deepak start two hours after Sandeep and cross Sandeep at the same instance
as Naimish does. After how many hours of Sandeep Naimish start?
(1) 6 (2) 4 (3) 8 (4) 3 (5) 2
Q.23. P, Q and R starts from the same place and travel in the same direction at speeds 30 km/hr, 40 km/hr and 60 km/hr
respectively. Q starts 2 hours after P, but Q and R overtakes P at the same instant. How many hours after P did R
start?
(1) 1 (2) 2 (3) 4 (4) 6 (5) 8
Q.24. A car and a truck start travelling towards each other from city A and city B respectively. After crossing each other
car takes 6 hours 40 minutes to reach at B and truck takes 3 hours and 45 minutes to reach city A. If speed of truck
is 36 Km/hr. then find the distance between city A and B.
(1) 225 Km (2) 320 Km (3) 350 Km (4) 315 Km (5) None of these
Q.25. After travelling 150 km car met with an accident and its speed reduced by 25% hence reached 45 min late to its
destination. Had this accident occoured 60 km after, it would have reached its destination only 30 minutes late.
Find the length of total journey.
(1) 300 Km (2) 270 Km (3) 330 Km (4) 420 Km (5) None of these

110 QUANTITATIVE APTITUDE


stportal.mahendras.org

EXERCISE Explanation
Q.1.(1) Here distance speed are known. Time is not 14
known we use relation. Required time = h r = 5 6 m in u te
7 +8
270
⇒ = 6 hr. So required answer = 8 : 56 am
45
Q.5.(1) Let the speed of the train be x km/hr and car be
480 y km/hr.
⇒ = 8 hr.
60 60 240
Then + = 4 hr.......(I)
x y
Total time = 14 hr.
750
Total distance ⇒ and 1 0 0 + 2 0 0 = 2 5 ...... (II)
14 x y 6
4
= 53
7
km/hr. 6 0 y + 2 4 0 x 6 (1 0 0 y + 2 0 0 x )
∴ =
4 25
Q.2.(1) Speed 5 : 7
Time 7 : 5 = 250y + 1000x = 400y + 800x

Speed and time are proportional to each another. 20x–16x=8y–5y

Difference in time 2 = 6 4x = 3y

1 = 3 minutes x:y=3:4

3 × 5 = 15 minutes Q.6.(3) Let the distance between A and B = D


Total distance covered by them = 2D
Q.3.(3) 18
Relative Speed = 7.5 × + 33 = 60
5
Time = 2 hrs.
2D = 60 × 2
When y meets x at C. y has walked the distance D = 60
AB + BC and x has walked the distance AC. t1
Q.7.(4) Speed of train Q = Speed of train P × t2
So, both x and y have walked together a distance
= 2 × AB = 2×78 = 156 km
39 / 2
The ratio of speed of x and y is 1.5 : 1.75 Speed of train Q = 80 ×
26 / 3
So that Speed of train Q = 80 × 3/2 = 120 kmph
150 : 175 = 6 : 7 Q.8.(1) x = first speed without stoppage
Hence the distance travelled by y = speed with stoppage
6 x −y
x = AC = ×156 stoppage time =
6 +7 x
60 − 40
6 =
× 1 5 6 = 72 km. 40
13
Q.4.(1) Ist 6 am.................. 10 am 20 1
= ⇒ × 6 0 = 2 0 minute
8 am ................. 11.30 am 60 3
Q.9.(2) Height covered in first minute= 20 metre
Decrease in height in next minute
= 6 metre
So, monkey climbs up (20 - 6)m in 2 minutes
Time = 4 : 3.5 = 8 : 7 14×3 = 42 m.
Speed = 7 : 8, Distance = 7×4=28 Monkey climbs up 42 m in 6 min
Distance Travelled by first person till 8 A.M. = 14 2×3 =6 min
QUANTITATIVE APTITUDE 111
stportal.mahendras.org
Remaining height = 60 - 42 = 18 192 192
⇒ = −2
20 m. climb up in 60 sec. x x −16

60 x = 48 km/hr.
18 m climb up in = × 1 8 = 54 second.
20 Q.15.(4) The person hears the second fire in 11 minutes
6 min + 54 sec = 6 min 54 sec instead of 12 minutes since the car is moving
towards the firing place.
1
Q.10.(3) Let Speed= 1- = 2/3, Speed = 2 : 3 ∴ The distance travelled by car in 11 minutes is
3
equal to distance covered by sound in 12–11=1
Time = 3 : 2 minutes.
⇒ 1 = 1 × 2 = 2 hr. ∴ Distance covered by car in 1 second
2 1 330 × 60
Distance 1- ⇒ = 2 hr. = =30 m.
3 3 11 × 60
So, total time is = 6 hr. 18
∴ Distance convered by car in 1 hour = 3 0 ×
Q.11.(1) = 108 km./hr. 5

Q.16.(4) Let total distance be D km and the normal speed


A fter 2 hrs.
be x km/h.

D D 2 D D 2
Then – = and – =
x x +3 3 x–2 x 3
Time difference = 8 - 6 = 2 hrs
D D D D 1 1 1 1
Distance moved by first bus before the second ⇒ –
x x +3
= –
x–2 x
⇒ –
x x +3
= –
x–2 x
bus starts]
= 60×2 = 120 km (x + 3 ) – x x – (x – 2) 3 2
⇒ = ⇒ =
x (x + 3 ) x (x – 2 ) x (x + 3 ) x (x – 2 )
Relative speed= 80 - 60 = 20 km/hr.
120 ⇒ 3(x–2)=2(x+3) ⇒3x–6=2x+6
Time = 2 0 = 6 hr. ⇒x=12 (km/hr.)
Time taken by second bus to catch first train- 8 Now, D – D = 2 ⇒ D = 2 ⇒ D = 2 × 6 0 = 4 0 km.
a.m. + 6 = 2 p.m. 12 15 3 60 3 3
Q.17.(2) Policeman gains = (10–8) km/hr. = 2 km/
Q.12.(5) Let he takes x minutes to reach at exact time hr. ⇒ He will gain 100 metre in 3 minutes.
50 (x - 5) = 40 (x + 10) Q.18.(4) Journey covered between 11 a.m. and 4.30 p.m.
50x - 250 = 40x + 400 5 3 11
= – =
6 8 24
10 x = 650 1
x = 65 Time taken = 4.30 p.m.–11 a.m. = 5 hours
2
Exact time is 1 hr. 5 minutes. 11
= hours.
Q.13.(4) Distance between Ramgarh and Devgarh 2

100 × 88 440 11 11
= = km ⇒ of journey is covered in = hours.
60 3 24 2
New speed = 110 km/hr. 3
∴ of journey is covered in
D is ta n ce 8
Required time =
Speed
440 60 11 24 3 9 1
= × = 80 minutes × × = hours = 4 hours
3 110 2 11 8 2 2
Q.14.(4) Let the speed of the faster train be x km/hr. 1
Journey was started at 11 a.m.– 4 hours
∴ Speed of the slower train 2
= 6.30 a.m.
= (x - 16) km/hr.

112 QUANTITATIVE APTITUDE


stportal.mahendras.org
Q.19.(2) Speed of train So Naimish start after 2 hours of Deepak and 4
D istance T ravel by sound hours (2+2) of Sandeep
= T im e taken by train to travel sam e distance
Q.23.(3) Let the time taken by P when he was overtaken
330 x 30 by Q and R be T hours.
= Time taken by Q
10 x 60
= 33 m / s = T – 2 hours
2 Since, distance travelled is same by the time Q
= 33 x 18 km / h overtakes P
2 5
30(T)= 40(T-2)
= 59 .4 km / h T = 8 hours

Q.20.(5) In race of 600m. Distance travelled by P = 30×8

A travels 600m. = 240 km 


R takes 4 hours to cover a distance of 240 km.
B travels 550 m. So, he starts 8 - 4 = 4 hours after P.
in race of 500m.
Q.24.(4) SC t
B travels 500 m. = B
SB tC
C travels 440 m.
∴ in race of 400 m.
SC 225 3
= =
440 SB 400 4
C travels = x 400 = 352m.
500
now 4=36   so   3=27
A travels = 400m. required distance
A beat C by = 400 - 352 = 48m. 20 15
= 27 × + 36 × = 315 Km
Q.21.(4) Data inadequate 3 4
We only find the ratio of the speed of both cars Q.25.(3) 3
but we can't find the actual speed. S 4 S

Q.22.(2) Let t be the time taken by Deepak to cross Sandeep 150 60

30×(t+2)=40×t Speed = 4 : 3, Time = 3 : 4


t = 6 hours Difference 1 = 45 So, 3 = 135 m.
Let T be the time taken by Naimish to cross Again Difference 1 = 30 So, 3 = 90 m.
Sandeep with Deepak 
In 45 minute car travel 60 km
40×6 = 60×T
So Speed = 80 km
T = 4 hours
Required Answer =150+ 2.25×80 = 330 km

QUANTITATIVE APTITUDE 113


stportal.mahendras.org

CHAPTER
PRoblemS
14 on Trains
This type of questions are totaly based on the concept of III. Concept of Time : Time is always constant (The time
time & distance & speed. Mainly problems based on trains which is taken by object I to cross object II)
have two objects.
Some important points
1. Object I: Those object who cross another object is
considered as object I (primarly trains are considered 1. When a moving train crosses a standing person i.e.
as Ist object but some times person also considered L1
as 1st object when person crosses another object like S1 =
T
stationary train or platform)
Ex. train, bus, person etc. Ex. Find the time taken by a train 150m long, running at
54 km/h in crossing an electrical pole.
2. Object II : Those object who are crossed by object I
train, platform, over bridge, tunnel, person, pole tree, L
Sol. S = (because pole has no breadth)
wall etc. T
General Rule L 1 5 0m 150
T= = 54 × 5 = = 10 sec
Concept of speed : S 15
m/s
18
I. When one object is stationary and another is still then
only speed of moving object is considered as S1=x (let) 2. When a moving train crosses a moving person in the
(S2 =0) same direction
(1) When both object are moving in same direction L1
S1 − S 2 = and L2 = 0
In this case relative speed is difference between T
speed of both objects that means relative speed Ex. A train 180m long is running at a speed of 65 km/h.
= S1 ~S2 In what time will it pass a man who is running at a
speed of 5 km/h in the same direction ?
A B

s k m /h r → 
s k m /h r → L1
1 2
Sol. S1 - S2 =
T
Relative Speed = s1-s2 (s1 >s2)
L1 180
T= = m/s= 10.8 sec
A B S1 − S 2 5
←
s1 k m /h r  ←
s 2 k m /h r  60 ×
18
(2) When both object are moving in opposite 3. When a moving train crosses a moving persons in the

direction. opposite direction

In this case relative speed is sum of both speed L1


S1 + S 2 =
of both object means relative speed = S1 + S2 T
B Ex. A train 150metres long is moving at a speed of 85
 A
s 1 km /hr
→ ←
s 2 km /hr
 km/hr. It will cross a man coming from the opposite

direction at a speed of 5 km./hr. in:
Relative Speed = s1+s2
Sol. According to question
A B
←
s1 k m /h r  

s2 km / hr S1 + S 2 =
L1
T
II. Concept of distance: Distance is always added of 5 150
both object with irrespective of direction but in some (8 5 + 5 ) × =
18 T
exception are there when any vertical object is taken
then distance of that object is considered as zero. T = 6 sec.
Ex. Length of pole, person and tree is zero. 4. When a moving train crosses a platform:
114 QUANTITATIVE APTITUDE
stportal.mahendras.org
L1 + L 2 Sol. Let the speed of each train be x m/sec.
S1 = , S2 = 0
T 120 + 120
Then, = 12
Ex. A train 140 m long is running at 60 km/h. In how x+x
much time will it pass a platform 260m long? 20  20 18 
∴x= m/sec. =  ×  km/hr.
L1 + L 2 L +L2 2  2 5 
Sol. S = ,t= 1
t s = 36 km/hr.
8. When a moving train crosses a person sitting in another
140 + 260 train:
=
5
60 × L1
18 S1 ± S 2 =
T
= 24 sec Ans.
L2 = 0
5. When a moving train crosses another stationary train:
S1+S2 (according to direction)
L +L2
S1 = 1 Ex. A goods train 150 m. long moving with speed 54 km/h.
T A man is sitting in a passanger train which is moving
S2 = 0 with 18 km/hr. in same direction. In how many seconds
the goods train crosses the man sitting in a passanger
Ex. A train 180 m long taken 20 sec. in crossing a train ?
stationary train 220 m long. The speed of the train
Sol. Let the length of the goods train is L.
is - (in km./h.)
L
L1 + L 2 180 + 220 400 S1 - S2 = (because man has no breadth)
Sol. S = = = t
t 20 20
5 1 5 0m
18 (54 - 18) =
= 20 × 5 = 72 km./h. 18 t
6. When a moving train crosses another moving train 5 1 5 0m
along same direction: ×36 =
18 t
L1 + L 2
S1 − S 2 = t = 15 sec
T
9. When a moving train crosses two different object:
Ex. Two trains travelling in the same direction at 54 km/
hr. and 36 km/hr. completely pass one another in 1 Ex. A train running at a speed of 30 km/h takes 18 sec to
minute. If the length of the first train is 125 metres, pass a railway platform and it takes 15 sec to pass a
the length of the second train is: man who is running at a speed of 6 km/h in the same
direction. Find the length of platform ?
Sol. According to question
Sol. Let the length of the train is L.
L + L2
S1 − S 2 = 1 and the length of the platform is P.
T
L +P
5 125 + L 2 S1 =
(5 4 − 3 6 ) × = t
18 60
5 L +P
5×60 = 125+L2 × 30 = 18
18
L2= 175 metres.
L + P = 150 ---- (I)
7. When a moving train crosses another moving train
Again
coming from opposite direction:
L1 + L 2
L
S1 + S 2 = S1 - S2 =
T T

Ex. Two trains are running in opposite directions with the 5 L
× ( 30 - 6) = 1 5
same speed. If the length of each train is 120 metres 18
and they cross each other in 12 seconds, the speed of
L = 100 meters
each train is :
QUANTITATIVE APTITUDE 115
stportal.mahendras.org
Length of train L = 100 m
40 FG 10 × 5 IJ = 5
Length of platform = 150 - 100 ⇒
Y ' s rate
=
H 3 24 K 6
= 50 m.
Ex. A train travelling at 36 km/hr. completely crosses
F 40 × 6 IJ km/hr. = 48 km/hr.
∴Y’s rate = G
another train, having half its length and travelling in
H 5 K
the opposite direction at 54 km/hr. in 12 seconds. If it Ex. A train starts from x and travel towards y at 10:00 am.
Another train B start from y and travel towards x at
1 11:00 am if they reach their destinations at 2:00 pm
also passes a railway platform in 1 minutes, the
2 and 5:00 pm. At what time will they meet?
length of the platform is: Sol. Time taken by A = 4 hour, Time taken by B = 6 hours.
Sol. Relative speed = (36+54)km/hr. = 90 km/hr. Let the distance = 4×6 = 24 km
and speed of A and B are 6 km/h & 4 km/h

FG
= 90 ×
5 IJ
m/sec. = 25 m/sec. 2 4 km
H 18 K A B
Let l metre be the length of slower train. 6 4

Fl+ l I Remaining distance = 24 - 6 = 18 km


G J
Now, G 2 J =12 or l = 200 m 6+4 =
18
T
GH 25 JK T = 1.8 hour
Also, speed of slower train T = 1hour 48 min


FG
= 36 ×
5 IJ
m/sec. = 10 m/sec.
12:48 pm
H 18 K Ex. A train overtakes two persons walking along a railway
track. The first one walks at 4.5 km/hr. the other one
Now, if x metre be the length of platform, then
walks at 5.4 km/h. The train needs 8.4 and 8.5 seconds
x + 200 respectively to overtake them. What is the speed of the
=90 or x = 700 metres.
10 train if the persons are walking in the same direction
10. When two trains start from two points X & Y towards as the train?
each other at the same time and after crossing they Sol. Length of train
take p & q second to reach Y & X respectively then (S tra in − 4 .5 ) × 5
× 8 .4 ---- (1)
=
ratio of 18
Length of train
Speed = q: p
5
Ex. Two trains x and y start from stations A and B = (S Train - 5.4) × 1 8 × 8.5 ---- (2)
towards B and A respec­tively. After passing each
(S − 4 .5 ) × 5 × 8 .4
other, they take 4 hours 48 minutes and 3 hours 20
18
minutes to reach B and A respec tively. If train x is
moving at 40 km/hr., the speed of train y is : - (S − 5 .4 ) × 5 × 8 .4
=
18
x ' s rate
Sol. = T im e taken by Y to reach A
y ' s rate T im e taken by X to reach B S = 81 km/h

116 QUANTITATIVE APTITUDE


stportal.mahendras.org

EXERCISE
Q.1. A train running at 40 km/h crosses a pole in 36 second. Find the length of train.
(1) 300 m (2) 200 m (3) 400 m (4) 500 m (5) 450 m
Q.2. A train running at the speed of 20 m/s takes 10 second to pass a man who is running at the speed of 5 m/s in the
same direction. Find the length of the train.
(1) 150 m (2) 200 m (3) 250 m (4) 300 m (5) None of these
Q.3. A train 200 m long takes 5 second to pass a standing man. Find the time taken by train in crossing a railway platform
of 240 m long.
(1) 18 sec. (2) 13 sec. (3) 11 sec. (4) 26 sec. (5) None of these
Q.4. A man is standing on railway bridge which is 100 m long. He finds that train crossed the bridge in 5 second but
himself in 3 sec. Find the length of the train.
(1) 100 m (2) 150 m (3) 200 m (4) 250 m (5) None of these
Q.5. A train running at a speed of 25 km/h takes 18 sec to pass a railway platform and it takes 9 sec to pass a man who
is running at the speed of 5 km/h in the same direction. Find the length of the platform.
(1) 77 m (2) 70 m (3) 75 m (4) 71 m (5) None of these
Q.6. Two trains are moving in opposite directions at speeds of 80 km/h and 70 km/h and their length is 1.20 km and
0.80 km respectively. Find the time taken by the slower train to cross each other.
(1) 48 sec (2) 50 sec (3) 42 sec (4) 44 sec (5) None of these
Q.7. Two trains each 200 m long move towards each other on parallel track with velocity 22 km/h and 28 km/h respectively.
What is the time taken when they meet each other?
(1) 20 sec (2) 24.8 sec (3) 28.8 sec (4) 30 sec (5) None of these
Q.8. Two trains are running 40 km/h and 20 km/h respectively in the same direction. The faster train completely passes
a man sitting in the slower train in 18 sec. Find the length of faster train.
(1) 105 m (2) 100 m (3) 90 m (4) 110 m (5) None of these
Q.9. A train with a speed of 90 km/h crosses a bridge in 36 seconds. Another train 100 m shorter crosses the same bridge
at 45 km/h. What is the time taken by the second train to cross the bridge?
(1) 32 sec (2) 64 sec (3) 40 sec (4) 50 sec (5) None of these
Q.10. A train moves pass a telegraph pole and a bridge 264 m long in 8 sec and 20 sec respectively what is the speed of
the train?
(1) 15m/s (2) 20 m/s (3) 25 m/s (4) 18 m/s (5) 22 m/s
Q.11. If a train runs at 40 km/h, it reaches its destination late by 11 minutes but if it runs at 50 km/h, it is late by 5 minutes
only. Find the correct time for the train to complete its journey.
(1) 19 minutes (2) 20 minutes (3) 21 minutes (4) 18 minutes (5) None of these
Q.12. Two trains, Train A and Train B crosses each other completely in 18 sec while travelling in opposite directions,
speed of train A is 72 km/hr and speed of train B is 54 km/hr. Length of train A is 170 m less than the train B. From
the statement given in the above question which of the following can be determined?
A) Length of train A
B) Length of train B
C) Time taken by train B to cross a 130 m length of bridge
D) Time taken by train A to cross a 70 m long canal
(1) All A), B), C) and D) (2) Only B) and C) (3) Only D) and B)
(4) Only A (5) Only C
Q.13. Train A started from station P towards station Q. At the same time, train B started from station R towards station
Q. All the three stations are in a straight line such that station Q is between station P and station R. Station Q is
equidistant from station P and station R. Distance between station P and station Q is 440 Km. If the speed of train
A and train B is _______Km/hrs and ______Km/hrs respectively, the distance between both the trains after five
hours is 580 km.

QUANTITATIVE APTITUDE 117


stportal.mahendras.org
Which of the following satisfies the two blanks given in the questions?
I. 28 km/hrs, 32 km/hrs
II. 40 km/hrs, 20 km/hrs
III. 25 km/hrs, 35 km/hrs
(1) None (2) Only II (3) Only III (4) Only I and II (5) All I, II and III
Q.14. A goods train and a passenger train are running on parallel tracks in the same direction. The driver of the goods
train observes that the passenger train coming from behind overtakes and crosses him completely in 80 seconds.
Whereas a passenger on the passenger train observes that the goods train crosses in 40 seconds. If the speeds of
the trains be in the ratio 1:3. Find the ratio of lengths of passenger train to goods train.
(1) 1:1 (2) 2:1 (3) 1 : 3 (4) 1:2 (5) 3∶1
Q.15. A train overtakes two person who are walking in the same direction in which the train is going, at the speed of 2
km/h and 4 km/h and passes them completely in 9 second and 10 sec respectively. Find the length of the train.
(1) 200 m (2) 50 m (3) 150 m (4) 250 m (5) None of these
Q.16. Two train X and Y start from station A and B towards B and A respectively after passing each other, they take 4
hours 48 minutes and 3 hours 20 min to reach B and A respectively if train X is moving at 40 km/h, Find the speed
of train Y.
(1) 60 km/h (2) 54 km/h (3) 64.8 km/h (4) 48 km/h (5) None of these
Q.17. A train travels a certain distance without stoppages with an average speed of 90 km/h. but with stoppages at an
average speed of 75 km/h. Find the number of minutes per hour that the train stops.
2
(1) 8 min (2) 10 min (3) 6 min (4) 15 min (5) 12 min
3
Q.18. Two trains are moving in the opposite directions on parallel tracks at the speeds of 63 km/hr and 94.50 km/hr
respectively. The first train passes a pole in 6 seconds whereas the second train passes a pole in 4 seconds.Find the
time taken by the trains to cross each other completely?
(1) 4.80 sec. (2) 4.40 sec. (3) 3.80 sec.
(4) Can't be determined (5) None of these
3
Q.19. A train after travelling 48Kms. meets with an accident and then proceeds at of its former speed and arrives at
4
its destination 92 minutes late. Had the accident occured 72 km, further, it would have reached the destination only
80 minutes late. Find the speed of the train ?
(1) 100 Km. ph (2) 120 Km. ph (3) 90 Km. ph (4) 150 Km. ph (5) None of these
Q.20. Two places A and B are 162 kms. apart. A train leaves A for B and at the same time another train leaves B for A.
The two trains meet after 6 hours. If the train travelling from A to B travels 8 kmph faster than the other, what is
the speed of the faster train ?
(1) 17.5 Kmph. (2) 18 kmph (3) 18.5 Kmph (4) Data inadequate (5) None of these
Q.21. Two trains at the same times from two stations and proceed towards each other at the rate of 30 km/h & 35 km/h
respectively. When they meet it is found that one train has travelled 60 km. more than the other . What is the distance
between two station?
(1) 880 (2) 720 (3) 780
(4) Can't be determined (5) None of these
Q.22. Two trains (X and Y) start from 'A' and 'B' respectively, towards each other. After they meet at point 'C' the train
1
'X' takes two hours to reach B while the trains 'Y' takes 4 hours to reach A. If the distance between places A
2
and B is 450 kms. How far should 'C' be from 'A' ?
(1) 180 km. (2) 270 km. (3) 225 km.
(4) 360 km. (5) None of these

118 QUANTITATIVE APTITUDE


stportal.mahendras.org

EXERCISE Explanation
Q.1.(3) When train crosses the pole L1 18
9= ×
L (because pole has no breadth) 25 − 5 5
S=
T
L1 = 50, L2 = 75m
40 × 5 L
=
18 36 Length of train = 50 m

L = 400 m Length of platform = 75 m


Q.2.(1) When train crosses a man travelling in same Q.6.(1) When they cross each other
direction
L1 + L 2
L S1 + S2 =
S1-S2 = T T
(8 0 + 7 0 ) × 5 = 1 2 0 0 + 8 0 0
L
(20 -5) = 18 T
10
L T = 48 sec
15 =
10 Q.7.(3) When they are moving in opposite direction
L = 150 m
L1 + L 2
Q.3.(3) According to question S1 + S2= T
L
S=
T
(2 2 + 2 8 ) × 5 =
200 + 200
18 T

200
S= = 4 0 m/s 50 × 5 400
5 =
18 T
when train crosses the platform
L1 + L 2 T = 28.8 sec
S=
T L
Q.8.(2) S1 - S2 =
210 + 240 T
40 =
T (4 0 − 2 0 ) × 5 = L
440 18 18
40 =
T L = 100 m
T = 11 second
L +100 L Q.9.(2) S = L +B
Q.4.(2) S= ........ (I) S= T
5 3 90 × 5 L + B
According to question =
18 36
L +100 L
=
5 3 L+B = 900
3L + 300 = 5L 45 × 5 L −100 + B
2L = 300 100 = T
L = 150m 45 × 5 900 −100
=
Q.5.(3) When it crosses platform 100 T

L1 + L 2 L1 45 × 5 800
S= , S −S =
T T M 18 T

25 × 5 L1 + L 2
= T = 64 sec
18 18
L
L1 + L2 = 125 Q.10.(5) S = T when crosses a pole

QUANTITATIVE APTITUDE 119


stportal.mahendras.org

L = 25×5 = 125 Km
S= ------ (1) Distance travelled by train B in 5 hours
8
L + 264
= 35×5 = 175 Km
S= Required distance = 880 – (125 + 175)
20
= 880 - 300 = 580 Km
L L + 264 This is satisfies the given condition.
⇒ 8 =
20 Q.14.(2) Let the speed of passenger train be 3v and that
5 L = 2L + 528 of goods train be v. as the ratio of their speeds be
1:3.
3L = 528
Also, suppose that the lengths of passenger train
L = 176 m
be P and goods train be G.
176 Then,
S= = 22 m/s
8
Q.11.(1) 3x 6x P
− =6 2v =
2 5 80
15x − 12x
=6 G
10 Again 2v =
40
x = 20
From equation (I)
3x On division
= t + 11
2 P 80
3 × 20 =
= t + 11 G 40
2
t = 30 – 11=19 minutes P:G=2:1

Q.12.(1) LA +LB = (72+54)×5/18×18=630


LB - LA = 170 Q.15.(2) (S − 2 ) × 5 L
= ..........(1)
LB = 400 m. LA = 230m 18 9
All A, B , C and D
Q.13.(5) From I:
(S − 4 ) × 5 = L ..........(2 )
Distance between station P and station R 18 10
= 440×2 = 880 km 9S − 1 8 = 1 0S − 4 0
Distance travelled by train A in 5 hours S = 22 km / hr
= 28×5 = 140 km
Distance travelled by train B in 5 hours 5 L
= 32×5 = 160 km 20 × =
18 9
Required distance = 880 – (140 + 160)
= 880 - 300 = 580 Km
L = 50m
This is satisfies the given condition.
From II: tim e1
Distance between station P and station R Q.16.(4) Speed of train Y = ×speed of train x
tim e 2
= 440×2 = 880 km
Distance travelled by train A in 5 hours 240 + 48
= × 40
= 40×5 = 200 km 180 + 20
Distance travelled by train B in 5 hours
= 20×5 = 100 km
Required distance = 880 – (200 + 100) 288 12
= × 40 = × 4 0 = 48 km/h
= 880 - 300 = 580 Km 200 10
This is satisfies the given condition.
From III: 90 − 75 15 × 60
Q.17.(2) × 60 = = 10 min
Distance between station P and station R 90 90
= 440×2 = 880 Km
Distance travelled by train A in 5 hours Q.18.(1) Length of Ist Train = S x T

120 QUANTITATIVE APTITUDE


stportal.mahendras.org
Speed of faster train = 9.5 +8 =17.5 km/h
= 6 3 x 5 x 6 = 1 0 5 m e tre
18
L e n gth o f IIn d T ra in = 9 4 .5 x 5 x4 = 1 0 5 M e tre Q.21.(3) Time=
Difference of the distance
18
T im e T a ke n to cro ss e a ch o th e r T ra in Difference of Speed
T o ta l le n gth
= 60
R e la tive sp e e d = 1 2 h rs
105 + 105 35 − 30
=
e6 3 +9 4 .5 jx 158 Reliance speed = 30+35 = 65 km/hr.
= 210 Total distance = Reliance speed + time
4 3 .7 5
= 4 .8 0 se co n d = 65 x 12 = 780 km.

Q.19.(2) 120 km/hr. Q.22. (2) According question
difference of 12 minute time comes when accident Train x ← d2 → ← 2 hours → Train y
occured after 1st accident point in 12 minute
train travel 24 km, hence speed of train A.....................................B
= 24 x 60 ← 4 1/2 hours → c ← d1 →
12
d T
= 120 km/hr. 1 = 1
d T
Q.20.(5) 17.5 km/hr. 2 2
d
Let the speed of slower train is x km/hr. 1 = 2 x2
d 9
then speed of faster train is x + 8 km/hr. 2
d
Formula used 1 =2
d 3
Relative S p e e d = D ista n ce
T im e
2
Total Distance value of ( 2 + 3 )
162 = 450 km.
(x + x + 8) = 6
2x = 27- 8 450
Distance A to C value of (3) = x3
5
x = 1 9 = 9 .5 km / h r . = 270 km.
2

QUANTITATIVE APTITUDE 121


stportal.mahendras.org

CHAPTER

15 Boat & Stream


Concept of Boat and Stream Depends on Concept of Speed, B s km /hr.
Time and Distance mainly it has 4 facts.
B oa t
(1) Still water : If the speed of the water in the river or
pond is zero it is called still water. It is denoted as Bs.

B s km /h C s km /h r.
B oa t
Bs-Cs = Us
Ex. If the speed of the boatman in still water is 10 km/hr
Still w ater and speed of the water (stream) is 4 km/hr then the

speed of the boatman in upstream.
(2) Stream: If the water of the river is flowing at a Sol. Us = Bs - Cs
constant speed it is called as stream. It is denoted as
Cs. Hence Us = 10-4 = 6 km/h
Important formulas:
(1) Speed of the boat in still water
C s km /hr. d o w n stre a m sp e e d + u p stre a m sp e e d
= 2
(3) Down Stream Speed: If a boat or swimmer goes along
the stream is called down stream speed. Ds + Us
Bs = 2
Down stream speed = speed of the boat + speed of the
current Ex. A man can goes 32 km/hr upstream & 36 km/h
downstream. Find the speed of man in still water.
Ds = Bs + Cs
Sol. Speed of man in still water
Bs
B oa t Ds + Us 32 + 36
(B s ) = =
B s+C s = D s 2 2

Cs = 34 km/h

(2) Speed of the current
Ex. If the speed of the boatman in still water is 10 km/hr
and speed of the water (stream) is 4 km/hr then speed d o w n stre a m sp e e d - u p stre a m sp e e d
=
of the boatman in downstream. 2

Sol. Since Ds = Bs + Cs Ds − Us
Cs =
Hence Ds = 10 + 4 = 14 km/hr. 2
Ex. A man can go 36 km/hr upstream & 48 km/h
(4) Up stream speed: If a boat or a swimmer moves downstream. Find the speed of current.
against the stream it is called upstream speed.
Ds − Us
Upstream speed = speed of the boat - speed of the Sol. Speed of current Cs =
2
current
48 − 36
Us = Bs - Cs = 6 km/hr
2

122 QUANTITATIVE APTITUDE


stportal.mahendras.org

EXERCISE
Q.1. A boat can travel a distance 10 km in an hour against the stream whereas 20 km in an hour with the stream. Hence
find the speed of boat in still water.
(1) 15 km/h (2) 22 km/h (3) 20 km/h (4) 44 km/h (5) 25 km/h
Q.2. The distance between two points is 36 km., boat rows in still water at 6 kmph. It takes 8 hour less to cover this
distance in downstream in comparison to that in upstream. The rate of stream is:-
(1) 3 kmph (2) 2 kmph (3) 2.5 kmph (4) 4 kmph (5) None of these
Q.3. Ram goes downstream with a boat to some destination and returns upstream to his original places in 6 hours. If the
speed of the boat in still water and the stream are 12 km/hr and 5 km/hr respectively, then find the distance of the
destination from the starting position.
(1) 25km (2) 26.67km (3) 33km (4) 29.75km (5) 20km
Q.4. When a person is moving in the direction of the current is 20km/hr, speed of the current is 5km/hr. Then find the
speed of the person against the current?
(1) 10km/hr. (2) 15km/hr. (3) 30km/hr. (4) 25km/hr. (5) 11km/hr.
Q.5. There are two places A and B which are separated by a distance of 100km. Two boats starts form both the places
at the same time towards each other. If one boat is going downstream and the other one is going upstream, if the
speed of A and B is 12km/hr. and 13km/hr. respectively. Find at what time will they meet each other.
(1) 10 hrs. (2) 4 hrs. (3) 8 hrs. (4) 6 hrs. (5) 7 hrs.
Q.6. A girl was travelling in a boat, suddenly wind starts blowing and blows her hat and started floating back downstream.
The boat continued to travel upstream for 12 more minutes before she realized that her hat had fallen off. She
turned back downstream and she caught her hat as soon as she reached the starting point. If her hat flew off exactly
2km from where she started. What is the speed of the water?
(1) 12 km/hr (2) 8 km/hr (3) 5 km/hr (4) 9 km/hr (5) 10 km/hr
Q.7. The ratio of the speed of the motor boat to that of the current of water is 36:5 . The boat goes along with the current
in 5 hours 10 minutes. How much time it will take to come back .
(1) 45/2 hr (2) 41/6 hr (3) 55/3 hr (4) 38/7 hr (5) 52/8 hr
Q.8. There is a road beside a river. Two friends started from a place A, moved to a temple situated at another place B and
then returned to A again. One of them moves on a cycle at a speed of 6 km/hr, While the other sails on a boat at a
speed of 8 km/hr. If the river flows at the speed of 6 km/hr, which of the two friends will return to place A first?
(1) Cyclist (2) Sailor (3) Both come at same time
(4) Anyother (5) None
Q.9. The different between downstream speed and upstream speed is 2 kmph and the total time taken during upstream
and downstream is 2 hours. What is the upstream speed, if the downstream and upstream distance are 2 km each?
(approx)
(1) 5.2 kmph (2) 3.7 kmph (3) 2.8 kmph (4) 1.4 kmph (5) 1.2 kmph
Q.10. A boat travels downstream from point A to B and comes back to point C half distance between A and B in 18 hours.
If speed of boat is still water is 7 km/hr and distance AB = 80 km, then find the downstream speed.
(1) 15 km/hr (2) 18 km/hr (3) 12 km/hr (4) 10 km/hr (5) 6 km/r
Q.11. A boat can row 18 km downstream and back in 8 hours. If the speed of boat is increased to twice its previous speed,
it can row same distance downstream and back in 3.2 hours. Find the speed of boat in still water.
(1) 9 km/hr (2) 5 km/hr (3) 4 km/hr (4) 8 km/hr (5) 6 km/hr
Q.12. A ship of length 300 m traveling from point A to B downstream passes a Ghat along the river in 18 sec, while in
return it passes the same Ghat in 24 sec. If the rate of current is 9 km/hr. Then what is the length of the Ghat?
(1) 50 m (2) 60 m (3) 80 m
(4) 100 m (5) Cannot be determined

QUANTITATIVE APTITUDE 123


stportal.mahendras.org
Q.13. A boat takes 25 hours for travelling downstream from point A to point B and coming back to point C midway
between A and B. If the velocity of the stream is 5 km/hr and the speed of the boat in still water is 10 km/hr, what
is the distance between A and B?
(1) 100 km (2) 122 km (3) 146 km (4) 178 km (5) 150 km
Q.14. A boat running at the certain speed downstream covers a distance of 4.8 kms. in 8 minutes. The same boat running
upstream covers the same distance in 9 minutes. What is the speed of the current?
(1) 2.4 kmph (2) 3 kmph (3) 2 kmph (4) 3.2 kmph (5) None of these
Q.15. The speed of a boat in still water is 6 kmph and that of current is 3 kmph. The boat starts from point A and rows to
point B and comes back to point A. It takes 12 hours during this journey. How far is point A from point B?
(1) 27 km (2) 25 km (3) 20 km (4) 30 km (5) None of these
Q.16. The speed of a boat in still water is 17.5 kmph and that of current is 2.5 kmph. The boat goes from X to Y in
downstream and returns to point Z. The whole journey takes 429 minutes. If the distance between Z and Y is 2/5
of the distance between X and Y. Find the total distance covered by the boat. (approximate to the nearest integer).
(1) 130 km (2) 140 km (3) 160 km (4) 120 km (5) 150 km
Q.17. The speed of the boat in still water is 16 kmph. and speed of the current is 2 kmph. It takes a total time of 6.5 hours
to row upstream from point A to point B and downstream from point B to C. The distance from point A to B is
two-third the distance between point B and C, what is the total distance travelled by the boat (Both upstream and
downstream)?
(1) 112 km (2) 98 km (3) 124 km (4) 90 km (5) 105 km
Q.18. A boy takes 3 hours 45 min to cover a distance of 15 km along the stream and 2 hours 30 min to cover the distance
fo 5 km against the stream. Find the speed of the current in km/hr.
(1) 2 km/h. (2) 1 km/h. (3) 3 km/h. (4) 4 km/h (5) 1/2 km/h
Q.19. A boy swims 40 km upstream and 55 km downstream in 13 hours. and he can also swims 30 km upstream and 44
km downstream in 10 hours. Then-
1. Find the speed of man.
2. Find the speed of stream.
(1) 8 kmph, 3 kmph (2) 16 kmph, 6 kmph (3) 11 kmph, 5 kmph
(4) 10 kmph, 6 kmph (5) 8 kmph, 2 kmph
Q.20. A man takes double time to swim upstream a certain distance as compared to the time taken to cover the same
distance in downstream. The ratio of speed of boat in still water to that of the speed of current is:
(1) 2 : 1 (2) 3 : 1 (3) 3 : 2 (4) 4 : 3 (5) 1 : 2
Q.21. A boat can travel 4.2 km upstream in 14 min. If the ratio of the speed of the boat in still water to the speed of the
stream is 7:1. How much time will the boat take to cover 17.6 km downstream?
(1) 42 minute (2) 48 minute (3) 40 minute (4) 44 minute (5) None of these

124 QUANTITATIVE APTITUDE


stportal.mahendras.org

EXERCISE Explanation
Q.1.(1) Speed of boat upstream 2 × 2 ×14
= = 3 .5 k m / h r
1 0k m 2 +14
= = 10 km/h
1h Q.9.(4) 2
Speed of baot downstream 2
+ =2
2 0k m x x +2
= = 20 km/h x2 − 2 = 0
1h
Speed of boat in still water x = 1 .4 1 4 k m p h
20 + 10
= = 15 km/h Q.10.(4) A to B is 80, so B to is 80/2 = 40 km
2
Q.2.(1) Speed of current = Cs kmph. Let speed of current = x km/hr So 80/(7+x) + 40/
(7-x) = 18 Solve, x = 3 km/hr
downstream speed =(6+Cs) kmph.
So downstream speed = 7 + 3 = 10 km/hr
upstream speed = (6-Cs) kmph.
Q.11.(5) Let speed of boat = x km/hr and that of stream
According to question.
= y km/hr
T1 - T2 = 8 hours.
18 18
36 36 + =8
∴ − =8 x +y x −y
(6 − C s ) (6 + C s )
when speed of boat becomes 2x km/hr:
 6 + Cs − 6 + Cs 
⇒ 36  =8 18 18
 (6 − C s )(6 + C s )  + = 3 .2
2x + y 2x − y
3 6 × 2C s
=8 x = 6 km/hr
(6 + C s )(6 + C s )
Q.12.(2) (S+9)×18 = (S-9)×24; S =63
Cs2 + 9Cs - 36 = 0, Cs = 3 kmph. 300+x = 72×5/18×18; x =60
2 xD 119 × 6 Q.13.(5) Downstream speed = 10+5 = 15 Upstream speed
Q.3.(4) T = 2 2 ⇒ D = = 29.75km
x -y 2 ×12 = 10-5 = 5
Now total time is 25 hours If distance between A
Q.4.(1) Speed of the person = 20 – 5 = 15km/hr Speed of
and B is d, then distance BC = d/2
the person against the current = 15 – 5 = 10km/hr.
Q.5.(2) Downstream = (12+x)km/hr Now distance/speed = time,
Upstream = (13-x)km/hr D D/2
So, + = 2 5 , D = 150 km
D ista n ce 15 5
Time = Q.14.(3) Let the speed of boat in still water = Bs kmph.
R e la tiv e S p e e d
∴ The speed of current = Cs kmph
Relative speed = 12 + x + 13 – x Hence, Ds = Bs + Cs
= 25 km/hr 4 .8 4 .8 × 6 0
= ⇒ = 36 kmph.
100 8 8
Time = = 4 hours
25 60
and
Q.6.(3) Distance = 2 km
4 .8 4 .8 × 6 0
Time = 2 × 12 = 24 mins Us = Bs - Cs = = = 32 kmph
9 9
2 60
Req. Answer = × 7 = 5 km / hr
24 Now, for speed of current
S1 T1 36 + 5 31 D s − U s 36 − 32
Q.7.(2) = -5 = x = =
S2 T2 36 6 2
2
41 Speed of current (Cs) = 2 kmph
x= = 6 hours 50 minutes
6 Q.15.(1) Downstream rate (Ds) = 6 + 3 = 9 kmph
Q.8.(1) Average speed of the cyclist =6 km/hr Upstream rate (Us) = 6-3 = 3 kmph
Average speed of sailor If the required distance be x km, then

QUANTITATIVE APTITUDE 125


stportal.mahendras.org

x x x + 3x 15
+ = 12 = 12 = km/h = 4 km/h.
9 3 9 3
3
4x = 12 × 9 4
upstream speed (Us)
12 × 9
x= 5
4 = km/h = 2 km/h.
1
x = 27 km. 2
2
Q.16.(1) Downstream speed of boat = 17.5+2.5 Speed of current
20 kmph Ds − Us 4 − 2
Upstream speed of boat = 17.5-2.5 = = = 1 km/h.
2 2
15 kmph. Q.19.(1) Let the upstream speed = x kmph.
Distance between X and Y = X km Let the downstream speed = y kmph.
2X 40 55
Distance between Y and Z = km. + = 13 ------- (1)
5 x y
3 30 44
Total time = 429 minutes ⇒ 7 hours. + = 10 ------- (2)
20 x y
143  D is ta n ce  Now, on solving from eq. (1) and (2)
or = hours.  = T im e 
20  Speed  We get
x 2x 143 y = 11 kmph., X = 5 kmph.
+ =
20 5 ×15 20 (i) Speed of boy
15 x + 8 x 143 Ds + Us x + y
= = =
60 4 2 2
23x = 143×15 11 + 5
143 ×15 ≈ = = 8 kmph
x= 93 km. 2
23 (ii) Speed of current
2x 7x
Total Distance= x + = Ds − Us x −y
5 5 = =
7 × 93 2 2
= ≈ 130 km.
5 11 − 5
= 3 kmph
Q.17.(5) Distance between B to C = x km 2
2x Q.20.(2) Let man's rate upstream be x kmph.
Distance between A to B = km Man's rate downstream be 2x kmph.
3
Downstream speed (Speed of still water) : (Speed of Stream)
= 16+2 ⇒ 18 kmph. 2x + x  2x − x 
Upstream speed = 16-2 ⇒ 14 kmph.
=  :  =3:1
 2   2 
According to the question Q.21.(4) Let the speed of the boat in still water and the speed
2x x of the stream is 7x and x
⇒ + = 6.5
3 ×14 18 Then, Downstream speed= 8x, upstream speed
= 6x
⇒ = 6.5 According to the question-
Upstream speed
13x = 6.5 × 126
4.2 ×60 = 18 kmph
6 .5 × 1 2 6 14
x=
13
u:v =7:1
x = 63
so D : U = 8 : 6
2x 5 x 5 × 63 Now 6 = 18
Total distance = +x= =
3 3 3 so 8 = 24
= 105 km. 17.6
Q.18.(2) downstream speed (Ds) Required answer = ×60 = 44 min.
24

126 QUANTITATIVE APTITUDE


stportal.mahendras.org

CHAPTER

16 INEQUALITY

Now, Multiplying Equation (2) by 2, we get,


Introduction
2(2x - 5y = -5) = > 4x - 10y = -10....(iii)
In this chapter, we will study about the problems based on
equations and methods of comparing their values. Inequality Now, from (i) and (iii)
basically deals with few signs. (<) "Less than"; (>) "greater 4x - 3y = 4
than"; (=) equal to; (<) "less than or equal to"; (>) "greater
than or equal to". 4x - 10y = -10

Pattern of questions asked in exams is as follows:- - + +

In the following question two equations are given. You have 7y = 14


to solve both equations and- y=2
Give the answer- ∴ x = 2.5
(i) x >y (ii) x > y (iii) x < y Hence x > y
(iv) x < y (v) x = y or the relation can’t be established. Quadratic equation: A quadratic equation is a second -
Variable : Variables are the values which do not remain fixed order polynomial equation in a single variable because it is a
and thus are denoted by English alphabets second-order polynomial equation, the fundamental theorem
of algebra identities that it has two solutions. These solutions
Ex. (a,b,c,m,n,o,p,q, r,x,y,z, etc.) may be both real or both complex.
Constant : Constant are the values which remain fixed. Quadratic formula :
Ex. (1,2,3,4 ........ and so on). For the standard equation ax2 + bx +c= 0
The power of any variable is always one and the variable has
x = −b ± b − 4a c
2

one or more than one is known as linear equation.


2a
Types of Equation:
(i) Linear Equation: A linear equation is an equation in Ex. I. x2 + x - 6 = 0
which each term is either a constant or the product of x2+3x-2x-6=0
a constant and the first power of a single variable.
x(x+3)-2 (x+3)=0
(a) Linear Equation with one variable:
(x+3) (x-2) = 0
Ex. (I) 6x + 5 = 4x - 15
x = - 3, 2
6x - 4x = - 20
II. y2 + 10y + 24 = 0
2x = - 20
y2 + 6y + 4y + 24 = 0
x = - 10
y(y+6) + 4 (y+6) = 0
(II) 4y - 4 = 5 + y
(y+6) (y+4) = 0
4y - y = 5 + 4
y = -6, - 4
3y = 9
Now, on comparing the values of x and y we get,
y=3
which means that x < y. x y

(b) Linear Equation with two variables - 3 > -6

Ex. (I) 4x - 5 = 3y -1 - 3 > -4


(II) 2x + 3 = 5y-2 2 > -6
4x - 3y = 4...... (i) 2 > -4
2x - 5y = -5.....(ii) Hence, we can say that x > y

QUANTITATIVE APTITUDE 127


stportal.mahendras.org
Example: y2 = 36
x2 + 2x = 7 y = +6, -6

x + 2x - 7 = 0
2 x>y
Ex. I. 2x+3y=14
a = 1, b = 2 c = -7
II. 4x+2y=16
With quadratic formula:
Sol. I. (2x+3y = 14)× 2 ⇒ 4x+6y = 28
−b ± b 2 − 4a c
x= 4x + 6y = 28
2a
4x + 2y = 16
(2 ) − 4 (1)( −7 )
2
−2 ±
x= (−)(−)(−)
2 (1)
0 + 4y = 12

x=
−2 + (1 6 )
2

= =
(
−2 ± 2 2 −1 ± 2 2 )
2 y=3
2 2
x = 2.5
= -1 ±2 2
x<y
D = b2 - 4ac
Ex. I. 20x2–x–12=0
Discriminant Rule: II. 20y2+27y+9=0
(1) If b - 4ac is less than 0, there is no real solution.
2
Sol. I. x = + 0.8, – 0.75
(2) If b2 - 4ac is equal to 0, then the equation has two equal II. y = – 0.6, –0.75
real solution.
0.8 > -0.6
(3) If b2 - 4ac is greater than 0, then the equation has two - 0.75 = - 0.75
different real solution.
-0.75 < -0.6
Important Examples :
Hence no relationship can be established.
Ex. In the following questions two equation numbered I
12
and II are given. You have to solve both equestions Ex. I. = x
and ----------- x
Give answer (1) if x >y 5

Give answer (2) if x ≥ y II. y 2



(8 ) 2 =0
y
Give answer (3) if x < y
Give answer (4) if x ≤ y 12
Sol. I. = x , 12=x
Give answer (5) if x = y or the relationship can not x
be established.
5
Ex. I. x2–24x+144=0
II. y 2

(8 ) 2 1 5

= 0 , y × y 2 – .8 2 = 0
2

II. 2y –52y+338=0
2
y
Sol. I. (x–12) = 0, x = +12
2

II. y2 – 26y+169=0 5 5

(y–13)2 = 0 y =8 2 2 , y=8
y = +13, x<y x>y
Ex. I. x + 18 = 144 − 49 3 7 12 25
Ex. × x= × y
10 4 6 15 24
12 × 4 3× 4 7 7 5
II.
4 - 4 =y Sol. x= y
8 6
y7 y7
21x = 20y
Sol. I. x+18=52
x<y
x=25–18=7
14 Ex. I. x + y = 16
II. 48–12=
y7 II. x2 + y2 + xy = 192

128 QUANTITATIVE APTITUDE


stportal.mahendras.org
Sol. I. (x+y) = 256 2
6y - 3y - 2y + 1 = 0
2

x +y + 2xy = 256 ....... (I)


2 2
3y (2y-1) - 1 (2y -1) = 0
From eq. (I) and (II) 1 1
y= ,
xy = 64 2 3
(x-y)2 = (x+y)2 - 4xy x<y
= 256 - 4 × 64 Ex. I. 2x – 1 – x + x = 5
=0 II. 3y – y – y – 10 = – 15
x-y = 0
Sol. I. 2x – 1 = 5
∴x=y 2x = 6
Ex. I. 4x2 + 8x = 4x + 8
x=3
II. y2 + 9y = 2y - 12
II. 3y – 2y = – 5
Sol. I. 4x2 + 4x - 8 = 0
y=–5
x2 + x - 2 = 0
x>y
x2 + 2x -x - 2 = 0
Ex. I. x2 = (38)2 × 52
x (x+2) - 1 (x+2) = 0
II. y2 = (38×5)2
x = -2,1
Sol. I. x = + 64
II. y2 + 7y + 12 = 0
II. y = + 7
y2 + 4y + 3y + 12 = 0
So, relationship can not be established.
y (y+4) +3 (y +4) =0
y = - 3, - 4 Ex. I. x 2 = 4 0 9 6
2
x>y II. y 2 =  3 3 4 3 
 
Ex. I. 2x2 + 40 = 18x Sol. From equation (I) and (II),
II. y2 = 13y - 42 x = +8
Sol. I. x - 9x + 20 = 0
2
y=7
x2 -5x - 4x + 20 =0 Relation can't be established
x (x-5) - 4 (x-5) =0
Ex. I. x – y = 9
x = 4,5
II. x + y = 7
II. y2 - 13y + 42 = 0
Sol. I. x = 8
y2 - 6y - 7y + 42 = 0
y=1
y (y-6) -7 (y-6) = 0
x>y
y = 6,7
Ex. (I) x where: 3x2 + 2x – 8=0
x<y
(II) y: where: 3y2 + 5y -12 =0
1 7
Ex. I. 6x + = x
2
4 4
2 2 Sol. x= -2 and ; y= -3 and ;
3 3
II. 12y2 + 2 = 10y So no relation
Sol. I. 12x2 - 7x + 1 = 0 Ex. (I) x where: 4x2 – 16x + 15 = 0
12x2 -4x -3x + 1 =0 (II) y where: 2y2 + y -6 = 0
4x (3x-1) -1 (3x-1) =0 3 5
Sol. x= and
1 1 2 2
x= ,
3 4
3
II. 6y2 - 5y + 1 = 0 y= -2 and
2

QUANTITATIVE APTITUDE 129


stportal.mahendras.org
Ex. It takes (X-2) men to do a work in 2Y days and it takes
New Pattern Inequality
(X+6) men to do the same work in Y days.
Ex. In each of the following question, a question is
followed by information given in two statement I: X II: Y
named as Quantity-I (Q1) and Quantity –II (Q2). You Sol. As the time is becoming half so means number of
have to study the information along with the question people have doubled; So (X+6)=2×(X-2)=>X=10
and compare the value derived from Quantity-I and Using M1D1=M2D2 the value of Y cannot be found. Y
Quantity-II and give answer- can take any value.
(1) Quantity I > Quantity II So we cannot determine a unique value of Y
(2) Quantity I ≥ Quantity II
Ex. A hemisphere of radius 4 cm is to be shipped in a
(3) Quantity II > Quantity I shipping box of rectangular shape. The dimension of
(4) Quantity II ≥ Quantity I the box are consecutive odd numbers.
(5) Quantity I = Quantity II or Relation cannot be I: Minimum volume of box required to have the
established shipment possible.
Ex. 90
0
II: 960 cm3
A
D
50 0
Sol. I: The minimum length of any side of rectangular box
should be greater than the diameter of the hemisphere.
B x0 Diameter of hemisphere=8; So for odd number,
y
0

20
0

40
0 C minimum length= 9, next side =11, next side=13
E Volume=9×11×13= 1287 cm3
I. angle x II. 1100 Ex. (I) No. of days in which A will work alone, given
Sol. In triangle BCE, angle y= 180-(40+20)= 120 A and B can complete work in 8 days, B and
Sum of exterior angle of a quadrilateral is 360 degree C can complete work in 12 days, C and A can
complete work in 8 days.
So in quadrilateral ABCD;
(II) No. of days in which A will work alone, given
50 + y +x+90=360 [y=120] A and B can complete work in 18 days, they
x=1000 started work together and after working for 6
days A left and B completed remaining work

Q1<Q2
in 24 days.
Ex. AB is parallel to DC and AD is parallel to BC. BD is
4 cm more than AC. AC and BD are the diagonal with Sol. LCM = 24
O as point of intersection. So
A B
24
90 0 A+B= =3
8
o 6 cm
24
D C B+C= =2
12
I: AB II: 9 cm
24
Sol. I: ABCD is a parallelogram as opposite sides are C+A= =3
8
parallel. In a parallelogram the diagonal are divided
in equal part at the point of intersection. 2 ( A+B+C) = 3+2+3 = 8
So AC = 2 OC = 12 BD=16 cm A+B+C = 4
means OB=8 and OA= 6 (A+B+C) – (B+C) = 4 – 2 = 2

in triangle AOB; use Pythagoras theorem to find 24


So A = = 12 days
AB=10 cm 2

Q1>Q2
A = 36 days

130 QUANTITATIVE APTITUDE


stportal.mahendras.org

EXERCISE
Q.1-20. In each of the following question, two equations Q.15. (I) 204x2 + 59 x + 2 = 0
(I) and (II) are given. You have to solve them and (II) 300y2 - 20y - 1 = 0
Give answer-
Q.16. (I) y2 + 6x - 40 = 0 
(1) If x>y (2) If x>y
(3) If x<y (4) If x<y (II) y2 + 45y + 476 = 0
(5) If x=y or relationship can not be established Q.17. (I)  x = 25
Q.1. (I) (8x + 21) – (6x + 23) = 0
2 2 4

(II) (7y2 – 14) – (10y2 – 17) = 0   (II) 16y2 - 40y + 25 = 0


Q.2. (I)  2x2 – 17x + 36 = 0 Q.18. (I) 4x2 + 9 - 12x = 0           
(II) 15y2 – 16y + 4 = 0 (II) 64y2- 80y + 25 = 0

Q.3. (I) 7x2 – 41x + 30 =  0 Q.19. (I) 34x2 + 11x = 3  


(II) 51y2 - 77y + 12 = 0 
(II)  7y2 – 26y + 24 = 0
1
Q.4. (I)
(1 9 6 )2 x + 324 = 172 Q.20. (I)

1
(II) (441) 2 y 2 − 288 = 1413
(II)
Q.5. (I)    x –  2 4 0 1 = 0
Q.21-40. In each of the following question, a question is
(II)  y – 7 = 0  followed by information given in two statement
Q.6. (I) 4 x 2 + 4 3 x + 3 = 0 named as Quantity-I (Q1) and Quantity –II (Q2).
You have to study the information along with
(II) 3 y 2 + 4 3 y + 4 = 0 the question and compare the value derived from
Q.7. (I) 16x2 – 8x + 1 = 0            Quantity-I and Quantity-II and give answer-
(1) Quantity I > Quantity II
(II)  100y2 – 20y +1 = 0
(2) Quantity I ≥ Quantity II
Q.8. (I) 14x2 - 53x + 14 = 0 (3) Quantity II > Quantity I
(II) 4y2 - 25 = 0 (4) Quantity II ≥ Quantity I
Q.9. (I) (x + 3)2-25=0 (5) Quantity I = Quantity II or Relation
(II) y -14y + 49 =0
2 cannot be established
Q.21. (I) The age of teacher, if the average age of 36
Q.10. (I) x2 -10x+25=0 students is 14. When teacher’s age is included
(II) y2 + 26y -169 =0  the average increases by 1.
Q.11. (I) x2 - 6.9x + 11.7 = 0 (II) The age of teacher, if the average age of 19
students is 35. When teacher’s age is included
(II) y2-10.2y + 10.8 = 0 the average increases by 0.5.
Q.22. (I) Profit Percentage , if Some articles were bought
1
Q.12. (I) 2x 2 + 20% of 15x − of 2.5 = 0 at 6 articles for Rs. 5 and sold at 5 articles for
50% Rs. 6.
1 (II) Profit Percentage, if 100 toys are bought at the
(II) 2y2- 50% of 36y + of 10 = 0
25% rate of Rs. 350 and sold at the rate of Rs. 48
per dozen.
Q.13. (I) x 2 − 11 11x + 110 = 0 Q.23. (I) On selling 17 balls at Rs. 720, there is a loss
equal to the cost price of 5 balls. The cost price
(II) y 2 − 13 13y + 156 = 0 of a ball is:
Q.14. (I) 18x + 13y = 8 (II) A man buys a cycle for Rs. 1400 and sells it at
(II) 27x + 12y = 12  a loss of 15%. The selling price is:

QUANTITATIVE APTITUDE 131


stportal.mahendras.org
Q.24. (I) A and B together can do a piece of work in 4 Q.32. Find the original consumption if
days. If A alone can do the same work in 6 days, (I) After increasing price by 20% a family now
then B alone can do the same work in? gets 5 kg less on that price.
(II) A can do a piece of work in 4 hours; B and C (II) After increasing price by 25%, a family in-
together can do it in 3 hours, while A and C creases its expenditure by 10% and gets 6 kg
together can do it in 2 hours. How long will B less than original consumption
alone take to do it? Q.33. Find the present age of A.
Q.25. (I) A man on tour travels first 160 km at 64 km/hr (I) Three years before, the ratio of ages of A and
and the next 160 km at 80 km/hr. The average B was 5:6. Three years hence this ratio will
speed of the tour is: become 6:7.
(II) A went from P to Q with the speed of 60km/ (II) Eleven years before the ratio of ages of A and
hr. and return back with the speed of 90km/hr. B was 1:3 and eleven years hence the ratio will
Find the average speed. become 1:2.
Q.26. (I) The ratio between the speeds of two trains is 7 Q.34. Find the sum.
: 8. If the second train runs 400 km in 4 hours, (I) If the Compound Interest for 2 years at 20%
then the speed of the first train is: rate of interest is Rs 1,320.
(II) Find the speed of a train which passes a tree in (II) If the amount on a sum for 2 years in which
12 seconds. The length of the train is 264m. rate of interest for 3 years makes a sum of 125
Q.27. (I) A and B started a business by investing Rs. to amount 216 is Rs 2880.
20000 and Rs. 35000 respectively. Find the Q.35. Find the distance if-
share of B out of an annual profit of Rs. 3520. (I) A man covers a distance in 15 hours. He cov-
(II) X and Y invested in a business. Their profit ratio ers first half at 12 kmph and second half at 15
is 2:3. If X invested Rs. 4000. Find the amount kmph.
invested by Y? (II) Two buses moves towards each other at a speed
Q.28. (I) The age of P is twelve times that of her daughter of 30 kmph and 40 kmph respectively. When
they meet it is found that faster bus covers 30
Q. If the age of Q is 3 years, what is the age of
km more than slower one.
P?
Q.36. (I) Selling price, if cost price is Rs 24,000 and
(II) The ratio between the present ages of A and B profit is 20%
is 2:3. 4 years ago the ratio between their ages
(II) Selling price, if cost price is Rs 24,000 and
was 5:8. What will be A’s age after 7 years? shopkeeper gained 16 2/3% after giving dis-
Q.29. (I) The difference between SI and CI compounded count of 25%
annually on a certain sum of money for 2 years Q.37. (I) Volume, if diameter of sphere is 14 cm
at 8% per annum is Rs. 12.80. Find the principal (II) Volume, if side of cube is 8 cm
(II) A sum fetched a total simple interest of Rs. 800 Q.38. If a & b are natural numbers and 6 ≥ a > b > 0.
at the rate of 8 %per annum in 5 years. What is (I) 5a2 b
the sum? (II) 6ab2
Q.39. (I) The shopkeeper sold an article at 10 % discount
Q.30. There are 5 Brown balls, 4 Blue balls & 3 black balls on marked price and he gains 20%. If the marked
in a bag .Four balls are chosen at random price of the article is Rs.400, then find the cost
(I) The probability of their being 2 Brown and 2 price.
Blue ball. (II) The shopkeeper marks the price of the Pen
Rs.350 and his profit % is 15%. Find the cost
(II) The probability of their being 2 Brown, 1 Blue price of the Pen, if he allows a discount of 8%?
& 1 blacks Q.40. (I) Rakesh can swim at 10 km/hrs in still water.
Q.31. If the quantity of milk in mixture is 10 litre then find The river flows at 6 km/hrs and it takes 6 hours
more upstream than downstream for the same
the quantity of water if distance. How far is the place?
(I) After selling it at Cost price, milkman saves (II) A boy can row 20 km/hrs in still water and the
25% river is running at 8 km/hrs. If the boy takes 4
hours to row to a place and back, how far is the
(II) If the ratio of Milk and water is 5:1 place?

132 QUANTITATIVE APTITUDE


stportal.mahendras.org

EXERCISE Explanation
Q.1.(5) I. 2x2 - 2 = 0 If, y2-10.2y+ 10.8 = 0
2x2 = 2 Then, y = 1.2 and 9
x2 = 1 So, x>y and y>x
Q.12.(3) On solving we get
x=±1
x = -2.5, 1
II. -3y2 + 3 = 0
y= 5 , 4
y2 = 1 x<y
y = ±1 Q.13.(5) If x2- 11 11x + 110 = 0
Hence , Relationship Cannot be established.
Then x = 10 11 and 1 11
Q.2.(1) x= 4.5, 4
If, y 2 − 13 13y + 156 = 0
y = 0.67, 0.4
x>y Then, y = 12 13 and 13
Q.3.(5) I. 7x - 35x - 6x + 30 = 0
2 So, x > y and x < y
7x (x-5) - 6 (x-5) = 0 Q.14.(3) x = 2/15 ; y = 47/135

6 x<y
x = 5,
7 Q.15.(3) x =   -0.25, -0.039
y =   +0.1, -0.033
II. 7y2 - 14y - 12y + 24 = 0
x< y
12
7y (y - 2) - 12 (y - 2) = 0, y = 2, Q.16.(1) x = -10, 4 ; y = -28, -17
7
Q.17.(1) x = 2.5 ; y = 1.25
\ Relation can't be established.
x>y
Q.4.(1) I. 14x + 18 = 172
Q.18.(1) 33 55
14x = 154, x = 11 x= ;y =
22 88
II. 21y2 = 1701, y2 = 81, y = +9
\ x > y x>y
Q.19.(4)
Q.5.(5) I. x = 2401, x = 49 3 1 4 3
x= ,− ;y = ,
17 2 3 17
II. y = 7, y = 49
\x=y x<y

3 3 2 2 Q.20.(1) I. , x = 80
Q.6.(3) x= , ; y =  ,
2 2 3 3
x<y
II. , y = 26
Q.7.(1) x = 0.25, 0.25 ; y = 0.1, 0.1
x>y
x>y
Q.21.(1) I. 14+ (37×1) =14+37=51yrs.
Q.8.(5) 7 2 5
x= , ,y = ± II. 35 + (20×0.5) =35+ 10=45yrs.
2 7 2
Q.9.(3) x=2 ,-8,     y= 7 ,7 11
Q.22.(1) I. Profit %ge = × 1 0 0 =44%
25
So, x<y
Q.10.(1) x= 5, 5,     y= -13, -13 0 .5 2
II. Profit %ge × 100 =14 %
3 .5 7
So, x>y
Q.23.(3) I. C.P. of 12 balls = S.P. of 17 balls = Rs.720.
Q.11.(5) If, x² - 6.9x + 11.7 = 0
720
Then x = 3.9 and 3 CP of 1 ball = =Rs. 60
12
QUANTITATIVE APTITUDE 133
stportal.mahendras.org
II. SP =85% of 1400 12 = 12×3 =36 years.
85 5x + 4 2
= × 1 4 0 0 =Rs. 1190. =
II.
100 8x + 4 3
1 1 2 15x+12 = 16x+8
Q.24.(5) I. B work= - = ⇒ 1 2 d a ys
x=4.
4 6 24
A’s age 4 yrs ago 5×4=20
Then A’s age after 7yrs is
1
II.  A’s 1hr work
. 20+4+7=31yrs.
4
1 Pr2
(B+C’s) 1 hr work . Q.29.(5) I. SI-CI= .
3 1002
1 P × 82
= 1 2 .8
(A+C’s) 1hr work . 2
2 1 0 0
1 1 7 6 4P 2
A+B+C 1hr work = + = . = 1 2 .8
4 3 12 2
1 0 0
7 1 1
B’s work = − = = 12 hours. P=Rs2000.
12 2 12
Pnr
160 160 9 II. SI=
Q.25.(3) I. Total time taken = + = hrs 100
64 80 2
320 P ×8 ×5
Then avg speed = ×2 800=
9 100
2
= 320 × = 71.11km/hr. 800 ×100
9 P= =Rs.2000.
40
2 × 60 × 90 4
II. =72km/hr.
5
c 2 ×4 c 2 60
150 Q.30.(3) I. = =
12
c4 495 33
Q.26.(1) I. Let the speed of two trains be 7x and 8x.
c 2 × 4 c 1 × 3 c 1 182 0
5
8
400 II. = =
= 100
12
c4 9
338 3 3
4
8x=100Þx=12.5. Q.31.(1) Quantity I.
Milk : Water
Then speed of first train =7×12.5
100    :   25
=87.5km/hr.
10=4  : 1=2.5
II. Length of the train = 264m. In Quantity II:
Time taken to pass the tree = 12 seconds. 5=10 ; 1=2
Quantity 1> Quantity 2
264
Speed of the train = m/sec = 22 Q.32.(3) Quantity I: 20% Increase= 1/5 
12
18 1
×T = 5
m/sec = 2 2 × km/hr = 79.2 km/hr. 1+5
5
T=30 Kg
Q.27.(3) I. Ratio 20:35=4:7
Quantity II: 
11 = 3520
                100
7 ?ÞRs2240.
110                        125
4000 2 125-110=15
II. =
y 3 15
×T = 6
y = 6000. 125
Q.28.(1) I. Ratio P:Q 12:1 T=50 Kg
1=3 II > I
134 QUANTITATIVE APTITUDE
stportal.mahendras.org
Q.33.(5) Quantity I:            A                B 7
3 years before       5                6 II. SP = × 2400 = 2800
6
3 years after          6                7
Difference in both case 6-5=1 and 7-6=1 4
Q.37.(1) I. Volume of sphere = × π × r3
1=6 3
6=30 R=7; Volume= 1437.33
Present age =30+3=33
Quantity II:                  A                B II. V=a3 = 83 = 512
11 years before             1                3 I > II
11 years after                1                2 Q.38.(2) Dividing Quantity I and Quantity II we get
Difference A=1-1=0 ; B= -1 I 5a
To make the difference same; multiply equation =
II 6b
2 by 2; we get
Quantity II:                  A                B Now b has to be > 0 and a has to be > b
11 years before             1                3 If a = 2, b = 1, I > II
11 years after                2                4
Similarly we will get I > II in all cases
Difference A=2-1=1; B=4-3=1
1=2 But a = 6 and b = 5
11=22 Then
Present age= 22+11=33 a=b
Q.34.(1) CI for 2 years on 20%=1/5 Quantity I > Quantity II
5             6
Q.39.(1) Quantity I:
5             6
5×5=25   6×6=36 Selling price of the article = 400×(90/100)
CI=36-25=11 =1320 = Rs.360
Sum=25=3000 Cost price of the article = 360×(100/120)
Quantity II: Find Rate = Rs.300
Cube root(125): Cube root (216)  Quantity II:
5:6 Selling price of the Pen = 350×(92/100)
6 −5 1
rate= × 1 0 0 =20% = = Rs.322
5 5
Cost price of the Pen = 322×(100/115)
5             6
5             6 = Rs.280
5×5=25   6×6=36 Quantity I > Quantity II
36=2880; hence 25=2000 Q.40.(3) x/(10-6) - x/(10+6) = 6
I > II
x/4 - x/16 = 6
2 × 12 × 15
Q.35.(3) I. D = × 15 = 200 3x/16 = 6
27
x = 32 km
Quantity II:
Quantity II:
Speed difference for 1 hour =40-30=10
Kmph; means in 1 hour faster bus will Speed of still water (x)
cover 10 km more than slower one; hence = 20 km/hrs, Speed of stream (y) = 8 km/hrs
to cover 30 km more it will take 3 hours. D = t×[(x2- y2)/2x]
Distance= Relative Speed × Time
= 4×[(202 - 82)/(2×20)]
D= 3× (30+40) =210 Km 
= 4×(400 - 64)/40
II > I
= 4×(336/40) = 33.6 km
120
Q.36.(1) I. SP = × 2400 = 2880 Quantity I < Quantity II
100

QUANTITATIVE APTITUDE 135


stportal.mahendras.org

Permutation &
CHAPTER

17 Combination
The study of permutations and combinations is concerned 7! 7! 7 × 6 × 5 × 4!
with determining the number of different ways of arranging (ii) 7
P3 = = = = 210
and selecting objects out of a given number of objects, ( 7 − 3 )! 4 ! 4!
without actually listing them. There are some basic counting
Note : Number of all permuations of n-things, taken all
techniques which will be useful in determining the number
at a time = n!
of different ways of arranging or selecting objects.
Circular Permutations : There are two cases of circular-
Factorial permutations:-
Factorial denoted by n or n ! (a) If clockwise and anti clock-wise orders are different,
Where n! = n. (n-1) (n-2)...............3.2.1 then total number of circular-permutations is given by
(n-1)!
eg. (I.) 5! = 5.4.3.2.1 = 120
(b) If clock-wise and anti-clock-wise orders are taken
(II) 6! = 6.5.4.3.2.1 = 720
as not different, then total number of circular-
from (i),and (II) we get permutations is given by (n-1)!/2
6! = 6.5.4.3.2.1 Combination
6! = 6.5!
(selection, committee, group)
in general n! = n (n-1) !
Each of the different groups or selections which can be
(III) 1! = 1 formed by taking some or all of a number of object called
(IV) 0! = 1 a combinations.
n! = n ×(n-1)! Suppose we want to select two out of three persons A, B,C.
Then possible selections are :
n is greater than 0
AB, BC, CA
1! = 1 ×(1-1)!
1! = 1×(0)! Note : AB and BA represent the same selection.

1 = 0! Suppose we want to select three out of three person A, B, C.


Then possible selections are ABC.
Permutation : (Arrangement)
Note : ABC and BAC and CAB are the same selection.
The different arrangements of a given number or things by
taking some or all at a time, are called permutation. Number of combinations : The number of combinations of
n things, taken r at a time is :
A permutation is an arrangement of objects in a definite order.
n n!
All permutations (or arrangements) made with the letters Cr =
of ‘abc’, taking two at a time are : (ab ba, ac, ca, bc, cb). b g
r !× n − r !

All permutation made with the letters a,b,c taking all at a Note : (i) If (n=r), n C r = 1 and n C 0 = 1
time are :
(abc, acb, bac, bca, cab, cba) Important points
n
Number of Permutations : Number of all permutations of C r = n C bn − r g ,
n things, taken r at a time, is given by :
n! 16 16 !
n
Pr = eg. C 13 =16 C 16 −13 =16 C 3 =
bn − r g! 3 ! × 13 !
e.g.
16 × 15 × 14 × 13 !
6! 6! 6 × 5 × 4! = = 560
(i) 6
P2 = = = = 30 3 × 2 × 1 × 13 !
( 6 − 2 )! 4 ! 4!
136 QUANTITATIVE APTITUDE
stportal.mahendras.org
Conclusion : The basic difference between permutation & Ex. In how many different ways can the letters of the
combination. word ELEPHANT be arranged so that vowels al-
ways occur together?
Permutation
There are three letters a,b,c and permutation made with them Sol. Vowels = E, E and A.
3!
by taking two at a time- a,b,c They can be arranged in= i.e. 3!/2! Ways
2!
ab
ac 6! × 3!
bc So total ways = = 2160
6-different w ays 2!
ba
ca Ex. In how many different ways can the lettes of the
cb word ‘ROSE’ be arranged so that vowels never come
* Here ab, is possible then ba is also possible in permutation. together ?
Combination : There are three person A, B, C we want to Sol. Ways (never)
select two out of three. The number of combination are : ABC
= Ways (total) - Ways (always)
AB
AC = 4! - 3! × 2! = 24 -12 = 12
BA
BC 3-different w ays Ex. In how many different ways can the word
CA ‘TOFFEE’arranged so as, vowels never come to-
CB gether ?
* Here AB is possible, then BA is not possible in combination Sol. Ways (never) = ways (Total) - ways (always)
because they represent same combination.
6! 4 !× 3 !
Result : In the above discussion, we observe that if AB is = − = 180 - 36= 144
2 !× 2 ! 2 !× 2 !
possible and BA is also possible then we apply permutation
and when AB is possible and BA is not possible then we Ex. How many words can be formed by re-arranging
apply combination. the letters of the word CURRENT such that C and
T occupy the first and last position respectively?
Permutation :
Sol. 5!
Ex. In how many different ways can the letters of the = 60
2!
word ROSE be arranged ?
Ex. In how many ways 4 Indians, 5 Africans and 7 Japa-
Sol. n = 4, r = 4
nese be seated in a row so that all person of same
By Formula : nationality sits together?
4 4! 4! 4 × 3 × 2 ×1 Sol. 4! × 5! × 7! × 3!
P4 = = = = 24
( 4 − 4 )! 0 ! 1
Ex. In how many ways 5 Americans and 5 Indians be
Trick : seated along a circular table, so that they are seated
4! = 4 × 3 × 2 × 1 = 24 in alternative positions?

Ex. In how many different ways can the letters of word Sol. 4!x5! =2880
‘Toffee’ arranged ?
Ex. How many necklace of 12 beads each can be made
Sol. Here, ‘F’ and ‘E’ repeated two times therefore- from 18 beads of different colours?
6!
6
P6 ( 6 − 6 )! 6! Sol. Here clock-wise and anti-clockwise arrangements are same.
= =
2
P2 × 2 P2 2! 2! 2 !× 2 ! Hence total number of circular–permutations:
×
( 2 − 2 )! ( 2 − 2 )!
P1 2
18
18 !
6 × 5 × 4 × 3 × 2 ×1 =
= = 180 2 × 12 6 × 24
2 ×1× 2 ×1
Ex. In how many ways can  15  people be seated
Trick : around two round tables with seating capacities
6! 6 × 5 × 4 × 3 × 2 ×1 of 7 and 8 people?
= = 180
2 !× 2 ! 2 ×1× 2 ×1 Sol. 15
C8×6!×7! + 15C7×6!×7!

QUANTITATIVE APTITUDE 137


stportal.mahendras.org

Combination 10 × 9 × 8
= × 5= 120 × 5 = 600
3 ×2
Ex. Find the value of 5c2.
Ex. From a group of 10 men & 5 women. 4 persons are
Sol. 5
c2 = 2 5 = 2 5 = 5 × 4 = 10 to be selected such that either 4 men or 4 women in
5 -2 ×3 2 the group. Find the different number of ways.
Ex. Find the value of n when nc2 = 105 ? 10 5
Sol. c4 + 5c4 =
4 6 + 4 1
10

Sol. n
c2 = 105
n 10 × 9 × 8 × 7
2 n-2 = 105 = + 5= 210 + 5 = 215
4 ×3 ×2
n (n- 1) = 210 { 142 < 210 < 152}
Ex. A committee of 5 members is to be formed out of 4
n = 15
men and 5 women.
Ex. There are 15 persons in a group. They shake hand
(i) In how many ways can a committee consisting
with each other. Find the different no. of hand shake.
of at least 1 woman be formed?
Sol. By formula
Sol. 5
c1 × 4c4 + 5c2 × 4c3 + 5c3 × 4c2 + 5c4 × 4c1 + 5c5 × 4c0
15 15 15 × 14
15
c2 = 2 1 5 – 2 = = = 105 = 5 × 1 + 10 × 4 +10 × 6 + 5 × 4 + 1 = 126
2 13 2
Ex. From a group of 10 men & 5 women 4 persons are to (ii) In how many ways can a committee consisting
be selected to form a committee. Find the different of 3 men and 2 women be formed ?
number of ways for selection. 4 5
Sol. 4
c3 × 5c2 = 3 1 × 2 3
15 15
Sol. 15
c4 = 4 1 5 -4 = 4
11
4 ×5 ×4
15 × 14 × 13 × 12 = = 40
= = 105 × 13 = 1365 2
4 ×3 ×2
Ex. From a group of 10 men & 5 women. 4 person is Ex. There are 6 men and 7 women. In how many ways
to be selected such that 3 men and 1 woman in the a committee of 4 members can be made such that a
group ? particular man is always to be excluded?
Sol. = 10c3 × 5c1 Sol. Here are total 13 people, a particular man is to be
10 5 10 5 excluded, so now 12 people are left to chosen from
= × = × and 4 members to be chosen. So ways are 12C4 = 495.
3× 7 1 4 3 7 4

138 QUANTITATIVE APTITUDE


stportal.mahendras.org

EXERCISE
Q.1. Find the number of different words that can be formed from the word 'SUCCESS'.
(1) 360 (2) 480 (3) 420 (4) 5040 (5) None of these
Q.2. In how many different ways can the letters of the word ‘LEADING’ be arranged in such a way that the vowels
always come together?
(1) 360 (2) 480 (3) 720 (4) 5040 (5) None of these
Q.3. In how many different ways can the letters of the word ‘CORPORATION’ be arranged so that the vowels always
come together?
(1) 810 (2) 1440 (3) 2880 (4) 50400 (5) 5760
Q.4. In how many diffirent ways can the alphabets of word "Equation" can be arranged taking 5 letters at a time
with the condition that the letters can be repeated?
(1) 32768 (2) 34569 (3) 37268 (4) 21724 (5) None of these
Q.5. In how many ways the alphabets of the word "CREATION" can be arranged so that all the consonants are alternate?
(1) 1052 (2) 952 (3) 1152 (4) 852 (5) None of these
Q.6. In how many ways the letters of the word "WOMEN" can be rearranged so that when constant occupies odd places,
vowel comes on even places ?
(1) 10 (2) 11 (3) 12 (4) 13 (5) 14
Q.7. In how many ways can 5 men, 3 women and 2 girls can be seated in a row so that the people of same gender are
not seated together ?
(1) 3718640 (2) 3729123 (3) 3628800 (4) 3620160 (5) None of these
Q.8. In how many ways can 7 boys and 5 girls be seated in a row so that no two of the girls can be together ?
(1) 9676800 (2) 9266420 (3) 9352980 (4) 3456780 (5) None of these
Q.9. A round table meeting is held between eight persons A, B, C, D, E, F G and H. In how many ways can they seated
so that A and D always sits diagonally opposite to each other?
(1) 1220 (2) 1330 (3) 1110 (4) 1440 (5) None of these
Q.10. How many ways can 4 apples be given in such a way to 3 girls if one girl can recieve 1 apple.
(1) 64 (2) 360 (3) 6 (4) 24 (5) None of these
Q.11. A group photograph of a family having 6 females and 20 males is to be taken the first row consist of women and the
second row consist of boys with 2 tallest boys standing at the extreme corners of the second row. Find the number
of arrangement.
(1) 2 × 18! × 6! (2) 18! × 6! (3) 3 × 6! × 18! (4) 9! × 14! (5) None of these
Q.12. Find the total number of employee code that can be formed by using two alphabets followed by 2 numbers and the
letters should be distinct.
(1) 61000 (2) 54000 (3) 42000 (4) 63000 (5) None of these
Q.13. 12 points lie on a circle. How many cyclic quadrilaterals can be drawn by using these points ?
(1) 595 (2) 495 (3) 394 (4) 295 (5) 410
Q.14. Two boxes A and B contain 5 balls each. We have to choose 6 balls in all of which at least 2 should be from Box A
and atleast 2 from Box B. In how many ways the selection can be made?
(1) 150 (2) 180 (3) 165 (4) 200 (5) None of these
Q.15. There are 5 boys and 4 girls, In how many ways 4 boys and 2 girls can be seated on 6 chairs?
(1) 6400 (2) 12500 (3) 21600 (4) 34300 (5) None of these
Q.16. There are 5 professor, 8 lecturer and 7 teacher. 6 people to be selected to make an interview board. In how many ways
can they selected if it contain equal number of professors, lecturers and teachers?
(1) 5880 (2) 4770 (3) 3450 (4) 2180 (5) None of these
Q.17. A committee of 5 persons is to be formed from 6 men and 4 women. In how many ways can this be done when at
most 2 women are included?
(1) 124 (2) 132 (3) 186 (4) 174 (5) None of these

QUANTITATIVE APTITUDE 139


stportal.mahendras.org
Q.18. A football team of 11 players is to be selected out of 16 players. 16 players consists of 2 goal keepers and 5 defenders
and rest forwards. In how many ways can it be selected so that it consist of 1 goal keeper and at least 4 defenders?
(1) 992 (2) 1100 (3) 1092 (4) 999 (5) None of these
Q.19. In a cricket tournament, there are 153 matches played. Every two team played one match with each other. The
number of team participating in the tournament is-
(1) 12 (2) 11 (3) 18 (4) 14 (5) 16
Q.20. A college has 10 volleyball players and 6 captain.Make a team of 6 members including captain. How many different
selections can be made ?
(1) 1134 (2) 1100 (3) 1300 (4) 1000 (5) None of these
Q.21. The total number of committee of 7 people is to be formed from 9 boys and 6 girls such that the boys are in the
majority and it has atleast 1 girl-
(1) 4914 (2) 2072 (3) 2076 (4) 3426 (5) None of these
Q.22. How many number plates of 3 digit can be formed with four digits 1,2,3 and 4 ?
(1) 27 (2) 24 (3) 8 (4) 20 (5) 16
Q.23. How many numbers of five digits can be formed with the digits 1,3,5 7 and 9 no digit being repeated ?
(1) 120 (2) 240 (3) 720 (4) 360 (5) 5040
Q.24. How many different 5 - digit numbers can be formed by using the digits of the number 713628459?
(1) 15210 (2) 15120 (3) 15180 (4) 45360 (5) 30240
Q.25. How many numbers of five digits can be formed with the digits 0,2,4,6 and 8 ?
(1) 24 (2) 48 (3) 96 (4) 120 (5) None of these
Q.26. How many even numbers of three digits can be formed with the digits 0,1, 2, 3, 4, 5 and 6 ?
(1) 30 (2) 105 (3) 75 (4) 180 (5) 150
Q.27. A round table conference is to be held between delegates of 15 companies. In how many ways can they be seated
if delegates from two MNCs may wish to sit together ?
(1) 14! (2) 14!×2! (3) 14!/2! (4) 14!×2!-1 (5) None of these
Q.28. A person has 12 friends out of which 7 are relatives. In how many ways can he invite 6 friends such that at least 4
of them are relatives ?
(1) 460 (2) 426 (3) 432 (4) 464 (5) 462
Q.29. How many words can be made of letters COURTESY which began with C and end with Y ?
(1) 120 (2) 540 (3) 420 (4) 720 (5) 1040
Q.30. Of 8 balls how many arrangement can be made in which 3 given balls are not all together ?
(1) 8!-6!×2! (2) 6!-8!×2! (3) 8!-6!×3! (4) 6!-8!×3! (5) None of these
Q.31. From a group of 7 men and 6 women, five persons are to be selected to form a committee so that at least 3 men are
there in the committee. In how many ways can it be done?
(1) 564 (2) 645 (3) 735 (4) 756 (5) None of these
Q.32. Out of 7 consonants and 4 vowels, how many words of 3 consonants and 2 vowels can be formed?
(1) 210 (2) 1050 (3) 25200 (4) 21400 (5) None of these
Q.33. A box contains 2 white balls, 3 black balls and 4 red balls. In how many ways can 3 balls be drawn from the box,
if at least one black ball is to be included in the draw?
(1) 32 (2) 48 (3) 64 (4) 96 (5) None of these
Q.34. In how many different ways can the letters of the word ‘DETAIL’ be arranged in such a way that the vowels occupy
only the odd positions?
(1) 32 (2) 48 (3) 36 (4) 60 (5) 120
Q.35. How many 4-letter words with or without meaning, can be formed out of the letters of the word, ‘LOGARITHMS’,
if repetition of letters is not allowed?
(1) 40 (2) 400 (3) 5040 (4) 2520 (5) 13
Q.36. The Indian Cricket team consists of 16 players. It includes 2 wicket keepers and 5 bowlers. In how many ways can a cricket
eleven be selected if we have to select 1 wicket keeper and at least 4 bowlers?
(1) 1029 (2) 1072 (3) 1040 (4) 1092 (5) None of these
140 QUANTITATIVE APTITUDE
stportal.mahendras.org
Q.37. In a small village, there are 87 families, of which 52 families have at most 2 children. In a rural development
program 20 families are to be chosen for assistance, of which at least 18 families must have at most 2 children. In
how many ways can the choice be made?
(1) 52C19×35C1+50C19×37C1+52C20 (2) 52C18×35C2+52C19×35C1+52C20
(3) 52C18×35C2+52C19×35C1-52C20 (4) 52C18×35C2-52C19×35C1+52C20
(5) None of these
Q.39. How many arrangements can be made out of the letters of the word COMMITTEE, taken all at a time, such that
the four vowels do not come together?
(1) 216 (2) 720 (3) 1260 (4) 4320 (5) None of these
Q.40. In how many ways can the letters of the word EDUCATION be rearranged so that the relative position of the
vowels and consonants remain the same as in the word EDUCATION?
(1) 4!×5! (2) 5!×3! (3) 4!×4! (4) 5!×5! (5) None of these
Q.41. 3 boys and 2 girls are to be seated in a row in such a way that two girls are always together. In how many different
ways can they be seated?
(1) 12 (2) 24 (3) 72 (4) 36 (5) 48
Q.42. If the letters of the word CHASM are rearranged to form 5 letter words such that none of the word repeat and the results
arranged in ascending order as in a dictionary what is the rank of the word CHASM ?
(1) 24 (2) 32 (3) 60 (4) 48 (5) 120
Q.43. How many arrangements of the letters of the word ‘BENGALI’ can be made if the vowels are to occupy only odd
places.
(1) 576 (2) 567 (3) 625 (4) 720 (5) None of these
Q.44. In how many ways 3 mathematics books, 4 history books, 3 chemistry books and 2 biology books can be arranged
on a shelf so that all books of the same subjects are together.
(1) 42472 (2) 41472 (3) 44725 (4) 45722 (5) 46427
Q.45. There are four bus routes between A and B; and three bus routes between B and C. A man can travel round-trip
in number of ways by bus from A to C via B. If he does not want to use a bus route more than once, in how many
ways can he make round trip?
(1) 72 (2) 44 (3) 32 (4) 19 (5) 24
Q.46. What is the total number of ways in which Dishu can distribute 99 distinct gifts among his 88 distinct girlfriends
such that each of them gets at least one gift?
(1) 172x8! (2) 144x8! (3) 36x8! (4) 72x8! (5) None of these
Q.47. From a total of six men and four ladies a committee of three is to be formed. If Mrs. X is not willing to join the
committee in which Mr. Y is a member, whereas Mr.Y is willing to join the committee only if Mrs Z is included. 
How many such committee are possible?
(1) 138 (2) 128 (3) 112 (4) 91 (5) None of these
Q.48. The number of ways which a mixed double tennis game can be arranged amongst 9 married couples if no husband
and wife play in the same is:
(1) 1514 (2) 1512 (3) 3024 (4) 3028 (5) None of these
Q.49. A basket contains 6 blue, 2 red, 4 green and 3 yellow balls. If 5 balls are picked at random find the number of ways
for selecting the balls such that at least one is blue?
(1) 2877 (2) 2651 (3) 2400 (4) 2600 (5) None of these
Q.50. There are eight boxes of chocolates, each box containing distinct number of chocolates from 1 to 8. In how many
ways four of these boxes can be given to four persons (one boxes to each) such that the first person gets more
chocolates than each of the three, the second person gets more chocolates than the third as well as the fourth persons
and the third person gets more chocolates than fourth person?
(1) 70 (2) 72 (3) 40 (4) 80 (5) None of these

QUANTITATIVE APTITUDE 141


stportal.mahendras.org

EXERCISE Explanation
Q.1.(3) No. of Permutation = n! / p! × q!, where p Q.8.(1) _ B _ B _ B _ B _ B _ B _ B _
= of one type , q = ( of another type ). Boys = 7!
No. of Permutation = 7!/ 3! × 2! Girls = 8p5
No. of Permutation = 420 7 ! × 8p5 = 9676800 ways
Q.2.(3) Then, we have to arrange the letters LNDG (EAI). Q.9.(4) A
Now, 5 (4 + 1 = 5) letters can be arranged in 5!
= 120 ways.
The vowels (EAI) can be arranged among
themselves in 3! = 6 ways. D
 Required number of ways = (120 x 6) = 720.
A and D can be seated in 2! ways
Q.3.(4) In the word 'CORPORATION', we treat the vowels
and others can be seated in 6! ways
OOAIO as one letter.
Total number of ways = 2! × 6! = 1440 ways
Thus, we have CRPRTN (OOAIO).
Q.10.(4) G1 G2 G3
This has 7 (6+1) letters of which R occurs 2 times
and the rest are different. ↓ ↓ ↓
Number of ways arranging these letters 4 × 3 × 2 = 24 ways
7! Q.11.(1) Difference = 2 × 18! × 6!
= = 2520
2! Q.12.(5) Out of 26 alphabets two distinct letters can be
arranged in 26p2 ways.
Now, 5 vowels in which ) occurs 3 times and the
5! The first digit of the number in 10 ways
rest are different, can be arranged in = 20 day
3! The second digit of the number in 10 ways
So, total number of ways
Required number of ways = (2520×20) = 50400
= 26p2 × 10 × 10 = 65000
Q.4.(1) The word formed will be of 5 letters and the letters
can be repeated. Q.13.(2) Any set of 4 points we get a cycle combination
quadrilateral number of ways of choosing 4 points
_ _ _ _ _
out of.
8×8×8×8×8
12 points is 12c4 = 495
85 = 32768 ways
Q.14.(4) Part A Part B
Q.5.(3) In one arrangement consonants will come on even
So,
places and in are arrangement they will come on
odd places (2 and 4), (3 and 3), (4 and 2),
therefore 2 × 4! × 4! = 1152 ways = 5c2 × 5c4 +5c3 × 5c3 + 5c4 × 5c2
Q.6.(3) Consonants comes on 1, 3, 5 place
= 10×5 + 10 × 10 + 5 × 10
So, total arrangement = 3! = 6 = 50 + 100 + 50 = 200 ways
Vowels comes on 2, 4 place Q.15.(3) 4 boys can be selected out of in 5c4 ways
So, total arrangement = 2! = 2 = 2 girls can be seated out of 4 in 4c2 ways
So, total arrangement = 6 × 2 = 12 ways Number of selecting = 5c4 × 4c2 = 30
Q.7.(4) Never together = All - together Ways of arranging = 6!
= 10 ! - 3! × 5! × 3! × 2! Total number of arrangement
= 3628800 - 8640 = 30 × 720 = 21600
= 3620160 ways Q.16.(1) 5c2 × 8c2 × 7c2 = 5, 880

142 QUANTITATIVE APTITUDE


stportal.mahendras.org
Q.17.(3) When at most 2 women are included, the committee Q.25.(3)
many consist of
ten thou- thousand hundreds tens units place
3 men & 2 women or sand place place place place
4 man 1 women
or
in 4P1 i.e. After flling up ten thousands place we are
5 men & 0 women 4 ways left with 4 digits including 0 and the number
So, 4c2 × 6c3 + 4c1× 6c4 + 6c5 (any one of blank places is 4. So, in 4P4 = 4! = 24 ways
of 2/4/6/8
120 + 60 + 6 = 186 ways.
Q.26.(2) Case (i) : When 0 occurs at units place :
Q.18.(3) According to the condition the team can be selected
in the following ways : Hundreds tens place units place
1 goalkeeper and 4 defender and 6 forward Place
or
6
P2 = 6×5 = 30 ways Only o, i.e. in 1
ways
1 goal keeper and 5 defender and 5 forward
Total such numbers = 30 × 1 = 30
So, 2c1 × 5c4 × 9c6 + 2c1 × 5c5 × 9c5
Case (ii) : When 0 does not occur at unit place :

840 + 252
After filling of After filling up any one of in 3
1092 ways
unit place we units place and ways
Q.19.(3) nc2 = 153 are left with hundreds place
Let n be the number of team participating 6 digit but 0 we are laft with
cannot occur at 5 digits ( includ-
n! hundred? place. ing O ; so in 5
= 153
n ! (n − 2 ) ! We are finally ways.

left with d digits,
On solving n = -17 and n = 18 so in P = 5 ways
n = 18 Total of such numbers = 5 × 5 × 3 = 75
Q.20.(1) 5 members can be selected from 10 players in req no. = 30+75 = 105
= 10c5 ways Q.27.(2) Since delegates from two multinational companies
will sit together, so considering these two delegates
1 captain can be selected from 6 players in
as one unit, there will be 13 + 1 = 14 delegates who
= 6c1 ways can be arranged in a circular table in 14! ways.
Total ways = 10c5 × 6c1 = 189 × 6 = 1134 ways The two delegates from the MNCs can be
arranged among themselves in 2! ways.
Q.21.(1) 6c1 × 9c6 +6c2 × 9c5 +6c3 × 9c4 = 4914 Using the product rule, the required no. of ways
Q.22.(2) Here, the order of arrangement of digits does = 14!×2!
matter.
Q.28.(5) No. of non-relative friends = 12-7 = 5
n
Pr = n! / (n-r)!
He may invite 6 friends in following ways:
n
Pr = 4! / (4-3)!
I : 4 relatives + 2 non-relatives Þ7C4 × 5C2
4
P3 = 4! / 1!
II : 5 relatives+ 1 non-relativesÞ 7C5 × 5C1
  P3 = 4! 4
III : 6 relatives + 0 non-relativesÞ 7C6
  P3 = 24 4
Required number of ways
Q.23.(1) the no. of digits = 5
= C 4 × C 2 + C 5 × C1 + C 6
7 5 7 5 7

Required no. = 5P5 = 5! = 120 = 35×10+21×5+7=350+105+7=462
Q.24.(2) Pr = n! / (n-r)!
n
Q.29.(4) Since first letter C and last letter Y are fixed. Therefore
9
P5 = 9! / (9-5)! we have to permutate only six letters between C and Y,
C O U R T E S Y
9
P5 = 9! / 4!
Therefore, total number of words = 6!
9
P5 = 15,120 
= 720 
QUANTITATIVE APTITUDE 143
stportal.mahendras.org
Q.30.(3) Arranging 8 balls in 8 places Also, the 3 consonants can be arranged at the
Total number of ways = 8!  remaining 3 positions.
Number of ways of these arrangements
Therefore, 
= 3P3 = 3! = 6
Number of ways in which 3
Total number of ways = (6 × 6) = 36
given balls are together= 6!.3!
Q.35.(3) 'LOGRAITHMS' contains 10 different letters.
Therefore,
Required number of words = Number of
Number of ways in which 3 given balls are not all arrangements of 10 letters, taking 4 at a time.
together = 8! - 6!.3!
= 10P4= (10 × 9 × 8 × 7) = 5040
Q.31.(4) We may have (2 men and 3 women) or (4 men and
1 women) or (5 men only) Q.36.(4) We are to choose 11 players including 1 wicket
keeper and 4 bowlers  or, 1 wicket keeper and 5
Required number of ways bowlers. 
= ( 7
) (
C 3 ×6 C 2 + 7
C 4 ×6 C1 + ) ( 7
C5 ) Number of ways of selecting 1 wicket keeper, 4
bowlers and 6 other players in 
2
C1 x 5C4 x 9C6 = 840 
7×6×5 6×5
= ×
 3 × 2 × 1 2 × 1 
+ ( 7
C 3 × C1 +
6
) ( 7
C2 ) Number of ways of selecting 1 wicket keeper, 5
bowlers and 5 other players in 2C1x5C5x9C5=252

 7 × 6 × 5   7 × 6 Total number of ways of selecting the team = 840


= 525 +  ×6 + + 252 = 1092
 3 × 2 × 1   2 × 1 
Q.37.(2) Hence, the total number of possible choices is 52C18
= (525 + 210 +21)= 756. × 35C2 + 52C19 × 35C1 + 52C20
Q.32.(3) Number of ways of selecting (3 consonants out of Q.39.(4) Hence, the required number of ways in which the
7) and (2 vowels out of 4) four vowels do not come together
9 4× 6
 7 × 6 × 5 4 × 3
( 7
C 3 ×4 C 2 ) = ×
 3 × 2 × 1 2 × 1 
= 210. = 2 2 2− 2 2 2

= 45360 - 2160 = 43200


Each group, each having 3 consonants and 2
Q.40.(1) Hence, the total number of ways = 4! × 5!
vowels = 210.
= 24 × 120 = 2880
Each group contants 5 letters.
Q.41.(5) 4!×2! = 48
Number of ways of arranging = 5!
Q.42.(2) The 5 letter word can be rearranged in 5!=120
= 5×4 ×3 × 2 × 1 = 120 Ways without any of the letters repeating.
Required number of ways = (210 × 120) The first 24 of these words will start with A.
= 25200 Then the 25th word will start will CA _ _ _. 
Q.33.(3) 2 W, 3 B, 4 R The remaining 3 letters can be rearranged in 3!=6
Ways.
Required probability = 3C1 × 6C1 + 3C2 × 6C1 + 3C3
i.e. 6 words exist that start with CA.

= 3 ×15 + 3 × 6 + 1
The next word starts with CH and then A, i.e., CHA
= 45 + 18 + 1 _ _. 
= 64 The first of the words will be CHAMS.
Q.34.(3) There are 6 letters in the given word, out of which The next word will be CHASM.
there are 3 vowels and 3 consonants. Let us mark
Therefore, the rank of CHASM will be 24+6+2= 32
these positions as under :
Q.43.(1) There are 7 letters in the word Bengali of these 3
(1) (2) (3) (4) (5) (6)
are vowels and 4 consonants.
Now, 3 vowels can be placed at any of the three There are 4 odd places and 3 even places. 3
places out 4, marked 1, 3, 5. vowels can occupy 4 odd places in  4P3 ways and
Number of ways of arranging the vowels 4 constants can be arranged in 4P4 ways.
= 3P3 = 3! = 6. Number of words =4P3 x 4P4= 24 x 24 = 576
144 QUANTITATIVE APTITUDE
stportal.mahendras.org
Q.44.(2) First we take books of a particular subject as one case (ii): If Mr. Y is not a member then we left
unit. Thus there are 4 units which can be arranged with (6+4−1) people.
in 4! = 24 ways. Now in each of arrangements, So total ways
mathematics books can be arranged in 3! ways,
history books in 4! ways, chemistry books in 3!
7
C1 + 9C3 = 91
ways and biology books in 2! ways. Thus the total Q.48.(2) Step I:  Two male members can be selected
number of ways = 4! × 3! × 4! × 3! × 2! = 41472 in 9C2=36
Q.45.(1) There are 4 bus routes from A to B and 3 routes Step II:  Having selected two male members, 2
from B to C. Therefore, there are 4 × 3 = 12 ways female members can be selected in 7C2=21 ways.
to go from A to C. It is round trip so the man will
Step III: Two male and two female members can
travel back from C to A via B. It is restricted that
arranged in a particular game in 2 ways.
man can not use same bus routes from C to B and
B to A more than once. Thus, there are 2 × 3 = 6 Total number of arrangements,36×21×2=1512 ways.
routes for return journey. Therefore, the required Q.49.(1) Total  number of balls = 6 + 2 + 4 + 3 = 15.
number of ways = 12 × 6 = 72.
Number of non-blue balls = 15 - 6 = 9
Q.46.(4) As every girl friend should get one gift.
Number of ways of selecting 5 non-blue balls
The number of ways  88  distinct gifts can be
selected is: 9C8= 9ways. = 9C5
The number of ways each GF gets one gift each Number of ways of selecting 5 balls out of 15 balls
out of these 8 selected gifts 8!. = 15C5
Total number of ways 8 gifts can be distributed Number of ways for selections such that atleast one
is 9×8! ball is blue.
Now the last one gift can be given to any of
the  8  GF hence the total number of ways of 1 5 c5 – 9 c5 = 3 0 0 3 − 1 2 6 = 2 8 7 7

distributing,
Q.50.(1) For each combination of 4 out of 8 boxes,
=9×8!×8=9×8!×8=72 × 8! ways.
the box with the greatest number has to be
Q.47.(4) case (i): As Mr. Y agrees to be in committee only given to the first person, the box with the
where Mrs. Z is a member. second highest to the second person and so on.
Now we are left with (6−1) men and(4−2) ladies The number of ways of giving 4 boxes to the 4
(Mrs. X is not willing to join). person is: 8C4= 70

QUANTITATIVE APTITUDE 145


stportal.mahendras.org

CHAPTER

18 PROBABILITY

Some useful facts 3


Probability of the even number =
If one dice is rolled randomly, the set of total possible 6
outcomes are {1, 2, 3, 4, 5, 6} and the set of all 1
P (E) =
outcomes even number is 2,4,6. 2
From the above example we design some definitions Results :
I. Experiment : An activity which when performed any (i) P (S) = 1...........(maximum probability is 1)
number of times under identical condition gives the (ii) P (φ) = 0..........(minimum probability is 0)
same (unique) outcome.
(iii) 0 < P (E) < 1
Ex. When we measure the three angles of any triangle we
always get their sum as 1800. (iv) P(E) + P ( E ) = 1
II. Random Experiment : An experiment is called a ie. P (E) = 1- P ( E ) ............Where E denotes not E
random experiment when it satisfies the following
two conditions: Ex. In a question there are 5-options in which one is right
and remaining 4 are wrong, if one option is to be
(1) It has more than one possible outcome. selected randomly, what is the probability that-
(2) It is not possible to predict the outcome in advance.
(I) The question will be right
Ex. Tossing a coin; tossing a dice.
(II) The question will be wrong
III. Sample space : The set of total possible outcomes
in an random experiment is known as sample space, 1
Sol. P (E) = .............(I)
denoted by S. 5
- For rolling a dice S = {1, 2, 3, 4, 5, 6} 4
P (E ) = ..................(II)
- For tossing a coin once, S = {H, T} 5

- If two coins are tossed, then Adding these equation (I) & (II), we get-
1 4
S = {HH, HT, TH, TT}

bg ej
P E +P E = +
5 5
IV. Event : Any subset of a sample space is called an
event, denoted by E.
bg ej
i.e. P E + P E = 1 ..........(Remember that)
In the above, E = {2, 4, 6} is an event and E ⊆ S
Ex. Find the probability of getting head when single coin
V. Probability of occurrence of an event(E) : is tossed.

Probability of any event(E) =


nEb g i.e. Sol. For single coin
n bS g n(E) = 1 {H}
P bE g =
nEbg
n bS g n(S) = 2 {H, T}

Where n (E) ⇒ Total number of favourable outcomes 1
P(E) =
n (S) ⇒ Total number of possible outcomes. 2
Ex. Find the probability of getting 1 head when two coins
P (E) ⇒ Probability of Event. are tossed simultaneously.
For rolling a dice
Sol. For two coins
n(S) = 4 {(H, H) (T, T) (H, T) (T, H)}
n(E) = 2 {(H, T) (T, H)}

146 QUANTITATIVE APTITUDE


stportal.mahendras.org

2 1 Sol. P(E) =
nE bg
P(E) = = n S bg
4 2
Ex. Find the probability of getting at least one head when LM1,3 2 ,6 2,2 OP
9 1
two coins are tossed simultaneously. MM 3,1 6 ,2 6,6 P =
4,4 P ,
36 4
Sol. For two coins

MN3,5 5,3
PQ
n(S) = 4 {(H, H) (T, T) (H, T) (T, H)}
Ex. Find the probability that sum is a prime no. & less
Here, n(E) = 3 {(H, H) (H, T) (T, H)} than 8 when two dice are thrown simultaneously.
1 + 2 + 4 + 6 13
Sol. P(E) = =
3 36 36
P(E) =
4 Note : Sum Ways No. of results
Ex. Find the probability of getting at most one head when
two coins are tossed simultaneously. 2 (1, 1) 1
Sol. n(S) = 4 {(H, H) (T, T) (H, T) (T, H)} 3 (1,2) (2,1) 2
n(E) = 3 {(T, T) (H, T) (T, H)} 5 (3,2) (2,3) (1,4) (4,1) 4
7 (1, 6) (6, 1) (4, 3) (3, 4)
3
P(E) = 4 DICE (2, 5) (5, 2) 6

Ex. Find the probability of getting a multiple of 3 when


Cards
one dice is thrown once. C A R D S (52 )

Sol. n(S) = 6 {1, 2, 3, 4, 5, 6} R e d (2 6) B lac k (2 6)

n(E) = 2 {3, 6}
D ia m o nd H e art C lu b Spad e
13 13 13 13
2 1
P(E) = = 2, 3, 4, 5, 6, 7, 8, 9, 10 – no. cards
6 3
→ 9×4 = 36
Ex. Find the probability of that number which is a multiple
of 2 when one dice is thrown once. Ace, King, Queen; Jack - Honour cards
Sol. n(S)=6 {1, 2, 3, 4, 5, 6,} → 4×4 = 16
n(E) = 3 {2, 4, 6} King, Queen; Jack - Face cards
3 1 → 3×4 = 12
P(E) = =
6 2 Ex. If from a pack of 52 cards, 1 card is drawn at random.
Ex. Find the probability that sum on both faces is 9 when Find the probability that the card is an Ace card.
two dice are thrown simultaneously. Solution :
Sol. 62 = 36 n (E ) 4
C1
P(E) = = 4 1
n(S) = 36 = =
n( S ) 52
C1 52 13
LM6, 3 OP Ex. If from a pack of 52 playing cards, 1 card is drawn at
MM3, 6 PP random. What is the probability that it is either a king
or queen?
MN5,4, 45 PQ
4 1
4
C 1 +4 C 1 8 2
= Sol. P(E) = = =
n (E) = 4, P(E) = 52
C1 52 13
36 9
Ex. Find the probability that sum is divisible by 4 when two
dice are thrown simultaneously.

QUANTITATIVE APTITUDE 147


stportal.mahendras.org

EXERCISE
Q.1-5. Study the given information carefully to answer the questions that follow.
An urn contains 4 green, 5 blue, 2 red and 3 yellow marbles.
Q.1. If four marbles are drawn at random, what is the probability that two are blue and two are red ?
10 9 17 2
(1) (2) (3) (4) (5) None of these
1001 14 364 7
Q.2. If eight marbles are drawn at random, what is the probability that there are equal number of marbles of each colour?
4 361 60 1
(1) (2) (3) (4) (5) None of these
7 728 1001 1
Q.3. If two marbles are drawn at random, what is the probability that both are red or at least one is red?
26 1 199 133
(1) (2) (3) (4) (5) None of these
91 7 364 191
Q.4. If three marbles are drawn at random, what is the probability that at least one is yellow ?
1 199 165 3
(1) (2) (3) (4) (5) None of these
3 364 364 11
Q.5. If three marbles are drawn at random, what is the probability that none is green ?
2 253 10 14
(1) (2) (3) (4) (5) None of these
7 728 21 91
Q.6. A person teaches to three children. What is the probality that no one among them will be a girl.
1 1 1 1
(1) (2) (3) (4) (5) 0
2 16 3 8
Q.7. The letter I, G, L, N & C are arranged to form the word CLING. What is the probability?
1 1 1 3
(1) (2) (3) (4) (5) None of these
120 64 36 5
Q.8. A bag contains 10 white balls and 16 black balls. Two balls are drawn in succession without replacement. What is
the probability that first is white and second is black?
18 17 35 37
(1) (2) (3) (4) (5) None of these
145 29 134 145
Q.9. Four boys and three girls sit in a row for interview. The probability that they will sit in alternate position is-
1 1 1 1 1
(1) (2) (3) (4) (5)
3 4
23 35 15 65
Q.10. What is the probability that number selected from number 1,2,3......100 is a prime number, when each number is
equally likely to be related?
9 8 11 1 10
(1) 3 0 (2) 3 5 (3) 3 0 (4) 4 (5)
30
Q.11. A bag contains 6 black, 5 white and 8 yellow balls. If four balls are picked at random, what is the probability that
all four of them are either black or any two out of the four are white?
3 925 359 11
(1) (2) (3) (4) (5) None of these
1292 3876 1927 387
1 1 3
Q.12. Ramesh play 3 sets of a badminton match. The probability of his winning the three sets are , , respectively.
7 5 4
What is probability that he will win atleast 1 set of the game?
1 7 13 23
(1) (2) (3) (4) (5) None of these
24 24 24 24
Q.13. What is probability of a head and even number when a coin is tossed and a dice is thrown?
1 1 1 1
(1) (2) (3) (4) (5) None of these
2 3 4 5
148 QUANTITATIVE APTITUDE
stportal.mahendras.org
Q.14. What is probability of getting a sum greater than 15 if three dice are rolled simultaneously?
(1) 5/54 (2) 15/216 (3) 11/216 (4) 17/216 (5) None of these
Q.15. The probability of sucess of A, B, C in an interview is 1/5, 1/4, 1/3 respectively. Find the probability of sucess of
atleast two?
(1) 1/6 (2) 3/5 (3) 3/4 (4) 2/5 (5) None of these
Q.16. Two team x and y are participating in quiz contest. The probability that team x will win is 5/8 and the probability
that team y will win is 1/5. What is the probability that either team x or team y will win the contest?
(1) 3/5 (2) 5/7 (3) 6/7 (4) 33/40 (5) 21/30
Q.17. A 5 digit number is formed by the digits 1, 2, 3, 4 and 5 without repitition. What is the probability that the number
formed is a multiple of 4?
(1) 1/2 (2) 1/3 (3) 1/4 (4) 1/5 (5) 1/6
Q.18. Two unbiased coins are tossed. What is the probability of getting at most one tail?
(1) 1/2 (2) 1/3 (3) 2/3 (4) 3/4 (5) 1/4
Q.19. Three unbiased coins are tossed, What is the probability of getting atleast 2 tail?
(1) 1/2 (2) 1/3 (3) 1/6 (4) 1/8 (5) 1/10
Q.20-22. There are 3 bags containing 3 colored balls – Red, Green and Yellow.
Bag 1 contains: 24 green balls. Red balls are 4 more than yellow balls. Probability of selecting 1 red ball is 4/13.
Bag 2 contains: Total balls are 8 more than 7/13 of balls in bag 1. Probability of selecting 1 red ball is 1/3. The ratio
of green balls to yellow balls is 1 : 2
Bag 3 contains: Red balls is equal to total number of green and yellow balls in bag 2. Green balls is equal to total number
of green and red balls in bag 2. Probability of selecting 1 yellow ball is 3/14.
Q.20. 1 ball each is chosen from bag 1 and bag 2, What is the probability that 1 is red and other is yellow?
(1) 15/128 (2) 21/115 (3) 17/135 (4) 25/117 (5) 16/109
Q.21. Some green balls are transferred from bag 1 to bag 3. Now probability of choosing a yellow ball from bag 3 becomes
3/16. Find the number of remaining balls in bag 1.
(1) 60 (2) 58 (3) 5 (4) 48 (5) 44
Q.22. Green balls in ratio 4: 1 from bags 1 and 3 respectively are transferred to bag 4. Also 4 and 8 red balls from bags 1
and 3 respectively. Now probability of choosing green balls from bag 4 is 5/11. Find the number of green balls in
bag 4?
(1) 12 (2) 15 (3) 10 (4) 9 (5) 11
Q.23-24. There are 3 people – A, B and C. Probability that A speaks truth is 3/10, probability that B speaks truth is 3/7
and probability that C speaks truth is 5/6. For a particular question asked, at most 2 people speak truth. All people
answer to a particular question asked.
Q.23. What is the probability that B will speak truth for a particular question asked?
(1) 7/1 (2) 14/33 (3) 4/15 (4) 9/28 (5) 10/33
Q.24. A speaks truth only when B does not speak truth, then what is the probability that C does not speak truth on a ques-
tion?
(1) 11/140 (2) 21/180 (3) 22/170 (4) 13/140 (5) None of these
Q.25. Four persons are chosen at random from a group of 3 men, 3 women and 4 children. What is the probability that
exactly 2 of them will be men?
(1) 1/9 (2) 3/10 (3) 4/15 (4) 1/10 (5) 5/12
Q.26-29. Rahul and Saif appear for an interview for two vacancies. The probability of Rahul's selection is 1/3 and that of
Saif's selection is 1/5 .
Q.26. Find the probability that both of them will be selected.
(1) 1/15 (2) 8/15 (3) 8/9 (4) 7/8 (5) None of these

QUANTITATIVE APTITUDE 149


stportal.mahendras.org
Q.27. Find the probability that none of them is selected.
(1) 1/15 (2) 8/15 (3) 8/9 (4) 7/8 (5) None of these
Q.28. Find the probability that at least one of them is selected.
(1) 1/15 (2) 8/15 (3) 8/9 (4) 7/15 (5) None of these
Q.29. Find the probability that only one of them is selected.
(1) 1/15 (2) 2/5 (3) 8/9 (4) 7/8 (5) None of these
Q.30. A committee of five persons is to be chosen from a group of 10 people. The probability that a certain married couple
will serve together is?
(1) 54/199 (2) 52/195 (3) 53/186 (4) 51/126 (5) None of these
Q.31. A man can hit a target once in 4 shots. If he fires 4 shots in succession, what is the probability that he will hit his
target?
(1) 1 (2) 1/256 (3) 81/256 (4) 175/256 (5) None of these
Q.32. An anti-aircraft gun can fire four shots at a time. If the probabilities of the first, second, third and the last shot hitting
the enemy aircraft are 0.7, 0.6, 0.5 and 0.4, what is the probability that four shots aimed at an enemy aircraft will
bring the aircraft down?
(1) 0.84 (2) 0.916 (3) 0.036 (4) 0.964 (5) None of these
Q.33-37.In a bilateral cricket series between India and Australia, the probability that India wins the first game is 0.4. If India
wins any game, the probability that it wins the next game is 0.3; otherwise the probability is 0.2.
Q.33. Find the probability that India wins the first two games.
(1) 0.08 (2) 0.32 (3) 0.18 (4) 0.12 (5) None of these
Q.34. Find the probability that India wins at least one of the first two games.
(1) 0.48 (2) 0.32 (3) 0.56 (4) 0.52 (5) 0.58
Q.35. Find the probability that India wins the first three games.
(1) 0.028 (2) 0.030 (3) 0.032 (4) 0.036 (5) 0.044
Q.36. Find the probability that India wins exactly one of the first three matches.
(1) 0.416 (2) 0.396 (3) 0.096 (4) 0.404 (5) 0.214
Q.37. Find the probability that India wins exactly one of the first two games.
(1) 0.20 (2) 0.40 (3) 0.44 (4) 0.36 (5) 0.28
Q.38. A number is to be chosen from the number - 4, - 3, -2, 0, 1, 2, 3, 4. What is the probability that the number is less
than 3?
5 3 3 1 3
(1) (2) (3) (4) (5)
7 7 4 3 5
Q.39. Two dice are rolled, find the probability that the sum is less than 13.
1
(1) (2) 1 (3) .75 (4) 1.25 (5) 0
2
Q.40. Two dice are tossed. The probability that the total score is a odd number-
1 5 1 7
(1) (2) (3) (4) (5) None of these
6 12 2 9

150 QUANTITATIVE APTITUDE


stportal.mahendras.org

EXERCISE Explanation
5 Q.13.(3) Probability of getting a head = 1/2
C 2 ×2 C 2 10
Q.1.(1) Required probability = =
14
C4 1001 When a coin is tossed, Probability of getting a even
4
C 2 × 2C 2 × 3C 2 × 5C 2 number when dice is tossed = 1/2
Q.2.(3) Required probability = 14
C8 1 1 1
180 60 Required probability = × =
= = 2 2 4
3003 1001 Q.14.(1) Sum of 18 - {6, 6, 6} - 1 way
Q.3.(1) Required probability =
2
C2
+ 1−
LM 12
C2 OP
Sum of 17 - {6, 6, 5} - 3 way
14
C2 MN 14
C2 PQ
Sum of 16 - {6, 6, 4}{6,5,5} - 3+3 = 6 way
1  66  1 25 26
== + 1 −  = + = Sum of 15-{6, 6, 3}{6,5,4}{5,5,5}-3+6+1= 10 way
9 1  9 1  91 91 91
Total way = 20 ways
11
C3 165 199
Q.4.(2) Required probability=1- = 1- = Required probability = 20/216 = 5/54
14
C3 364 364
Q.15.(1) Required probability
10
C3 30 4 1 1  1 3 1  1 1 2  1 1 1 
Q.5.(5) Required probability = = 5 × 4 × 3  + 5 × 4 × 3  + 5 × 4 × 3  +  4 × 3 × 5 
14
C3 91        

4 3 2 1 10 1
Q.6.(4) Probability of the child of not a girl = 1/2 = 60 + 60 + 60 + 60 = =
60 6
1 1 1 1
Required probability = × × =
2 2 2 8 Q.16.(4) Winning probability of team x = 5/8
Q.7.(1) Favourable case = 1
winning probability of team y = 1/5
Total case = 5!
required probability = 5/8 + 1/5 = 33/40
1 1
Probability = =
5 ! 120 Q.17.(4) Favourable event : To be a multiple of 4, the last
2 digits of the number have to be divisible by 4 is
10
C1 16
C1 they must be 12, 24, 32 or 52. and semaining place
Q.8.(5) Required probability = ×
26
C1 25
C1 can be filled in 3! ways

10 16 16 So total favourable event = 4 × 3!


= × =
26 25 65 Total event = 5!
4 !× 3 !
Q.9.(2) Required probability = 4 × 3! 1
7! Required Probability = =
5! 5
4 × 3 × 2 × 3 × 2 ×1 1
= = Q.18.(4) Favourable cases = [HH,TH,HT] = 3
7 × 6 × 5 × 4 × 3 × 2 ×1 35
Q.10.(4) Favourable cases = 2,3,5,7,11,13,17, 19,23,29,31, Total cases = [HH,TT,TH,HT] = 4
37,41,43,47,53,59,61,67,71,73,79, 83,89,97. Required probability = 3/4
Total case = 100, Required probability = 25/100 = 1/4
Q.19.(1) Favourable cases = [TTH,THT,HTT,TTT] = 4
6
C 4 + 5 C 2 × 14 C 2
Q.11.(2) Required probability = Total cases=[TTT,HHH,THT,HTT,HTH,HHT] = 8
19
C4

6 × 5 × 4 × 3 5 × 4 14 ×13 19 ×18 ×17 ×16 925 Required probability = 4/8 = 1/2


= + × / =
1×2 ×3 × 4 2 2 ×1 4 × 3 × 2 ×1 3876 Q.20-22.

Q.12.(5) Probability of not winning all the three sets Red Green Yellow Total
 1  1  3 6 4 1 6 Bag-1 16 24 12 52
=  1 − 7  ×  1 − 5  ×  1 − 4  = × × = Bag-2 12 8 16 36
7 5 4 35
6
Bag-3 24 20 12 56
29
Probability of winning atleast one set= 1- =
35 35

QUANTITATIVE APTITUDE 151


stportal.mahendras.org
Q.20.(4)  1 6 1 6   1 2 1 2 
 52 × 36  +  52 + 36  1 2 4 1 2
    Q.29.(2) × + × =
5 3 5 3 5
16 1 25
+ = Q.30.(4) Five persons is to be chosen from a group of 10
117 13 117
people = 10C5 = 252
Q.21.(5) 12 3 , x=8
= Couple Serve together = 8C3 x 2C2 = 56
56 + x 16
Probability = 102/252 = 51/126
Number of balls in bag 1 = 52-8 = 44 Q.31.(4) (1−81/256) = 175/256
Q.22.(3) Let number of green balls be 4 k and k Q.32.(4) The probability that none of the four shots hit the
5 k 5 aircraft is given by: 
=
5 k + 12 11 (1−0.7)(1−0.6)(1−0.(5) (1−0.
12 (4)=0.3×0.4×0.5×0.6=0.036
k = =2
6 So, the probability that at least one of the four
hits the aircraft: =1–0.036=0.964
Number of green balls = 5×2 =10
Q.33.(4) P(Win first game)× P(Win second game)
Q.23.(4) In any case B speaks truth. Now at most 2 people
speak truth for 1 question = 0.4×0.3=0.12

So case 1: B and A speaks truth Probability Q.34.(4) P(won at least 1 game)= 1- P(won no games)
3 3  5 3 =1- [(1-0.(4) × (1-0.(2) ] (0.(2) = 0.52
= × × 1 −  =
7 10  6  140 Q.35.(4) (4) 0.4× 0.3 × 0.3= 0.036
Case 2: B and C speaks truth Probability Q.36.(4) Case1- India wins first game and loses second and third
3  3 5 5
= × 1 −  × = Case 2= Lose + Win + Lose
7  10  6 20
Case 3= Lose + Lose+ Win P
Case 3: Only B speaks truth Probability
3  3   5 1 = P1+P2+P3 = 0.404
= × 1 −  × 1 −  =
7  10   6  120 Q.37.(2) Case 1= Won first × Lost Second
= 0.4 × (1-0.(3) = 0.4 × 0.7=0.28
3 1 5 9
Add the three cases = + + = Case 2= Lost First × Won second
140 20 20 28
Q.24.(1) Case 1: B does not speak truth, A speaks truth So A = (1-0.(4) × 0.2 = 0.6×0.2=0.12
speaks truth here Probability that C does not speak
truth = (3/10) × (1 – 3/7) × ( 1- 5/6) = 1/35 P= 0.28+0.12=0.40

Case 2: B speaks truth So A does not speak truth Q.38.(3) Favourable case = 2,1,0,-2,-3,-4
here Probability that C does not speak truth Total cases = -4,-3,-2,-1,0,1,2,3,4
= ( 1- 3/10) × 3/7 × ( 1- 5/6) = 1/20 6 3
Probability = =
So total = 1/35 + 1/20 = 11/140 8 4
Q.39.(2) Favourable case = 36
Q.25.(2) 2 men means other 2 woman and children So prob. Total case = 36
C2 ×7 C2 3
3
36
= 10
= Probability = =1
C4 10 36
Q.40.(3) Favourable case = (1,2) (1,4) (1,6) (4,1) (4,3) (4,5)
Q.26.(1) 1 1 1 (2,1) (2,3) (2,5) (5,2) (5,4) (5,6) (3,2) (3,4) (3,6)
× = (6,1) (6,3) (6,5)
3 5 15
Total cases = 36

4 2 8 18 1
Q.27.(2) × = Probability = =
5 3 15 36 2

8 7
Q.28.(4) 1 − =
15 15
152 QUANTITATIVE APTITUDE
stportal.mahendras.org

CHAPTER

19 DATA
SUFFICIENCY
Sol. Let the two-digit number be 10x+y
Introduction
Then, y-x = 4..........(i)
Data sufficiency has recently become a favourite topic for
many of the recent examinations. In this type of questions, I. x + y = 10 .........(ii)
usually a question is given followed by two or three Solving (i) with (ii), we get
statements. These two or three statements contain data or x = 3, y = 7
some pieces of information, using which, the question can
Number = 10 × 3 + 7 = 37
possibly be solved. You are required to judge whether the
data given is sufficient to answer the question or not. II. |10x+y-10y-x| = 36
Format of Study : Data sufficiency questions are not new or, |9x - 9y| = 36 or, |x-y| = 4
topics in themselves. They may be covering any of the topics we wouldn’t get the value of x and y.
already covered; for example : percentage, time and work, Therefore, only statement I alone is sufficient to
algebra, time and distance profit and loss, S.I., C.I., average answer the question.
etc. Hence you should treat these questions as old-type only.
Only these questions are asked in a different pattern and not Ex. What is the value of m - n ÷ 37 ?
the conventional pattern. I. m is the largest possible six-digit number and n
Helping Hands: When you are attempting a question of data is the smallest possible six-digit number.
sufficiency you should follow a systematic approach as laid II. The difference between m and n is known.
down below. This approach being a systematic one, will save Sol. I. m = 999999, n = 100000 ∴ ? = m - n ÷ 37
your time. Also in case you are stuck up at any point, it will
= 999999 - 100000 ÷ 37
help your chances of guessing a correct answer because it
narrows down the possible answers from 5 to 3 or 2. = 999999 - 2702.70 = 997296.30
II. m - n = known, but neither the value of ‘m’
To understand this approach let us first look at the way in
is known nor the value of ‘n’ is known. So,
which such questions are usually asked : (1) Two statements
we cannot find the value of m-n ÷ 37 by this
D.S. (2) Three statements D.S.
statement.
Direction : The questions given below contain two Ex. What is the average daily wages of a worker who
statements giving certain data. You have to decide whether works five days if he made Rs. 80 for the first day?
the data given in the statements are sufficient for answering
I. The worker made a total of Rs. 400 for the first
the question ? Mark answer-
four days of work.
(1) If statement I alone is sufficient but statement II II. The worker made 20% more each day than he
alone is not sufficient. did on the previous day.
(2) If statement II alone is sufficient but statement Sol. Only II statement is sufficient.
I alone is not sufficient. Ex. What is the difference between the present ages of
(3) If each statement alone (either I or II) is mother and her daughter?
sufficient. I. Ratio of the age of the daughter eight years hence
to the present age of the mother is 3 :4.
(4) If statement I and II together are not sufficient.
II. Ratio of the present age of the daughter to that
(5) If both statement together are sufficient, but of the mother is 11 : 20.
neither statement alone is sufficient. Sol. Statements I and II will give us the separate equations
Ex. In a two-digit number, the digit at unit's place is 4 more for mother and daughter. Therefore, both statements
than the digit at tens place.Find the two digit number. together are sufficient.
I. Sum of their digits is 10. Ex. What is the population of state A?
I. After an increase in the population of state A by
II. The difference between the number and the
12% it becomes 627.20 lakhs.
number obtained by interchanging the po­sition
II. Ratio of population of state A to that of state B
of the digits is 36.
is 4:5.
QUANTITATIVE APTITUDE 153
stportal.mahendras.org
Sol. I. Population of State (4) All I, II and III
627 .20 (5) Answer can not be given even using all the
A= × 100 = 560 lakhs three statements.
112
.
II. Ratio never determine value therefore only I Sol. Answer can not be given even using all the three
statement is sufficient. statements.
Ex. What is the rate of simple interest per annum? Ex. What will be the sum of ages of mother and daughter
I. The sum trebles in 20 years at simple interest. after 6 years?
II. The difference between the sum and the simple I. Mother's present age is 24 years more than the
interest earned after 10 years is Rs. 1000. daughter's present age.
100 II. 4 years ago, the ratio of ages of mother and
Sol. I. b g
R = 3 −1 ×
20
= 10% daughter was 5:1.
II. Here the sum is not given. III. Five times the daughter's present age is 16 years
more than the mother's present age.
Therefore, statement I alone is sufficient.
(1) Any two (2) Only I and either II or III
Ex. What is the speed of a running train which takes 6
(3) Only I and II (4) Only II and III
seconds to cross a signal post?
(5) None of these
I. The length of the train is 90 m. Sol. From statement (I) and (II),
II. The train takes 18 seconds to cross a platform180 M = D + 24 _________(i)
m long. M = 5 D ___________(ii)
90 18 From equation (i) and (ii),
Sol. I. ×
Speed of the train = = 54 km/hr
6 5 D = 10 years
II. Quicker Method : Length of train
M = 34 years
Length of platform From statement (I) and (III),
= × T im e
diff. in tim e M = D+24 __________(iii)
taken to cross a signal post
5D = M + 16 _________(iv)
180
= × 6 = 90 m From equation (iii) and (iv),
18 − 6
D = 10 years
90 18 M = 34 years
Speed of the train = × = 54 km/hr.
6 5 Therefore, only I and either II or III is sufficient.
Either I or II is sufficient. Ex. What is the length of the platform?
Ex. A train crosses another train running in the op­posite I. A train running at the speed of 25 km./hr. crosses
direction in x seconds. What is the speed of the train? a platform in 18 seconds.
I. Both the trains are running at the same speed. II. The train crosses a man moving in opposite
II. The first train is y cm long. direction at the speed of 5 km./hr. in 12 seconds.
Sol. The length of the other train is not given in any of the III. The train crosses a pole in 14.4 seconds.
statements. (1) All I, II and III (2) Any two
Both the statements are not sufficient.
(3) Only II and either I or III (4) Only II and III
Three Statement Data Sufficiency
(5) None of these
Direction.The questions given below contain three
statements giving certain data. You have to decide whether Sol. From statement (I) and (II),
the data given in the statements are sufficient for answering 5 25
the question ? Relative speed = 25+5 = 3 0 × = m./sec.
18 3
Ex. What is the average of three numbers?
25
I. The largest number is 20 more than the smallest Distance = ×12 = 100 meter
number. 3
II. The sum of the largest and the smallest number Length of train = 100 m.
is twice the middle number. Length of train + length of platform
III. The difference between first two numbers is 10.
25 × 5
(1) Only I and III = ×18
18
(2) Only II and III
Length of platform = 25 m.
(3) Any two
154 QUANTITATIVE APTITUDE
stportal.mahendras.org

EXERCISE
Q.1-15. The questions given below contain two statements giving certain data. You have to decide whether the data given
in the statements are sufficient for answering the question ? Mark answer-
(1) If statement I alone is sufficient but statement II alone is not sufficient.
(2) If statement II alone is sufficient but statement I alone is not sufficient.
(3) If each statement alone (either I or II) is sufficient.
(4) If statement I and II together are not sufficient.
(5) If both statements I and II together are sufficient, but neither statement alone is sufficient.
Q.1. Is 12 is one of the number in 13 consecutive even numbers.
Statement I. Difference between first and least numbers is 24.
Statement II. Average of the set is zero.
Q.2. In a school, 35% girls and 19% boys participated in singing. Number of students in the school is 700. How many
boys are there in the school?
Statement I. In school number of boys are 200 more than number of girls
Statement II. In singing 45 more boys participated than girls
Q.3. Is a>0?
Statement I. (a+b)² < (a-b)²
Statement II. 2a+2b < 2a-2b
Q.4. What is the rate of interest per annum?
Statement I. A sum becomes double to itself at SI in 4 years
Statement II. The difference between CI and SI on an amount of Rs. 10,000 in 2 years is Rs.625
Q.5. What is the circumference of a circle?
Statement I. Area of circle is half of the area of square.
Statement II. Ratio of the perimeters of square and circle is 2:1
Q.6. What is the present age of Vijay?
Statement I. Present age of Vijay and his wife are in the ratio of 13:11
Statement II. After 5 years his age is 6 years more than his wife.
Q.7. The average weight of A, B, C, D and E is?
Statement I. Average weight of A, B and C is 30 kg.
Statement II. Average weight of C, D and E is 40 kg.
Q.8. Is c >0?
Statement I. (b² -4ac) > 0
Statement II. 4ac <0
Q.9. A car dealer sold 80% of the cars in its inventory during September. What was the total revenue from the sales of
cars?
Statement I. All but 60 cars in the showroom inventory were sold during September.
Statement II. Cars were sold for an average price of Rs.4.5 lakh during September.
Q.10. Did Incumbent P get over 60% of the vote?
Statement I. Challenger M got 39% of the vote
Statement II. Incumbent P got 40000 of the total votes cast
Q.11. How much money did X invest?
Statement I. An increase in the Simple Interest from 39/8 % to 41/8 % per annum increases his yearly income
by 25%.
Statement II. The sum invested gets doubled, when invested at 8% per annum for 25/2 year.
Q.12. For a certain bottle and cork, what is the price of the cork?
Statement I. The combined price of the bottle and the cork is Rs.95
Statement II. The price of the bottle is Rs.75 more than the price of the cork.
Q.13. A man sales mobile sets as profit of 20%. How much total amount he gains in profit?:

QUANTITATIVE APTITUDE 155


stportal.mahendras.org
Statement I. He sales 20 mobile sets.
Statement II. He sales each mobile set on Rs. 18000
Q.14. What is the ratio of investments of P, Q and R ?
Statement I. Total investments is Rs.12000 in which P’s contribution is Rs.4500.
Statement II. Ratio of investments of P to Q is 4 :3 and Q to R is 3:1.
Q.15. What is Ravi’s present age ?
Statement I. Ravi is three years older than Sita.
Statement II. The ratio between Sita and Renuka’s age is 3:4.
Q.16-28. Which information are necessary to answer the questions asked-
Q.16. The following information is given about a four sided polygon. Which of the above facts are sufficient to deter-
mine the dimensions of the polygon?
I. The polygon is a rectangle
II. The area of the polygon is given to be 100 m2.
III. One side of the polygon is 8 m.
IV. All the adjacent sides are at right angles to each other.
(1) Any two (2) All together are not sufficient
(3) II, III and (I or IV) (4) (I or IV) and (II or III)
(5) None of these
Q.17. What is the length of the train X?
I. Two trains X and Y running in opposite directions and cross each other in 25 seconds.
II. The length of the train Y is 320m and the difference between the speeds of trains X and Y is 14 km/h.
III. The ratio of the speed of the trains is 19 : 26.
(1) Only II and III (2) Only I and III (3) Any two
(4) All three together are not sufficient (5) None of these
Q.18. Find the two digit number.
I. The product of the two digits of the two digit number is 20.
II. The difference between the two digits of the two digit number is 1.
III. The sum of the two digits of the two digit number is 9.
(1) None of these (2) II and III together are sufficient
(3) III and (I or II are sufficient) (4) All three together are sufficient
(5) All three together are not sufficient
Q.19. How much is monthly profit of a company?
I. Company decided to give bonus with the salary to its employees. Bonus is 12% of the monthly profit of the
company.
II. On the day of salary distribution, company changed its mind and gave a total of Rs.24000 as bonus which
was 125% of what the company had decided earlier.
III. Company gives an amount of Rs.15000 as bonus which is 3/4th of total monthly profit.
(1) None of these (2) II and III together are sufficient
(3) III and (I or II are sufficient) (4) All three together are sufficient
(5) All three together are not sufficient
Q.20. Is integer x divisible by 6?
(I) x is the product of three consecutive positive integers
(II) x2 is divisible by 36
(III) x2 is divisible by 4
(1) I or II (2) II and III (3) None of these
(4) All three together are sufficient (5) All three together are not sufficient

156 QUANTITATIVE APTITUDE


stportal.mahendras.org
Q.21. If a = 5, what is value of b + c?
(I) a is the average of the set {a, b, c}
(II) a is the median of the set {a, b, c}
(III) ba + bc = 5
(1) Only II (2) II and III (3) Only I
(4) All three together are sufficient (5) Any one
Q.22. What is the second number out of three numbers?
(I) Sum of the three numbers is 182
(II) The ratio of first and second number is 3 : 4 and second to third number is 2 : 3.
1
(III) Middle number is 33 % greater than the first number and lesser by the same percent from third number.
3
(1) Only II and III (2) Only II (3) Only III (4) Only I (5) None of these
Q.23. x, y and z together will finish the work in how many days?
(I) x does half as much work as y and y does half as much work as x and z does together in one day.
(II) y does the whole work in 40 days.
(III) x and z do the whole work in 20 days.
(1) Only I or II or III (2) Either I and II or I and III (3) Only II and III
(4) All are necessary (5) None of these
Q.24. In how many days 9 men and 15 women can the work be done ?
(I) 6 men and 5 women can complete the work in 6 days.
(II) 3 men and 4 women can complete the work in 10 days.
(III) 18 men and 15 women can complete the work in 2 days.
(1) Only I (2) Only II (3) Any two (4) All I, II and III (5) None of these
Q.25. Which is the smaller of two numbers?
I. The difference between these two numbers is one-fourth of the larger number.
II. The sum of these two numbers is 49.
III. The difference between these two numbers is 7.
(1) Only I and II (2) Only I (3) Either I or II and III (4) All (5) None of these
Q.26. What is the ratio of the present ages of Rohan and his father?
I. Five years ago Rohan’s age was one-fifth of his father’s age.
II. Two years ago the sum of ages of Rohan and his father was 36 years.
III. The sum of the ages of Rohan, his mother and his father is 62 years.
(1) Only II and III (2) Only I and II (3) Only I and III (4) All (5) None of these
Q.27. What is the rate of interest per annum?
I. The amount becomes Rs. 11025 with compound interest after 2 years.
II. The same amount with simple interest becomes Rs. 11000 after two years.
III. The amount invested is Rs. 10000
(1) Any two (2) III and either I or II (3) I and either II or III
(4) II and either I or III (5) All
Q.28. In how much time will the train cover the distance between city ‘Y’ from city ‘X’?
I. The train crosses another train of equal length of 200 m running in opposite direction in 15
seconds.
II. The train leaves city ‘Y’ at 7.15 am for City ‘X’ situated at a distance of 560 km.
III. The 200 m long train crosses a signal pole in 10 seconds.
(1) Only II and III (2) Only I and III (3) Only I and II (4) All (5) None of these

QUANTITATIVE APTITUDE 157


stportal.mahendras.org

EXERCISE Explanation
Q.1.(2) Average of the set is zero: in that case 12 must be From Statement I, Increased Rate of interest =
there in the set. (41/8) – (39/8) = 2/8 =1/4
Q.2.(1) 700= B+G---------(i) x×(1/4)×(1/100) = 25 ⇒ x=10000
B - G = 200 ---------(ii) From Statement II, we can’t find x.
adding (i) and (ii) Q.12.(5) if the data in both the statements I and II together
B = 450 are necessary to answer the question.

Q.3.(5) If the data given in both statements I & II together From Statement II, Let the price of cork be x, then
are necessary to answer the question the price of the bottle = (x+75)

From II b<0 From Statement I, combined price bottle and cork


= Rs.95
And from I ab<0 then a>0
x+(x+75) = 95 ⇒ 2x + 75 = 95
Q.4.(3) If the data either in Statement I alone or Statement
II alone are sufficient to answer the question ⇒ x = 10

R =25% can be determined from either of the Q.13.(3) SP = 18000, Profit = Rs. 3000
statements CP = 15000
Q.5.(4) If the data given in Both Statements I & II are not Total profit = Rs. 3000 × 20 = Rs. 60000
sufficient to answer the question Q.14.(2) Only II sufficient
No values of radius and side can be find out from 4:3:1=P:Q:R
above statements
Q.15.(4) Nither I nor II are sufficient
Q.6.(5) If the data given in both statements I & II together
are necessary to answer the question From I and II we cannot find the exact Sita’s
V/W = 13/11 present age
V = w+5 Q.16.(3) From the statement IV, it is clear that the poly-
V=39. gon is a rectangle. Thus, statements I and IV are
similar.
Q.7.(4)
Q.8.(4) If the data given in Both Statements I & II are not To find the dimensions of the polygon, either I,
sufficient to answer the question Statement I II and III or II, III and IV are sufficient.
b² > 4ac Q.17.(5) From statement (1),
c could be either positive or negative x+y
V1+V2 =
Q.9.(5) Let total no of cars in September = x 24
From statement (2),
From Statement I – 20% of x = 60 (Cars that are
not sold) V1 V2 = 14

x = 300 From statement (3)


80% 0f 300 =240 V1 = 1 9
From Statement II – Price of a car V2 26

= Rs.4.5 lakh
Now we have 3 variables and 3 equations so all
From Statement I & II – The total revenue from 3 statements together are sufficient.
the sales of cars = 240 × 4.5 lakh = 1080 lakh
Q.10.(1) If the data in statement I alone is sufficient to Q.18.(5) From statement (1),
answer the question. xy = 20
From Statement I, P got 61% of the vote which is
From statement (2)
over 60%.
Q.11.(1) If the data in statement I alone is sufficient to x>y=1
answer the question. From statement (3),
Invested amount – x
x+y=9
158 QUANTITATIVE APTITUDE
stportal.mahendras.org
Here its seems that answer can be find but on We can not reach any certain conclusion from
solving we get two 2 digit numbers that is 54 and statement (III)
45. Hence all three together are not sufficient Q.21.(3) From statement (I)
Q.19.(1) From statements (I) and (II), b +c
a=
2
125% of 12% of profit = 24000 b+c =2a = 10
From statement (III) but from statement (II) and (III) we can not reach
3/4 of profit = 15000 any certain conclusion.
Hence statement III or (I and II) together are suf- Q.22.(5)
ficient. Q.23.(2)
Q.20.(1) From statement (I) Q.24.(3)
Q.25.(3)
Any product of three consecutive positive intigers
Q.26.(2)
will be always divisible by 6.
Q.27.(4)
From statement (II)
Q.28.(1) From III, we can get the speed of train and in
If x2 is divisible by 36 then x will be always divis- statement II distance is given. Therefore we can
ible by 6. find the required time from II and III.

QUANTITATIVE APTITUDE 159


stportal.mahendras.org

CHAPTER
DATA
20
INTERPRETATION
It is difficult when we find 64% of 75 but if we find
Data Interpretation
3
By interpretation of data we mean understanding, organising ( 75% = )
4
and drawing appropriate conclusions from the given data. 3
In these days, Data Interpretation is an important aspect of 75% of 64 = × 64 = 3 × 16 = 48
4
almost every competitive examination. Usually, a table or a
We easily get the required answer.
bar diagram or a pie-chart or a graph is given and candidates
are asked questions that test their ability to analyse the-data (4) % profit of a company x in various years are given
given in those forms. Through these questions, examiner below in the form of line graph-
makes an attempt to check your ability to calculate faster and 30
to comprehend complex and voluminous data.
25 25%
Unorganised and haphazard data does not make any sense
more so to top management for whom time is a very valuable 20 20%
P ro fit P e rce n t

and rare commodity. Hence, any data, be it daily production


figures, daily sales Figures, financial performance or 15
productivity, will have to be presented in a concise manner 10 12%
at the same time being precise so that top management can 10% 7%
study it with least of effort and time thus also facilitating 5
5%
faster decision making,
0
Types of D.I. : 2 01 1 2 01 2 2 01 3 2 01 4 2 01 5 2 01 6
(1) Tabulation (2) Bar Graphs
Ex. In which year company x got maximum profit ?
(3) Pie- charts (4) Line-Graphs
Sol. (According to the question, most of the candidate
(5) Mixed Graph (6) Missing DI
answered year 2016 (25%) but it is not necessary)
(7) Puzzle DI that 25% is maximum out of all given years then
Useful Tips : the value of 25% is also maximum (it may be),
(1) To solve D.I., first we learn some useful topic; because (% P is always calculated on its expenditure,
percentage profit & loss, Ratio, Average, because (expenditure is not given).
these topics are used in D.I. Ans. CND
(2) In, income- expenditure type of D.I. Ex. In which year, company x got maximum % profit?
Expenditure is equivalent to C.P. Sol. 2016 (25%)
& Income is equivalent to S.P. Solved Examples :
25
(3) In percentage- 25% of 80 = × 80 = 20 Table
100
A table is one of the simplest and most convenient means of
80 summarising data and presenting them in a meaningful way,
it is equivalent to- 80% of 25 = × 25 = 20
100 as all the numerical values are given directly and any errors
For example : Suppose a student got 80% out of that may arise due to discrepancies in analysing or interpreting
maximum mark 75. What is the marks obtained by the data expressed in graphical form are eliminated. Here,
that student ? it should be borne in mind that data expressed in the tabular
4 4 form may be expressed in most of the other graphical forms.
80% of 75 = × 75 ...(∴ 80% = ) = 60
5 5 So, we can express or summarise the voluminous data in any
but, suppose another student got 64% marks out of one of the forms as we wish. So, the ways of expression do
maximum marks 75. What is the marks obtained by not matter. The various graphs can be drawn from the original
that student ? piece of data which is in the form of table.

160 QUANTITATIVE APTITUDE


stportal.mahendras.org
Q.1-5. Study the following table carefully and answer Number of tickets sold of movie B in Lucknow
the questions given below. =27000
Number of tickets sold in a week of five movies Required difference=27000-21000=6000
in the multiplexes in six different cities Q.3.(4) Total number of tickets of movie C sold in all
(number in thousands) cities=35+21+19+32+26+20=153 (thousand)
Movie® 153000
City¯
A B C D E So, required average= = 25500
6
Mumbai 20 15 35 26 18 Q.4.(1) Number of tickets sold of movie E in Chennai
Delhi 17 19 21 25 28 = 34 (thousand)
Kolkata 32 24 19 21 17 Number of tickets sold of movie A in Mumbai
Chennai 18 21 32 28 34
= 20 (thousand)
Hyderabad 16 34 26 29 22
34
Lucknow 15 27 20 35 26 required percentage = ×100 = 170
20
Q.5.(1) Total number of tickets sold in Mumbai
Q.1. The number of tickets of movie B sold in
Hyderabad was approximately, what per cent of = 114 (thousand)
the total number of tickets of the same movie sold Total number of tickets sold in Delhi=110 (thousand)
in all the cities together?
Total number of tickets sold in Kolkata
(1) 15 % (2) 18 % (3) 12 %
= 113 (thousand)
(4) 20 % (5) 24 %
Total number of tickets sold in Chennai
Q.2. What is the difference between the number of
tickets of movie D sold in Kolkata and the number = 133 (thousand)
of tickets of movie B sold in Lucknow? Total number of tickets sold in Hyderabad
(1) 7500 (2) 7000 (3)14000 = 127 (thousand)
(4) 9000 (5) None of the above
Total number of tickets sold in Lucknow=123
Q.3. What is the average number of tickets of movie C (thousand). Minimum tickets sold in Delhi.
sold in all the six cities?
Bar graph
(1) 15500 (2) 2550 (3) 24000
(4) 25500 (5) None of the above Bar graph is probably the most widely used method of
diagrammatic representation of data. A bar chart is a graph
Q.4. The number of tickets of movie E sold in Chennai that consists of a number of rectangles (called bars) whose
is what per cent of number of tickets movies A sold length or height varies with the magnitude repsresented but all
in Mumbai? bars are of equal width. The bars may be arranged vertically
(1) 170 (2) 70 (3) 30 or horizontally.
(4) 130 (5) None of the above Q.1-3. Read the following graph carefully and answer the
Q.5. In which city was the total number of tickets of all questions given below:-
the five movies together sold the minimum? Profit earned by three companies in different years
(1) Delhi (2) Chennai (3) Lucknow (in crore Rs.)
(4) Kolkata (5) None of these
Explanation : Q.1-5.
Q.1.(5) Total number of tickets of movie B sold in all the
cities together
=15+19+24+21+34+27 =140 (thousand)
Number of tickets sold of movie B in Hyderabad
=34 (thousand)
34
Required percentage= ×100 = 24
140
Q.2.(5) Number of tickets sold of movie D in Kolkata
=21000
QUANTITATIVE APTITUDE 161
stportal.mahendras.org
Q.1. In which of the following years was the difference
between the profit earned by company B and Central angle
company A maximum?
o
(1) 2006 (2) 2004 (3) 2008  P e rce n ta g e P a rt o f T o ta l V a lu e 
=  × 3 6 0
(4) 2005 (5) None of these  100 
Q.2. What was the percent increase in the profit earned Q.1-5. Study the information carefully and answer the
by company C from 2006 to 2007 ? questions that follow:
(1) 40 (2) 45 (3) 56 The following pie-chart shows the percentage of
(4) 50 (5) None of these people in Lucknow who are interested to buy
different company mobiles.
Q.3. In which of the following years was the total profit
earned by all the three companies minimum? Total Number of peoples = 65000
(1) 2004 (2) 2003 (3) 2006
(4) 2005 (5) None of these
Explanation : Q.1-3.
Redmi Vivo
Q.1.(1) In 2006
20% 21%
150
Q.2.(4) Percentage increase = × 1 0 0 = 50%
300
Q.3.(2) In 2003 Motorola Oppo
12% 14.50%
Pie-Chart
Apple I
Pie-Charts (also called Circle-Charts) are used
to represent the relative sizes of component in phone, 3%
Samsung
an aggregate. Pie-Chart is a circle broken down 30%
into component sectors. Therefore, in pie-chart
different data are represented by sectors. Pie- Q.1. What is the difference between the number of
Chart is generally used on a percentage basis people interested to buy Samsung and the total
and not on an absolute basis. Different sectors of number of people interested to buy Oppo, Motorola
a Pie-Chart represent various component parts. and Apple i Phone together?
Each component value is expressed either as a (1) 750 (2) 650 (3) 605
percentage of respective total or as the central
(4) 625 (5) 675
angle of the respective total.
Q.2. The number of people interested to buy Apple
o
 V a lu e o f th e se c to r  i Phone is what percentage of the number of
Central angle =  × 360o 
to ta l V a lu e  people interested to buy Motorola? (Calculate
approximate percentage)
C e n tra l A n g le (1) 12% (2) 30% (3) 37%
Value of the Sector = × T o ta l V a lu e
360o (4) 21% (5) 40%
[Since the angle at the centre of the circle is 360°, Q.3. What is the ratio of people interested to buy Apple
the total magnitude of the various components is i Phone to the number of people interested to buy
taken to be equal to 360°] Oppo?
Total magnitude of the various components is equal (1) 5 : 29 (2) 17 : 5 (3) 6 : 22
to 100%. Therefore, 100% = 360° (4) 7 : 29 (5) 29 : 5
o o
Q.4. The number of people interested to buy Redmi
 360   18  is approximately what per cent of the number of
1% =  = 3 .6 o =  
 1 0 0   5  people interested to buy Oppo?
Hence, the percentage of the component parts can (1) 148% (2) 118% (3) 130%
be converted into degrees by multiplying each of (4) 125% (5) 138%
them by (3.6)°. Q.5. The number of people interested to buy Motorola
Percentage Part of Total Value is approximately what per cent of the people
interested to buy Vivo, Redmi and Oppo together?
 C e n tra l A n g le  (1) 14% (2) 32% (3) 22%
=  × 100 %
360o  (4) 10% (5) 28%
162 QUANTITATIVE APTITUDE
stportal.mahendras.org
Q.2. What is the difference between the number of
Explanation : Q.1-5.
vehicles manufactured by Company Y in 2014 and
65000 2015 ?
Q.1.(2) Required difference= × (3 0 − 2 9 ) = 650
100 (1) 21000 (2) 22000 (3) 23000
Q.2.(4) Number of people interested to buy Apple i Phone (4) 24000 (5) None of these
6 5 0 0 0 × 2 .5 Q.3. What is the average number of vehicles
= = 1 6 2 5 = 1625
100 manufactured by Company X over the given
Number of people interested to buy Motorola period?
65000 ×12 (1) 119133 (2) 119233 (3) 119333
= = 7800 (4) 119433 (5) None of these
100
Q.4. In which of the following years, the difference
1625
Required % = 7 8 0 0 × 1 0 0 = 20.83% = 21% between the production of Companies X and Y was
the maximum among the given years ?
2 .5 25 25 (1) 2011 (2) 2012 (3) 2013
Q.3.(1) Required ratio = = = = 5 : 29
1 4 .5 1 4 5 145 (4) 2014 (5) None of these
Q.4.(5) Number of people interested to buy Redmi Q.5. The production of Company Y in 2014 was
= 65000 × 20/100 = 13000 approximately what percent of the production of
Company X in same year ?
Number of people interested to buy Oppo
(1) 163% (2) 164% (3) 165%
= 65000 × 14.5/100 =9425 (4) 166% (5) None of these
Required % = 13000/9425 × 100 = 137.93
Explanation : Q.1-5.
= 138%
Q.1.(2) Please note that line graph values are given in
Q.5.(3) thousands.
Line Graph Total production of Company X from 2011 to 2016
The line graph is one of the simplest ways of =119000+99000+141000+78000+120000+15900
expressing the data. There could be one or more 0 = 716000
lines depicting different sets of data. X-axis is
usually the category axis and Y-axis is usually the Total production of Company Y from 2011 to 2016
value axis. Therefore, in line graph there are two = 139000+120000+100000+128000+107000+148
axes representing two different variables. It is also 000 = 742000
known as XY-Chart or -XY Graph.
Difference = 742000 - 716000 = 26000
Q.1-5. Study the following line graph and answer the
questions based on it. Q.2.(1) Required Difference = 128000-107000 = 21000
Vehicles made by two companies Q.3.(3) Average number of vehicles manufactured by
Company X
X Y
= 1\6(119000+99000+141000+78000+120000+15
180
160
9000) = 119333
159
148
140 139 141 Q.4.(4) For 2011 = (139000 - 119000) = 20000
128
120 119 120 120
100 99 100
107 For 2012 = (120000 - 99000) = 21000
80 78 For 2013 = (141000 - 100000) = 41000
60
40 For 2014 = (128000 - 78000) = 50000
20
0
For 2015 = (120000 - 107000) = 13000
12011
997 2012
1998 2013
1999 2014
2000 22015
001 2 02016
02
For 2016 = (159000 - 148000) = 11000
YE A R
Clearly the difference was maximum in year 2014.
Q.1. What is the difference between the two companies
in the given years ? Q.5.(5) Required Percentage
(1) 16000 (2) 26000 (3) 28000 128000
= × 1 0 0 = 1 6 4 .1 %
(4) 30000 (5) None of these 780000
QUANTITATIVE APTITUDE 163
stportal.mahendras.org
(1) 88% (2) 94% (3) 123
Combined Graph
(4) 80% (5) 73%
Here combination of two or more than two graphs
is given like table-line chart, pie chart with table, Q.5. The population of City B which is below poverty
line chart with bar graph and double pie chart with line is approximately what percent more/less than
table graph etc. the population of City D which is below poverty
line?
Q.1-5. Read the following informations carefully to
answer the questions asked- (1) 73% (2) 79% (3) 67%
Given pie-chart shows the percentage distribution (4) 52% (5) 85%
of population and the table shows the percentage
Explanation : Q.1-5.
population below poverty line in these cities.
Q.1.(5) Population of City C which is above poverty line
Total Population = 180 Lakhs
8 65
= 180 × × = 9 .3 6
A
100 100
F 10
18 Q.3.(5) Population of City A which is above poverty line

B
10 52
20 = 180 × × = 9 .3 6
100 100
E Population of City D which is below poverty line
22
C
8 13 40
D
= 180 × × = 9 .3 6
13 100 100
 
Required ratio=1:1
Population below
City Q.4.(2) Population of City G which is above poverty line
poverty line
A 48%
B 45% 9 50
= 180 × × = 8 .1 0
C 35% 100 100
D 40% Population of City A which is below poverty line
E 55%
F 45%
G 50% 10 48
= 180 × × = 8 .6 4
Q.1. What is the population of City C which is above 100 100
poverty line? (in lakhs)
8 .1 0
(1) 8.44 (2) 10.24 (3) 9.15 Required percent= ×1 0 0 % ≈ 9 4 %
(4) 7.96 (5) None of these 8 .6 4
Q.2. What is the difference between the population of Q.5.(1) Population of City B which is below poverty line
City E which is below poverty line and that which
is above poverty line? (in lakhs)
20 45
(1) 2.96 (2) 3.24 (3) 3.96 = 180 × × = 1 6 .2 0
(4) 2.24 (5) None of these 100 100
Q.3. What is the ratio of the population of City A which Population of City D which is below poverty line
is above poverty line to the population of City D
which is below poverty line?
(1) 3:2 (2) 2:3 (3) 4:5 13 40
= 180 × × = 9 .3 6
(4) 5:4 (5) None of these 100 100
Q.4. The population of City G which is above poverty 1 6 .2 − 9 .3 6
line approximately what per cent of the population Required percent= ×1 0 0 % ≈ 7 3 %
of City A which is below poverty line?
9 .3 6

164 QUANTITATIVE APTITUDE


stportal.mahendras.org
Q.5. The total number of employees in Company P was
Missing DI
3 times the total number of employees in Company
This type of DI is mostly in table form where some O. If the difference between the number of Arts
values are missing and some values are given with graduate employees in Company P and that in
some instruction along the table. Company O was 180, what was the total number of
Q.1-5. Study the table and answer the given questions. employees in Company O?
Data related to the number of employees in five (1) 1200 (2) 1440 (3) 720
different companies in December 2017. (4) 900 (5) 1080

Percent Percent of Percent


Explanation : Q.1-5.
Total
Company of Science Commerce of Art Q.1.(3) Total number of employees in company N = 700
Employee
Graduates Graduates Graduates Percentage of Science graduate employees
M 1050 32% – – = [100 – (31 + 40)] = 29%
N 700 – 31% 40%
Now, percentage difference between Arts graduate
O – 30% 30% –
and science graduate employees
P – – 40% 20%
Q – 35% 50% – 11% of 700 = 77
Note:
Therefore, difference = 77
(I) Employees of the given companies can be
Q.2.(2) The percentage of Arts graduate employees in
categorised only in three types: Science graduates,
Company Q = 100 – 35 – 50 = 15%
Commerce graduates and Arts graduates
Now, the percentage of Arts graduate employees
(II) A few values are missing in the table (indicated –).
and Commerce and Arts = 50 + 15 = 65%
A candidate is expected to calculate the missing
value, if it is required to answer the given question, Average = 312
on the basis of the given data and information. Therefore, the total number of employees in
Q.1. What is the difference between the number of commerce and Arts = 2 × 312
Arts graduate employees and Science graduate
Let the total employees in Company Q be x
employees in Company N?
Then, 65% of x = 2 × 312
(1) 87 (2) 89 (3) 77
(4) 81 (5) 73 x = 960

Q.2. The average number of Arts graduate employees Q.3.(1) The percentage of commerce graduate and Arts
and commerce graduate employees in Company Q graduate employees in company M
was 312. What was the total number of employees = 100 – 32 = 68%
in Company Q?
Now, the percentage of Arts graduate employees
(1) 920 (2) 960 (3) 1120
(4) 1040 (5) 1080 = =28%

Q.3. If the ratio of the number of Commerce graduate


employees to that of Arts graduate employees in the percentage of Commerce graduate employees
Company M was 10 : 7, what was the number of
Arts graduate employees in M? = =40%
(1) 294 (2) 266 (3) 280 The number of arts graduate employees in company
(4) 308 (5) 322
M= =294
Q.4. The total number of employees in Company
N increased by 20% from December 2017 to Q.4.(5) The number of employees in company N in
December 2018. If 20% of the total number of December 2017 = 700
employees in Company N in December 2018 were The number of employees in company N in
Science graduates, what was the number of Science December 2018
graduate employees in company N in December
2018? =

(1) 224 (2) 266 (3) 294 Number of Science graduate employees in
(4) 252 (5) 168 company N in December 2018

QUANTITATIVE APTITUDE 165


stportal.mahendras.org
Q.3. What is the total number of female employees who
= work on Ops in Company A and B together?
Q.5.(4) The percentage of Arts graduate employees in (1) 681 (2) 781 (3) 689
company O = 100 – 30 – 30 = 40% (4) 649 (5) 788
The percentage difference between Arts graduate Q.4. What is the difference between the average number
employees in company O and P of males working in ‘Admin’ in both the companies
together and average number of females working
= 40 – 20 = 20% in ‘Other Departments’ in both the companies
Now, let the number of employees in company O together?
be x (1) 26 (2) 36 (3) 16
Then, x × 20% = 180 (4) 24 (5) 14
x = 900 Q.5. In company B, what is the respective ratio between
the total number of employees (both male and
Puzzle DI female) who work in ‘Admin’ and the total number
A puzzle is a set of information that is given in of employees (both male and female) who work in
paragraph form. In a puzzle, no tables or graphs are ‘Other Department’ in the same company?
given. You have to read the information and draw
(1) 2 : 3 (2) 1 : 3 (3) 1 : 4
a table or caselet accordingly by means of which
organize the given data to solve the questions. (4) 3 : 5 (5) 1 : 5

Q.1–5. This data is regarding total number of employees Explanation : Q.1-5.


working in Administration (admin), Operations For company A , Total = 2000
(Ops.) and other departments of corporate divisions
of Companies A and B. Male (1400) Female (600)
The total number of employees working in both the Ops 840 285
companies together is 4800. The respective ratio Admin 70 144
of number of employees in Companies A and B is Other 490 171
5 : 7. Each employee works in only one of the 3
Departments i.e. “ops”, “Admin” and “others”. For Company B –, Total = 2800
In company A, 70% of the total employees are Male (2240) Female (560)
males. 60% of the total male employees work Ops 1456 364
in ‘Ops’ out of the remaining male employees,
Admin 196 49
1/8th work in ‘Admin’. Out of the total female
employees, 24% work in ‘Admin’ and 5/8th of the Other 588 147
remaining female employees work in ‘Ops’. 490
Q.1.(4) Required % = × 100 = 35%
In company B, 80% of the total employees are 1400
males. 65% of the total male employees work in
‘Ops’. Number of male employees who work in 49
Q.2.(2) Required % = × 100 = 8.75%
other departments in Company B is 20% more 560
than the male employees who work in ‘Other Q.3.(4) Required no. of female = 285 + 364 = 649
Departments’ in company A. Number of female
employees who work in Ops in Company B are 171 + 147 70 + 196
Q.4.(1) Required difference = −
less than the number of male employees who work 2 2
for ‘Ops’ in the same company by 75%. Out of the
= 159 – 133 = 26
remaining female employees, 1/4 work in ‘Admin’.
Q.5.(2) Required Ratio = (196 + 49) : (588 + 147)
Q.1. What per cent of the total number of male employees
in company A work in ‘other’ departments? = 245 : 735 = 1 : 3
(1) 45 (2) 25 (3) 30 DI Based on Topics
(4) 35 (5) 40
Now a days in exams new type of DI questions are asked in
Q.2. What per cent of the total number of female which we have to apply the fundamentals which we learned
employees in company B work in administration in previous chapters.
department?
(1) 18.5 (2) 8.75 (3) 14 Q.1-5. Study the data carefully and answer the questions
(4) 16 (5) 19 given below. 
166 QUANTITATIVE APTITUDE
stportal.mahendras.org
The table shows the distance travelled by five dif- Boat Speed of Speed of
ferent boats upstream and downstream in same time boat stream
and the speed of stream. A 24 12
Down- B 24 8
Upstream Speed of C 40 15
Boat stream
distance stream D 25 10
distance
E 40 20
A 96 288 12
Ratio = (24 + 24) : (25 + 40)= 48 : 65
B 120 240 8
C 100 220 15 Q.2.(5) Speed of Boat C in still water
D 150 350 10 = 40 × 110% = 44 km/h
E 180 540 20 Speed of stream = 15 × 120% = 18 km/h
Q.1. Find the ratio of the speed of Boats A and B together
Time taken by Boat C to cover the distance of 91
in still water to the speed of Boats D and E together
km upstream
in still water.
(1) 23:25 (2) 34:41 (3) 48:65
(4) 13:17 (5) None of these = 91
= 3 .5 hours
Q.2. If the speed of Boat C in still water is increased by 44 −18
10% and the speed of stream is increased by 20%, 210 210
Find the time taken by Boat C to cover the distance Q.3.(4) Required answer= + = 1 4 hours
40 − 20 40 + 20
of 91 km upstream.
Q.4.(1) Speed of the Boat B in still water = 24 km/h
(1) 2.5 hours (2) 3 hours (3) 4.5 hours
(4) 6 hours (5) None of these
5
Q.3. The distance between point P and point Q is 210 Speed of the Boat F in still water= 2 4 × = 30
4
km. Boat E travels from point P to Q and comes
back. What is the time taken by Boat E to cover the Let the speed of stream = x km/h
total distance? According to the question,
(1) 10 hours (2) 12 hours (3) 12.5 hours
(4) 14 hours (5) 15 hours 126 81 15
+ =
Q.4. The ratio of the speeds of the Boat B to the Boat F 30 + x 30 − x 2
in still water is 4 : 5. If the Boat F travels 126 km
distance downstream and 81 km distance upstream
in 7 hours 30 minutes. What is the speed of stream On solving, Speed of stream = 12 km/h
of Boat F?
Q.5.(5) Speed of Boat B and D in still water together
(1) 12 Km/hr. (2) 14 Km/hr. (3) 15 Km/hr.
= (25 + 24) = 49 km/h
(4) 10 Km/hr. (5) None of these
Q.5. The speed of Boat B and D in still water together Speed of stream of Boat B and D together
is approximately how much percent more than the
speed of stream of the same boats together? = 18 km/h
(1) 125% (2) 80% (3) 150% Required answer
(4) 190% (5) None of these 49 −18
= × 1 0 0 % ≈ 1172
70%
Explanation : Q.1-5. 18
Q.1-5. Study the following information carefully and
answer the given questions :
96 288
Q.1.(3) = The following table represents time taken (in hours)
a −12 a +12
by different pipes to fill a cistern. Some values are
On solving a=24 Km/h. missing.

similarly for others we can get this table


QUANTITATIVE APTITUDE 167
stportal.mahendras.org
Q.5. Three pipes A, D and F together can fill the cistern
Pipe Time taken to fill the cistern
in 8 hours. Find the time taken by F alone to fill the
A 24 cistern?
(1) 20 hours (2) 15 hours
B --
(3) 18 hours (4) 12 hours
C --
(5) None of these
D 30 Explanation : Q.1-5.
E 45
4 4 5 5
Q.1.(4) + + + =1
F -- 24 x x 30
Q.1. If A and C are kept open for 4 hours then A is 27 1
replaced by D and kept open for 5 more hours, the x= = 13 h
2 2
tank is filled. In how many hours pipe C alone can
fill the cistern? Q.2.(4) x + x + 2 0 =1
30 45 45

3 1
(1) 12 hours (2) 11 hours on solving above equation, we get
4 4
x = 10
Q.3.(4) Ratio of C, F and B to fill the cistern=1 :2 :4
1 1
(3) 9 hours (4) 13 hours
2 2 1 1 1 1
+ +
x 2x 4 x = 48
(5) None of these x = 84
Q.2. Two pipes D and E are opened simultaneously to Required answer=2×84=168 hours
fill the cistern. After how much time should D be
closed so that E alone can fill the cistern in another 1 1 3
20 hours? Q.4.(2) Part of tank filled in 2 hours = + =
24 30 40
(1) 8 hours (2) 15 hours (3) 12 hours
(4) 10 hours (5) None of these 3 39
Part of tank filled in 26 hours = × 13 =
40 40
Q.3. If C takes half of the time taken by F to fill the
cistern and F takes half of the time taken by B to fill 39 1
the cistern and all of them working together can fill Remaining part = 1 − =
40 40
the cistern in 48 hours, What is the time taken by F
Time taken by A to fill remaining part
to fill the cistern?
(1) 120 hours (2) 144 hours (3) 180 hours 1 3
= × 24 =
(4) 168 hours (5) None of these 40 5
Q.4. Two pipes A and D can fill the cistern. If they are
3 3
opened on alternative hours and if pipe A is opened Total time = 26 + = 26
first, in how many hours will the cistern be full? 5 5

Q.5.(1) 1 + 1 + 1
5 3 24 30 x
(1) 24 hours (2) 26 hours
8 5
1
=
8
1 1
(3) 27 hours (4) 26 hours
3 2 on solving above equation, we get
(5) None of these
x = 20

168 QUANTITATIVE APTITUDE


stportal.mahendras.org

EXERCISE
Q.1-5. Study the following table and answer the questions that follow:
The table represents the total sales value (in lakhs) of five books P, Q, R, S and T across eight bookstores A, B,
C, D, E, F, G and H.
BOOKS BOOK STORES
A B C D E F G H
P 560 590 210 670 560 680 420 460
Q 550 560 890 230 820 610 520 230
R 450 290 540 530 500 520 560 410
S 230 240 560 400 430 200 210 480
T 230 400 410 240 200 360 500 470
Q.1. What is the ratio the total sales values of Book P across all the bookstores and Book S across all the bookstores?
(1) 83 : 55 (2) 85 : 53 (3) 83 : 53 (4) 89 : 55 (5) None of these
Q.2. The sale of the book R from Bookstore H is what per cent of the total sale of Book R across all the bookstores
(rounded off to two places after the decimal)?
(1) 21.12 (2) 17.19 (3) 7.43 (4) 3.04 (5) 10.79
Q.3. What is the average sale (in lakh) of Bookstore E?
(1) 502 (2) 504 (3) 512 (4) 540 (5) None of these
Q.4. Book Q constituted approximately what per cent of the total sales of Bookstore C?
(1) 34 (2) 39 (3) 32 (4) 23 (5) 28
Q.5. What is the total sales value (in lakh) of Bookstore D?
(1) 2510 (2) 2670 (3)2900 (4) 2070 (5) None of these
Q.6-10. The table shows the Cost Price of 5 products divided in 3 costs: Production Cost, Transportation Cost and Pack-
aging Cost, the selling price, profit/loss and profit%/loss%. Some values are missing. Find the answers based on
information in table.

Production Transportation Packaging Selling Profit/loss


Product Profit/loss
cost cost cost price %
A 80 16 -- 300 -- 5%
B 100 20 8 -- -- 30% profit
C 90 -- 20 -- 100 --
D 60 12 30 -- -- --
E 120 20 -- 110 -- 10% loss

1
Q.6. If the percentage of profit on selling product A is 9 % , then what is its cost of packaging?
11
(1) 174 (2) 164 (3) 179 (4) 156 (5) None of these
Q.7. What is the difference between the selling price of products B and C, if the cost of transportation of C is Rs 16 and
both are sold at profit?
(1) 54.90 (2) 61.12 (3) 69.44 (4) 59.60 (5) None of these
Q.8. Suppose all the prices are given for per kg of a product. What amount of product B will have to add to 54 kg of
product E such that the resultant product has cost price of Rs 125.
(1) 60 (2) 45 (3) 50 (4) 40 (5) None of these
Q.9. What is the percentage profit (approximate) on selling product D if its selling price is 80% of the selling price of
B?
(1) 20% (2) 30% (3) 25% (4) 22.5% (5) 33%

QUANTITATIVE APTITUDE 169


stportal.mahendras.org
Q.10. If 2 kg of A, 3 kg of C and 4 kg of E are sold, then what will be the final profit/loss% (approximate) on selling these
given. Transportation cost of C as Rs 10 and profit of 5% on selling A?
(1) 15% (2) 18% (3) 12% (4) 20% (5) 10%
Q.11-14. Study the following information carefully and answer the given questions.
The following bar graph shows the selling price of an article (In Rs.) and the profit % earned by selling the article.

700 650 45

600 40
550
35
500

(In Percentage)
30
384 392
(In Rupees)

400 25
288 290
300 20
15
200
10
100 5
0 0
P Q R S T U
Axis Title

Selling Price P%

Q.11. The cost price of article P is approximately what percentage more/less than the cost price of article U?
(1) 23 % less (2) 14 % less (3) 14 % more (4) 8 % more (5) 23 % more
Q.12. Manoj sold an article S to Kamran who again sold it at 25 % profit. Find the difference between the profits earned
by Manoj to that of Kamran?
(1) Rs. 14 (2) Rs. 18 (3) Rs. 24 (4) Rs. 30 (5) None of these
Q.13. Rajesh marked an article Q, which is Rs. 60 above the cost price. How much discount percentage should be given
on marked price to earn the given profit?
(1) 5% (2) 8% (3) 12% (4) 17.5% (5) None of these
Q.14. Find the average cost price of the article P, Q, R and S together.
(1) Rs. 250 (2) Rs. 350 (3) Rs. 300 (4) Rs. 330 (5) None of these
Q.15-19. Read the given bar graph and answer the following questions.

N um ber of students tak ing fresh education loan from different


B ank s
60 00

50 00

40 00 20 08
20 09
30 00
20 10
20 00 20 11
20 12
10 00

0
SBI PNB BOB UCO OBC

170 QUANTITATIVE APTITUDE


stportal.mahendras.org
Q.15. If 23% from UCO in 2009 and 20% from PNB in 2010 have defaulter then, find the total number of defaulter of
UCO and PNB taken together?
(1) 630 (2) 650 (3) 600 (4) 750 (5) 840
Q.16. In 2007, no of defaulters in SBI was 5%. However each year no of defaulters increases by 10% in number. What
will be the difference between the number of defaulters of SBI in the year 2009 and 2012?
(1) 1500 (2) 2000 (3) 1325
(4) 1456 (5) Cannot be determined
Q.17. In which of the following years, the difference in number of students taking loan from Bank BOB from the previ-
ous year is highest?
(1) 2008 (2) 2009 (3) 2010 (4) 2012 (5) None of these
Q.18. If on average, Rs. 175000 per student's education loan sanctioned by OBC bank all over the year. What will be
total amount sanctioned by OBC in all given years?
(1) Rs. 1055600000 (2) Rs. 1055800000 (3) Rs. 1620000000 (4) Rs. 1050000000 (5) None of these
Q.19. What is the ratio of Number of students taking Education Loans from SBI and BOB together in all the years and
the total no of students taking Education loans in 2010 and 2011 together?
(1) 8 : 5 (2) 5 : 7 (3) 7 : 5 (4) 9 : 7 (5) None of these
Q.20-24. The following Line chart gives the ratio of the amounts of imports by a Company to the amount of exports from
that Company over the period from 2011 to 2017. Answer the following questions based on following Line graph.
Ratio of value of imports to export

2011 2012 2013 2014 2015 2016 2017

Q.20. In how many of the given years were the exports more than imports ?
(1) 1 (2) 2 (3) 3 (4) 4 (5) None of these
Q.21. The imports were minimum proportionate to the exports of the Company in the year
(1) 2013 (2) 2011 (3) 2012 (4) 2016 (5) None of these
Q.22. If the imports of a company in 2012 was Rs. 272 crores, the exports from the company in 2012 was
(1) Rs 120 Crores (2) Rs 220 Crores (3) Rs 320 Crores (4) Rs 420 Crores (5) None of these
Q.23. What was the percentage increase in imports from 2013 to 2014 ?
(1) 70 (2) 72 (3) 74 (4) Data Inadequate (5) None of these
Q.24. If the imports in 2014 was Rs. 250 crores and the total exports in years 2014 and 2015 together was Rs 500 crores,
then the imports in 2015 was
(1) 320 Crore (2) 420 Crore (3) 520 Crore (4) 620 Crore (5) None of these

QUANTITATIVE APTITUDE 171


stportal.mahendras.org
Q.25-28. Study the following graph carefully and answer the questions given below it.
Profit percent earned by two companies in various years.

Profit percent of two companies in


various years

60
55 55
50
45 45 45
40 40
35
30
25

2011 2012 2013 2014 2015 2016

A B

Q.25. In the year 2013 the income of company A was Rs. 261000 and income of company B was Rs. 256500. Find the
difference of their expenditures.
(1) 15000 (2) 12000 (3) 10000 (4) 17500 (5) None of these
Q.26. Company A spent Rs. 185000 in the year 2011. Find out the profit of company A in that year after 18% tax payment
on total income?
(1) 28580 (2) 32450 (3) 27380 (4) 25290 (5) None of these
Q.27. If the expenditure of both the companies A and B in the year 2015 was same, then what was the ratio between the
incomes of company A to Company B?
(1) 8 : 7 (2) 7 : 8 (3) 5 : 4 (4) 4 : 5 (5) None of these
Q.28. In which of the following years was there maximum percentage of growth/decline with respect to the previous year
in case of company A?
(1) Years 2012 (2) Years 2015 (3) Years 2011
(4) Years 2016 (5) Can not be determined
Q.29-33.Study the graphs carefully to answer the questions that follow.
Total number of children in 6 different schools and the percentage of girls in them
35 00

30 00
N um ber of childrens

25 00

20 00

15 00

10 00

50 0

0
P Q R S T U
S chool

172 QUANTITATIVE APTITUDE


stportal.mahendras.org

50
45
40
P ercentage of girls

35
30
25
20
15
10
5
0
P Q R S T U
S chool

Q.29. What is the total percentage of boys in schools R and U together? (rounded off to two digits after decimal)
(1) 78.55 (2) 72.45 (3) 76.28 (4) 75.83 (5) None of these
Q.30. What is the total number of boys in school T?
(1) 500 (2) 600 (3) 750 (4) 850 (5) None of these
Q.31. The total number of students in school R, is approximately what per cent of the total number of students in school
S?
(1) 89 (2) 75 (3) 78 (4) 82 (5) 94
Q.32. What is the average number of boys in schools P and Q together?
(1) 1425 (2) 1575 (3) 1450 (4) 1625 (5) None of these
Q.33. What is the respective ratio of the number of girls in schools P to the number of girls in school Q?
(1) 27 : 20 (2) 17 : 21 (3) 20 : 27 (4) 21 : 17 (5) None of these
Q.34-38. The annual rate of interest offered by the two Companies P and Q over the years are shown by the line graph
provided below.

Rate of interest
12
10
9.5
10 9 9
8 8 8 8
7.5
8 7
6.5 6.5
6
6
4
4

0
2006 2007 2008 2009 2010 2011 2012

Company P Company Q

QUANTITATIVE APTITUDE 173


stportal.mahendras.org
Q.34. A sum of Rs. 4.75 lakhs was invested in Company Q in 2009 for one year. How much more interest would have
been earned if the sum was invested in Company P?
(1) 8400 (2) 9200 (3) 9500 (4) 10250 (5) None of these
Q.35. If two different amounts in the ratio 8:9 are invested in Companies P and Q respectively in 2012, then the amounts
received after one year as interests from Companies P and Q are respectively in the ratio?
(1) 5:2 (2) 2:5 (3) 4:3 (4) 3:4 (5) None of these
Q.36. In 2010, a part of Rs. 30 lakhs was invested in Company P and the rest was invested in Company Q for one year.
The total interest received was Rs. 2.43 lakhs. What was the amount invested in Company P? (in lakhs)
(1) 15 (2) 17.5 (3) 16 (4) 12 (5) 18
Q.37. An investor invested a sum of Rs. 12 lakhs in Company P in 2008. The total amount received after one year was
re-invested in the same Company for one more year. The total appreciation received by the investor on his invest-
ment was?
(1) 225600 (2) 248500 (3) 292000 (4) 265000 (5) None of these
Q.38. An investor invested Rs. 5 lakhs in Company Q in 2006. After one year, the entire amount along with the interest
was transferred as investment to Company P in 2007 for one year. What amount will be received from Company
P, by the investor?
(1) 520450 (2) 580425 (3) 575655 (4) 624500 (5) None of these
Q.39-43. Study the following bar graph and answer the given questions.

R atio betw een production and consum ption of w heat, rice and
pulses in different years in india
2
1.8
1.6
1.4
1.2
1
0.8
0.6
0.4
0.2
0
20 12 20 13 20 14 20 15 20 16

W hea t R ice P ulses

Q.39. If in the year 2013 the production of rice is 15 ton and in the year 2014 the consumption of pulses is 14 ton. Find
the ratio between consumption of rice in the year 2013 and production of pulses in the year 2014.
(1) 7 :5 (2) 5 : 7 (3) 4 : 5 (4) 9 : 7 (5) 7 : 9
Q.40. If in the year 2014 the consumption of rice and wheat is same. Find the ratio of production of rice and wheat in the
same year.
(1) 3:7 (2) 4 : 7 (3) 7 : 4 (4) 7 : 3 (5) Data inadequate
Q.41. If in the year 2012 the production of pulses is 21 ton and consumption of rice is 20 ton. How much % the consump-
tion of pulses in 2012 is more/less than production of rice in same year.
(1) 24% (2) 25% (3) 45% (4) 60% (5) 55%
Q.42. If  export=production-consumption then find the % change of export of rice in the year 2015 as compare to previ-
ous year.
(1) 100% (2) 50% (3) 50% (4) No change (5) Data inadequate

174 QUANTITATIVE APTITUDE


stportal.mahendras.org
Q.43. If production of wheat and consumption of Rice in 2012 is 17.5 ton and 12.5 ton. Find the diiference between
consumption of wheat and production of rice in same year.
(1) 2 ton (2) 3 ton (3) 3.5 ton (4) 2.5 ton (5) 1.5 ton
Q.44-48. Read the following information carefully and answer the questions given below.
In a city, the account holders have their accounts in 5 different banks viz. Allahabad Bank, Dena Bank, Andhra
Bank, Corporation Bank and UCO Bank. The total number of account holders is 2050. 24% of total account hold-
ers have account in Allahabad Bank. One-fifth of the total account holders have account in Corporation Bank. 16%
of total account holders have account in Andhra Bank. Remaining account holders have account in either UCO
Bank or Dena Bank. The number of accounts in Dena Bank is 182 less than the number of accounts in UCO Bank.
Q.44. What is the total number of accounts in UCO Bank, Andhra Bank and Corporation Bank?
(1) 1027 (2) 1157 (3) 1057 (4) 957 (5) None of these
Q.45. What is the respective ratio of number of accounts in Corporation Bank and that in UCO Bank and Dena Bank
together?
(1) 2 : 1 (2) 1 : 2 (3) 3 : 4 (4) 7 : 6 (5) None of these
Q.46. The number of accounts in Andhra Bank is approximately what percent of the total number of accounts in Alla-
habad Bank and Corporation Bank together?
(1) 63 (2) 26 (3) 46 (4) 56 (5) 36
Q.47. What is the total number of accounts in Dena Bank and UCO Bank?
(1) 820 (2) 800 (3) 840 (4) 860 (5) None of these
Q.48. The number of accounts in Andhra Bank is approximately what percent more than that in Dena Bank?
(1)10 (2) 8 (3) 12 (4) 3 (5) 5
Q.49-53. There are five students who appeared for RBI Grade B exam. Paper consists of 100 questions with 1 mark for
each correct answer and 0.25 marks for each wrong answer.
Students Questions attempted Right Questions Wrong Questions Marks obtained

Aditya 78 – – 70.5
Puskar 92 76 – –
Anshuman 98 – 36 –
Alka – 30 – 27.25
Avanish 56 – – 53.50

Q.49. Difference between total right number of questions of all students together and total wrong no. of questions of all
students together is
(1) 141 (2) 161 (3) 223 (4) 156 (5) None of these
Q.50. Marks obtained by Aditya and Puskar together is what % of the marks obtained by Anshuman, Avanish and Alka
together? (rounded off to 2 decimal places)
(1) 106.54% (2) 91.16% (3) 95.20% (4) 96.71% (5) 101.71%
Q.51. If the penalty of wrong answer is 0.33 then marks obtained by Aditya, Anshuman and Puskar together is
(1) 192.21 (2) 224.19 (3) 190.86 (4) 219.14 (5) 194.22
Q.52. If the passing % marks in the exam is 50 marks than at least how many questions has to be answered right by
Puskar? (He attempted 92 questions)
(1) 58 (2) 56 (3) 59 (4) 55 (5) 60
Q.53. What is the percent of marks obtained by all of them together?
(1) 59.03% (2) 53.15% (3) 52.53%
(4) 45.05% (5) 55.25%

QUANTITATIVE APTITUDE 175


stportal.mahendras.org
Q.54-58. Study the following information and answer the questions asked:

The premises of an office are to be renovated. The renovation is in terms of flooring. Certain areas are to be floored
either with tiles or wooden flooring. All rooms/halls and pantry are rectangular. The area to be renovated comprises
of a hall for customer care executive measuring 24 m by 18 m, team leader’s room measuring 12 m by 15 m, a
pantry measuring 18 m by 16 m, a server room measuring 21 m by 14 m and cafeteria area measuring 26 m by 22
m. The total area of the office is 1950 square meters. The cost of wooden flooring is 175/- per square meter and
the cost of tiles flooring is 160/- per square meter. The cafeteria area, server room and pantry are to be floored with
tiles. Team leader’s room and the hall for customer care executive are to be floored with wooden flooring. No other
area is to be renovated in terms of flooring.

Q.54. What is the difference between the total cost of wooden flooring and total cost of tiles flooring (in Rs.)?

(1) 120800 (2) 84375 (3) 77540 (4) 62500 (5) None of these

Q.55. If the four walls and ceiling of the team leader’s room (the height of the room is 8 meter) are to be painted at the
cost of 180/- per square meter, how much will be the total cost of renovation of the team leader’s room including
the cost of flooring? (Note-you have not to consider any left area for door or window)

(1) 142560 (2) 112800 (3) 108600 (4) 132000 (5) None of these

Q.56. If the remaining area of the office is to be carpeted at the rate of 175/- per square meter, how much will be the
increment in the total cost of renovation of office premises?

(1) 32200 (2) 30450 (3) 39750 (4) 44800 (5) None of these

Q.57. What is the approximate percentage area of the office that is not to be renovated?

(1) 11.11% (2) 10.52% (3) 9.44% (4) 6.94% (5) 12.35%

Q.58. What is the total cost of renovation of the hall for customer care executive and the cafeteria area?

(1) 162820 (2) 167120 (3) 159080 (4) 175200 (5) None of these

Q.59.-63. Read the following pie-chart carefully and answer the questions asked.
       Population distribution of states in two years
       Total population in 2016=50 Lakhs

Tamilnadu
Karnataka
12%
15%

Andhra
Pradesh
9% Uttarpradesh
20%
Maharastra
11%

Kerala
8% Bihar
25%

176 QUANTITATIVE APTITUDE


stportal.mahendras.org
         Total population in 2017=56 Lakhs

Karnataka Tamilnadu
15% 20%

Andhra
Pradesh
8%
Uttarpradesh
Maharastra 12%
12%

Kerala
10% Bihar
23%

Q.59. What is the respective ratio of population of Tamil Nadu in 2016 to that of 2017?
(1) 8:13 (2) 17:29 (3) 15:28 (4) 15:31 (5) None of these
Q.60. What was the approximate difference between percent increase in population of Bihar and Kerala from 2016 to
2017?
(1) 32% (2) 41% (3) 28% (4) 37% (5) 51%
Q.61. What is difference between population of Tamil Nadu  in 2016 and Karnataka in year 2017?
(1) 280000 (2) 240000 (3) 180000 (4) 320000 (5) None of these
Q.62. If in 2018 population of Bihar is increased by 15% whereas the population of Uttar Pradesh is decreased by 12%
as compared to previous year, Find the sum of population of these two states in 2018.
(1) 2072560 (2) 2284320 (3) 3122450 (4) 1844620 (5) None of these
Q.63. Find the difference of central angle of Uttar Pradesh and Kerala in 2016.
(1) 17.20 (2) 32.40 (3) 44.40 (4) 30.80 (5) None of these
Q.64.68.Study the following information carefully to answer the questions asked.
The following bar graph represents the exports (in Cr Rs.) and imports (in Cr Rs.) of five countries in 2016.
Import and Export of five countries

Import Export

350
296
300 280
270 264
240
250
200
200
156 150
144
150
100
100

50

0
Pakistan Bangladesh China Bhutan India

QUANTITATIVE APTITUDE 177


stportal.mahendras.org

Ratio of import Ratio of export


Country
(2016 to 2017) (2016 to 2017)
Pakistan 27:25 2:3
Bangladesh 39:41 12:11
China 37:32 5:3

Bhutan 10:11 18:19

India 15:16 7:5

Q.64. Find the respective ratio between Import of China in 2017 and export of Bhutan in 2017.
(1) 32:17 (2) 32:19 (3) 35:23 (4) 19:17 (5) None of these
Q.65. Imports of all the countries together in 2016 is what percent of the exports of all the countries together in 2016?
(1) 112.57% (2) 108.59% (3) 106.22% (4) 123.15% (5) 119.62%
Q.66. Find the sum of the imports of the Pakistan and Bangladesh in 2017 and exports of the China and Bhutan in 2017.
(1) Rs. 626 crore (2) Rs. 616 crore (3) Rs. 606 crore (4) Rs. 636 crore (5) None of these
Q.67. Find the difference between the sum of Import and Export of  India in 2016 and that in 2017. (in crore)
(1) 70 (2) 62 (3) 74 (4) 80 (5) None of these
Q.68. Import of China in 2017 is approximately what percent more/less than the export of the same country in same
year?
(1) 221% (2) 427% (3) 369% (4) 327% (5) 411%
Q.69-73. Read the following informations carefully to answer the questions asked-
There are three sections in class 12 and students are given two choices weather to choose Science or Art as one of
optional subject. Related informations are given as below-
4
(1)  In section C there are total 66 girls out of which  45 %   girls choose Arts which is 20 less than number of
9
boys choosing Arts in same section.
(2)  Ratio of boys choosing Science to boys choosing Arts in section C is 24:25.
(3)  In section B, 12 more boys and 9 more girls choose Science as compared to section C.
(4)  In section B, 40 boys choose Arts which is 20% less than number of girls choosing Arts in same section.
(5)  Overall 188 boys and 121 girls choose science.
(6)  Total number of students taking all three sections together is 604.
(7)  Ratio of boys and girls choosing Art in section C is 11:14 respectively.
Q.69. Find the total number of girls who choose art.
(1) 120 (2) 145 (3) 150 (4) 175 (5) None of these
Q.70. Find the total number of students in section B.
(1) 175 (2) 195 (3) 205 (4) 185 (5) None of these
Q.71. Find difference of boys who choose Science in section A and number of girls who choose art in section B.
(1) 20 (2) 25 (3) 30 (4) 15 (5) None of these
Q.72. In section A number of boys is approximately how much percent less/more than the number of girls in same sec-
tion?
(1) 22.7% (2) 30.2% (3) 25.6% (4) 21.2% (5) 24.4%
Q.73. Find the ratio of girls choosing Art and Science respectively.
(1) 135:111 (2) 150:121 (3) 73:51 (4) 19:13 (5) None of these

178 QUANTITATIVE APTITUDE


stportal.mahendras.org

EXERCISE Explanation
Q.1.(1) Total sales values of Book P across all the SP = 110, loss% = 10%
bookstores
So CP of E = 100/90 × 110 = Rs 122 (2/9)
= 560 + 590 + 210 + 670 + 560 + 680 + 420 +
460 = 4150 Using method of alligation
Total sales value of Book S across all the
bookstores (x kg)…………………………(54 kg)
= 230 + 240 + 560 + 400 + 430 + 200 + 210 Rs 128 Rs 122 (2/9)
+480 = 2750
Ratio = 4150: 2750= 83 : 55 Rs 125
Q.2.(5) Total sales values of Book R across all (25/9) (3)=25:27
bookstores
= 450 + 290 + 540 + 530 + 500 + 520 + 560 + So
410 = 3800
Here 27=54
Sales of book R from Bookstore H is 410.
percent of books from Bookstore H 1=2
410 25=50
= × 1 0 0 = 10.789 =10.79
3800
Q.9.(2) SP of B = 130/100 × 128= Rs 166.4
Q.3.(1) Average sales of Bookstore E
SP of D = 80/100 × 166.4 = Rs 133.12
= (560+820+500+430+200)/5= 502
Q.4.(1) Total sales of Bookstore C= 210 + 890 + 540 + CP of D = 60+12+30 = Rs 102
560 + 410 = 2610
So profit% = 31.12/102 × 100 = 30%
Sales value of Q in Bookstore C = 890
890 Q.10.(4) CP (2 kg of A + 3 kg of C + 4 kg of E) = 2×286
percent of Q in C = 2 6 1 0 × 1 0 0 = 34%
+ 3×120 + 4×122 = Rs 1420

Q.5.(4) Total value of Bookstore D = 670 + 230 + 530 + Similarly SP of (2 kg of A + 3 kg of C + 4 kg of


400 + 240 = 2070 E) = 2×300 + 3×220 + 4×110 = Rs 1700
1
Q.6.(3) SP = 300, profit% = 9 %
11 So profit %
11 280
So CP = 300 × = 275 = × 100% ≈ 20%
12 1420
So cost of packaging = 275– (80+16) = Rs 179 Q.11-14.

Q.7.(4) CP of B = 100+20+8 = Rs 128 Article CP SP


P 500 650
30% profit Q 240 288
R 300 384
So SP of B = 130/100 × 128 = Rs 166.4
S 280 392
CP of C = 90+16+20 = Rs 126 T 250 290
U 440 550
Profit = Rs 100
500 − 440
Q.11.(3) Required answer = × 100% ≈ 14%
So SP of C = 126+100= Rs 226 440
Q.12.(1) Profit earned by Manoj =392-280=112
So difference = 226 –166.4 = Rs 59.6
Profit earned by Kamran =490-392=98
Q.8.(3) CP of B = 100+20+8 = Rs 128
Required answer=112-98=14 Rs.
For CP of E:
QUANTITATIVE APTITUDE 179
stportal.mahendras.org
Q.13.(5) CP=240 Let the exports from the company in 2012 was x
then,
MP=240+60=300
272 272
= 0.85 Þ x = Þ x = 320
SP=288 x 0 .8 5
300 − 288 Note: Please not that we are given the ratio of
Discount % = × 100% = 4%
300 imports to exports, so export will in denominator
Q.14.(4) Required answer Q.23.(4) For calculating the percentage we will need
500 + 240 + 300 + 280 1320 value of exports, imports etc. We are only given
= = = 330
4 4 with the ratio. So data in Inadequate
Q.15.(1) Students taking loan from UCO in 2009 = 1000 Q.24.(2) The Ratio of imports to exports for the years
Defaulters (UCO) = 23% of 1000 = 230 2014 and 2015 are 1.25 and 1.40 respectively.

Person taking loan from PNB in 2010= 2000 Let the exports in the year 2014

Defaulters (PNB) = 20% of 2000 = 400 = Rs. x crores

Total desired defaulters = 230 + 400 = 630 Then, the exports in the year 2015

Q.16.(5) Cannot be determined because no. of students = (500-x) crores


taking a loan from SBI in 2007 is unknown. 250
⇒ 1 .2 5 =
Q.17.(2) From graph, it is clear that in 2009, difference x
between no. of students taking a loan is highest
as compared to previous year. [because 1.25 is 2014 ratio]

Q.18.(5) No. of students taking education loan from OBC 250


⇒x= = 2 0 0 crore
bank all over the year 1 .2 5
= 1000 + 1000 + 1500 + 2000 + 1500 Thus the exports in the year 2015 were 500 -
= 7000 200 = 300 crore
Total loan amount sanctioned over the years Let the imports in the year 2015
= 7000 × 1,75,000 = Rs y crore
= Rs. 1,22,50,00,000 y
Then 1.40 =
Q.19.(3) SBI : 2500 + 3000 + 4500 + 4000 + 5000 300
= 19000 => y = 1.40×300 = 420 crore
BOB : 2500 + 3500 + 4000 + 4500 + 5000 Q.25.(3) Required answer
= 19500 100 100
= 256500 × – 261000 ×
Total no. of students taking loan in 2010 135 145
= 13500 =190000-180000 = 10000
Total no. of students taking loan in 2011 = 14000
Q.26.(3) Income=140% of 185000=259000
Desired ratio
1 9 0 0 0 +1 9 5 0 0 3 8 5 0 0 7 Income after tax deduction=82% of
= 1 3 5 0 0 +1 4 0 0 0 = 2 7 5 0 0 = 5 259000=212380

Required answer=212380-185000=27380
Q.20.(4) Clearly the exports are more than the imports
implies that the ratio of value of imports to Q.27.(2) Required ratio = 7 : 8
exports in less than 1.
So years are 2011, 2012, 2013 and 2016. So Q.28.(5) Can not be determined
these are four years
Q.29.(5) Number of boys in school R and U together
Q.21.(1) Clearly from the line graph we can judge it is
minimum in year 2013. 2 0 0 0 × 7 7 .5 8 7 .5
+ 1000 ×
100 100
Q.22.(3) We are given with the ratio of imports and
exports in the line graph. = 1550 + 875 = 2425
180 QUANTITATIVE APTITUDE
stportal.mahendras.org
∴Required percentage (30 - x) lakhs.
2425 Total interest received from the two Companies
= × 1 0 0 = 8 0 .8 3 %
3000 after 1 year

Q.30.(3) Number of boys in school T = Rs. [(7.5% of x) + {9% of (30 - x)}] lakhs

1250 × 60 2.7–1.5% of x = 2.43
= = 750
100
1.5% of x = 0.27
Q.31.(1) Total number of students in school R x = 18
= 2000 Q.37.(1) Amount received from Company P on investing
Rs. 12 lakhs after one year
Total number of students in school S
= Rs. [12 + (8% of 12)] lakhs
= 2250
∴Required percentage = Rs. 12.96 lakhs.
2000 Amount received from Company P on investing
×100 ≈ 89
2250 Rs. 12.96 lakhs after one year

Q.32.(2) Required average = Rs. [12.96 + (10% of 12.96)] lakhs


1  25000 + 60 3000 × 55  = Rs. 14.256.
=  + 
2 100 100 
Appreciation received on investment during the
period of two years
1 1 = Rs. (14.256 - 12) lakhs
= (1 5 0 0 + 1 6 5 0 ) = × 3 1 5 0 = 1 5 7 5
2 2
= Rs. 2.256 lakhs
Q.33.(3) Required ratio
= Rs. 225600.
2500 × 40 3000 × 45
= : Q.38.(2) Amount received from Company Q after one year
100 100
on investment of Rs. 5 lakhs
= 25 ×40 : 30 × 45
= Rs. [5 + (6.5% of 5)] lakhs
= 100 : 135 = 20 : 27
Q.34.(3) Required difference=(10% of 4.75)-(8% of 4.75) = Rs. 5.325 lakhs.
lakhs=9500 Amount received from Company P after one year
Q.35.(3) Let the amounts invested in 2012 in Companies on investment of Rs. 5.325
P and Q be Rs. 8x and Rs. 9x respectively. = Rs. [5.325 + (9% of 5.325)] lakhs
Then, interest received after one year from = Rs. 5.80425 lakhs
48 = Rs. 580425
Company P=6% of 8x = x
100
and interest received after one year from Company
Q.39.(2) 3
15 : 14 × =5:7
36 2
Q= x
100
Q.40.(3) 1.4 : 0.80 = 7 : 4
48 Q.41.(2) PP : PC = 7 : 10
x
4 RP : RC = 6 : 5
Required ratio = 100 =
36 3
x For pulse For rice
100
7 = 21 5 = 20
Q.36.(5) Let Rs. x lakhs be invested in Company P in 2010,
10 = 30 6 = 24
the amount invested in Company Q in 2010 = Rs.

QUANTITATIVE APTITUDE 181


stportal.mahendras.org
Required percent pulse more 7 0 .5 +7 2
30 − 24 = 5 3 +2 7 .2 5 +5 3 .5 0 × 1 0 0
× 1 0 0 = 25%
24 = 106.54%
Q.42.(5) Data inadequate Q.51.(3) Required marks = (72 + 76 + 62) – 0.33 (6 + 16
+ 36) = 190.86
Q.43.(4) Difference = 17.5 - 15 = 2.5 ton
Q.52.(3) By options
Q.44.(5) Allahabad Bank - 492
Let right Questions = 59
Dena Bank - 319
marks =92−14(92−59)=50.75
Andhra Bank - 328
Q.53.(5) Required %
Coporation Bank - 410
7 0 .5 +7 2 +5 3 +2 7 .2 5 +5 3 .5 0
UCO Bank - 501 = ×100
500
Required number = 501 + 328 + 410
= 55.25%
= 1239
Q.54-58.
Q.45.(2) Allahabad Bank - 492
Dena Bank - 319 Type of
Portion Dimension Area
Andhra Bank - 328 flooring
Coporation Bank - 410 Hall (C.C.E.) 24×18 432 Wooden
UCO Bank - 501 T.L. Room 12×15 180 Wooden
Ratio = 410 : 820 = 1 : 2 Pantry 18×16 288 Tiles
Q.46.(5) Allahabad Bank - 492 Server Room 21×14 294 Tiles
Dena Bank - 319 Cafeteria 26×22 572 Tiles
Andhra Bank - 328 Q.54.(3) Required answer = 1154×160 - 612×175=77540
Q.55.(5) Required answer = 180×175+{2(12+15)×8
Coporation Bank - 410
+180}×180=141660
UCO Bank - 501
328 Q.56.(1) Required answer = 175 × (1950 - 1766) = 32200
Percent = × 1 0 0 = 36
902
Q.57.(3) Required answer = 1950 − 1766 × 100 ≈ 9.44%
Q.47.(1) Allahabad Bank - 492 1950
Dena Bank - 319 Q.58.(2) Required answer = 432×175+572×160=167120
Andhra Bank - 328 Q.59.(3) Required answer 12% of 50 : 20% of 56
Coporation Bank - 410
Ratio = 15 : 28
UCO Bank - 501
Total = 501 + 319 = 820 1288 − 1250
Q.60.(4) Bihar = ×100 ≈ 3%
Q.48.(4) Allahabad Bank - 492 1250

Dena Bank - 319
560 − 400
Andhra Bank - 328 Kerala = ×100 = 40%
400
Coporation Bank - 410
UCO Bank - 501 Required answer = 40 - 3 = 37
9 Q.61.(2) Required answer 15% of 56 lakhs - 12% of 50
Percent increase = ×100
319 lakhs=240000
= 3 (approx) Q.62.(1) Required answer (115% of 23% + 88% of 12%)
Q.49.(3) Required difference = (72 + 76 + 62 + 30 + 54) of 56 lakhs=2072560
– (6 + 16 + 36 + 11 + 2) Q.63.(5) Required answer
= 294 – 71 = 223 18
Q.50.(1) Required % (20-8) × =43.20
5

182 QUANTITATIVE APTITUDE


stportal.mahendras.org
Q.64.(2) 19
Country Import Export Export of BHUTAN in 2017 = ×144 = 152
18
2016 2017 2016 2017 Required sum = 250 + 164 + 60 + 152
Pakistan 270 250 240 360 = Rs. 626 crore
Bangladesh 156 164 264 242 Q.67.(1) Sum of Import and Export of INDIA in 2016
China 296 256 100 60 = 430

Bhutan 220 242 144 152 Sum of Import and Export of  INDIA in 2017

India 150 160 280 200 = 360


Required difference = 430 – 360= 70
Import of China in 2017= 256
Q.68.(3) Import of CHINA in 2017 = 256
Export of Bhutan in 2017= 152
Export of CHINA in 2017 = 60
Required ratio = 32:19
Required percentage
Q.65.(3) Imports of all the countries together in 2016
= 1092 256 − 60
= ×100% ≈ 327
Exports of all the countries together in 2016 60
= 1028
Q.69.(3)
Required percentage
Boys Girls
=
1092 Section
× 100 = 106.22 Science Art Science Art
1028
A 80 55 40 70
25 B 60 40 45 50
Q.66.(1) Import of Pakistan in 2017 = 270 250
27
C 48 50 36 30
Import of Bangladesh in 2017 = 164 Required answer = 70+50+30 = 150

41 Q.70.(2) Required answer=195


= ×156 = 164 Q.71.(3) Required answer=80-50=30

39
Q.72.(1) Required answer
3
Export of China in 2017 = ×100 = 60 135 −110
5 =  ×100% = 22.7%
110
Q.73.(2) Required answer=150:121

QUANTITATIVE APTITUDE 183


stportal.mahendras.org

CHAPTER

21 MENSURATION
Regular Ploygon :- A polygon that has all sides and all
Mensuration
interior angles equal.
Mensuration is the branch of mathematics which deals with
the study of different geometrical shapes, their area and Convex Polygon :- A convex polygon is defined as a polygon
volume. In the broadest sense, it is all about the process of with all its interior angle less than 180o.
measurement. It is based on the use of algebraic equations B
and geometric calculations to provide measurement data
regarding the width, depth and volume of a given object or A C
group of objects. While the measurement results obtained by
the use of mensuration are usually considered very accurate. D
There are two types of geometric shapes :
Concave Polygon :- A polygon that has one or more interior
1. 2D : Those figures have two dimension is called 2D figures. angle greater than 180o.
like- square, rectangle, parallelogram, rhombus,
triangle, trapezoid, circle etc. D C
2. 3D : Those figures have three dimension are called
3D figures. like- cube, rectangular prism (cuboid),
cylinder, cone, sphere, hemisphere, prism, pyramid A B
etc.
Properties of polygon :-
Polygon :- A polygon is a closed, plane figure bounded by
‘n’ straight lines (n > 3). Each of the n line segments forming (n = number of sides in polygon)
the polygon is called its sides. 1. Sum of interior and exterior angle is 180o.
C
2. Sum of interior angles of polygon is (n–2)×180o.
B D
3. Sum of the exterior angles of polygon is 360o.
4. Each interior angle of regular polygon is
A E
Type of polygon No. of side (n − 2 ) × 1 8 0
Triangle 3 n
Quadrilateral 4 5. Each exterior angle of regular polygon
Pentagon 5
Hexagon 6 3600
=
Heptagon 7 n
6. Number of diagonals of a polygon
Octagon 8
Nonagon 9 n (n − 3 )
=
Decagon 10 2

184 QUANTITATIVE APTITUDE


stportal.mahendras.org

[Plane Figures]
Plane figures are flat two dimenstional shape. A plane figure can be made of straight lines, curved lines, or both straight and
curved lines. Ex. Square, Rectangle and Triangle etc.
In mathematics there are many plane figures which is listed below along with formulas.

S. NO. Name Figure Nomenclature Area Perimeter


l → length
Rectangle b → breadth l×b = lb 2l+2b = 2(l+b)
1. d b d → diagonal
l 2 2
d= l +b
a
2. Square (i) a×a = a2 a+a+a+a = 4a
a → side
a d → diagonal d2
a d
(ii)
d = a 2 2
a
1
Triangle a+b+c = 2s
3. a, b and c are three (i) 2 ×b×h
(Scalene)
sides of triangle and
a c s is semiperimeter, (ii) s (s − a )(s − b )(s − c )
h
where (Heron’s formula)
a +b +c
b s =  2

b is the base and h
is the altitutde of tri-
angle
h = height or altitude

Equilateral a → equal 1 3a
4. (i) ×a×h
triangle sides 2
h → height or
a h a 3 2
altitude (ii) a
3 4
h = a
a 2


Isosceles a → equal 1 2a+b
5.
triangle sides (i) ×b×h
2
b → base
a a h → height or 1
h (ii) 4
× b × 4a 2 − b 2
altitude
4a 2 − b 2
b h =
2
Right angled b → base 1 b+h+a
6. ×b×a
triangle a → altitude/height 2
a h
h = h y p o t e n u s e
900 h = a2 + b2
b

QUANTITATIVE APTITUDE 185


stportal.mahendras.org

S. NO. Name Figure Nomenclature Area Perimeter

a → equal sides 2
7. Isosceles right angled a b 1 2 b 2a + b
b → other side a =
triangle 2
900 b = a 2 4
a

D C
AC is the diagonal and
8. Quadrilateral h1 h1, h2 are the altitudes on AB+BC+CD+AD
1
h2 AC from the vertices D × A C × (h 1 + h 2 )
and B respectively. 2
A B

9. Parallelogram a and b are side adjacent


to each other. 2(a+b)
h→ distance between a×h
the parallel sides.

a
10. Rhombus D C a- length of each side of
d 1 90 0 rhombus
a 90 0 d1 and d2 are the diago- 1 4a or 2 d 12 + d 22
90 0 a × d1 × d 2
nals
d 2 90
0 2
d1 → BD
A a B d2 → AC
1 2 2
a= d1 + d 2
2

11. Trapezium D b C
a and b are parallel sides  a + b  × h AB+BC+CD+AD
to each other and h is the  2 
h perpendicular distance
between parallel sides.
A a B

a
12. Regular hexagon a 3 3 2 6a
a
a
a → length of each side 2
a a
a

a a
13. Regular
a a a → length of each side (
2a 2 1 + 2 ) 8a
octagon
a a
a a

Circle πr2
14. r → radius of the circle 2πr (called as cir-
cumference)

186 QUANTITATIVE APTITUDE


stportal.mahendras.org
S. NO. Name Figure Nomenclature Area Perimeter

15. Semicircle 1 2 36
r → radius of the circle πr r (π+2) = r
2 7

16. Quadrant r → radius 1/4 πr2 1 25


πr+2r = r
2 7

R→ outer π(R2 – r2) (outer) →2πR


17. Ring or circlular path (shaded R
(inner) →2πr
region) radius
r
r →inner radius

O→ centre of the  θ 
circle πr2  3 6 0 o  l+2r
18. Sector of a circle
O r → radius
θ
r l→ length of the arc
l θ→ angle of the sec-
A
tor
B
l  θ 
l → 2πr 
 3 6 0 o 

Area of segment
19. Segment of a circle θ → angle of the sec-
O tor ACB (minor seg- 2r  πθ o + sin  θ  
r  360 2
 r → radius ment)
A B AB → chord  πθ sin θ 
= r2  3 6 0 o − 
ACB → arc of the 2 
C
circle
l l → length 2(l+b) - 4w
(l+b – w)w
20. Pathways running parallel W b → breadth
inside to sides of a rectanlge w → width of the path
W W
(road)
b
W

(l+b+2w) 2w (inner) →2 (l+b)


21. Outer path W l → length (outer) →2 (l+b+4w)
l b → breadth
bW w → width of the path

W l → length (Other)→2(l+b)
22. Inner path (l+b–2w) 2w
W b → breadth (inner)→2(l+b-4w)
b
w → width of the path
l

QUANTITATIVE APTITUDE 187


stportal.mahendras.org

Solid Figures
Solid figures are three dimensional objects what this mean is that solid figures which have a length a width and a height
(depth).
Ex. Computer, Laptop, phone etc.
In mathematics there are many solid figures among few are is listed below along with formulas.
S. NO. Name Figure Nomenclature Volume Curved/ Total
Lateral surface
surface area area
1. Cuboid l → length lbh 2(l+b)h 2(lb+bh+hl)
h
b → breadth
b
l h → height

a
2. Cube a→ e d g e / s i d e a3 4a2 6a2
a
a
r

3. Right circular h r→ radius of πr2h 2πrh 2πr(h+r)


cylinder base
h→ height of
the cylinder

l h r→ radius
4. Right circular 1 2 πrl πr(l+r)
h→ height πr h
cone 3
r
l → slant height
l = r2 + h2
r
r → smaller radiusπ 2
(r +Rr+R2) π(r+R)l lateral sur-
Frustum of a h l R→ layer radius 3
5. face area +
cone l→ slant height π[R2+r2]
h→ height
R
l= ( R − r ) + h2
2

4 3
6. Sphere r→ radius πr 4πr2 4πr2
r 3

7. Hemisphere r 2 3
πr 2πr2 3πr2
r→ radius 3

4 inner = 4πr2 4π[R2 + r2]


r→inner radius
8. Spherical shell π[R3 – r3] outer= 4πR 2
R r R→ outer radius 3

188 QUANTITATIVE APTITUDE


stportal.mahendras.org

area of base × perimeter of lateral surface


h—height
height base × height area + 2(area
9. Right triangular of base)
prism

1 lateral surface
10. Right pyramid ×area of 1
3 ×perimeter area + area of
S la nt
he igh t
2 base
base × height of base × slant

height

3 ( sid e )
1 2
(h ) = × 6 × sid e 1 3 3 2
× 2 × (sid e )
(sid e )
3
3
12
11. Tetrahedron 1 4
r= × 6 × sid e
12
1
R = × 6 × sid e
4
(Where h height, r
inradius R circum-
radius)

Ex. A cylinder whose base of circumference is 6 m can Sol. 4 a = 2 (l+b) = 2 (14+20) = 68


roll at a rate of 3 rounds per second. How much
a = 17 = r
distance will the cylinder cover in 9 seconds?
1 22
Sol. Distance covered in one round = 2πr = 6 m A= × × 1 7 × 1 7 = 4 5 4 .1 4
2 7
Distance covered in 1 second = 3 × 6 = 18 m
Ex. What is the minimum amount of cardboard required
So distance covered in 9 seconds = 18×9= 162 m
to make a closed box of dimension 26 cm × 20 cm
Ex. If a square, circle and rectangle has same perimeter × 7 cm?
then which one of them has the maximum area?
Sol. Area of required cardboard
Sol. In such case the area in descending order is: Circle>
Square> Rectangle = 2 [26 × 20 + 26 × 7 + 20 × 7]

Ex. The perimeter of a square is equal to the perimeter = 2 (520+ 182 + 140)
of a rectangle of length 14 cm and breadth 20 cm. = 2 × 842
Find the area of a semicircle (approx.) whose radius
is equal to the side of the square. (in sq.cm.) = 1684 cm2

QUANTITATIVE APTITUDE 189


stportal.mahendras.org

EXERCISE
Q.1. What will be the area of trapezium whose parallel sides are 22 cm and 16 cm long, and the distance between them
is 11 cm?
(1) 209 cm2 (2) 282 cm2 (3) 265 cm2 (4) 179 cm2 (5) 302 cm2
Q.2. The perimeter of a rectangle is 42 m. If the area of the square formed on the diagonal of the rectangle as its side is
1
1 % more than the area of the rectangle, find the longer side of the rectangle.
12
(1) 19 m (2) 16 m (3) 9 m (4) 5 m (5) 12 m
Q.3. At the rate of Rs. 2 per sq m, cost of painting a rectangular floor is Rs 5760. If the length of the floor is 80% more
than its breadth, then what is the length of the floor?
(1) 25 m (2) 72 m (3) 67 m (4) 56 m (5) 46 m
Q.4. A 7 m wide path is to be made around a circular garden having a diameter of 7 m. What will be the area of the path
in square metre?
(1) 298 (2) 256 (3) 308 (4) 365 (5) 387
Q.5. The perimeter of a rectangle of length 62 cm and breadth 50 cm is four times perimeter of a square. What will be
the circumference of a semicircle whose diameter is equal to the side of the given square?
(1) 36 cm (2) 25 cm (3) 29 cm (4) 17 cm (5) 16 cm
Q.6. If the four equal circles of diameter is 6 meter touches its adjacent circle, then find the area of the region bounded
by the four circles.
(1) (35 - 9π) m2 (2) (36 - 8π) m2 (3) (36 - 9π) m2 (4) (25 - 9π) m2 (5) None of these
Q.7. The respective ratio of curved surface area and total surface area of a cylinder is 4:5. If the curved surface area of
the cylinder is 1232 cm2. What is the height?
(1) 14 cm (2) 28 cm (3) 7 cm (4) 56 cm (5) 24 cm
Q.8. A rectangular sheet of paper, when halved by folding it at mid-point of its longer side, results in a rectangle, whose
longer and shorter sides are in the same proportion as the longer and shorter sides of the original rectangle. If the
shorter side of the original rectangle is 2 unit, what is the area of the smaller rectangle?
(1) 1 2 sq. unit (2) 2 2 sq. unit (3) 4 2 sq. unit (4) Data inadequate (5) None of these
Q.9. Perimeter of a square and rectangle is equal. If length and breadth of rectangle are 12 cm and 10 cm respectively.
The area of square is what % more than that of rectangle?
(1) 5/6 % (2) 3/4 % (3) 2/3 % (4) 1% (5) None of these
Q.10. What is the volume of a cylinder whose curved surface area is 1408 cm and height is 16 cm?
2

(1) 7715 cm3 (2) 9340 cm3 (3) 8722 cm3 (4) 7346 cm3 (5) 9856 cm3
Q.11. The radii of two cylinders are in the ratio 3 : 2 and their curved surface areas are in the ratio 3 : 5. What is the ratio
of their volumes?
(1) 8 : 11 (2) 5 : 9 (3) 7 : 4 (4) 9 : 10 (5) 13 : 7
Q.12. A square park has a 2 m wide cross road in middle of it. If the side of park is 100 m then find the remaining area
of the park.
(1) 9650 m2 (2) 9596 m2 (3) 9600 m2 (4) 9604 m2 (5) None of these
Q.13. A car has wheels of diameter 70 m. How many revolutions can the wheel complete in 20 minutes if the car is
travelling at a speed of 110 m/s?
(1) 550 (2) 580 (3) 630 (4) 640 (5) 600
Q.14. A clock has its minute hand of length 7 cm. What area will it swept in covering 10 minutes?
(1) 32.17 cm2 (2) 25.67 cm2 (3) 45.45 cm2 (4) 41.23 cm2 (5) None of these
Q.15. The height of the cone is 24 cm and radius of cone is 7 cm . Find its volume.
2

(1) 1200 cm3 (2) 1232 cm3 (3) 1240 cm3 (4) 1260 cm3 (5) 1262 cm3

190 QUANTITATIVE APTITUDE


stportal.mahendras.org
Q.16. If the radius of a sphere increases by 4 cm then the surface area increases by 704 cm . The radius of the sphere
2

initially was?
(1) 5 (2) 4 (3) 6 (4) 8 (5) 10
Q.17. If the perimeter of square, circle, rectangle are equal. Then whose area is largest?
(1) Circle (2) Square (3) Rectangle
(4) All are equal (5) Can not determined
Q.18. A rectangular plot of grass is 50 m long and 40 m broad. From the center of each side a path of 3 m wide goes
across the center of the opposite side. Find the area of path?
(1) 270 (2) 280 (3) 251 (4) 261 (5) None of these
Q.19. The circumference of a circular garden is 1320 m.Find the area. Outside the garden, a road of 2 m width runs
around it. What is the area of this road and calculate the cost of gravelling it at the rate of 50 paise per sq. m .
(1) 2500.15 m2, Rs.1500.15 (2) 2652.57 m2, Rs.1326.285 (3) 2541.14 m2, Rs.1600.47
(4) 3245.78 m2, Rs.2000 (5) 4157.12 m2, Rs.1452.11
Q.20. A square shape of park of area 23,104 sq. m is to be enclosed with wire placed at heights 1,2,3,4 m above the
ground . Find required length of the wire ,,if its length required for each circuit is 10% greater than the perimeter
of the field ?
(1) 2675.2 m (2) 2145.12 m (3) 2750 m (4) 2478.11 m (5) 2400.5 m
Q.21. If the radius of the circular field is equal to the side of a square field .If the difference between the area of the
circular field and area of the square field is 5145 sq. m ,then calculate the perimeter of the circular field?
(1) 421 m (2) 315 m (3) 310 m (4) 308 m (5) 300 m
Q.22. A rectangular plot has a concrete path running in the middle of the plot parallel to the parallel to the breadth of the
plot. The rest of the plot is used as a lawn ,which has an area of 240sq. m. If the width of the path is 3m and the
length of the plot is greater than its breadth by 2m ,what is the area of the rectangular plot(in m )?
(1) 410 m (2) 288 m (3) 250 m (4) 300 m (5) 320 m
1
Q.23. A rectangular tank of length 3 7 m internally , 12 m in breadth and 8 m in depth is full of water.Find the weight
3
of water in metric tons, given that one cubic metre of water weighs 1000kg.
(1) 3584 metric tons (2) 4500 metric tons (3) 4101 metric tons (4) 3870 metric tons (5) 5721 metric tons
Q.24. The length and the breadth of a rectangular table are increased by 1 m each and due to this the area of the table
increased by 27 sq. m. But if the length is increased by 1 m and breadth decreased by 1 m, area is decreased by 7
sq. m. Find the perimeter of the table.
(1) 45 m (2) 52 m (3) 60 m (4) 72 m (5) None of these
Q.25. The perimeter of a square is twice the perimeter of a rectangle. If the perimeter of a square is 140cms and the
length of the rectangle is 20cm. Find the breadth of the rectangle?
(1) 18 (2) 20 (3) 15 (4) 12 (5) None of these
Q.26. A room is 7.5 m long, 5.5 m broad and 5 m high. What will be the expenditure in covering the walls by paper 40
cm broad at the rate of 80 paise per metre ?
(1) Rs. 255.5 (2) Rs. 260 (3) Rs. 282.25 (4) Rs. 244 (5) None of these
Q.27. 2
Total area of a circle and a square is equal to 6350 m . The radius of a circle is 35 m. What is the difference of the
circumference of the circle and area of square ? 
(1) 2260 m2 (2) 1280 m2 (3) 3280m2 (4) 2180 m2 (5) 2280 m2
Q.28. If the length of a rectangular plot is increased by 20% and the breadth of the plot is reduced by 20%, the area of
the plot is decreased by 8 m.2. What is the original area of the rectangular plot?
(1) 184 m.2 (2) 196 m.2 (3) 200 m.2 (4) 225 m.2 (5) None of these
Q.29. The ratio between the radius and height of a cone is 3:4.What is the curved surface area of the cone?
(1) 16 m2 (2) 15 m2 (3) 12 m2 (4) Data inadequate (5) None of these
Q.30. If the perimeter of a circle is increased by 30%. What will be the effect on the area of the circle?
(1) 48% increase (2) 52% increase (3) 50% decrease (4) 69% increase (5) None of these

QUANTITATIVE APTITUDE 191


stportal.mahendras.org

EXERCISE Explanation
Q.1.(1) Area of a trapezium = 1/2 (sum of parallel sides) Q.6.(3)
× (perpendicular distance between them)
1
= ( 22 + 16) × 11 = 209 cm2 3m 3m
2

Q.2.(5) Let the sides of the rectangle be l and b


respectively.
From the given data,
 1
( ) = 1 + 112  lb
2 3m 3m
l2 + b 2

Area of shaded region = 6×6 – π (3)2
 13 
=> l2 + b2 =  1 +  lb ............(1) Area of shaded region = (36 - 9π) m2
 12 
Q.7.(2) 4x = curved surface area = 1232
l+b = 21 .................(2)
x = 308
On solving above equations we get
5x = total surface area = 1540
lb = 108
Again l + b = 21 curved surface area = 2πrh
Hence longer side = 12 m total surface area = 2πr(r + h)
Q.3.(2) Let the length and the breadth of the floor be l m 2πr(r + h) = 1540
and b m respectively.
2πr2 + 2πrh = 1540
l = b + 80% of b = l + 0.8 b = 1.8b
5760 2πr2 = 1540 – 1232
Area of the floor = = 2880 sq m
2 r = 7; h = 28
l×b = 2880 Q.8.(2) According to the question,

i.e., l ×
l 2x : 2 : : 2 : x
= 2880
1.8 x= 2
l = 72 Area of the smaller rectangle

Q.4.(3) Area of the path = Area of the outer circle – Area = 2 × 2 = 2 2 = 2 sq. unit
of the inner circle Q.9.(1) Perimeter of rectangle= 2(l + b) = 22 × 2 = 44
2 2
7  7 Perimeter of square= perimeter of rectangle
= π  + 7 − π   = 308 sq m
2  2 = 44
Q.5.(1) Let the side of the square be a cm. Perimeter of Side of square= 11
the rectangle = 2(62 + 50)
Area of square= 121
= 224 cm
Perimeter of the square = 56 cm i.e. Area of rectangle= 12 × 10 = 120
4a = 56  121 − 100  5
Required% =  120  × 100 = 6 %
So a = 14
Q.10.(5) 2πrh = 1408, h = 16
Diameter, d of the semicircle = 14 cm
so r = 14
Radius = 7 cm.
Volume=π r2h =(22/7) × 14 × 14 × 16= 9856 cm3
Circumference of the semicircle=πr + 2r= r(π+2)
r1 3 3
22 Q.11.(4) = or 1= × rr22
7( + 2 ) = 36 cm r2 2 2
7
192 QUANTITATIVE APTITUDE
stportal.mahendras.org

Again Area of the road = p×(212)2-p×(210)2
CSA1 2π r1h1 3 = p×422×2 = 2652.57 m2
=
CSA2 2π r2 h2 = Therefore , cost of gravelling
5
= 2652.57 × 0.5 = Rs.1326.285
So h1/h2 = 2/5
Q.20.(1) Perimeter = 23,104 × 4 = (152 × 4) m
Volume1/ Volume2
= πr12h1/ πr22h2 152  110 
Length of each circuit = × 
4  100 
= 9/10
Q.12.(4) Remaining area of the park The wire goes around 4 times ,so the total length
of the wire required
= 100×100-(100×2+100×2-2×2) =9604 cm2
110 × 20 × 60  110 
= 152 × 4 ×  × 4 = 2675.2 m
Q.13.(5) Number of revolutions=
22
= 600
 100 
70 ×
7
Q.21.(4) Let the radius of the circular field and the side of
Q.14.(2) Length will be the radius, the square field be r Then,
so r = 7cm π × r 2 − r 2 = 5145
Minute hand covers 360 in 60 minutes So in 10 => r 2  ( 22 − 7 )  = 5145
o

minutes it covers = 60o 7
 
θ

r= × πr 2
=> r = 49 m
360
Therefore , circumference of the circular field
= 2×p×r = 308m

So area covered = 60 × 22 × 7 × 7 = 25.67 cm2
360 7 Q.22.(2) Let width be x m and length be (x+2)m
Area of path = 3x sq. m
1
Q.15.(2) Volume= π×r2 × h x(x+2) – 3x = 240
3
=> x2 – x – 240 = 0
1 22
× × 7 × 7 × 2 4 = 1 2 3 2cm 3 => x(x – 16) +15 (x – 16) = 0
3 7
=>(x – 16)(x + 15) = 0

=>x = 16
Q.16.(1) 4 π (r + 4 ) 2 − r 2  = 7 0 4 Length = 16 + 2 = 18m
On solving above equation Therefore , Area of plot = 16 × 18 = 288sq. m
r = 5cm  1 3
Q.17.(1) Circle Q.23.(1) Volume of water = 37   × 12 × 8m
3
Q.18.(4)  112 
Weight of water =   × 12 × 8 × 1000
50 m 3 

= 3584metric tons.
Q.24.(2) Let original length = l
3m 40 m breadth = b,
So area = lb
When l and b increased by 1: (l+1)(b+1)
= lb + 27
area of road = 3×50 + 3×40 −3² = 270 −9=261 Solve, l + b = 26
Q.19.(2) Circumference of the garden = 2×p×R = 1320 When l increased by 1, b decreased by 1: (l+1)
R= 210m (b-1) = lb – 7
Outer radius = 210 +2= 212 m Solve, l – b = 6

QUANTITATIVE APTITUDE 193


stportal.mahendras.org
Now solve both equations, Required difference = a2- 2pr
l = 16, b = 10
22
Perimeter = 2(16+10)=52m 2500 - 2 × ×35
7
Q.25.(3) 4 a = 2 × 2(l+b) = 2280
140 = 4 (20+b) Q.28.(3) Let original length and breadth be l and b
60 = 4 b respectively.
b = 15 cm Original area = l × b
Q.26.(2) Area of four walls New area = 1.2 l × 0.8 b
= 2 × 5 (7.5 + 5.5) = 130 m2
= 0.96 l × b
Area of required paper = 130 m2
According to question
Breadth of the paper = 40 cm = 0.4 m
⇒ Length of the paper 0.04 l × b = 8
=130/0.4= 325 m l × b = 8 ÷ 0.04
⇒ Cost of paper at 80 paise per meter = 200 m.2
= 325 × 0.80 = Rs.260
Q.29.(4) Data inadequate
Q.27.(5) πr 2 + a 2 = 6350
30 × 30
22 Q.30.(4) Req.% = 30+30+
× (35) + a 2 = 6350
2
100
7 = 69% increase
a 2 = 6350 − 3850
a = 2500
a = 50 meter

194 QUANTITATIVE APTITUDE

You might also like